Vous êtes sur la page 1sur 343

Differential Calculus

Partial Differentiation
(Partial Differential Coefficient)
Prepared by:
Dr. Sunil
NIT Hamirpur (HP)
(Last updated on 01-08-2009)
Latest update available at: http://www.freewebs.com/sunilnit/

(37 Solved problems and 00 Home assignments)

Introduction
Partial differentiation is the process of finding partial derivatives. A partial
derivative of several variables is the ordinary derivative with respect to one of the
variables when all the remaining variables are held constant. All the rules of
differentiation applicable to function of a single independent variable are also
applicable in partial differentiation with the only difference that while differentiating
(partially) with respect to one variable, all the other variables are treated
(temporarily) as constants.

Differential Coefficient:
If y is a function of only one independent variable, say x, then we can write
y = f(x).
Then, the rate of change of y w.r.t. x i.e. the derivative of y w.r.t. x is defined as
dy
 Lim
y
 Lim
y  y   y  Lim f x  x   f ( x )
dx x  0 x x  0 x x  0 x
where y is the change or increment of y corresponding to the increment x of the
independent variable x.
Partial Differentiation: Partial Differential Coefficient Prepared by: Dr. Sunil, NIT Hamirpur 2

Partial Differential Coefficient:


Let u be a function of x and y i.e. u = f(x, y).
Then the partial differential coefficient of u (i.e. f(x, y) w.r.t. x (keeping y as constant) is
defined and written as
u f x  x , y   f ( x , y) f
 Lim  u x  fx  .
x x  0 x x
Similarly, the partial differential coefficient of u (i.e. f(x, y) w.r.t. y (keeping x as
constant) is defined and written as
u f x , y  y   f ( x , y) f
 Lim  uy  fy  .
y y 0 y y
Similarly, we can find

 2u   u   2 u   u   2 u   u   2 u   u 
2
   , 2
 
 
 ,    ,   .
x x  x  y y  y  xy x  y  yx y  x 

 2u  2u
Also, it can be verified that  .
xy yx
Notation:
u f
The partial derivative is also denoted by or f x ( x , y, z) or fx or Dxf or
x x
f1 (x, y, z) , where the subscripts x and 1 denote the variable w.r.t. x which the partial
differentiation is carried out.
u f
Thus, we can have   f y x, y, z   f y  D y f  f 2 x, y, z  etc.
y y
The value of a partial derivative at a point (a, b, c) is denoted by
u u
  f x a , b, c  .
x x a , y  b , z c x a ,b,c 
Partial Differentiation: Partial Differential Coefficient Prepared by: Dr. Sunil, NIT Hamirpur 3

Geometrical Interpretation of partial derivatives:


(Geometrical interpretation of a partial derivative of a function of two variables)
z  f ( x , y) represents the equation of surface in xyz-coordinate system. Let APB
be the curve, which is drawn on a plane through any point P on the surface parallel to the
xz-plane.
As point P moves along the curve APB, its coordinates z and x vary while y remains
constant. The slope of the tangent line at P to APB represents the ‘rate at which z changes
w.r.t. x’.
z-axis B z-axis D
P P

A C

x-axis y-axis
O O

y-axis x-axis

Figure 1 Figure 2
z
Thus  tan  = slope of the curve APB at the point P (see fig.1).
x
z
Similarly,  tan  = slope of the curve CPD at the point P (see fig.2).
y
Higher Order Parallel Derivatives:
Partial derivatives of higher order, of a function f(x, y, z) are calculated by
successive differentiate. Thus, if u = f(x, y, z) then

 2u  2f   f   2u  2f   f 
     f xx  f11 ,      f yx  f 21 ,
x 2 x 2 x  x  xy xy x  y 

 2u  2f   f   2u  2f   f 
    xy  f  f 12 ,      f yy  f 22 ,
yx yx y  x  y 2 y 2 y  y 

 3u    2 f      f 
      f yzz  f 233 ,
z 2 y z  zy  z  z  y 

 4u    3 f       2 f 
     f zzyx  f 3321 .
xyz 2 x  yz 2  x  y  z 2 
Partial Differentiation: Partial Differential Coefficient Prepared by: Dr. Sunil, NIT Hamirpur 4

f
The partial derivative obtained by differentiating once in known as first order partial
x

 2f  2f  2f  2f
derivative, while , , , which are obtained by differentiating twice are
x 2 y 2 xy yx

known as second order derivatives. 3rd order, 4th order derivatives involve 3, 4, times
differentiation respectively.

 2f  2f
Note 1: The crossed or mixed partial derivatives and are, in general, equal
yx xy

 2f  2f
 .
yx xy
i.e. the order of differentiation is immaterial if the derivatives involved are continuous.
Note 2: In the subscript notation, the subscript are written in the same order in which
differentiation is carried out, while in '' notation the order is opposite, for example

 2u   u 
    f xy .
yx y  x 
Note 3: A function of 2 variables has two first order derivatives, four second order
derivatives and 2nd of nth order derivatives. A function of m independent variables will have
mn derivatives of order n.

Now let us solve some problems related to the above-mentioned topics:

y  2u  2u
Q.No.1.: If u  tan 1  , then prove that   0.
x x 2 y 2

y
Sol.: Here u  tan 1  .
x
u
Since  the p. d. coefficient of u w. r. t. x (keeping y as constant)
x
1  y  y
  2   2 .
y  x  x  y2
2
1
x2


 2u
    

  u     y  x 2  y 2 .0  2x  y 

 2 xy
....(i)
x 2  2 2
x  x  x  x  y 
x 2  y2
2
 
x 2  y2  
2
Partial Differentiation: Partial Differential Coefficient Prepared by: Dr. Sunil, NIT Hamirpur 5

u
Similarly,  the p. d. coefficient of u w. r. t. y (keeping x as constant)
y

1 1 x
   2 .
y  x  x  y2
2
1
x2


 2u

  u    x  x 2  y 2 .0  2 y x 
    2  
 2 xy  ....(ii)
y 2  2
y  y  y  x  y 
x 2  y2
2
x 2  y2     2

Adding (i) and (ii), we get

 2u  2u 2xy 2xy
    0.
x 2
y 2
x 2
y 2 2
 x 2
 y2 
2

This completes the proof.

 2u  2u
Q.No.2.: If u  f x  ay   x  ay  , then prove that  a 2. .
y 2 x 2
Sol.: Here u  f x  ay   x  ay  .

u 2u
  f x  ay   x  ay  and  f x  ay   x  ay 
x x 2
u
Also  f x  ay a   x  ay  a 
y

2u
and
y 2  
 f   x  ay  a 2    x  ay  a  .
2

 2u 2  u
 
2
 a f x  ay    x  ay   a . 2 .
2
 
y 2 x

 2u  2u
  a 2. .
y 2 x 2
This completes the proof.

xy3
Q.No.3: Show that Lim does not exist.
x , y 0,0  x 2  y 6

xy3 xy3 0.y3


Sol.: Now Lim = Lim = Lim = Lim 0 = 0 . ...(i)
x , y 0,0  x 2  y 6 x 0 x 2  y6 y 0 0  y 6 y0
y0
Partial Differentiation: Partial Differential Coefficient Prepared by: Dr. Sunil, NIT Hamirpur 6

xy3 xy3 x. 0
Again Lim = Lim = Lim = Lim 0  0 ....(ii)
x , y  0,0  x 2  y 6 x 0 x 2  y6 x 0 x 2  0 x 0
y0

Let x , y   0,0  along the curve x  my3 , where m is a constant.

xy3 my3.y3 y6 m m
 Lim = Lim 2 6 = m Lim 6 2  2 Lim1  2 . (iii)
x , y 0,0  x  y
2 6 y 0 m y  y 6 y 0 y ( m  1) m  1 y 0 m 1

From (i) and (ii) given limit is zero as x , y   0, 0  separately.


But from (iii) limit is not zero, but is different for different values of m.
Hence the given limit does not exist.

x2y
Q.No.4: Show that Lim does not exist.
x , y 0,0  x 4  y 2

x2y x2y 0.y


Sol.: Now Lim = Lim = Lim = Lim 0  0 . ...(i)
x , y  0,0  x 4  y 2 x 0 x 4  y 2 y 0 0  y 2 y 0
y0

x2y x2y x 2 .0
Again Lim = Lim = Lim = Lim 0  0 . ....(ii)
x , y 0,0  x 4  y 2 x 0 x 4  y 2 x 0 x 4  0 x 0
y0

Let x , y   0,0  along the curve x  my ,where m is a constant.

x2y my.y y2 m m
 Lim = Lim = m Lim  Lim 1  (iii)
x , y 0,0  x 4  y 2 y 0 m 2 y 2  y 2 y 0 y 2 ( m 2  1) m 2  1 y 0 m2  1
From (i) and (ii) given limit is zero as x , y   0, 0  separately.
But from (iii) limit is not zero, but is different for different values of m.
Hence the given limit does not exist.

y2  x 2
Q.No.5: If f x , y   , find the limit of f(x, y) when approaches origin (0, 0) along
y2  x 2
the line y = mx, where m is constant.
Sol.: Let x, y   0,0 along the curve y  mx where m is a constant.

y2  x 2 m 2 x 2  .x 2 m 2  1 x 2 m2  1 m2  1
 Lim = Lim = Lim  Lim 1  . Ans.
x , y 0,0  y 2  x 2 x 0 m 2 x 2  x 2 m 2  1 x 0 x 2 m 2  1 x 0 m2  1
Partial Differentiation: Partial Differential Coefficient Prepared by: Dr. Sunil, NIT Hamirpur 7

1  2 u  2u  2 u
Q.No.6.: If u  , where r 2  x 2  y 2  z 2 . Show that    0.
r x 2 y 2 z 2

Sol.: Since r 2  x 2  y 2  z 2 .
r r x
Differential partially w. r. t. x , we get 2r  2x   .
x x r
1
Now here u  ,
r
u 1 r 1 x x
Differential partially w. r. t. x , we get  2   2.   3 .
x r x r r r
2
r 3 3 2 x 2
r .1  x.3r . r  3r .
 2u x  
3 2 2
r   r  3rx  3x  1
 2  ...(i)
x r6 r6 r6 r5 r3
 2u 3y 2 1
Similarly, 2
 5
 ...(ii),
y r r3

 2u 3z 2 1
2
 5
 ...(iii)
z r r3
Adding (i), (ii) and (iii), we get

 2u
x 2

 2u
y 2

 2u
z 2

3
r 5
x 2

 y2  z2 
3
r 3

3
r 5
.r 2 
3
r 3

3
r 3

3
r3
 0.

This completes the proof.

Q.No.7: If u = xyz, find d 2u .


Sol.: We know that if u = f(x, y, z), then

u u u     
du  dx  dy  dz   dx  dy  dz u
x y z  x y z 

 d 2  ddu 
2
             
  dx  dy  dz  dx  dy  dz u   dx  dy  dz  u
 x y z  x y z   x y z 

 2 2 2 2 2 2 
 dx 2 2  dy 2 2  dz 2 2  2dxdy  2dydz  2dzdx u
 x y z xy yz zx 
Partial Differentiation: Partial Differential Coefficient Prepared by: Dr. Sunil, NIT Hamirpur 8

 2u 2  u
2
2  u
2
 2u  2u  2u
 2
dx   2
dy   2 dz 2  2 dxdy  2 dydz  2 dzdx (i)
x y z xy yz zx

Here u  xyz
u u u
 yz ,  zx ,  xy .
x y z

 2u  2u  2u
   0.
x 2 y 2 z 2

 2u  2u  2u
 z,  x,  y.
xy yz zx
2
 From (i), we have d u  2zdxdy  2xdydz  2 ydzdx .

u u
Q.No.8: Evaluate and , when (a) u  x y and (b) xy  yu  ux  1 .
x y

Sol.: (a) Given u  x y . ...(i)


Differentiate (i) partially w. r. t. x and y separately, we get
u

x x
 y
 
x  yx y 1 and
u

y y
 y
 
x  x y log x . Ans.

1  xy
(b) Given xy  yu  ux  1  u x  y   1  xy  u  ...(ii)
xy
Differentiate (ii) partially w. r. t. x and y separately, we get

u

  1  xy  x  y  y   1  xy .1
    
1  y2  
x x  x  y  x  y 2 x  y 2

and
u

  1  xy  x  y  x   1  xy .1
    
1  x2 . Ans.

y y  x  y  x  y 2 x  y 2
 2u  2u
Q.No.9: Verify that  , where u is equal to
xy yx

 x 2  y2 
(i) logy sin x  x sin y  , (ii) log ,
 xy 
 
x  y x
(iii) log tan   and (iv) x 2 tan 1   y 2 tan 1  .
y x y
Partial Differentiation: Partial Differential Coefficient Prepared by: Dr. Sunil, NIT Hamirpur 9

Sol.:(i) Here u  logy sin x  x sin y  . ...(i)


Differentiate (i) partially w. r. t. x, we get
u

y cos x  sin y  . ....(ii)
x y sin x  x sin y 
Differentiate (ii) partially w. r. t. y, we get

 2u   u  y sin x  x sin y cos x  cos y   y cos x  sin y sin x  x cos y 


  . (iii)
yx y  x  y sin x  x sin y 2
Differentiate (i) partially w. r. t. y, we get
u

sin x  x cos y  . (iv)
y y sin x  x sin y 
Differentiate (iv) partially w. r. t. x, we get

 2u   u  y sin x  x sin y cos x  cos y   sin x  x cos y y cos x  sin y 


   . (v)
xy x  y  y sin x  x sin y 2
 2u  2u
Hence from (iii) and (v), we get  .
xy yx
This completes the proof.
 x 2  y2 
(ii) Here u  log . ...(i)
 xy 
 
Differentiate (i) partially w. r. t. x, we get

u
 2
1 
xy2 x   x 2  y 2 y

1 x 2 y  y3

x 2  y2
x x  y 2
.
xy 2 x 2  y2
.
xy 
x x 2  y2
.
 (ii)

xy
Differentiate (ii) partially w. r. t. y, we get

 2u
 
  
  u  x 3  y 2 x  2 y   x 2  y 2 2xy  
 
4x 3y

4 xy
. (iii)

yx y  x 
x3  y2x  2
 
x 3  y2 x
2
 
x 2  y2 2

Differentiate (i) partially w. r. t. y, we get

u

1 
xy2 y   x 2  y 2 x
 2
1 xy 2  x 3

y2  x 2
y x 2  y 2
.
xy 2 x  y2
.
xy 
y x 2  y2
.
 (iv)

xy
Differentiate (iv) partially w. r. t. x, we get
Partial Differentiation: Partial Differential Coefficient Prepared by: Dr. Sunil, NIT Hamirpur 10

 2u
 
  
  u  yx 2  y3  2x   y 2  x 2 2xy 
 
 4xy3

4 xy
. ..(v)
xy x  y  yx 2  y3
2
  yx 2  y3 2
x 2
y 
2 2

 2u  2u
Hence from (iii) and (v), we get  .
xy yx
This completes the proof.
x
(iii) Here u  log tan  . ....(i)
y
Differentiate (i) partially w. r. t. x, we get
x
sec 2
u 1 x 1 y
 . sec 2 .  . ....(ii)
x tan x y y y tan x
y y
Differentiate (ii) partially w. r. t. y, we get
x   x x   x
2
y tan .  sec 2   sec 2 .  y tan 
 u   u  y y  y y y  y
   
yx y  x  x
y 2 tan 2
y
x x x x
x sec 2 tan  3x sec 2 tan 2
y y y y
 . (iii)
3 2 x
y tan
y
Differentiate (i) partially w. r. t. y, we get
x
sec 2
u 1 x x  x y
 .sec 2 .   .
2
. (iv)
y tan x y y  y tan x
2

y y
Differentiate (iv) partially w. r. t. y, we get
x   x x   x
2
y 2 tan .  x sec 2   x sec2 .  y 2 tan 
 u   u  y x  y y y  y
   
xy x  y  x
y 4 tan 2
y
Partial Differentiation: Partial Differential Coefficient Prepared by: Dr. Sunil, NIT Hamirpur 11

x x x x
x sec 2 tan  3x sec 2 tan 2
y y y y
 . (v)
x
y3 tan 2
y

 2u  2u
Hence from (iii) and (v), we get  .
xy yx
This completes the proof.

 y x
(iv) Here u  x 2 tan 1   y 2 tan 1  . (i)
x  y
Differentiate (i) partially w. r. t. x, we get
 
 
u 1  y   1 y 1  1 
 x2 .    2 x tan  y 2
.  
x 2
y  x2   x x2  y 
1  1 2
x2  y 

x2y y y3 1 y x 2 y  y3 y
  2 x tan 1  2  2 x tan   2x tan 1  y . (ii)
x 2  y2 x x y 2 x x y 2 2 x

Differentiate (ii) partially w. r. t. y, we get

 2u   u  1 1 2x 2 2x 2  x 2  y 2 x 2  y 2
  2x.    1   1   2 (iii)
yx y  x  y2  x  x 2  y2 x 2  y2 x  y2
1
x2
Differentiate (i) partially w. r. t. y, we get
 
 
u 1 1  x 1  x 
x 2
.  2 y tan 1  y 2 . 

 .
y 2
y x  y x2  y 2 
1 2  1 2 
x  y 


x3
 2 y tan 1
x
 2
xy 2

x 3  xy 2
 2 y tan 1 x

x x 2  y2  
 2 y tan 1
x
2 2 2 2 2 2 2
x y y x y x y y x y y

u x
  x  2 y tan 1 . (iv)
y y
Differentiate (iv) partially w. r. t. x, we get
Partial Differentiation: Partial Differential Coefficient Prepared by: Dr. Sunil, NIT Hamirpur 12

 2u   u    1 x  1 1 2 y2 x 2  y2
   x  2 y tan   1  2y 2
.  1 2  . (v)
xy x  y  x  y x y x  y2 x 2  y2
1
y2

 2u  2u
Hence from (iii) and (v), we get  .
xy yx
This completes the proof.
2
 3 3 3

 
Q.No.10:If u  log x  y  z  3xyz , show that  
  
  u 
9
x  y  z 2
.
 x y z 
2
          u u u 
Sol.: Since     u         .
 x y z   x y z  x y z 


Here u  log x 3  y3  z3 .  .(i)
Differentiate (i) partially w. r. t. x ,y and z separately, we get

u


3 x 2  yz
,
u

3 y 2  xz
and
u


3 z 2  xy
,
 
x x 3  y3  z3  3xyz y x 3  y3  z3  3xyz z x 3  y3  z3  3xyz

     

 u u u  3 x 2  y 2  z 2  xy  yz  zx 3 
 x y z 
3 3
 3
x  y  z  3xyz


x  y  z 
.

     3    3    3   3 
Hence              
 x y z  x  y  z  x  x  y  z  y  x  y  z  z  x  y  z 
3 3 3
  
x  y  z  2
x  y  z  2
x  y  z 2
9
 .
x  y  z 2
2
     9
Hence     u  .
 x y z  x  y  z 2
This completes the proof.

Q.No.11: If u  e xyz , show that


 3u
xyz

 1  3xyz  x 2 y 2 z 2 e xyz . 
Sol.: Here u  e xyz . Now
u

z z
 xyz
e  
 e xyz xy .
Partial Differentiation: Partial Differential Coefficient Prepared by: Dr. Sunil, NIT Hamirpur 13


 2u

  u   xyz
  
yz y  z  y
  
e xy  xy e xyz xz  e xyz x  x 2 yze xyz  e xyz x  x 2 yz  x e xyz   

And hence
 3u

   2u   2
 
xyz x  yz  x

x yz  x e xyz  2xyz  1e xyz  x 2 yz  x e xyz yz    

 2xyz  1  x 2 y2z 2  xyz e xyz  x 2 y 2z 2  3xyz  1 e xyz .   
This completes the proof.


Q.No.12: If u  z  1  2xy  y 2 
1 / 2
, prove that

(i) x
z
x
y
z
y
 y 2z3 , (ii)
 
x 
1  x 2 u 

  2 u 
y   0.
x  y  y 
 

Sol.: (i) Here z  1  2 xy  y 2 
1 / 2
. ....(i)
Differentiate (i) partially w. r. t. x and y separately, we get
z
x
1

  1  2 xy  y 2
2

3 / 2
 2 y   y1  2 xy  y 2 
3 / 2
.

and
z
y
1

  1  2 xy  y 2
2

3 / 2
 2x  2 y   x  y 1  2xy  y 2 
3 / 2
.

Hence x
z
x
y
z
y
 x  y 1  2 xy  y 2

  3 / 2  
  y x  y  1  2xy  y
2
 3 / 2 



 1  2 xy  y 2  xy  xy  y   y z .
3 / 2 2 2 3

This completes the proof.


  u    2 u 
(ii) To show: 
x 

1  x2  y   0.
x  y  y 


Here u  1  2 xy  y 2  1 / 2
. ....(i)
Differentiate (i) partially w. r. t. x and y separately, we get
u
x

 y 1  2 xy  y 2 
3 / 2
and
u
y

 x  y  1  2 xy  y 2  3 / 2
.

Now
 

x 

1  x2 
u   
 
x  x 
1  x 2 y 1  2xy  y 2   
3 / 2 

Partial Differentiation: Partial Differential Coefficient Prepared by: Dr. Sunil, NIT Hamirpur 14



 y 1  x 2

x 1  2xy  y  2 3 / 2

 1  2xy  y 2  3 / 2 
x
 
1  x2 




 y 1  x 2   32 1  2xy  y  2 5 / 2
 2 y   1  2xy  y 2 
3 / 2
 2x 
 
 3y 1  x 2
 y
  
2x   3y  3x 2 y  2 x  4 x 2 y  2 xy 2 
  y 
 
 1  2xy  y 2 5/2

1  2xy  y 2
3/ 2



  1  2 xy  y 2 5/2
 
 

 
1 
 x 2 u 
 
y 3y  2x  x 2 y  2xy2
.

x  x  1  2xy  y2 
5/ 2

Again
  2 u    2
y  
y  y  y 
y x  y  1  2 xy  y 2  
3 / 2 




y
 
xy 2  y3 1  2xy  y 2 
3 / 2


 1  2 xy  y 2 2xy  3y  xy  y   32 1  2xy  y   2x  2y
3 / 2 2 2 3 2 5 / 2

2 xy  3y 3xy  y x  y  2 xy  3y 1  2 xy  y   3xy  y x  y 


2 2 3 2 2 2 3
  
1  2xy  y  1  2xy  y 
2 3/ 2
1  2xy  y 
2 5/ 2 2 5/2

2xy  4x 2 y 2  2xy3  3y 2  6xy3  3y 4  3y 2 x 2  6xy3  3y 4



1  2xy  y  2 5/2

2
2 xy  3y  x y  2 xy  y3y  2x  x y  2xy 
2 2 3 2 2
 
1  2xy  y  1  2xy  y 
2 5/2 2 5/ 2

   1  x   .
  u  2   u  2
or  y
y  y x 
 x 

Hence
 

x 

1  x2 
u    2 u 
 y   0.
x  y  y 

This completes the proof .


 
Q.No.13: If u  tan 1 
xy
 1 x  y 
2 2
 , prove that
 2u
x y
 1  x 2  y2  
3 / 2
.
 
 xy 
Sol.: Here u  tan 1  . ...(i)
 1  x 2  y2 
 
Partial Differentiation: Partial Differential Coefficient Prepared by: Dr. Sunil, NIT Hamirpur 15

Differentiate (i) partially w. r. t. y, we get

z   xy 
 tan 1  
y y  1  x 2  y2
   

1
1  x 2  y 2 .x  xy. .2 y
2 2
1 2 1 x  y

1
2 2
x y
.
1  x 2
 y2 
2 2
1 x  y


1  x 2
 y2  .

x 1  x 2  y 2  xy2  
x  x 3  xy 2  xy2
1  x 2  y2  x 2 y 2
1  x 2

 y2 1  x 2  y2 1  x  y  x y  1  x 2 2 2 2 2
 y2

x  x3 x 1  x  2
 
1  x 2
 y 2  x 2 y2 1  x 2  y2  1  x   y 1  x  1  x  y
2 2 2 2 2

x
 . ...(ii)
1  y  1  x
2 2
 y2
Differentiate (ii) partially w. r. t. x ,we get

 2z   z    x 
   
 
xy x  y  x  1  y 2 1  x 2  y 2 
   
 

1  x 2  y 2 1  y 2 .1  x  1  y 2 .  2x
  

 2 1  x 2  y 2 

1  x 2  y2 1  y2  x 2 1  y2     
 2
 
1  y 2 1  x 2  y2
2
1  y 2 1  x 2  y2 1  x 2  y 2   

1  y 1  x  y  x  
2 2
1  y 2 2 2 2

1
.
1  y  1  x  y  1  y  1  x  y 
2 2 2 2 3/ 2 2 2 2 2 3/ 2
1  x 2
 y2 
3/ 2

 2z 1
Hence  .
xy 1  x 2  y 2  3/ 2

This completes the proof.

Q.No.14: If z 2  t 2  4x  y2  0 and z3  t 3  2x 3  3y  0 ;
z t
Evaluate and .
x x
Partial Differentiation: Partial Differential Coefficient Prepared by: Dr. Sunil, NIT Hamirpur 16

Sol.: Here z 2  t 2  4x  y2  0 and z3  t 3  2x 3  3y  0 .


Differentiate partially the given equations w. r. t. x, considering z and t as function of
x, we get
z t
2z  2t 4  0
x x
z t
and 3z 2  3t 2  6 x 2  0 .
x x
z t
Solve these equations simultaneously for and .
x x
z t
x x 1
 
 
2 t.  6 x 2  4.3.t 2  12z 2  12zx 2 6zt 2  6tz 2
.

z t
x x 1

 
 

12t t  x 2 12z x 2  z 6 tzt  z 
.

z t
x 1 x 1
Considering

12 t t  x 2

6 tzt  z  
and 
12z x  z 6 tzt  z 
2
.
 
We get 

z 12t t  x 2

2 x2  t
and
  
t 12z x 2  z 2 x 2  z
  . Ans.
   
x 6 tzt  z  zz  t  x 6 tzt  z  t t  z 
x 2
2
ke 4a y
u  2u
Q.No.15: If u  , then prove that  a2 2 .
y y x
x 2
x2 x2
4a 2 y  2   
ke u k 4a 2
y x   k   1 .e 4a y
2
Sol.: Here u  , then  .e
y y y  4a 2 y 2   2 y 3 / 2 
   
x 2
4a 2 y
 x2 1 
 ke  2 5 / 2  3/ 2  .
 4a y 2 y 

x 2 x 2
u k 4a 2
y

 2x 

kx 4a 2 y
Also  e  4a 2 y  e
x y   2a 2 y 3 / 2
Partial Differentiation: Partial Differential Coefficient Prepared by: Dr. Sunil, NIT Hamirpur 17

x 2 x 2 x 2
 u2
k 4a 2 y kx 
4a 2 y  2x  4a 2 y
 x2 1 
and   .e  .e  ke   
x 2 2 3/ 2
2a y 2 3/ 2
2a y  4a 2 y  4 5/ 2
 4a y 2a 2 y 3 / 2 
 

 2u
1 u u 2
2 u
 2  2 , hence a .
x a y y x 2
This completes the proof.
r2
n
 1   2   
Q.No.16: If   t e 4t , find what value of n will make r  .
r 2 r  r  t
r2

Sol.: Here   t n e 4t . ....(i)
Differentiate (i) partially w. r. t. r, we get
 r2    r2  r2 r2
   n  4 t     2r  1 
 t e  t n . e 4 t   t n .e 4 t      t n 1r.e 4 t .
r r   r    4t  2
   
r2
 1 
r2   t n 1r 3.e 4 t . ...(ii)
r 2
Differentiate (ii) partially w. r. t. r, we get
 r2   r2 
  2     1 n 1 3  4 t  t n 1   3  4 t 
r   t r e  r e
r  r  r  2  2 r  
   
 r2 r2  n 1  4  r 
2
t n 1  2  4 t   2r   t  r 
 3r  e 4 t 
3 2
 3r .e  r .e 4 t      
2   4t  2  2t  
   
 r2 
1   2   t n 1  2 r 4   4 t 
 2 r    2  3r  e  ...(iii)
r r  r  2r  2t  
 
 r2  r2
  r2 
r2 r2
 r2 
   n  4 t  
Now  t e  t n .e 4 t  2   nt n 1.e 4 t  e 4 t  t n  2  nt n 1 
t t    4t   4 
   

r2
  n 1  r 2 
e 4t  t   n   . ...(iv)
  4t  
Partial Differentiation: Partial Differential Coefficient Prepared by: Dr. Sunil, NIT Hamirpur 18

 r2  r2
  
1   2    t n 1  2 r 4   4 t   r2
But 2  r    2  3r  e   e 4 t  t n 1   n  
r r  r  t 2r  2t    4t 
  
 

1  2 r 4   r 2  1  r 2   r 2 


  3r      n  .    3      n 
2r 2  2 t   4 t  2  2 t   4 t 
3 3
  n . Hence n   . Ans.
2 2

Q.No.17: If u  Aegx sinnt  gx  , where A , g , n are positive constants, satisfies the

u  2u n
heat conduction equation   2 , then prove that g  .
t x 2
or

u  2u
The equation   2 refers to the conduction of heat along a bar without radiation,
t x
n
show that if u  Ae gx sin nt  gx , where A , g , n are positive constants then g  .
2

u
Sol.: Here u  Aegx sinnt  gx  ,we have  Ae  gx cos nt  gx n .
t

Also
u
x

 A e  gx  g sin nt  gx   e  gx cos nt  gx  g  

 A g  e gx sinnt  gx   e gx cosnt  gx  
 Age gx sinnt  gx   cosnt  gx 

and
 2u
x 2

  Ag e gx cosnt  gx  g   sin nt  gx  g 

 sinnt  gx   cosnt  gx e gx  g 


 Age gx  g cosnt  gx   sinnt  gx   sinnt  gx   cosnt  gx 

 Agegx  g 2 cosnt  gx   2Ag2egx cosnt  gx  .

u  2u
Also given   2  Aegx cosnt  gx n  2Ag2e gx cosnt  gx 
t x
Partial Differentiation: Partial Differential Coefficient Prepared by: Dr. Sunil, NIT Hamirpur 19

n n
 g2  . Hence  g  .
2 2
This completes the proof.

Q.No.18: (a) Show that at the point for surface x x y y z z  const. , where x = y = z

 2z 1
 .
xy x log(ex )

 3u
(b) If u  e xyz ; find the value of .
xyz

Sol.: (a) Given x x y y z z  const. , where x = y = z.


Taking log both sides, we get
x log x  y log y  z log z  log c
Differentiating z partially w. r. t. x [keeping y as constant] , we get
z 1  log x z 1  log y
1  log x   1  log z  z 0   . Similarly,  .
x x 1  log z y 1  log z

 2z   z    z  z   1  log x   1  log y 
Now            
xy y  x  z  x  y z  1  log z   1  log z 

1  log z .0  1  log x  1


 z  1  log y    1 1  log x 1  log y 
 
1  log z 2
 1  log z  z 1  log z 3
Since x = y = z,

 2z 1 1  log x 2 1 1 1
     .
xy x 1  log x 3 x 1  log x  x log e  log x  x log(ex )

 2z 1
Hence  . This completes the proof.
xy x log(ex )

(b) Here u  e xyz .

Now
u

z z
 xyz
e  
 e xyz xy .


 2u

  u   xyz
  
yz y  z  y
   
e xy  xy e xyz xz  e xyz x  x 2 yze xyz  e xyz x  x 2 yz  x e xyz  
Partial Differentiation: Partial Differential Coefficient Prepared by: Dr. Sunil, NIT Hamirpur 20

And hence
 3u

   2u   2
 
xyz x  yz  x
  
x yz  x e xyz  2 xyz  1e xyz  x 2 yz  x e xyz yz  
  
 2xyz  1  x 2 y 2z 2  xyz exyz  x 2 y2z 2  3xyz  1 e xyz . Ans. 
 2z  2 z  2z
Q.No.19: If z  xf x  y   ygx  y  , show that 2   0.
x 2 xy y 2

Sol.: Since z  xf x  y   ygx  y  . ...(i)


z
  xf / x  y   f x  y   yg / x  y  .
x

 2z
and  2
 f / x  y   xf // x  y   f / x  y   yg // x  y  . ...(ii)
x
z
Also  xf / x  y   yg / x  y   gx  y  .
y

 2z
and  2
 xf // x  y   yg// x  y   g / x  y   g / x  y  . ...(iii)
y
z
Now since  xf / x  y   f x  y   yg / x  y  .
x

 2z
  xf // x  y   f / x  y   g / x  y   yg // x  y  . ...(iv)
xy

 2z  2z  2 z
Putting these values in 2  , we get
x 2 xy y 2

 2z  2z  2 z
2   0 . This completes the proof.
x 2 xy y 2

y z x u u u
Q.No.20: If u    , then show that x y z  0.
z x y x y z
y z x
Sol.: Since u    . ...(i)
z x y
Differentiate (i) partially w. r. t. x, y and z separately, we get

u  1 z  u  1 x  u  1 y 
   2  ,    2  and    .
x  y x  y  z y  z  x z 2 
Partial Differentiation: Partial Differential Coefficient Prepared by: Dr. Sunil, NIT Hamirpur 21

u u u 1 z  1 x  1 y 
Hence x y z  x   2   y  2   z  2   0 .
x y z y x  z y  x z 
This completes the proof.
u u
Q.No.21: If u  eax  byax  by  , then prove that b a  2abu .
x y

Sol.: Since u  eax  byax  by . ....(i)


Differentiate (i) partially w. r. t. x and y separately, we get
u
 eax  by a.ax  by   e ax  by . / ax  by a ,
x
u
and  eax  by b.ax  by   eax  by . / ax  by  b 
y
u u
Now b a  2abeax  byax  by   2abu .
x y
This completes the proof.
Q.No.22: If x  r cos  , y  r sin  , then show that
x r  1 x
(i)  , (ii) r  .
r x x r 

Sol.: (i) Given x  r cos  , y  r sin   x 2  y 2  r 2 ......(i)


Differentiating (i) w. r. t. x partially (keeping y as constant), we get
r r r
2 x  0  2r r  x  r cos    cos  ......(ii)
x x x
x
Also since x  r cos    cos  . ....(iii)
r
x r
Comparing (ii) and (iii) ,we get  . Ans.
r x
This completes the proof.
 1 x
(ii) To show : r  .
x r 
y y
Now since x  r cos  , y  r sin   tan      tan 1
x x
Partial Differentiation: Partial Differential Coefficient Prepared by: Dr. Sunil, NIT Hamirpur 22

 1 y y
  . 2   2 .
x y  x  x  y2
2
1
x2

  y   y  y
Now r  r. 2   r.  . ...(i)
x 2   r2 
x y  r

x 1 x y
since x  r cos    r sin     sin    . .....(ii)
 r  r
 1 x
Comparing (i) and (ii) ,we get r  . This completes the proof.
x r 
Q.No.23: If x  r cos  , y  r sin  , prove that

1   r    r  
2 2
 2r  2r
(i)  
      
x 2 y 2 r  x   y  

 2  2
(ii)  0 x  0, y  0
x 2 y 2
Sol.: (i) Given x  r cos , y  r sin  .
[By looking at the answer we find that we need partial derivative of r w. r. t. x and y.
Therefore, let us express r as an explicit function of x and y]
Squaring and adding x  r cos  , y  r sin  ; we find that

r 2  x 2  y2 i.e. r  x 2  y 2 . ...(i)
Differentiating (i) w. r. t. x partially (keeping y as constant), we get
r 1 2
 x  y2
x 2
 1 / 2

.2 x  x 2  y 2  1 / 2
.x 
2
x
2

x
r
. ...(ii)
x y
Similarly, differentiating (i) w. r. t. y partially (keeping x as constant), we get
r 1 2
 x  y2
y 2
 
1 / 2

.2 y  x 2  y 2 1 / 2
.y 
2
y
2

y
r
. .(iii)
x y
Again differentiating(ii) w. r. t. x partially (keeping y as constant), we get
  r x
r x   x r  r  x r  x.
 2r  x x x x 
2 2 2
r  r x  y .
    
x 2 x  r  r2 r2 r2 r3 r3
Again differentiating(iii) w. r. t. y partially (keeping x as constant), we get
Partial Differentiation: Partial Differential Coefficient Prepared by: Dr. Sunil, NIT Hamirpur 23


2 y   y  r  r  y r r  y. y 2 2 2
r
 r  y y y y r  r y  x .
2
   2
 2
 2
y x  r  r r r r3 r3

 2r  2r y2 x2 x 2  y2 r2 1
L.H.S.= 2
 2
 3
 3
 3
 3
 .
x y r r r r r

1  r   r   1  x 2 y 2  1  x 2  y 2  1  r 2  1
2 2
R.H.S.=         2  2    2    2  .
r  x   y   r  r r 
 r 
 r  r  r  r
 
 L. H.S. R. H.S. This completes the proof.
y
(ii) It is given that x  r cos  , y  r sin  . Dividing ,we get tan  
x
y
   tan 1 . ...(i)
x
Differentiating (i) w. r. t. x partially (keeping y as constant), we get
  y 1  y 1  y  y
 tan 1   
2  2    2
2
. ...(ii)
x x x y x  x  y  x  x  y2
1 1
x2 x2
Again differentiating (ii) w. r. t. x partially (keeping y as constant), we get

 2

 

y  x 2  y 2 0    y .2x
 
 2xy  . ...(iii)
x 2  2 2
x  x  y 
x 2  y2
2
x 2  y2     2

Differentiating (i) w. r. t. y partially (keeping x as constant), we get


  y 1  y 1 1 x
 tan 1       . ...(iv)
y y x y 2 y  x  y2  x  x 2  y2
1 1
x2 x2
Again differentiating (iv) w. r. t. y partially (keeping x as constant), we get

 2

  x  x 2  y 2 0  x .2 y
 

 
2xy  . ...(v)
y 2 y  x 2  y 2  x 2  y2
2
x 2  y2    
2

Adding (iv) and (v), we get

 2  2 2xy 2 xy
L.H.S.      0  R.H.S. This completes the proof.
x 2
y x  y 
2
x 2
y 2 2 2 2 2

Q.No.24: If u  f ax 2  2hxy  by2  and v  ax 2  2hxy  by2  , prove that
Partial Differentiation: Partial Differential Coefficient Prepared by: Dr. Sunil, NIT Hamirpur 24

  v    v 
u   u  .
y  x  x  y 


Sol.: Given u  f ax 2  2hxy  by2  ...(i)


and v   ax 2  2hxy  by2  ....(ii)
Differentiating (ii) partially w. r. t. x and y separately, we get
v
x
 
  / ax 2  2hxy  by 2 .2ax  2hy    / .2ax  2hy 

v
y
 
 / ax 2  2hxy  by 2 .2by  2hx   / .2by  2hx 

Now L.H.S.=
  v  
u  
y  x  y

f ./ .2ax  2hy  
 f / .2by  2hx ./ .2ax  2hy  f .// .2by  2hx 
. 2ax  2hy  f ./ .2h

 2ax  2hy 
. 2by  2hx . f / /  f//  2h.f ./ .  ....(iii)

R.H.S.=
  v  
 u  
x  y  x

f ./ .2by  2hx  
 f / .2ax  2hy./ .2by  2hx   f .// .2ax  2hy
. 2by  2hx   f ./ .2h

 2ax  2hy  
. 2by  2hx . f / /  f//  2h.f ./ .  (iv)

  v    v 
From (iii) and (iv), we have u    u  . This completes the proof.
y  x  x  y 

 
Q.No.25: If u  x 2  y 2 f t  , where t = x y, prove that

 2u
xy
 
 x 2  y 2 tf // t   3f / t  
   
Sol.: Given u  x 2  y 2 f t   x 2  y2 f xy   x 2f xy   y2f xy  . (i)
Differentiating (i) partially w. r. t. x and y separately, we get
u
x
  
 2 x.f xy   x 2 .f / xy y  y 2 .f / xy y  2 xf xy   x 2 yf / xy   y3f / xy 


 2u

 2u

  u  

xy yx y  x  y

2 xf xy   x 2 yf / xy   y3f / xy  
Partial Differentiation: Partial Differential Coefficient Prepared by: Dr. Sunil, NIT Hamirpur 25

  
 2xf / xy x  x 2 y.f // xy x  x 2 .f / xy   y3.f // xy x  3y 2 .f / xy  
 2x f t   x yf
2 / 3 //
t   x 2f / t  y3xf // t   3y2f / t 

 3x 2f / t   3y2f / t   x 3 y  y3x f // t  
  
 3 x 2  y 2 f / t   xy x 2  y 2 f // t  
  
 x 2  y 2 tf // t   x 2  y 2 3f / t  
Hence
 2u
xy
  
 x 2  y 2 tf // t   3f / t  . This completes the proof.

Q.No.26: If u and v are functions of x and y defined by x  u  e  v sin u ,


u v
y  v  e  v cos u , then prove that  .
y x

Sol.: Given x  u  e  v sin u and y  v  e  v cos u .


Differentiating both the equations partially w. r .t. x and y separately, we get

1
u
x
 e  v cos u
u
x
 v 
 e  v    sin u  1 
 x 
u
x
v

1  e  v cos u  e  v sin u
x
 (i)

0
u
y
 e  v cos u
u
y
 v 
 e  v    sin u  0 
u
1  e  v cos u  e  v
v

sin u  (ii)
 y  y y

0
v
x
 e  v ( sin u )
u
x
 v 
 e  v    cos u  0 
 x 
v
x
u
1  e  v cos u  e  v sin u
x
  (iii)

1
v
y
 e  v ( sin u )
u
y
 v 
 e  v    cos u  1 
v
y
u
1  e v cos u  e  v sin u
y
  (iv)
 y 


Multiplying (i) by e v sin u and (iii) by 1  e  v cos u and then adding, we get 
v e v sin u
 (v)
x 1  e 2 v

 
Multiplying (ii) by 1  e v cos u and (iv) by e  v sin u and then adding, we get

u e  v sin u
 (vi)
y 1  e  2 v
From (v) and (vi), we get
Partial Differentiation: Partial Differential Coefficient Prepared by: Dr. Sunil, NIT Hamirpur 26

u v
 . This completes the proof.
y x
2
 z z   z z 
Q.No.27: If zx  y   x  y , show that     41 
2 2
  .
 x y   x y 

x 2  y2
Sol.: Since zx  y   x 2  y 2  z  . ...(i)
xy
Differentiating (i) partially w. r. t. x and y separately, we get

 
z x  y .2x  x 2  y 2 .1 x 2  y 2  2 xy
 
x x  y 2 x  y 2
 
z x  y .2 y  x 2  y 2 .1 y 2  x 2  2xy
 
y x  y 2 x  y 2
2 2
 z z   x 2  y 2  2 xy y 2  x 2  2xy 
Now L.H.S.=       
 x  y  x  y 2 
2
 x y 
2 2 2
 ( x 2  y 2  2xy)  ( y 2  x 2  2xy)   2x 2  2 y 2   2x  y x  y  
       
 x  y 2   x  y  
2
 x  y 
2

2
 2x  y   4x  y 2
   . (ii)
 x  y   x  y2
 z z 
R.H.S.= 41 

   41 

x 2  y 2  2 xy

 
y 2  x 2  2 xy 
 x y   x  y 2 x  y 2 

 ( x 2  y 2  2 xy)  ( x 2  y 2  2xy)  ( y 2  x 2  2 xy)   x 2  y 2  2 xy 


 4   4  
 x  y 2   x  y 
2


4x  y 2
 . (iii)
x  y 2
From (ii) and (iii), we have L.H.S.=R.H.S. This completes the proof.

 3u  3u
Q.No.28: If u  x y , show that  .
x 2y xyx

Sol.: Since u  x y . (i)


Partial Differentiation: Partial Differential Coefficient Prepared by: Dr. Sunil, NIT Hamirpur 27

 3u
For , first differentiate (i) partially w. r. t. y and then twice w. r. t. x
x 2y
u
  x y log x . Now differentiate twice w. r. t. x, we get
y

 2u 1
 x y .  log x.yx y 1  x y 1  y log x y 1  x y 1 1  y log x  and
xy x

 3u    2u  y
  . y  1x y  2  x y 1.  x y  2 1  y log x y  1  y . (ii)
  1  y log x 
x 2y x  xy  x

 3u
For , first differentiate (i) partially w. r. t. x , then y and then x
xyx
u
  yx y 1 . Now differentiate partially w. r t. y, we get
x
 2u   u 
  y.x y 1 log x  x y 1  1  y log x x y 1 .
yx y  x 
Now again differentiate partially w. r. t. x, we get

   2u 
  x 1  y log x y  1  y .
y2
 (iii)
x  yx 

 3u  3u
Hence from (ii) and (iii),  . This completes the proof.
x 2y xyx

x2 y2 z2
Q.No.29: If    1 , where u is a function of x , y , z ; prove that
a2  u b2  u c2  u
2 2 2
 u   u   u   u u u 
         2 x  y  z  .
 x   y   z   x y z 

x2 y2 z2
Sol.: Since    1.
a2  u b2  u c2  u
Now differentiate partially w. r. t. x, we get

a 2
  u   u 
 u .2 x  x 2    y 2    z 2  
 x    x  
 u 
 x   0
a2  u
2
 
b2  u
2

c2  u
2
 
Partial Differentiation: Partial Differential Coefficient Prepared by: Dr. Sunil, NIT Hamirpur 28

a 
2  u   u 
 u .2 x  x 2   y 2   z 2  
 u 
 x    x    x   0

  a2  u
2
   
b2  u
2
c2  u
2

a  u .2x  x  ux  y  ux  z  ux 
2 2 2 2

  
a  u  2 2
b  u  c  u 2 2 2 2

 u   u   u 
x 2   y2   z2  
2x  x    x    x 
 2

a u

a2  u
2
 
b2  u
2
 
c2  u
2
  
2x  x2 y2 z2   u 
 

a2  u

 
 a2  u
  b
2

2
u  
2
 2  x 
c  u   
2

u 2x  x2 y2 z2
 

x
 2

a  u  a 2  u   b
2

2
u  
2
 2
c  u 
2

u 2y  x2 y2 
z2
 ,
Similarly
y
 2
  
b  u  a 2  u  b2

2
u  
2
 2
c 2  u  
u 2z  x2 y2  z2
 2    

z c  u  a 2  u

   b
2 2
u  
2 2 2
c u  
2 2 2
 2 x   2 y   2z 
2 2
 u   u   u 
Now L.H.S.=         
2  2

   
a  u   b 2  u   c2  u 

     
2
 x   y   z   x2 y2 z2 
   
 a 2  u 2 b2  u 2 c2  u 2 
       
4

 x2 y2 z2 
   

 a2  u
  b
2 2
u  
2 2 2
c  u  
Partial Differentiation: Partial Differential Coefficient Prepared by: Dr. Sunil, NIT Hamirpur 29

 2x 2y 2z 
2  x.  y.  z. 
 u
R.H.S.= 2 x y
u u 
 z   
 2
a u
2 2
b u
2 2 2
c  u       
 x y z   x2 y2 z2 
   
 a 2  u 2 b2  u 2 c2  u 2 
       
4

 x2 y2 z2
   

 a2  u
  b
2 2
u  
2 2 2
c  u  
= L.H.S.
2 2 2
 u   u   u   u u u 
Hence          2 x  y  z  . This completes the proof.
 x   y   z   x y z 

Q.No.30: If v  x 2  y 2  z 2   1 / 2
. Show that
2v
x 2

 2v
y 2

 2v
z 2
0 .

Sol.: Since v  x 2  y 2  z 2  
1 / 2
, we have
v
x
1

  x 2  y2  z2
2

3 / 2

.2 x   x x 2  y 2  z 2  3 / 2
.

and
2v
x 2

 

x 

 x x 2  y2  z2 3 / 2   2 2
   1. x  y  z
2
 3 / 2  3

 x   x 2  y2  z 2
 2

5 / 2 
.2 x 
 


  x 2  y2  z2  x
5 / 2 2
 y 2  z 2  3x 2  x 2  y 2  z 2    2x
5 / 2 2
 y2  z2  ..(i)

Similarly,
 2v
y 2

 x 2  y2  z2   x
5 / 2 2
 2y2  z2 .  ...(ii)

and
2v
z 2

 x 2  y2  z2   x
5 / 2 2
 y 2  2z 2 .  ...(iii)

Adding (i), (ii) and (iii), we have

 2v
x 2

 2v
y 2

2v
z 2

 x 2  y2  z 2 
5 / 2
0  0 .

This completes the proof.


Partial Differentiation: Partial Differential Coefficient Prepared by: Dr. Sunil, NIT Hamirpur 30

Q.No.31: If V  r m , r 2  x 2  y 2  z 2 , then show that

Vxx  Vyy  Vzz  mm  1r m  2 .

r r x
Sol.: Since r 2  x 2  y 2  z 2  2r  2x  
x x r
V x
Now V  r m   mr m 1.  mxr m  2 and
x r

 2V  r   x
 2
 m r m  2  x m  2 r m 3   m r m  2  x m  2r m 3 
x  x   r


 2V
x 2

 m r m  2  m  2 x 2 r m  4 .  ......(i)

Similarly,
 2V
y 2

 m r m  2  m  2 y 2 r m  4  ......(ii)

and
 2V
z 2
 
 m r m  2  m  2 z 2 r m  4 . .....(iii)

Adding (i), (ii) and (iii), we get

  
Vxx  Vyy  Vzz  m 3r m  2  m  2 r 2 r m  4  m r m  2 3  m  2   mm  1r m  2 .
This completes the proof.
Q.No.32: If u  logtan x  tan y  tan z  , then prove that
u u u
sin 2 x  sin 2 y  sin 2z  2.
x y z

Sol.: Here u  logtan x  tan y  tan z  . ....(i)


Differentiate (i) partially w. r. t. x, y and z separately, we get

u sec 2 x u sec 2 y u sec 2 z


 ,  and  .
x tan x  tan y  tan z y tan x  tan y  tan z z tan x  tan y  tan z
u u u
Now L.H.S.= sin 2 x  sin 2 y  sin 2z
x y z
1 1 1
2 sin x cos x. 2
 2 sin y cos y. 2
 2 sin z cos z.
cos x cos y cos 2 z

tan x  tan y  tan z
Partial Differentiation: Partial Differential Coefficient Prepared by: Dr. Sunil, NIT Hamirpur 31

2tan x  tan y  tan z 


  2 = R.H.S.
tan x  tan y  tan z 
This completes the proof.

Q.No.33: If u 

xy x 2  y 2  ; u0,0  0 , show that  2u

 2u
at
x0
.
x 2  y2 xy yx y0

Sol.: For x , y   0,0 , u x , y  



xy x 2  y 2  ( given ) ....(i)
x 2  y2
Differentiating (i) partially w. r. t. x, we get

u
  2

  xy x 2  y 2 
  y

  x 3  xy 2 
   y 
 
 x 2  y 2 3x 2  y 2  x 3  xy 2 .2x 

  
x x  x  y 2  x  x 2  y 2  
 x 2
 y 2 2


 
 3x 4  2 x 2 y 2  y 4  2 x 4  2 x 2 y 2   x 4  4x 2 y2  y4 
 y   y 

 
x 2
 y 2 2
 


 x 2
 y 2 2
 
 
 For x , y   0,0 ,
u
 u x x , y  

y x 4  4x 2 y 2  y 4
.
 ...(ii)
x 
x 2  y2
2

For
u
0,0 , let us consider u 0,0   Lim u x ,0   u 0,0   Lim 0  0  0 .
x x x 0 x x 0 x

u
which exists.  0,0   0 .
x
  u 
For the existence of u yx 0,0 , i.e.
y  x  0,0 

  u  u 0, y   u x 0,0  y  0
Consider    Lim x  Lim  1 , which exists.
y  x  0,0  y 0 y y  0 y

  u 
  1 . ...(iii)
y  x  0,0 

Again because for x , y   0,0 , u x , y  



xy x 2  y 2  ( given ) ....(i)
2 2
x y
Differentiating (i) partially w. r. t. x, we get

u
 

  xy x 2  y 2   x   yx  y3 
2
  x 
   
 x 2  y 2 x 2  3y 2  x 2 y  y3 .2 y 


y y  x 2  y 2  y  x 2  y 2  

2
x y 2 2
  

Partial Differentiation: Partial Differential Coefficient Prepared by: Dr. Sunil, NIT Hamirpur 32

 x 4  2 x 2 y 2  3y 4  2 x 2 x 2  2 y 4   x 4  4x 2 y2  y4 
 x   x 


2
x y 2 2
 



2
x y  2 2 
 
 For x , y   0,0 ,
u
 u y x , y  
 
x x 4  4x 2 y 2  y 4
. ...(iv)
y 2

x y 
2 2

For
u
0,0 , let us consider u 0,0  Lim f 0, y   f 0,0  Lim 0  0  0 .
y y y  0 y y 0 y

u   u 
which exists.  0,0  0 . For the existence of u xy 0,0 , i.e.  
y x  y  0,0 

  u  u y x,0  u y 0,0 x  0
Consider    Lim  Lim  1 , which exists.
x  y  0,0  x 0 x y 0 x

  u 
    1. ...(v)
x  y  0,0 

 2u  2u x0
 From (iii) and (v), we get  at .
xy yx y0

i.e. u yx 0,0  u xy 0,0 .

This completes the proof.


r2
 1   2 θ  θ
Q.No.34: If θ  t n e 4t , find the value of n which will make r  .
r 2 r  r  t
r2

Sol.: Given θ  t n e 4t .
r2 r2
θ 
4 t .
2r  1 n 1 
 t n .e     rt e 4t
r  4t  2
r2
θ 1 
 r2   r 3.t n 1e 4 t
r 2
 r2 r2  r2
 
  2 θ  1 n 1  2  4 t   2r  1 r2 
r  t 3r e  r 3e 4 t      t n 1r 2e 4 t 3  
r  r    4 t  2
2
   2t 
Partial Differentiation: Partial Differential Coefficient Prepared by: Dr. Sunil, NIT Hamirpur 33

r2
1   θ  1   r2 
 2  r 2   t n 1e 4 t   3 
r r  r  2  2t 
 
r2 r2 r2
θ    r2    r2 
Also  nt n 1e 4 t  t n e 4 t . 2   t n 1e 4 t . n  2  .
t  4t   4 t 
  
1   2 θ  θ
Since r  is given
r 2 r  r  t
r2 r2
1   r2    r2 
 t n 1e 4 t .  3   t n 1e 4 t . n  
2  2t   4 t 
  

r2 3 r2 3
  n  n   . Ans.
4t 2 4t 2

 2u  2u 1
Q.No.35: If u  f ( r ) , where r 2  x 2  y 2 , prove that 2
  f ' ' r   f ' r  .
2
x y r

Sol.: Given r 2  x 2  y 2 . (i)


r r x
Differentiating partially w. r. t., we get 2r  2x  
x x r
r x
Similarly,  .
y r
u r x
Now u  f ( r )   f ' r .  f ' r 
x x r
Differentiating again w. r. t. x, we get

 2u1  1 r  x r
 f ' r   x.  2 f ' r   f ' ' r .
x 2
r  r x  r x

     
  x uvw   vw x u   uw x v   uv x w 
 

1 x x x x 1 x2 x2
 f ' r   2 . f ' r   .f ' ' r .  f ' r   3 f ' r   2 f ' ' r 
r r r r r r r r
r2  x2 x2 y2 x2
 f ' r   f ' ' r   f ' r   f ' ' r  [using (i)]
r3 r2 r3 r2
Partial Differentiation: Partial Differential Coefficient Prepared by: Dr. Sunil, NIT Hamirpur 34

 2u x2 y2
Similarly,  f ' r   f ' ' r 
y 2 r3 r2

 2u  2u x 2  y2 x 2  y2 r2 r2
   f ' r   f ' ' r   f ' r   f ' ' r 
x 2 y 2 r3 r2 r3 r2

 2u  2u 1
  f ' ' r   f ' r  . Hence prove.
x 2 y 2 r

Q.No.36: If x  e r cos θ cosr sin θ  and y  er cos θ sin r sin θ  ,


x 1 y y 1 x
prove that  . ,  . .
r r θ r r θ

 2x 1 x 1  2 x
Hence deduce that    0.
r 2 r r r 2 θ 2

Sol.: Given x  e r cos θ cosr sin θ  .


x
  e r cos θ . cos θ. cos r sin θ   e r cos θ . sin r sin θ . sin θ
r

 e r cos θ cos θ cosr sin θ   sin θ sinr sin θ 

 er cos θ cosθ  r sin θ  (i)


x
 e r cos θ . r sin θ . cosr sin θ   e r cos θ . sin r sin θ .r cos θ
θ

 rer cos θ sin θ cosr sin θ   cos θ sinr sin θ 

 rer cos θ sinθ  r sin θ  (ii)

Also y  er cos θ sin r sin θ 


y
 e r cos θ . cos θ. sin r sin θ   e r cos θ . cosr sin θ . sin θ
r

 e r cos θ sin θ cosr sin θ   cos θ sinr sin θ 

 e r cos θ sinθ  r sin θ  (iii)


y
 e r cos θ . r sin θ . sin r sin θ   e r cos θ . cosr sin θ .r cos θ
θ

 rer cos θ cos θ cosr sin θ   sin θ sinr sin θ 


Partial Differentiation: Partial Differential Coefficient Prepared by: Dr. Sunil, NIT Hamirpur 35

 rer cos θ cosθ  r sin θ  (iv)


x 1 y
From (i) and (iv), we get  . (v)
r r θ
y 1 x
From (ii) and (iii), we get  . (vi)
r r θ

 2x 1 y 1  2 y
From (v), we get  .  .
r 2 r 2 θ r rθ
x y
From(vi), we get  r
θ r

 2x 2y 2y
 2  r  r
θ rθ rθ

 2x 1 x 1  2 x 1 y 1  2 y 1 y 1  2 y
  2 2  2.  .  .  .  0.
r 2 r r r θ r θ r rθ r 2 θ r rθ
x  a 2
1  4y
Q.No.37: Prove that if f ( x , y)  .e , then f xy  f yx .
y

x  a 2 1  x  a 
2
 
1 4y 4y
Sol.: Given f ( x , y)  .e y 2 .e .
y

1  x  a 
2

f 
4y   x  a 2 
fx  y 2 .e .  
x x  4 y 

1  x  a  x  a 2
2
3
  2x  a   1 2 
y 2 .e 4y
.    y  x  a .e 4y
 4y  2

3  x  a  1  x  a 
2 2

f 1 
4y

4y   x  a 2 
fy   y 2 .e y 2 .e .  
y 2 y  4 y 

x  a 2  3  x  a 
2
 x  a 2 
3 1
 
  
1 1
  y 2 .e 4 y  2  y 1 x  a 2
 e 4 y . y 2  y 2 . 2
 2 4y  4
 

  f 
f xy   
x  y 
Partial Differentiation: Partial Differential Coefficient Prepared by: Dr. Sunil, NIT Hamirpur 36

3   x  a  x  a 2 
2

  x  a 2 
 
 
1 2 e 
.  2  y x  a   e .2 y x  a 
4y 1 2 4y 1
 y  . 
4  x  4 y  
 
3  x  a 
2


1
4
y

2 .e 4y  2x  a 

4y
  
 2  y 1 x  a 2  2 y 1 x  a 
 

3  x  a 
2


1
4
y

2 .e 4y
.
xa  1
y  2
 2 
  2  y  x  a   2 
1


5 x a 2
1    x  a 2 
 y 2 x  a .e 4y
3  .
4  2 y 

 5  x a  3  x  a  
2 2

f yx
  f  1  3 2
     x  a  y .e 4y

 y 2 .e 4 y
.
 x  a 2

y  x  2

2 4 y 
2

5  x  a 
2


1
x  a .y

4y
 x  a 2 
2 .e
 3  
4  2 y 

5 x a 2
1    x  a 2 
 y 2 x  a .e 4y
3  .
4  2 y 

Hence f xy  f yx .

1  2z 1  2z
Q.No38.: Find the value of  2 2 when a 2 x 2  b 2 y 2  c2 z 2  0 .
a x
2 2
b y

Sol.: Here a 2 x 2  b 2 y 2  c2 z 2  0  c2 z 2  a 2 x 2  b 2 y2
1 2 2
 z2 
c 2 
a x  b2 y2  (i)

Differentiating (i) partially w.r.t. x, we get


z 1 z a 2  x 
2z  .2a x 
2
   (ii)
x c 2 z c2  z 
Differentiating (ii) partially w.r.t. x, we get
Partial Differentiation: Partial Differential Coefficient Prepared by: Dr. Sunil, NIT Hamirpur 37

 z 
z.1  x 
2z a 2  x  a
2
 a 2  x  a2  a2x2  a2
    z  x     z    2 2 2 
c2 z 2  a 2 x 2 
x 2 c 2  z2 2 2
 cz  c2  z  c2z2  c 2
z  c z .c z
 

a2

c 4 z3
b2 y 2   a 2 x 2  b 2 y 2  c 2 z 2  0 

 2z a 2b2 y2
 2  4 3 (iii)
x c z
 2z a 2b2 x 2
Similarly,   4 3 (iv)
y2 c z

1  2 z 1  2 z 1 a 2 b2 y2 1 a 2 b 2 x 2 1
Consider  2 2  2 . 4 3  2 . 4 3  4 3  b 2 y 2  a 2 x 2 
a x
2 2
b y a c z b c z cz
1
 
c2 z 2
4 3
cz
  a 2 x 2  b 2 y 2  c 2 z 2  0 

1 2z 1 2z 1
 2 2  2 . Ans.
a x
2 2
b y cz

NEXT TOPIC

Homogeneous Functions and Euler’s Theorem

*** *** *** *** ***


*** *** ***
***
Differential Calculus

Partial Differentiation
(Homogeneous Functions and Euler’s Theorem)
Prepared by:
Dr. Sunil
NIT Hamirpur (HP)
(Last updated on 01-08-2009)
Latest update available at: http://www.freewebs.com/sunilnit/

(25 Solved problems and 00 Home assignments)


Homogeneous Expression:

An expression of the form a 0 x n  a1x n 1y1  a 2 x n  2 y 2  ...  a n y n ,


where each term of degree ‘n’ , is called Homogeneous expression in x and y and of
degree or order ‘n’.
Homogeneous Function:
If this expression equal to some quantity ‘u’ , then ‘u’ is called Homogeneous
Function in x and y of degree ‘n’ .
Now u  a 0 x n  a 1 x n 1 y1  a 2 x n  2 y 2  ...  a n y n

  y y
2
y 
n

 x n a 0  a 1    a 2    .....  a n   
 x x  x  

 y
 u  x nf   .
x
x
Also, we can write a Homogeneous Function in x and y of degree 'n' as u  y n f   .
 y
Similarly, a Homogeneous Function in x ,y and z of degree ‘n’ can be written as
Partial Differentiation: Homogeneous Equation and Euler’s Theorem 2
Prepared by: Dr. Sunil, NIT Hamirpur

y z x z x y
u  x n F ,  or u  y n F ,  or u  z n F ,  .
x x  y y z z
Here 'u' is dependent variable and x, y, z are independent variables.
Euler’s Theorem:
Statement: If ‘u’ is a homogeneous function of x and y of degree ‘n’, then
u u
x y  nu .
x y
Proof: Given ‘u’ is a homogeneous function in x and y of degree ‘n’.
 y
Then we may write u  x n f   . (i)
x
Differentiating (i) partially w.r.t. x [keeping y as constant], we get
u  y  y   y
  x n f       2   nx n 1f  
x x  x  x
 y  y
  x n  2 yf     nx n 1f   .
x x
Similarly, differentiating (i) partially w.r.t. y [keeping x as constant], we get
u y1  y
 x n f    x n 1f   .
y xx x
u u   y  y    y 
Now x y  x  x n  2 yf     nx n 1f     y  x n 1f    
x y  x  x    x 

  y 
 n  x n f     nu .
  x 
This completes the proof.
Extension of Euler’s Theorem:
Statement: If ‘u’ is a homogeneous function of x and y of degree ‘n’, then show that

 2u  2u  2u
x2  2 xy  y 2 2  n n  1u .
x 2 xy y
Proof: Since ‘u’ is a homogeneous function in x and y of degree ‘n’ then
u u
x y  nu . [by Euler’s Theorem] ..... (i)
x y
Differentiating (i) partially w. r. t. x [keeping y as constant], we get
Partial Differentiation: Homogeneous Equation and Euler’s Theorem 3
Prepared by: Dr. Sunil, NIT Hamirpur

 2 u u  2u u
x   y n .
x 2
x xy x
Multiplying by x, we get

 2u
u  2u u 2
2 u  2u u
x 2
2
x  xy  nx x 2
 xy  n  1x . ......(ii)
x x xy x x xy x
Again, Differentiating (i) partially w. r. t. y [keeping x as constant], we get

 2u  2 u u u
x y 2  n .
yx y y y

Multiplying by y, we get

 2u  2u u u  2u  2u u
xy  y2 2  y  ny  xy  y 2 2  n  1y . .....(iii)
yx y y y yx y y

Adding (ii) and (iii), we get

 2u  2u  2u  u u 
x2  2xy  y 2 2  n  1 x  y   n  1nu  n n  1u .
x 2 xy y  x y 

This completes the proof.


Now let us solve some problems related to the above-mentioned topics:

Q.No.1.: Verify Euler’s theorem, when z  x 3  3x 2 y  y3 .

Sol.: Since z  x 3  3x 2 y  y3
z z
  3x 2  6xy and  3x 2  3y 2 .
x y

x
z
x
y
z
y
     
 x 3x 2  6 xy  y  3x 2  3y 2  3 x 3  3x 2 y  y3  3z .

 y y 
3
 y
Also z  x 1  3       x 3f   .
3
  x   x   x

 z is a homogeneous function of x and y of degree 3.


z z
 By Euler’s theorem, we get x y  nz  3z .
x y
Hence, Euler’s theorem is verified.
Partial Differentiation: Homogeneous Equation and Euler’s Theorem 4
Prepared by: Dr. Sunil, NIT Hamirpur

1/ 2
 x1 / 3  y1 / 3  u u 1
Q.No.2.: If u   1 / 2  , then prove that x y   u.
 x  y1 / 2  x y 12

1/ 2 1/ 2
 1 / 3 y1 / 3      y 1 / 3 
1/ 2  x 1    1    
 x1 / 3  y1 / 3    x1 / 3    1 / 6   x  
Sol.: Here u   1 / 2     x  1 / 2 
 x  y1 / 2   x1 / 2 1  y1 / 2     y  

  x1 / 2    1   x  
   
1/ 2
  y 1 / 3 
1    
1 / 12  x  y
x  x 1 / 12f   .
 1/ 2 
x
1   y  
  x  
1
 u is a homogeneous function of x and y of degree  .
12
u u 1
 By Euler’s theorem, we have x y  nu   u .
x y 12
Hence the result.
 y u u
Q.No.3.: If u  f   , then show that x y  0.
x x y

y y
Sol.: Here u  f    x 0f   .
x x
 u is a homogeneous function of x and y of degree 0.
Hence by Euler’s theorem, we have
u u
x y  nu  0.u  0 .
x y
Hence the result.

y u u   y 
Q.No.4.: If u  xyf   , then show that x y  2 xyf   .
x x y   x 
 y  y  y   y
Sol.: Here u  xyf    x 2  f    x 2F  .
x  x  x  x
 u is a homogeneous function of x and y of degree 2.
Hence by Euler’s theorem, we have
Partial Differentiation: Homogeneous Equation and Euler’s Theorem 5
Prepared by: Dr. Sunil, NIT Hamirpur

u u   y 
x y  nu  2.u  2 xyf   .
x y   x 
Hence the result.
x  y u u
Q.No.5.: If u  sin 1   tan 1  , find the value of x y .
y x x y

   
 x   y    1   y    y
Sol.: Here u  sin 1   tan 1   x 0 sin 1    tan 1   x 0f   .
y x  y  x  x
 x 
 u is a homogeneous function of x and y of degree 0.
Hence by Euler’s theorem, we have
u u
x y  nu  0u  0 .
x y
Hence the result.
Q.No.6.: Verify Euler’s Theorem on homogeneous functions in the following cases:-

(i) f x , y  
x
1/ 4
 y1 / 4 , (ii) u  f x , y, z   x  y  z
x
1/ 5
 y1 / 5 
 x 2  y2  x  y
(iii) z  tan 1 , (iv) u  sin 1   tan 1 
 y  y x
 
y
(v) z  x 4 log  .
x
  y 1 / 4 

Sol.: (i) Here f x , y  


x 1/ 4
 y1 / 4  x
1    
1 / 20   x    x1 / 20f  y 
x 1/ 5
y 1/ 5
 
 y 
1    
1/ 5 
 
x
  x  
1
 f x , y  is a homogeneous function of x and y of degree .
20
Hence by Euler’s theorem, we have

f f 
1 x1 / 4  y1 / 4 
x
x
y
y
 nf 

20 x1 / 5  y1 / 5
.
 (i)
Partial Differentiation: Homogeneous Equation and Euler’s Theorem 6
Prepared by: Dr. Sunil, NIT Hamirpur

Again since f x , y  
x
1/ 4
 y1 / 4 
x
1/ 5
 y1 / 5 
Differentiating partially w. r. t x and y respectively, we get

f
x 1/ 5 1
 
 y1 / 5  x  3 / 4   x1 / 4  y1 / 4
4 
  15 x 4 / 5 



x x 1/ 5
 y1 / 5  2

Multiplying by x, we get

f
x 1/ 5 1

 y1 / 5  x1 / 4   x1 / 4  y1 / 4
4 
  15 x 1/ 5 


x  (ii)
x x 1/ 5
y 1/ 5 2

f
x1/ 5 1
4
 


1
5

 y1 / 5  y  3 / 4   x1 / 4  y1 / 4  y  4 / 5 



y x 1/ 5
 y1 / 5  2

Multiplying by y, we get

f
x1/ 5 1

 y1 / 5  y1 / 4   x1 / 4  y1 / 4
4 
  15 y 1/ 5 


y  (iii)
y x 1/ 5
 y1 / 5  2

Adding (ii) and (iii), we get

f f
x
1/ 5
 y1 / 5 14 x 1/ 4
 
 y1 / 4  x1 / 4  y1 / 4 15 x
1/ 5
 y1 / 5 
x y 
x y x 1/ 5
 y1 / 5 2


1 x1 / 4  y1 / 4 


20 x1 / 5  y1 / 5  (iv)

Hence, the Euler’s Theorem is verified.


  y 1 / 2  z 1 / 2  y z
(ii) Here u  f x , y, z   x  y  z  x 1/ 2
1         x1 / 2f  , 
  x   x   x x

1
 u  f x , y, z  is a homogeneous function of x ,y and z of degree .
2
Hence by Euler’s theorem, we have
Partial Differentiation: Homogeneous Equation and Euler’s Theorem 7
Prepared by: Dr. Sunil, NIT Hamirpur

x
u
x
y
u
y
z
u
z
1
 nu  u 
2
1
2
 x y z . 
Again since u  f x , y, z   x  y  z
Differentiating partially w. r. t x , y and z respectively, we get
u 1 u 1 u 1
 ,  and 
x 2 x y 2 y z 2 z

x
u
x
u
y z
y
u
z
x
1
2 x
y
1
2 y
z
1

2 z 2
1
 x y z . 
Hence, the Euler’s Theorem is verified.

 x 2  y2   2 
1    y0 tan 1  x x
(iii) Here z  tan    1  y0f   .
 y    y  y
   
 z is a homogeneous function of x ,y and z of degree 0.
Hence by Euler’s theorem, we have
z z
x y  nz  0.z  0 .
x y

 x 2  y2 
1  .
Again since z  tan
 y 
 
Differentiating partially w. r. t x and y respectively, we get

z 1 1  1  xy 1
 .2 x   2
x
1
2
x y
.
2 y 2
2 x y
2 

x  2y 2
.
x  y2
2  
y2

y 1 
 .2 y  x 2  y 2 
z

1 2 x y
2 2

 2
1 y2  x 2  y2  
y
1
.
x 2  y2 

y2 
 x  2y
2
.

x 2  y2
y2  
 


1  x  2

x 2
 2y2 . x 2  y2
.
Partial Differentiation: Homogeneous Equation and Euler’s Theorem 8
Prepared by: Dr. Sunil, NIT Hamirpur

z z 
 x 2
xy 1  
  y
1  x2 
  0.
 
x
x
y
y


 x  2y2
.
x 2
 y 2

  x 2  2y2


.
x 2
 y 2


Hence, the Euler’s Theorem is verified.
   
 x   
y   1   
y  y
(iv) Here u  sin 1   tan 1   x 0 sin 1    tan 1   x 0f   .
y x  y  x  x
 x 
 u is a homogeneous function of x and y of degree 0.
Hence by Euler's theorem, we have
u u
x y  nu  0u  0 .
x y

x  y
Again since u  sin 1   tan 1  .
 y x
Differentiating partially w. r. t x and y respectively, we get
u 1 1 1 y 1 y
 .  2
. 2    .
x
x
2 y y x  y2  x 2 x  y2
2

1    1
x2
y

u 1 x 1 1 x x
 . 2   .   2 .
y 2
 y  y 2 x y y2  x 2 x  y2
x 1 2
1    x
y
 

u u  1 y   x x 
x y  x  2   y    0.
x y  y2  x 2 x  y2   y y2  x 2 x 2  y2 
   
Hence, the Euler’s Theorem is verified.
y y
(v) Here z  x 4 log   x 4f   .
x x
 z is a homogeneous function of x and y of degree 4.
Hence by Euler's theorem, we have
z z y
x y  nz  4z  4x 4 log  .
x y x
Partial Differentiation: Homogeneous Equation and Euler’s Theorem 9
Prepared by: Dr. Sunil, NIT Hamirpur

y
Again since z  x 4 log  .
x
Differentiating partially w. r. t x and y respectively, we get

z y 1  y y z 1  1  x4
 4 x 3. log   x 4 .  2   4 x 3. log   x 3 and  x 4.   
x x yx  x y yx y
x x
Hence, the Euler’s Theorem is verified.

x 4  y4 u u
Q.No.7.: If u  log , show that x y  3.
xy x y

x 4  y4 u x 4  y4
Sol.: Here u  log e  .
xy xy

eu is a homogeneous function of x and y of degree 3.

Hence by Euler’s theorem, we have x


 
 eu
y
 eu  
 neu  3e u .
x y
u u
 xe u  yeu  3e u .
x y
u u
Hence x y  3.
x y
Hence the result.
 x y
Q.No.8.: If u  sin 1  , show that u   y u .
 x y x x y
 
  y 1 / 2 
 x y 1    
 x y
Sol.: Here u  sin 1   sin u     x   x    x 0f  y  .
0
 x y  x  y   y 
1/ 2
x
   
1    
  x  
 sin u is a homogeneous function of x and y of degree 0.
Hence by Euler’s theorem, we have
 sin u   sin u 
x y  n sin u  0.sin u  0 .
x y
u u u u
 x cos u  y cos u 0 x y  0.
x y x y
Partial Differentiation: Homogeneous Equation and Euler’s Theorem 10
Prepared by: Dr. Sunil, NIT Hamirpur

u y u
Hence  .
x x y
Hence the result.
 y  y
Q.No.9.: If u  x      , show that
 x  x

u u  y
(i) x y  x   .
x y  x

 2u  2u  2u
(ii) x 2  2 xy  y2 2  0 .
x 2 xy y

y y
Sol.:(i) Here u  x      u1  u 2 (say)
x x
u u u u u u
Then x y x 1y 1x 2 y 2 .
x y x y x y

Now since u1 and u 2 are homogeneous function of x and y of degree 1 and 0


respectively. Then by Euler’s theorem, we have
u1 u y u u
x  y 1  nu1  x  and x 2  y 2  nu 2  0 .
x y x x y

u u u u u u y
Hence x y  x 1  y 1  x 2  y 2  x   0 .
x y x y x y x
u u  y
x y  x  .
x y x
Hence the result.
(ii) Again, since u1 and u 2 are homogeneous function of x and y of degree 1 and 0
respectively. Then, by extension of Euler’s theorem, we have

 2u  2u  2u y
x2  2 xy  y 2 2  n n  1u  11  1.x   0 .
x 2 xy y x

2 u 2 u 2  u
2
Hence x2  2 xy  y 0 .
x2 xy y2
Hence the result.
Partial Differentiation: Homogeneous Equation and Euler’s Theorem 11
Prepared by: Dr. Sunil, NIT Hamirpur


Q.No.10.: If u  x 2  y 2 
1/ 3
, prove that x 2
 2u
x 2
 2xy
 2u
xy
 2u 2
 y2 2   u .
y 9
1/ 3
  y 2 
Sol.: Here u  x  y  2

2 1/ 3
x 2/3
1    
 y
 x 2 / 3f   .
  x   x

2
 u is a homogeneous function of x and y of degree .
3
Hence by extension of Euler’s theorem, we have

 2u  2u 2
2 u 22  2
x 2
2
 2 xy y 2
 n n  1u    1   u .
x xy y 33  9

Hence the result.


 x 2  y2 
Q.No.11.: If u  sin 1  , then show that x u  y u  tan u .
 xy  x y
 
 y2 
 x 2  y2   x 2  y2  1  2 
Sol.: Here u  sin 1   sin u ( z, say)     x  x   x1f  y  .
 xy   xy   y  x
   
 1 x 
 
 z is a homogeneous function of x and y of degree 1.
z z
Then, by Euler’s theorem, we have x y  nz  z .
x y
u u
 x cos u  y cos u  sin u .
x y
u u
x y  tan u .
x y
Hence the result.
1/ 2
 x1 / 3  y1 / 3 
Q.No.12.: If u  sin 1  1 / 2  , then prove that
 x  y1 / 2 

u u 1
(i) x y   tan u .
x y 12

(ii) x 2 u xx  2xyu xy  y 2 u yy 
tan u
144
13  tan 2 u . 
Partial Differentiation: Homogeneous Equation and Euler’s Theorem 12
Prepared by: Dr. Sunil, NIT Hamirpur

1/ 2
 x 1 / 3  y1 / 3 
1
Sol.: (i) Here u  sin  1 / 2 1/ 2 
x  y 
1/ 2 1/ 2 1/ 2
 1/ 3  y1/ 3      y 1/ 3    y 1 / 3 
 x 1  1/ 3    1     1    
 sin u    x   x
  x 1/ 6   1/ 2 
 1 / 12  x  y
x  x 1 / 12f   .
 1/ 2  y1/ 2      1/ 2 
 x 1  1/ 2    1   y   1   y  
x
  x     x     x  

1
 sin u is a homogeneous function of x and y of degree  .
12
Then, by Euler’s theorem, we have
 (sin u )  (sin u ) 1
x y  n sin u   sin u
x y 12
u u 1
 x cos u  y cos u   sin u
x y 12
u u 1
x y   tan u . ...(i)
x y 12
Hence the result.
1
(ii) Differentiating (i) w. r. t. x partially, we get xu xx  u x  yu xy   sec2 u u x 
12
Multiplying by x , we get
1
x 2 u xx  xu x  xyu xy   sec2 u xu x  . .....(ii)
12
Differentiating (i) w. r. t. y partially, we get

xu xy  u y  yu yy  
1
12
 
sec2 u u y .

Multiplying by y , we get

yxu xy  yu y  y 2 u yy  
1
12
 
sec2 u yu y . ......(iii)

Adding (ii) and (iii), we get


Partial Differentiation: Homogeneous Equation and Euler’s Theorem 13
Prepared by: Dr. Sunil, NIT Hamirpur

x 2 u xx  2 xyu xy  y 2 u yy  
1
12
    1
 
sec 2 u xu x  yu y  xu x  yu y    sec 2 u  1 xu x  yu y  
 12 
1  1  1 1
  sec 2 u  1 tan u   sec 2 u tan u  tan u
 12  12  144 12


1
144
  1
1  tan 2 u tan u  tan u 
12
1
144
tan u 1  tan 2 u  12 

1
144

tan u 13  tan 2 u . 
Hence the result.

Q.No.13.: If u 
x 2
 y2 m
y  y
 x     , show that
2m  1 x x

 2u  2u
 
2
2 2 u m
x 2
 2 xy y  2m x 2  y 2 .
x xy y 2

Sol.: Here u 
x 2
 y2 m
 y  y
 x      u1  u 2  u 3 (say)
2m  1 x x
u u u u u u u u
Then x y  x 1 y 1 x 2 y 2 x 3 y 3 .
x y x y x y x y

Now since u1 , u 2 and u 3 are homogeneous function of x and y of degree 2m , 1 and 0


respectively. Then by Euler’s theorem, we have
u1 u u u u u
x  y 1  nu1  2mu1 , x 2  y 2  nu 2  u 2 and x 3  y 3  nu 3  0 .
x y x y x y
u u u u u u u u
Hence x y  x 1  y 1  x 2  y 2  x 3  y 3  2m.u1  1.u 2 .
x y x y x y x y

Again, since u1 , u 2 and u 3 are homogeneous function of x and y of degree 2m, 1 and 0
respectively.
Then, by extension of Euler’s theorem, we have

x 2

 2u
2 xy
 2u
 y
2
2 u
 n n  1u  2 m 2 m  1u1  11  1u 2  2 m 2 m  1 x 2  y2  
m

x 2 xy y 2 2m  1

Hence x 2
 2u
x 2
 2 xy
 2u
xy
 2u
 y 2 2  2m x 2  y 2
y
 
m
.

Hence the result.


Partial Differentiation: Homogeneous Equation and Euler’s Theorem 14
Prepared by: Dr. Sunil, NIT Hamirpur

Q.No.14.: If u  sin  
x  y , show that

x
u
x
y
u 1

y 2
  
x  y cos x  y . 

Sol.: Here u  sin x  y . 
  y 1 / 2 
 sin 1 u  z, say     y
x  y  x1 / 2 1      x1 / 2f   .
 x 
  x

1
 z is a homogeneous function of x and y of degree .
2
z z 1
Then, by Euler’s theorem, we have x y  nz  z .
x y 2

x
1 u
y
1 u 1 1
 sin u 
1
 x y . 
1  u 2 x 1  u 2 y 2 2

x
u
x
y
u 1

y 2
 
x  y 1  sin 2  x y   1
2
 x y  cos 2  
x y .

Hence x
u
x
y
u 1

y 2
  
x  y cos x  y . 
Hence the result.

Q.No.15.: If z  sin 1 x 2  y 2  , then show that x 2 z xx  2 xyz xy  y 2 z yy  tan 3 z .


 

y2 y
Sol.: Here z  sin 1 x 2  y 2   sin z u , say   x 2  y 2  x 1  2  xf   .
  x x
 u is a homogeneous function of x and y of degree 1.
u u
Then by Euler’s theorem, we have x y  nu  u .
x y
z z z z
 x cos z  y cos z  sin z  x y  tan z . (i)
x y x y
Differentiating (i) w. r. t. x partially, we get

xz xx  z x  yzxy  sec 2 zz x 

Multiplying by x , we get

x 2 z xx  xz x  xyz xy  sec 2 zxz x  . ......(ii)


Partial Differentiation: Homogeneous Equation and Euler’s Theorem 15
Prepared by: Dr. Sunil, NIT Hamirpur

Differentiating (i) w. r. t. y partially, we get

 
xz xy  z y  yz yy  sec2 z z y

Multiplying by y , we get

 
yxzxy  yzy  y 2 z yy  sec 2 z yzy . ......(iii)

Adding (ii) and (iii), we get

   
x 2 z xx  2 xyzxy  y 2 z yy  sec2 z xz x  yzy  xz x  yzy  sec2 z(tan z)  tan z .

   
 tan z sec2 z  1  tan z tan 2 z  tan 3 z .
Hence the result.
u w w
Q.No.16.: If u  iv  x  iy 2 , and w  , prove that x y  0.
v x y

Sol.: Here u  iv  x  iy 2  u  iv  x 2  y 2  2ixy .

  y 2 
1    
u x 2  y2 0  x    x 0f  y  .
Thus w   x  
v 2 xy  y  x
 2 
 x 
 w is a homogeneous function of x and y of degree 0.
Then, by Euler’s theorem, we have
w w
x y  nw  0.w  0 .
x y
w w
Hence x y  0.
x y
This completes the proof.

Q.No.17.: If u  iv  ax  iby 3 ,show that


u u
(i) x y  3u ,
x y
v v
(ii) x y  3v ,
x y
w w u
(iii) ( x y  0 where w  .
x y v
Partial Differentiation: Homogeneous Equation and Euler’s Theorem 16
Prepared by: Dr. Sunil, NIT Hamirpur

Sol.: (i) Here u  iv  ax  iby 3  u  iv  (a 3x 3  3ab 2 xy 2 )  i(b3 y3  3a 2bx 2 y) .

 3 2 y 
2
y
Thus u  (a x  3ab xy )  x a  3ab     x 3f   .
3 3 2 2 3
  x   x

 u is a homogeneous function of x and y of degree 3.


Then, by Euler’s theorem, we have
w w
x y  nw  3.w .
x y
w w
Hence x y  3w .
x y
This completes the proof.

(ii) Here u  iv  ax  iby 3  u  iv  (a 3x 3  3ab 2 xy 2 )  i(b3 y3  3a 2bx 2 y) .

 3  y 3  y   y
Thus v  (b y  3a bx y)  x b    3a 2 b   x 3f   .
3 3 2 2 3
  x   x  x

 v is a homogeneous function of x and y of degree 3.


Then, by Euler’s theorem, we have
v v
x y  nv  3.v .
x y
v v
Hence x y  3v .
x y
This completes the proof.

(iii) Here u  iv  ax  iby 3  u  iv  (a 3x 3  3ab 2 xy 2 )  i(b3 y3  3a 2bx 2 y) .

 3 2 y 
2 
 a  3 ab   
u a 3x 3  3ab 2 xy 2 0  x    x 0f  y  .
Thus w   3 3  x  
v b y  3a 2 bx 2 y  3
y  x
 y
 b    3a b  
3 2 
  x   x  
 w is a homogeneous function of x and y of degree 0.
Then, by Euler’s theorem, we have
w w
x y  nw  0.w  0 .
x y
Partial Differentiation: Homogeneous Equation and Euler’s Theorem 17
Prepared by: Dr. Sunil, NIT Hamirpur

w w
Hence x y  0.
x y
This completes the proof.


Q.No.18.: If v  x 2  y 2  z 2 
1 / 2
,then show that x
v
x
v
y z
y
v
z
 v .

1 / 2
  y 2  z 2 
 2
Sol.: Here v  x  y  z 2

2 1 / 2 1
 v  x 1       
y z
 x 1f  , 
  x   x   x x

 v is a homogeneous function of x ,y and z of degree 1 .


Hence by Euler’s theorem, we have
v v v
x y z  nu  (1).v   v .
x y z
Hence the result.


Q.No.19.: If u  sin 1 x  y , prove that 
 2u  2u  2 u  sin u. cos 2u
x2  2xy  y2 2  .
x 2 xy y 4 cos3 u

  y 1 / 2 

Sol.: Here u  sin 1 x  y  sin u      x 
 y
x  y  x1 / 2 1      x1 / 2f   .
  x

1
 sin u is a homogeneous function of x and y of degree
2
 (sin u )  (sin u ) 1
 By Euler’s theorem, x y  n sin u  sin u
x y 2
u u 1 u u 1
 x cos u  y cos u  sin u .  x  y  tan u . ...(i)
x y 2 x y 2
Differentiating (i) w. r. t. x partially, we get
1 2
xu xx  u x  yu xy  sec u u x 
2
Multiplying by x , we get
1 2
x 2 u xx  xu x  xyu xy  sec u xu x  ......(ii)
2
Differentiating (i) w. r. t. y partially, we get
Partial Differentiation: Homogeneous Equation and Euler’s Theorem 18
Prepared by: Dr. Sunil, NIT Hamirpur

xu xy  u y  yu yy 
1 2
2
sec u u y  
Multiplying by y , we get

yxu xy  yu y  y 2 u yy 
1 2
2
 
sec u yu y ......(iii)

Adding (ii) and (iii), we get

x 2 u xx  2 xyu xy  y 2u yy 
1 2
2
   1
 

sec u xu x  yu y  xu x  yu y   sec 2 u  1 xu x  yu y 
2 
1  1  1 1
  sec 2 u  1 tan u   sec 2 u tan u  tan u
2  2  4 2


1

tan u sec2 u  2 tan u  1
  2  
 2
 tan u 1  2 cos u 
4 4  cos2 u  4 cos2 u

 2u  2u  2 u  sin u. cos 2u
 x2  2xy  y2 2  .
x 2 xy y 4 cos3 u
Hence the result.
 x 3  y3 
1
Q.No.20.: If V  tan  , prove that
 2 x  3y 
 
 2V  2V  2V
x2  2 xy  y 2 2  sin 4V  sin 2V .
x 2 xy y

  y 3 
 3 3 1    
1  x  y  x 3  y3 2  x    x 2f  y   z (say).
Sol.: Here V  tan  tan V  x  
 2 x  3y  2 x  3y  y  x
   2  3 
x
 
 z is a homogeneous function of x and y of degree 2.
z z
Then by Euler’s theorem, we have x y  nz  2z .
x y
V V
 x sec 2 V  y sec 2 V  2 tan V
x y
V V 2 tan V sin V
x y  2
2 . cos 2 V  2 sin V cos V  sin 2V . ....(i)
x y sec V cos V
Differentiating (i) partially w. r. t. x [keeping y as constant], we get
Partial Differentiation: Homogeneous Equation and Euler’s Theorem 19
Prepared by: Dr. Sunil, NIT Hamirpur

 2V V  2V V
x  y  cos 2V.2 .
x 2 x xy x
Multiplying by x, we get

 2V V  2V V
x2 x  xy  2 x cos 2V
x 2 x xy x

 2V  2V V
 x2  xy  2 cos 2V  1x . ..(ii)
x 2 xy x
Again, Differentiating (i) partially w. r. t. y [keeping x as constant], we get

 2V  2 V V V
x y 2   cos 2V.2 .
yx y y y

Multiplying by y, we get

 2V 2
2 V V V
xy y 2
y  2 y cos 2V
yx y y y

 2V  2V V
 xy  y 2 2  2 cos 2V  1y . .....(iii)
yx y y

Adding (ii) and (iii), we get

 2V  2V  2V  V V 
x2  2 xy  y 2 2  2 cos 2V  1 x y   2 cos 2V  1sin 2V
x 2 xy y  x y 

 2 sin 2V cos 2V  sin 2V  sin 4V  sin 2V .

2  2V
 2V 2
2 V
Hence x  2 xy y  sin 4V  sin 2V .
x 2 xy y 2
Hence the result.
y z u u u
Q.No.21.: If u  x n f  ,  , show that x y z  nu .
x x x y z

y z y z
Sol.: Here u  x n f  ,  .Let  t1 ,  t 2
x x x x

 u  x n f t1, t 2   x n f
Differentiating partially w. r. t x , y and z respectively, we get
Partial Differentiation: Homogeneous Equation and Euler’s Theorem 20
Prepared by: Dr. Sunil, NIT Hamirpur

u  f t f t 2  n  f   y  f   z 
 nx n 1f  x n  . 1  . n 1
  nx f  x   2    2 
x  t1 x t 2 x   t1  x  t 2  x 
u f f
  nx n 1f  yx n 1  zx n 1 ,
x t1 t 2

u  f t f t 2  n  f  1   n 1 f
 xn  . 1  .   x     x ,
y  t1 y t 2 y   t1  x  t1

u  f t f t 2  n  f  1   n 1 f
 xn  . 1  . x     x
z  t1 z t 2 z   t 2  x  t 2
u u u  f f   n 1 f   n 1 f 
x y z  x nx n 1f  yx n 1  zx n 1   yx   z x .
x y z  t1  t 2  t1  t 2

u u u
Hence x y z  nx n f t1, t 2   nu .
x y z
Hence the result.
1 1 log x  log y u u
Q.No.22.: If u  2
  2 2
show that x  y  2u  0 .
x xy x y x y

  y 
 log  
1 1 log x  log y 1  x    x  2f  y  .
Sol.: Here u  2    x  2 1    
x xy 2
x y 2  y y2  x
 x 1 2 
 x 
 u is a homogeneous function of x and y of degree  2 .
u u
Hence by Euler’s theorem, we have x y  2u .
x y
u u
Hence x y  2u  0 .
x y

 x 5  y5  z5 
Q.No.23.: If f x , y, z   log  , show that x f  y f  z f  4 .
 xyz  x y z
 
 x 5  y5  z5  x 5  y 5  z5
Sol.: Here f x , y, z   log  f
e  ...(i)
 xyz  xyz
 
Partial Differentiation: Homogeneous Equation and Euler’s Theorem 21
Prepared by: Dr. Sunil, NIT Hamirpur

  y 5  z 5 
1       
f
e x 4  x   x    x 4f  y , z  .
 
 y z   x x 
 1  
 x x 

 ef is a homogeneous function of x, y and z of degree 4.


Hence by Euler’s theorem, we have

x
 
 ef
y
 
 ef
z
 ef 
 4ef .
x y z
f f f f f f
 xef  yef  zef  4ef  x y z  4.
x y z x y z
Hence the result.
 x 2  y2  z2 
Q.No.24.: If u  sin  , show that x u  y u  z u  0 .
 xy  yz  zx  x y z
 
  y 2  z 2 
1       
 x 2  y2  z2 
Sol.: Here u  sin    x sin   x   x    x 0f  y , z  .
0
 xy  yz  zx   y y z z  x x
    .  
x x x x
 
 u is a homogeneous function of x ,y and z of degree 0.
Then by Euler’s theorem, we have
u u u
x y z  nu  0.u  0 .
x y z
Hence the result.
Q.No.25: Given that F(u) = V(x , y , z), where V is a homogeneous function of x , y , z of
u u u F(u )
degree n , then prove that x y z n / .
x y z F (u )
Sol.: Here V(x , y , z) is a homogeneous function of x , y , z of degree n , then by Euler's
V V V F(u ) F(u ) F(u )
theorem x y z  nV  x y z  nF(u )
x y z x y z
u u u
 xF(u )  yF(u )  zF(u )  nF(u )
x y z
Partial Differentiation: Homogeneous Equation and Euler’s Theorem 22
Prepared by: Dr. Sunil, NIT Hamirpur

u u u F(u )
x y z n / .
x y z F (u )
Hence the result.

NEXT TOPIC

Total Differentials,
Explicit Function, Implicit Functions
and
Total Differential Coefficient

*** *** *** *** ***


*** *** ***
***
Differential Calculus

Partial Differentiation
(Total Differentials, Explicit Function, Implicit Functions
and Total Differential Coefficient)
Prepared by:
Dr. Sunil
NIT Hamirpur (HP)
(Last updated on 01-08-2009)
Latest update available at: http://www.freewebs.com/sunilnit/

(18 Solved problems and 15 Home assignments)

Explicit Function:
A function, where the dependent variable say y, is expressed in terms of the
independent variable say x, then that function is called explicit function.
Example: y  4x 3  3x 2  5x  9 .

Implicit Functions:
A function, where one of the various variables cannot be expressed explicitly in
terms of the other variables, then that function is called implicit function.
Example: Consider the relation x 3  y3  3axy  0 .

dy
In this case, we obtain by differentiating throughout w.r.t. x.
dx
Partial Differentiation: Total Differential and Total Differential Coefficient 2
Prepared by: Dr. Sunil, NIT Hamirpur

Total Differentials:
Let u be a function of x and y i.e. u = f(x , y).
Then the total differential of u is defined and written as
u u
du  dx  dy .
x y
Similarly, if u be a function of x , y and z i.e. u = f(x, y, z).
Then the total differential of u is defined and written as
u u u
du  dx  dy  dz .
x y z
dy
Evaluation of for an implicit function:
dx
Let u = f(x, y) be an implicit function  u  f ( x, y)  0 or const.
 du  0 . ....(i)
Also when u be a function of x and y, i.e. u = f(x , y),
u u
then the total differential of u is defined and written as du  dx  dy . ....(ii)
x y
u
u u dy u
From (i) and (ii), we get dx  dy  0    x   x .
x y dx u uy
y
Total Differential Coefficient:
Let u  f (x, y) , where x  t  , y  t  .
Then u is ultimately a function of t.
Then the total differential coefficient of u w.r.t. t is defined and written as
du u dx u dy
 .  . .
dt x dt y dt

Similarly, if u  f ( x, y, z) , where x  t  , y  t  and z  t  .


Then the total differential coefficient of u w.r.t. t is defined and written as
du u dx u dy u dz
 .  .  . .
dt x dt y dt z dt
du u dx u dy
Remark: u  f (x, y) and t = x, then from  .  . , we have
dt x dt y dt
Partial Differentiation: Total Differential and Total Differential Coefficient 3
Prepared by: Dr. Sunil, NIT Hamirpur

du u u dy
  . ,
dx x y dx
du
where is the total differential coefficient of u w.r.t. x.
dx

Now let us solve some problems related to Total Differentials and Total Differential
Coefficient:

Q.No.1: Find the total differential of u in the following cases:-

(i) u  x  y and (ii) u  log x 2  y 2 . 


Sol.: (i) Since u  x  y

u 1 u 1
  and  .
x 2 x  y y 2 x  y

u u 1
Then du  dx  dy  dx  dy  .
x y 2 xy


(ii) Since u  log x 2  y 2   u
 2
2x
x x  y 2
and
u
 2
2y
y x  y 2
.

u u 2xdx  2 ydy
Then du  dx  dy  .
x y x 2  y2
dy
Q.No.2: If x 3  y3  3axy , find .
dx
Sol.: Given x 3  y3  3axy  0 . So let u  x 3  y3  3axy  0 .
u u
  3x 2  3ay and  3y 2  3ax .
x y

u
dy
  x   2

3x 2  3ay ay  x 2 
Hence
dx u 3y  3ax  2
y  ax 
.

y

Q.No.3: If u  sin 1 x  y  , where x = 3t, y  4 t 3 . Prove that the total differential


coefficient of u w. r. t. t is equal to 3 1  t 2 
1 / 2
.

Sol.: Given u  sin 1 x  y  , where x = 3t, y  4 t 3 .


Partial Differentiation: Total Differential and Total Differential Coefficient 4
Prepared by: Dr. Sunil, NIT Hamirpur

u 1 u 1 du du
  ,  ,  3 and  12 t 2 .
x 1  x  y 2 y 1  x  y 2 dx dy

Then total differential coefficient of u w. r. t. t is


du u dx u dy
 .  .
dt x dt y dt


du

1
.3 
1 2
.12 t 
3 1  4t 2   

3 1  4t 2 
dt 1  x  y 2 1  x  y 2 1  3t  4 t 3  
2
1  4t  1  t 
2 2 2


du
dt

 3 1  t2  1 / 2
.

 
Q.No.4: If u  sin x 2  y 2 , where a 2 x 2  b 2 y 2  c 2 , find the total differential
coefficient of u w. r. t. x.

 
Sol.: Given u  sin x 2  y 2 , where a 2 x 2  b 2 y 2  c 2 .

Let f  a 2 x 2  b 2 y 2  c 2 .

f f dy f a 2x
  2a 2 x ,  2b 2 y and  x  2 .
x y dx fy b y

Since we know the total differential coefficient of u w. r. t. x. is


du u u dy
  . .
dx x y dx


du
dx
   a 2x 
 b y
 
 a2 
 2x cos x 2  y 2  2 y. cos x 2  y 2 .  2   21  2  x cos x 2  y 2 .
 b 
 
   
Q.No.5: Find the total differentials in the following cases:


(a) u  2 x 2  4 y3 , (b) u  tan 
3 x
y
.

u u
Sol.: Since we know the total differential of u is du  dx  dy .
x y


(a) Here u  2 x 2  4 y3  3
.


u
x
 2
 2
 3 2 x 2  4 y3 .4 x   12 x 2x 2  4 y3 ,  
Partial Differentiation: Total Differential and Total Differential Coefficient 5
Prepared by: Dr. Sunil, NIT Hamirpur

and
u
y

 3 2 x 2  4 y3   12y   36y 2x
2 2 2 2
2
 4 y3 .

Hence du 
u
x
u
y
2
 2
dx  dy  12x 2x 2  4 y3 dx  36 y 2 2x 2  4 y3 dy  

 12 2 x 2  4 y3  xdx  3y dy . Ans.
2 2

x u  x  1  u  x  x 
(b) Here u  tan .  sec2  .  and  sec2  .  2  .
y x  y y y  y y 

u u  x  1   x  x 
Hence du  dx  dy  sec2  . dx  sec2  .  2 dy
x y  y  y  y  y 

 x   ydx  xdy 
 sec2  . 2
 . Ans.

y
  y 

 x
Q.No.6: Find the total differential coefficient of u  sin  , where x  et ,
 y

y  t 2 w. r. t. t.
or
x du
Given u  sin   , where x  et , y  t 2 , find as a function of t.
 y dt
Verify your result by direct substitution.

du u dx u dy  x1  x  x 
Sol.: We have  .  .   cos  .e t   cos   2 2 t
dt x dt y dt  yy  y  y 

 et  et  et  et  t  2  t  et 
 cos 2 . 2  2 cos 2 . 3   3 e cos 2  .
   
t  t t  t  t  t 
     

x  et 
Also u  sin    sin 2  .
y t 
 

du  e t  t 2e t  e t .2 t  t  2  t  et 
  cos 2 .   3  e cos  as before.
dt t  t 4
 t   t2 
   

Q.No.7: If u  x log xy , where x 3  y 3  3xy  1  0 , find total differential coefficient of


u w. r. t. x.
Partial Differentiation: Total Differential and Total Differential Coefficient 6
Prepared by: Dr. Sunil, NIT Hamirpur

Sol.: Given u  x log xy , where x 3  y 3  3xy  1  0 .


Since we know that total differential coefficient of u w. r. t. x is
du u u dy
  . . .....(i)
dx x y dx

u  1  u  1  x
Now  x. .y   1. log xy  1  log xy . Also  x. .x   .
x  xy  y  xy  y
f
3 3 dy x 3x 2  3y 
x2  y   
Let f  x  y  3xy  1 , then
dx

f
 2
3y  3x
 2

y x
.
  
y
u u dy
Substituting the values of , and in (i), we get
x y dx

du u u dy  x   x 2  y 
 1  log xy    .  2
x x2  y  
  .
dx x y dx  y   y  x 
 1  log xy 
y y2  x
. Ans.
 
Q.No.8: Find the total differential coefficient of x 2 y w. r. t. x where x and y are

connected by x 2  xy  y 2  1 .

Sol.: Let u  x 2 y , where x and y are connected by x 2  xy  y 2  1 .


Since we know that total differential coefficient of u w. r. t. x is
du u u dy
  . . .....(i)
dx x y dx
u u
Now  2xy . Also  x2 .
x y

Let f  x 2  xy  y 2  1 ,

f
then
dy
  x  
2x  y    2x  y  .
dx f 2 y  x  x  2 y 
y
u u dy
Substituting the values of , and in (i), we get
x y dx
Partial Differentiation: Total Differential and Total Differential Coefficient 7
Prepared by: Dr. Sunil, NIT Hamirpur

du u u dy  2 x  y  2 xyx  2 y   x 2 2 x  y 
  .  2 xy  x 2 .   
dx x y dx  x  2y  x  2 y 

 

du x xy  4 y 2  2 x 2 
. Ans.
dx x  2 y 
Q.No.9: (i) If f x , y   0 , y, z   0 ,
f  dz f 
show that . .  . .
y z dx x y

(ii) If the curves If f x , y   0 and x , y   0 touch,


f  f 
show that at the point of contact .  . .
x y y x

Sol.: (i) Given f x , y   0 , y, z   0  df  0 and d  0 .


f f  
i.e. dx  dy  0 and dy  dz  0 .
x y y z

f 
dy dz y
   x ....(i) and  . .....(ii)
dx f dy 
y z

  
f dy f f dy dz f f  
z . dz   f
(i)  .   . .   .
y dx x y dz dx x y    dx x
 
y 

f  dz f 
 . .  . .
y z dx x y

(ii) Let the curves f x , y   0 and x , y   0 are touching at a point a , b  .


Now the slope of the tangent of the curve f x , y   0 at point of contact is

f
dy
  x . ......(i)
dx f
y
and the slope of the tangent of the curve x , y   0 at point of contact is
Partial Differentiation: Total Differential and Total Differential Coefficient 8
Prepared by: Dr. Sunil, NIT Hamirpur


dy
  x . .......(ii)
dx 
y

Now since the curves f x , y   0 and x , y   0 are touching so that their slope of the
tangents are same.
f  f 
Hence from (i) and (ii), at the point of contact .  . .
x y y x
dx dy dz
Q.No.10: If x 2  y 2  z 2  2 xyz  1 . Show that    0.
1  x2 1  y2 1  z2

Sol.: Given x 2  y 2  z 2  2 xyz  1 .

Let u  x 2  y 2  z 2  2 xyz  1  0 .
Here u be an implicit function  du  0 . ...(i)
Here u be a function of x , y and z, i.e. u = f(x, y, z).
u u u
Then the total differential of u is du  dx  dy  dz
x y z
u u u
 dx  dy  dz  0 . [by (i)] ....(ii)
x y z
u u u
Evaluate: , and .
x y z

Since u  x 2  y 2  z 2  2 xyz  1 .
u u u
  2x  yz ,  2y  xz  and  2z  xy  .
x y z

Hence (ii) becomes x  yz dx  y  xz dy  z  xy dz  0 . ....(iii)


Find: x  yz, y  xz  and z  xy  .

Since we have given x 2  y 2  z 2  2 xyz  1

 x 2  2 xyz  1  y 2  z 2  x 2  2 xyz  y 2 z 2  1  y 2  z 2  y 2 z 2

 
 x  yz 2  1  y 2 1  z 2   x  yz  1  y 1  z  .
2 2

Similarly, y  xz   1  x 1  z 
2 2
Partial Differentiation: Total Differential and Total Differential Coefficient 9
Prepared by: Dr. Sunil, NIT Hamirpur

and 1  x 1  y  .
z  xy   2 2

Hence (iii) becomes 1  y 2 1  z 2 dx  1  x 2 1  z 2 dy  1  x 2 1  y 2 dz  0 .

Last step: Dividing by 1  x 2 1  y 2 1  z 2  , we get

dx dy dz
  0.
1 x 2
1 y 2
1  z2

du
Q.No.11: If u  x 2  y 2 , where x  a cos t , y  b sin t . Find and verify the result.
dt
Sol.: Given u  x 2  y 2 .
du u dx u dy
We have  .  .  2 x.a  sin t   2 y.b cos t  2yb cos t  xa sin t 
dt x dt y dt

Also u  x 2  y 2  a 2 cos 2 t  b 2 sin 2 t .


du
  a 2 2 cos t  sin t   b 2 2 sin t cos t   2 a cos t a sin t   b sin t b cos t 
dt
 2yb cos t  xa sin t  as before.

Q.No.12: If ax 2  by 2  cz 2  1 and lx  my  nz  0 , then prove that


dx dy dz
  .
bny  cmz clz  anx amx  bly
dy dz
Also find and .
dx dx
Sol.: Let f  ax 2  by 2  cz 2  1 and   lx  my  nz .
 f  0  df  0 and   0  d  0 .
f f f
Now df  dx  dy  dz  2axdx  2bydy  2czdz  axdx  bydy  czdz  0 ....(i)
x y z
  
Also d  dx  dy  dz  ldx  mdy  ndz  0 ...(ii)
x y z
Solving (i) and (ii), we get
dx dy dz
  .
bny  cmz clz  anx amx  bly
Partial Differentiation: Total Differential and Total Differential Coefficient 10
Prepared by: Dr. Sunil, NIT Hamirpur

dx dy dx dz
Now consider  and  .
bny  cmz clz  anx bny  cmz amx  bly
dy clz  anx dz amx  bly
  and  . Ans.
dx bny  cmz dx bny  cmz
dy dz
Q.No.13: If x 2 y  e x  x sin z  0 and x 2  y 2  z 2  a 2 . Find and .
dx dx
Sol.: Let f  x 2 y  e x  x sin z  0 and   x 2  y 2  z 2  a 2  0 .
 f  0  df  0 and   0  d  0 .

Now df 
f
x
f f

dx  dy  dz  2 xy  e x  sin z dx  x 2dy  x cos zdz  0
y z
 ....(i)

  
Also d  dx  dy  dz  2xdx  2 ydy  2zdz  xdx  ydy  zdz  0 ...(ii)
x y z
Solving (i) and (ii), we get
dx dy dz

2
x z  yx cos z 2

x cos z  z 2 xy  e  sin z x
 y2xy  e
 x

 sin z  x 3
.

dx dy

Now consider 2
x z  yx cos z 
x cos z  z 2 xy  e x  sin z
2

dx dz
and 2
x z  yx cos z


y 2xy  e  sin z  x 3
x
 .

We get 

dy x 2 cos z  z 2xy  e x  sin z
and

dz y 2 xy  e x  sin z  x 3
 .Ans.
 
dx x 2 z  yx cos z dx x 2 z  yx cos z
dy log x
Q.No.14: If x y  e x  y , then prove that  .
dx 1  log x 2

Sol.: Given x y  e x  y .
Taking log on both sides, we get

log x y  log e x  y  y log x  x  y  y log x  x  y  0 .


Let u  y log x  x  y . ....(i)
Differentiate (i) partially w. r. t. x and y separately, we get
u y yx u
  1  and  (log x  1) .
x x x y
Partial Differentiation: Total Differential and Total Differential Coefficient 11
Prepared by: Dr. Sunil, NIT Hamirpur

u yx
dy xy y log x
Hence   x  x   . ...(i) [ x  y  y log x ]
dx u log x  1 x log x  1 x 1  log x 
y
y y y y 1
Now since y log x  x  y  log x  1   1  log x  1  
x x x x 1  log x

y
Substituting the value of in (i), we get
x
dy log x
 .
dx 1  log x 2

This completes the proof.


dy
Q.No.15: Using partial differentiation, find when x y  y x  C .
dx
Sol.: Given x y  y x  C  x y  y x  C  0

Let f x , y   x y  y x  C .....(i)
Differentiate (i) partially w. r. t. x and y separately, we get
f f
 yx y 1  y x log y and  x y log x  xy x 1 .
x y

dy fx
 

y x y 1  y x 1 log y 
But
dx fy 
x x y 1 log x  y x 1
. Ans.

du
Q.No.16.: Find as a total derivative and verify the result by direct substitution if
dt

u  x 2  y 2  z 2 and x  e 2t , y  e 2 t cos 3t , z  e 2t sin 3t .


Sol.: Here u is a function of x, y, z are and x, y, z are in turn functions of t. Thus u is a
function ‘t’ via the intermediate variables x, y, z. Then
du u dx u dy u dz
  
dt x dt y dt z dt

  
 2 x.2e 2 t  2 y. 2e 2 t cos 3t  3e 2 t sin 3t  2 z 2e 2 t sin 3t  3e 2 t cos 3t 
Rewriting in terms of x, y, z
 2 x.2.x  2.y2 y  3.z   2z2z  3y 
Partial Differentiation: Total Differential and Total Differential Coefficient 12
Prepared by: Dr. Sunil, NIT Hamirpur


 4 x 2  y2  z2 
or in terms of t
du
dt
  
 4 e 4 t  e 4 t cos 2 3t  sin 2 3t  8e 4 t

Verification by direct submission:

u  x 2  y 2  z 2  e 4 t  e 4 t cos 2 3t  e 4 t sin 2 3t  2e 4 t
du
 8e 4 t .
dt

Q.No.17.: Find the total differential coefficient of x 2 y w.r.t. x when x, y are connected

by x 2  xy  y 2  1 .

Sol.: Let u  x 2 y , then the total differential is


u u
du  dx  dy
x y
Thus the total differential coefficient of u w.r.t x is
du u u dy
 
dx x y dx
du dy
 2 xy  x 2
dx dx
From the Implicit relation f  x 2 xy  y  1 , we calculate

dy f x 2x  y
 
dx f y x  2y

du dy  2 x  y  
so  2 xy  x 2 .  2 xy  x 2   
dx dx  x  2 y  

du x 2 2x  y 
 2 xy  .
dx x  2 y 
Q.No.18.: The altitude of the right circular cone is 15 cm and is increasing at 0.2 cm/sec.
The radius of the base is 10 cm and is decreasing at 0.3 cm/sec. How fast is
the volume changing?
Partial Differentiation: Total Differential and Total Differential Coefficient 13
Prepared by: Dr. Sunil, NIT Hamirpur

Sol.: Let x be the radius and y be the altitude of the cone. So volume V of the right
1
circular cone is V  x 2 y .
3
Since x and y are changing w.r.t time t, differentiate V w.r.t. t.
dV V dx V dy
 
dt x dt y dt

1  dx dy 
  2 xy  x2 
3  dt dt 
dx dy
It is given that x = 10, y = 15,  0.3 and  0.2 , substituting these values
dt dt
dV 1

  2.10.15 0.3  10 2 0.2 
dt 3
 70
3

 cm3/sec

70
i.e, volume is decreasing at the rate of .
3

Home Assignments
du x
Q.No.1.: Find when u  sin   and x  e t , y  t 2 . Verify the result by direct
dt  y
substitution.

t2  et 
Ans.: e t cos 2  .
t3 t 
 
du
Q.No.2.: Find given u  sin 1 x  y  , x = 3t, y  4 t 3 . Verify the result by direct
dt
substitution.


Ans.: 3 1  t 2 1 / 2
du
Q.No.3.: If u  x 3 ye z where x = t, y  t 2 and z  In t , find at t = 2.
dt

Ans.: 6t 5 ; 192.
Partial Differentiation: Total Differential and Total Differential Coefficient 14
Prepared by: Dr. Sunil, NIT Hamirpur

du y
Q.No.4.: Find , if u  tan 1   and x  e t  e  t and y  e t  e  t .
dt x
2
Ans.:
e 2t
 e 2t

Q.No.5.: If x, y are related by x 2  y 2  2 and u  tan x 2  y 2 , find   du


dx
.


Ans.: 4 x sec 2 2 x 2  2 . 
y du
Q.No.6.: If u  tan 1   and y  x 4 find at x = 1.
x dx

3x 2 3
Ans.: 6
; at x = 1.
1 x 2

Q.No.7.: In order that the function u  2 xy  3x 2 y remains constant. What should be the
rate of change of y (w.r.t. t) given that x increases at the rate of 2cm/sec at the
instant when x = 3 cm and y = 1 cm.
dy 32 32
Ans.:   cm / sec ; y must decrease at the rate of cm/sec.
dt 21 21
Q.No.8.: Find the rate at which the area of a rectangle is increasing at a given instant
when the sides of the rectangle are 4 ft and 3 ft and are increasing at the rate of
1.5 ft/sec. and 0.5 ft/sect respectively.
Ans.: 6.5 sq. ft/sec.
dz dz
Q.No.9.: Find (a). and (b). , given z  xy 2  x 2 y , y = In x.
dx dy
Ans.: (a). Here x is the independent variable
dz z z dy
   y 2  2xy  2 y  x
dx x y dx
(b). Here y is the independent variable
dz z z dx
   xy 2  2 x 2 y  2xy  x 2
dy y x dy
Q.No.10.: Find the differential of the function f ( x, y)  x cos y  y cos x .
Ans.: df  cos y  y sin x dx  x sin y  cos x dy

Q.No.11.: Find the differential of the function u x , y, z)   e xyz .


Partial Differentiation: Total Differential and Total Differential Coefficient 15
Prepared by: Dr. Sunil, NIT Hamirpur

Ans.: du  e xyz yzdx  zxdy  xydz  .


du
Q.No.12.: Find for the functions u  x 2  y 2 , x  e t cos t , y  e t sin t at t = 0.
dt
du
Ans.: 2e 2 t cos 2 t  sin 2 t  ; At t = 0, 2
dt
du
Q.No.13.: Find for the functions u  Inx  y  z  ; x  e  t , y  sin t , z  cos t .
dt
cos t  sin t  e  t
Ans.:
cos t  sin t  e  t

Q.No.14.: Find
du
dt
 
for the functions u  sin e x  y , x  f t  , y  gt  .

Ans.:
du
dt
     
 cos e x  y e x f t   cos e x  y g t  .

du
Q.No.15.: Find for the functions u  x y when y  tan 1 t , x  sin t .
dt
1
Ans.: y.x y 1 cos t  x y In x. .
1 t2

NEXT TOPIC

Transformation of independent variables (Composite Functions),

Jacobian, Properties of Jacobians

*** *** *** *** ***


*** *** ***
***
Differential Calculus

Partial Differentiation
[Transformation of independent variables
(Composite Functions), Jacobian, Properties of
Jacobians]
Prepared by:
Dr. Sunil
NIT Hamirpur (HP)
(Last updated on 01-08-2009)
Latest update available at: http://www.freewebs.com/sunilnit/

(27 Solved problems and 00 Home assignments)


Composite function:
If u  f x 1 , x 2 , x 3 ,............. and the independent variables x 1 , x 2 , x 3 ,......... are

further functions of other variables t 1 , t 2 , t 3 ,........ .

by the relations, x 1  t 1 , t 2 , t 3 ,............... , x 2  t 1 , t 2 , t 3 ,........... etc.

Then u is said to be a composite function of the variables t 1 , t 2 , t 3 ,........ ,

For example if u to be function of x, y, i.e. u  f x , y  and further if x ,y are function of


t1, t 2 , i.e. x  t 1 , t 2  and if y   t1, t 2 , .
Then u is a composite function of variables t1, t 2 ,
Partial Differentiation: Composite Functions Prepared by: Dr. Sunil, NIT Hamirpur 2

Transformation of independent variables:


Now the necessary formulae for changing of independent variables are obtained:
u u x u y u u x u y
 .  . ,  .  . ,…………
t1 x t1 y t1 t 2 x t 2 y t 2

Further, if u  f x , y  and if t1  f1 x, y  and t 2  f 2 x , y  .


Then the transformation equations are
u u t1 u t 2
 .  . ,
x t1 x t 2 x
u u t1 u t 2
 .  . .
y t1 y t 2 y
Expansion:
Extending the above results, we may obtain.
In case u  f x , y, z  and x  1 t1, t 2 , t 3  , y  2 t1, t 2 , t 3  , z  3 t1, t 2 , t 3  .
Then the transformation equations are
u u x u y u z
 .  .  . ,
t1 x t1 y t1 z t1
u u x u y u z
 .  .  . ,
t 2 x t 2 y t 2 z t 2
u u x u y u z
 .  .  . .
t 3 x t 3 y t 3 z t 3

Further, if u  f x, y, z  , t1  f1x , y, z  t 2  f 2 x, y, z  and t 3  f3 x , y, z  . Then

u u t1 u t 2 u t 3
 .  .  . ,
x t1 x t 2 x t 3 x
u u t1 u t 2 u t 3
 .  .  . ,
y t1 y t 2 y t 3 y
u u t1 u t 2 u t 3
 .  .  . .
z t1 z t 2 z t 3 z
Partial Differentiation: Composite Functions Prepared by: Dr. Sunil, NIT Hamirpur 3

Jacobian:
Definition: If u and v are functions of two independent variables x and y, then the
u u
x y u v u v
determinant  .  . ,
v v x y y x
x y
is called the functional determinant or Jacobian of u, v with respect to x, y, and is
 u, v   u , v 
denoted by the symbol J  or .
 x, y   x, y 

Similarly, if u, v, w are functions of three independent variables x, y, z, then the Jacobian


u u u
x y z
 u , v, w   u, v, w  v v v
of u, v, w with respect to x, y, z is J    .
 x, y, z   x, y, z  x y z
w w w
x y z
Properties of Jacobians:
I. If u, v are functions of r, s where r, s are functions of x, y
 u , v   u, v   r, s 
then   .
 x , y   r, s   x , y 
Proof: Since u, v are composite functions of x, y
u u r u s
  .  .  u r rx  u ss x ,
x r x s x
u u r u s
 .  .  u r ry  u ss y ,
y r y s y
v v r v s
 .  .  v r rx  vss x ,
x r x s x
v v r v s
 .  .  v r ry  vss y .
y r y s y

u , v  r, s  u r u s rx ry
Now   . .
r, s  x, y  v r vs s x sy

Interchanging rows and columns in the second determinant, we get


Partial Differentiation: Composite Functions Prepared by: Dr. Sunil, NIT Hamirpur 4

u u
ur u s rx sx u r rx  u ss x u r ry  u ss y x y  u , v 
.    .
vr vs ry sy v r rx  vss x v r ry  vss y v v  x , y 
x y

II. If J1 is the Jacobian of u, v, with respect to x, y and J 2 is the Jacobian of x, y, with


 u , v   x , y 
respect to u, v, then J1J 2  1 i. e.   1.
 x , y   u , v 
Proof: Let u = u(x, y) and v = v(x, y), so that u and v are functions of x, y.
Suppose on solving for x and y, we get x =  (u, v) and y =  (u, v).
u u x u y
1 .  .  u x x u  u y yu
u x u y u
u u x u y
0 .  .  u x x v  u y yv
v x v y v
v v x v y
0 .  .  vx x u  vy yu
u x v y u
v v x v y
1 .  .  vx x v  vy yv
v x v y v

u , v  x , y  u x uy xu xv
Now   . .
 x , y   u , v  v x v y yu yv

Interchanging rows and columns in the second determinant, we get


ux uy xu yu u x x u  u y yu u x x v  u y yv 1 0
.    1.
vx vy x v yv vx x u  v y yu vx x v  vy yv 0 1

Now let us solve some more problems:

y  r, θ 
Q.No.1.: If r  x 2  y 2 , θ  tan 1 , evaluate .
x  x, y 
y
Sol.: Given r  x 2  y 2 , θ  tan 1 .
x
r x r y
Now  , 
x x 2  y2 y x 2  y2
Partial Differentiation: Composite Functions Prepared by: Dr. Sunil, NIT Hamirpur 5

θ 1  y  y  1 1 x
  2    2
2 2
,    2
x y  x  x y y y  x  x  y2
2
1 1
x2 x2

r r x y
 r, θ  x y 2 2
x 2  y2
   x y
 x, y  θ θ y x
 2
x y x  y2 x 2  y2

x2 y2 x 2  y2 1
    .
x 2
y 
2 3/ 2
x 2
y 
2 3/ 2
x 2
y 
2 3/ 2 x 2  y2
Q.No.2.: If x  r sin  cos  , y  r sin  sin  , z  r cos θ ,
 x , y, z 
show that  r 2 sin  .
 r, , 

x x x
r   sin  cos  r cos  cos   r sin  sin 
 x , y, z  y y y
Sol.:   sin  sin  r cos  cos  r sin  cos 
r, ,  r  
z z z cos   r sin  0
r  
Taking out common factor (r from second column and r sin θ from third column)
sin  cos  cos  cos   sin 
2
 r sin  sin  sin  cos  sin  cos 
cos   sin  0

Expanding by third row


 cos  cos   sin  sin  cos   sin  
 r 2 sin cos   sin  
 cos  sin  cos  sin  sin  cos  

   
 r 2 sin  cos  cos  cos 2   cos  sin 2   sin  sin  cos 2   sin  sin 2  
 r2 sin cos   sin   r
2 2 2
sin 
u u u
Q.No.3.: If u  f y  z, z  x, x  y  , prove that    0.
x y z

Sol.: Suppose u1  y  z, u 2  z  x, u 3  x  y . (i)

 u  f y  z, z  x, x  y  becomes u  f u1, u 2 , u 3  . (ii)


Partial Differentiation: Composite Functions Prepared by: Dr. Sunil, NIT Hamirpur 6

From (i) and (ii) we conclude that u is composite function of x, y, z.


u u u1 u u 2 u u 3
  .  .  . (iii)
x u1 x u 2 x u 3 x

u1 u 2 u 3
Now  0,  1 , 1
x x x
u u u
 (iii) becomes   . (iv)
x u 2 u 3
u u u
Similarly   , (v)
y u 3 u1
u u u
and   . (vi)
z u 3 u 2
Adding (iv), (v) and (vi), we get
u u u
   0, which is the required result.
x y z
Q.No.4.: If w  f ( x, y) , x  r cos θ , y  r sin θ ,
2 2 2 2
 w  1  w   f   f 
show that    2        .
 r  r  θ   x   y 
Sol.: The given equations define w as a composite function of r and θ .
w w x w y w w
 .  .  . cos θ  .sin θ
r x r y r x y
w f f
  cos θ  sin θ  w  f (x, y) (i)
r x y
w w x w y w
Also  .  .   r sin θ   w r cos θ 
θ x θ y θ x y
1 w f f
   sin θ  cos θ . (ii)
r θ x y
Squaring and adding (i) and (ii), we get
2 2 2 2
 w  1  w   f   f 
   2        .
 r  r  θ   x   y 
dz
Q.No.5.: If z  x 2  y 2 and x 3  y3  3axy  5a 2 , find the value of ,
dx
Partial Differentiation: Composite Functions Prepared by: Dr. Sunil, NIT Hamirpur 7

when x  y  a .
Sol.: The given equation are of the form z  f (x, y) and x , y   c .
 z is the composite function of x.
dz z dx z dy z z dy
  .  .   . (i)
dx x dx y dx x y dx

Now
z 1 2
 x  y2
x 2
  1 / 2
.2 x 
x
x 2  y2
z y
Similarly, 
y x 2  y2

Also, differentiating x 3  y3  3axy  5a 2 , we get

3x 2  3y 2 .
dy
dx
 3ay  3ax.
dy
dx
 0  y2  ax
dy
dx

  x 2  ay   
dy x 2  ay
  2
dx y  ax

dz x y  x 2  ay 
 From (i), we get    
dx 2  y 2  ax 
x 2  y2 2
x y  

 dz  a a a2  a2
 dx  x a   .
a2  a2
 0.
ya a2  a2 a2  a2

du
Q.No.6.: If u  xe y z , where y  a 2  x 2 , z  sin 2 x , find .
dx
Sol.: Here u is a function of x, y and z while y and z are functions of x.
du u dx u dy z dz
  .  . 
dx x dx y dx y dx

 e y z.1  xe y z.
2

1 2
a  x2 
1 / 2
 2x   xe y .2 sin x cos x

 x 2z 
 e y z   x sin 2 x  . Ans.
 a2  x2 

 y   z   x 
Q.No.7.: If φx , y, z   0 , show that        1 .
 z  x  x  y  y z
Partial Differentiation: Composite Functions Prepared by: Dr. Sunil, NIT Hamirpur 8

Sol.: The given relation defines y as a function of x and z. Treating x as constant


φ

 y
    z .
 z  x φ
y
The given relation defines z as a function of x and y. Treating y as constant
φ
 z 
    x .
 x  y φ
z
φ
 x  y
Similarly,    .
 y  y φ
x

 y   z   x 
Multiplying, we get        1 . Hence prove.
 z  x  x  y  y z

2z 2z 2z 2z


Q.No.8.: Prove that    ,
x 2 y 2 u 2 v 2
where x  u cos α  v sin α , y  u sin α  v cos α .
or
By changing the independent variables u and v to x by means of the

 2z  2z
relations x  u cos α  v sin α , y  u sin α  v cos α , show that 
u 2 v 2
 2z  2z
transforms into  .
x 2 y 2
Sol.: Here z is a composite function of u and v
z z dx z dy z z
 .  .  cos α  sin α
u x du y du x y

  
 z    cos α   sin α  z 
  
 cos α  sin α . (i)
u  x y  u x y

z z dx z dy z z
Also  .  .   sin α  cos α
v x dv y dv x y
Partial Differentiation: Composite Functions Prepared by: Dr. Sunil, NIT Hamirpur 9

  
 z     sin α   cos α  z 

  sin α
 
 sin α . (ii)
v  x y  v x y

Now we shall make use of the equivalence of operations as given by (i) and (ii)

 2z   z      z z 
     cos α  sin α  cos α  sin α 
u 2 u  u   x y  x y 

 2z  2z  2z  2z
 cos 2 α  cos α sin α  sin α cos α  sin 2 α 2
x 2 xy yx y

 2z  2z  2z
 cos 2 α  2 cos α sin α  sin 2 α 2 . (iii)
x 2 xy y

 2z   z      z z 
      sin α  cos α   sin α  cos α 
v 2 v  v   x y  x y 

 2z  2z  2z  2z
 sin 2 α  sin α cos α  cos α sin α  cos 2 α 2
x 2 xy yx y

 2z  2z  2z
 sin 2 α  2 cos α sin α  cos 2 α 2 . (iv)
x 2 xy y

 2z  2z  2z  2z
Adding (iii) and (iv), we get    . Hence prove.
u 2 v 2 x 2 y 2
u u u
Q.No.9: If u  f r, s  , r  x  y , s  x  y , prove that  2 .
x y r

Sol.: Since u  f r, s  and r, s are the function of x and y.


 u is the composite function of x and y.
u u r u s u u u  r s 
 .  .
x r x s x
  
x r s  x  1 and x  1 (i)
 

u u r u s u u u  r s 
 .  .      1 and  1 (ii)
y r y s y y r s  y y 
Now by adding (i) and (ii), we get
u u u u u u
    
x y r s r s
u u u
   2.
x y r
Partial Differentiation: Composite Functions Prepared by: Dr. Sunil, NIT Hamirpur10

Hence this proves the result.


Q.No.10: If u  f x , y  , x  r cos  , y  r sin  , then
2 2 2 2
 u   u   u  1  u 
         2   .
 x   y   r  r   
Sol.: Here u is a composite function of r and 
So we have
u u x u y x y
 .  . since  cos ,  sin 
r x r y r r r
u u
 cos   sin 
x y
By squaring, we get
2 2 2
 u   u  2  u  2  u   u 
     cos     sin   2 .  cos .sin  . (i)
 r   x   y   x   y 
Similarly we can get
u u x u u x y
 .  . since   r sin ,  r cos 
 x  y   
u u
 r .sin   r . cos 
x y
By squaring, we get

 u 
2  2
 u 
2
 u   u  
 2  u  2 2 
    r   . sin    
 y  . cos   2  .  sin  cos 
   
  x     x   y  

2 2 2
1  u   u   u   u   u 
 2      sin 2     cos 2   2 .  sin  cos  . (ii)
r     x   y   x   y 
Now by adding (i) and (ii), we get
2
   
2 2 2
 u  1  u   u  2 2  u  2 2
   2      cos   sin     cos   sin 
 r  r     x   y 
2 2 2 2
 u  1  u   u   u 
    2         .
 r  r     x   y 
Hence this proves the result.
Partial Differentiation: Composite Functions Prepared by: Dr. Sunil, NIT Hamirpur11

Q.No.11:If z be a function of x and y, and u and v be two other variables, such that
u  x  my , v  y  mx . Show that

 2z  2z
  u z  v z  , assuming that z is a function of u and v.
2 2

2
 2  2  m2 2 2
x y  
Sol.: Let us assume that z is a function of u and v.
z z u z v z z z z
 .  .  .  . m     m
x u x v x u v u v
z
Let  f . Since f is a composite function of x and y. Noting that f is also a function of
x
u and v.
f f u f v
  .  .
x u x v x
 2z   z  u   z  v  z 
x 2
 .    .
u  x  x v  x  x  By putting f  x 
 
  z z  u   z z  v
 .   m .     m .
u  u v  x v  u v  x

 2z   2z  2 z  u   2z  2z  v
   m    m . . (i)
x 2  u 2 uv  x  vu v 2  x

z z u z v z z
Similarly  .  .  m 
y u y v y u v

 2z   z  u   z  v
 . .   .
y 2 u  y  y v  y  y

 2z   2z  2z  u   2z  2 z  v
 .m    m   . . (ii)
y 2  u 2 uv  y  uv v 2  y
By adding (i) and (ii) we get,
  2z    2z  u   2 z  2 z   v   2 z  2 z 
     m      m
 x 2   y 2  x  u 2 uv   x  uv v 2 
    

 u   2z  2z   v   2z  2z 
   m 2      m  2 
 y  v uv   y  uv v 
Partial Differentiation: Composite Functions Prepared by: Dr. Sunil, NIT Hamirpur12

 2z  2z  2z  2z  2z  2z
   2  m  m  m2 2
x 2 y 2 u 2 uv uv v

 2z  2z  2z  2z
 m2  m  m  2 2
v 2 uv uv v

 2 

2 
 2z  2z 2
  z   z .
    2
 m
x 2 y 2  u 2 v 2 
 
Hence this proves the result.

Q.No.12: If z  f u , v  and u  x 2  2 xy  y 2 and v = y. Show that

x  y  z  x  y  z  x  y  z .
x y v
Sol.: Clearly z is a composite function of x and y
z z u z v z z
  .  .   2x  2y  z 0
x u x v x x u v
z z
  2x  y  . (i)
x u
Also
z z u z v
 .  .
y u y v y
z z
   2x  2y  z 1
x u v
z z z
  2x  y   . (ii)
y u v
Taking L. H. S. we get
z z
 x  y.  x  y 
x y
z z z
 x  y x  y  .2  x  y  2x  y  
u u v
z z z
 x  y x  y  .2  x  y  2x  y   x  y 
u u v
z z z
 x  y x  y  .2  x  y x  y  .2  x  y 
u u v
Partial Differentiation: Composite Functions Prepared by: Dr. Sunil, NIT Hamirpur13

z
 x  y  = R. H. S..
v
Hence this proves the result.

 2u  2u
Q.No.13: Transform the equation   0 into polar co-ordinates.
x 2 y 2
Sol.: The relations connecting Cartesian co-ordinates (x, y) with polar co-ordinates r,  

are x  r cos  , y  r sin  . Squaring and adding, we get r 2  x 2  y 2 .

y
Dividing, we get   tan 1 
x
y
 r  x 2  y 2 and   tan 1 
x
r x r cos 
   cos  and
x x 2
 y2  r

 1  y  y y r sin  sin 
     
x
1
y2  x 2  x 2
 y2 
2 x 2  y2 r2 r
x2
u u r u  u sin  u
  .  .  . cos   .
x r x  x r r 
 2u   u   f u
  .   where f 
x 2 x  x  x x

 2u f sin  f   u  sin    u 
 2
 cos   .  cos     .  
x r r  r  x  r   x 
 u sin  u  sin    u sin  u 
 cos   cos   .  .  cos   . 
r  r r   r   r r  

 2 u cos . sin   2 u cos . sin   2 u sin 2   2 u


 cos 2   .  .  2 . 2
r 2 r r r r  r 
sin 2  u cos . sin  u cos . sin  u
  .  . . (i)
r r r2  r2 
Similarly, we get

 2u  2u 2 cos .sin   2 u cos2   2u


 sin 2   .  2
y 2 r 2 r r r 2
Partial Differentiation: Composite Functions Prepared by: Dr. Sunil, NIT Hamirpur14

cos 2  u cos . sin  u cos . sin  u


  .  . . (ii)
r r r2  r2 
Adding (i) and (ii) we get

 2u
x 2

 2u
y 2
 2
 cos   sin  2
 r 2u
2

cos 2   sin 2   2 u
r2 2

cos 2   sin 2  u
r r

 2u 1  2u 1 u
 2
 2 2

r r  r r

 2u  2u  2u 1  2u 1 u
2
 2
 2
 2 2
  0.
x y r r  r r

Hence this proves the result.

Q.No.14: If v  r 3 and r 2  x 2  y 2  z 2 then show that

 2v  2v 2v  2v 2 v
    .
x 2 y 2 z 2 r 2 r r

v v r x
Sol.: Let  .  3r 2 .  3rx
x r x r
 2v
 2  3r  3x.
r x 3 r2  x2
 3r  3x.  .
  (i)
x x r r
Similarly we can find

 2v


3 r 2  y2  (ii)
2
y r

 2v


3 r 2  z2
.
 (iii)
2
z r
By adding (i), (ii) and (iii), we get

 2v

 2v

 2v


3 3r 2  x 2  y 2  z 2

 
3 3r 2  r 2

3  4r 2 
 12r . (iv)
2 2 2
x y z r r r

By differentiating v  r 3 w. r. t. r, we get
dv
 3r 2 .
dr
d 2v
Again differentiating, we get  6r
dr 2
Partial Differentiation: Composite Functions Prepared by: Dr. Sunil, NIT Hamirpur15

d2v 2 dv 2 2
 Let R. H. S.  .  6 r  .3r  6r  6r  12r . (v)
dr 2 r dr r
Hence from (iv) and (v), we get

 2v  2v  2v  2v 2 v
2
 2
 2
 2

x y z r r r

Hence this proves the result.


Q.No.15: If z  f x , y  , x  u cos   v sin  , y  u sin   v cos  , prove that

 2z  2z  2z  2z
   .
x 2 y 2 u 2 v 2
Sol.: Since z is a composite function of u and v
z z x z y z z
Thus  .  .  . cos .  .sin   f
u x u y v x y

 2z   z  f f x f y
Now,     .  .
u 2 u  u  u x u y u

 2z  2 2   2 2 
  cos   cos   z  sin   z   sin  cos   z  sin   z 
u 2  x 2 xy   xy y 2 
 

 2z  2z  2z  2z
  cos2   2 cos  sin   sin 2 z 2 . (i)
u 2 x 2 xy y
z z x z y z z
Similarly,  .  .   sin   cos  g
v x v y v x y

 2z   z  f f x f y
     .  .
v 2 v  v  v x v y v

  2z  2z    2z  2z 
  sin   sin  2  cos   cos   sin   cos  2 
 x xy   xy y 
 

 2z  2z  2z  2z
  sin 2   2 cos  sin   cos 2  2 . (ii)
v 2 x 2 xy y
Now by adding (i) and (ii), we get

 2z  2z
 xz  cos yz  xz  yz .
2 2 2 2
  cos2   sin 2  2
  sin 2 
u 2 v 2 2 2 2 2
Partial Differentiation: Composite Functions Prepared by: Dr. Sunil, NIT Hamirpur16

Hence this proves the result.

 dp   dt   dv 
Q.No.16: If f p, t, v   0 . Prove that          1 .
 dt  v  c  dv  p  c  dp  t  c

Sol.: When v  c then


f1  f p, t, v   f p, t, c   f p, t   0

 dp   f1 / t
Now    (i)
 dt  v c f1 / p

 dt   f 2 / v
Similarly    (ii)
 dv p c f 2 / t

 dv   f3 / p
and    (iii)
 dp  t  c f3 / v

Multiplying (i), (ii) and (iii), we get

 dp   dt   dv   f1 / t  f 2 / v  f3 / p
          
 dt  v  c  dv  p  c  dp  p  c f1 / p f 2 / t f3 / v

 f   f   f 
     1    3 
 p  v, t 0  p  v c  p  t  c

f1 f 2 f f
Similarly  and 2  3
t t v v
Thus, we get

 dp   dt   dv  f 1 f
         1   2  1 = R. H. S..
 dt  v  c  dv  p  c  dp  t  c t f1 / p v

Hence this proves the result.

Q.No.17: If f u , v   0 , u  x  my  mz and v  x 2  y 2  z 2 . Hence show that

y  mx   ny  mz  z  z  nx  z  0.


x y
Sol.: Since f is a composite function of x, y, and z. Then we have
f f u f v f f f
 .  .     2x (i)
x u x v x x u v
f f u f v f f f
and  .  .  m  2y (ii)
y u y v y y u v
Partial Differentiation: Composite Functions Prepared by: Dr. Sunil, NIT Hamirpur17

f f f
and  n  2z (iii)
z u v
Solving (i) and (ii), we get
f f f f
y x m 
f x y f x y
 and 
u y  mx v 2 xm  2y

 f f   f f 
y x  m  
f f f x y  x y 
  n  2z  n  2 z
z u v  y  mx   2mx  y  
   
   
f f f f f
 y  mx   ny  nx  mz  z
z x y x y
f f f
 y  mx   ny  mz   z  nx 
z x y
f / x f / y
 y  mx   ny  mz  z  nx  0
f / z f / z
 z f / x 
z z  x   f / z 
 y  mx   ny  mz   z  nx   0 .  
x y  z   f / y 
 y f / z 
Hence this proves the result.
Q.No.18.: If z  f x, y  , x  u  v , y  uv , prove that
z z z
(i) u  v   u v .
x u v
z z z
(ii) u  v    .
y v u
Sol.: Here z is a composite function of u and v
z z x z y z z z z
Hence  .  .  1  v  v (i)
u x u y u x y x y
Similarly we get
z z x z y z z z z
 .  .  1  u  u (ii)
v x v y v x y x y
Partial Differentiation: Composite Functions Prepared by: Dr. Sunil, NIT Hamirpur18

z z z z z z z z z
Let u v u  uv  v  uv u v  u  v  .
u v x y x y x x x
Hence this prove the (i) relation.
Let us subtract (ii) from (i), we get
z z z z z z z
  u  v  u  v  .
v u x y x y y
Hence this proves the (ii) relation.
x y z
Q.No.19.: If z  f r, s, t  and r  , s  and t  , prove that
y z x
u u u
x y z  0.
x y z

u u r u s u t u 1 u  z  u
Sol.: Here  .  .  .  .  .0     2 .
x r x s x t x r y s  x  t
1 u z u
  .
y r x 2 t
Similarly, we get
u 1 u x u u 1 u y u
  2 and  
y z s y r z x s z 2 s

u u u x u z u y u x u z u y u
x y z      
x y z y r x t z s y r x t z s
u u u
x y z  0.
x y z
Hence this proves the result.

 2z  2z
Q.No.20: If z  f x, y  and x  r cos  , y  r sin  express the equation  0
x 2 y 2
in terms of r  . Is the equation in terms of r and  valid at r  0 .

Sol.: Let x  r cos  and y  r sin   r  x 2  y 2

y
And   tan 1 
x
Partial Differentiation: Composite Functions Prepared by: Dr. Sunil, NIT Hamirpur19

r x
   cos 
x x 2  y2
 y y sin 
  2
 2 2

x  x 2  y 2  x y r
 
u u r u  u sin  u
  .  .  . cos   .
x r x  x r r 
 2u   u  f u
    where f 
x 2 x  x  x x

 2u f sin  f   u  sin    u 
 2
 cos .   cos      
x x r  r  x  r   x 
 u sin  u  sin    u sin  u 
 cos .  cos   .  .  cos .  . 
r  r r   r   r r  

 2u 2 sin . cos   2u sin 2   2 u sin 2  u


 cos 2   .  2 . 2  .
r 2 r r. r   r r
sin  cos  u sin . cos  u
 .  . . (i)
r2  r2 
Similarly, we get

 2u  2u 2 sin . cos   2 u cos2   2 u cos2  u


 sin 2   .  2 . 2 .
y 2 r 2 r r. r  r r

sin  cos  u sin . cos  u


 .  . . (ii)
r2 r r2 r
By adding (i) and (ii), we get

 2u  2u
  r u  r1 .sin  u  1r .sin  ur
2 2
  sin 2   cos 2  . 2
  cos 2  . 2
  cos 2  .
x 2 y 2 2 2 2

 2u 1  2 1 u
  .  .
r 2 r 2 2 r r
From this equation, we get

 2z z  2 z
r2 r   0.
r 2 r 2
When r = 0 then we have
Partial Differentiation: Composite Functions Prepared by: Dr. Sunil, NIT Hamirpur20

 2z
 0 . Thus the equation is valid.
2
Hence this proves the result.

Q.No.21: If x  u  v  w , y  u 2  v 2  w 2 , z  u 3  v3  w 3 then prove that


u vw
 .
x u  v u  w 
Sol.: Let x  u  v  w
By differentiating w. r. t. x, we get
x u v w u v w
      1 (i)
x x x x x x x
Also y  u 2  v 2  w 2
Again by differentiating partially w. r. t. x, we get
u v w u v w
0  2u  2v  2 w u v w 0 (ii)
x x x x x x

and z  u 3  v3  w 3
Again by differentiating partially w. r. t. x, we get
u v w u v w
0  3u 2  3v 2  3w 2  u2  v2  w2 0 (iii)
x x x x x x
u v w
Let a,  b and c
x x x
Putting these values in (i), (ii) and (iii), we get
a bc 1 (iv)
ua  vb  wc  0 (v)

u 2a  v 2 b  w 2c  0 (vi)
a  b  c  1  wa  wb  wc  w (vii)

ua  vb  wc  0  wua  wvb  w 2c  0 (viii)


Now subtracting (v) from (vii), we get
w  u a  w  v b  w
Now subtracting (vi) from (viii), we get

wu  u a  wv  v b  0 i. e.
2 2
Partial Differentiation: Composite Functions Prepared by: Dr. Sunil, NIT Hamirpur21

w  u a  w  v b  w  vw  u a  vw  v b  wv (ix)


and u w  v a  vw  v b  0 (x)
By solving, we get
vw
vw  u a  u w  v a  vw  v  u w  v a  vw  a 
v  u w  v 
u uw
Hence  .
x u  v u  w 
Hence this proves the result.
Q.No.22: If x  cosh . cos  , y  sinh .sin  then show that

 x, y  1
J   cosh 2  cos 2 .
 ,   2
x x
 x, y    x . y  x . y
Sol.: Let J   
 ,   y y    
 
x x
  sinh  cos  ;   cosh .sin 
 
y x
and  cosh  sin  ;  sinh . cos 
 

 x, y 
 J   sinh . cos 
. sinh . cos   cosh .sin 
. cosh .sin 
 ,  

 cos 2 .sinh 2   cosh 2  sin 2 

ex  ex ex  ex
Now here sinh   and cosh  
2 2
2 2 2 2
2e x  e  x  2e x  x e x  e x 1
 cosh    
4 4 2
2 2
2 ex  ex 1
and sinh   
4 2

 x, y   x 2  e x 2 1   x 2  ex 2 1 
2 e  2 e
J   cos    sin   
 ,    4 2   4 2
   
Partial Differentiation: Composite Functions Prepared by: Dr. Sunil, NIT Hamirpur22

2 2


ex  ex
4
 1
 
cos 2   sin 2   cos 2   sin 2 
2

2 2 2 2
ex  ex 1 1 ex  ex 1
  cos 2  .  cos 2
4 2 2 2 2

1  e x  e  x  1
2 2
 e x  e x 
 .  cos 2   .cos 2h  cos 2   cosh 
2 2  2
 2 
 
Hence this proves the result.
yz zx xy  u, v, w 
Q.No.23.: If u  , v , w , show that  4.
x y z  x , y, z 
u yz v zx w xy
Sol.: Here  2,   2 and  2
x x y y z z
u z v x w y
 ,  and 
y x z y x z
u y v z w x
and  ,  and 
z x x y y z
 Taking L. H. S., we get
 u , v, w   u , v, w 
J  
 x, y, z  x , y, z 

u u u yz z y yz
  z y
x y z x 2 x z x
v v v z zx x 1 zx
   2
 z  x
x y z y y y x yz y
w w w y x xy xy
 2 y x 
x y z z z z z

1  yz  zx xy   
    x 2   z xy  yx   yzx  zx 
x y z  x  zy  
1
  yzx  yzx   z 2xy   y2zx 
x yz
1
 0  2xyz  2xyz  1 4xyz = 4 = R. H. S..
xyz x yz
Hence this proves the result.
Partial Differentiation: Composite Functions Prepared by: Dr. Sunil, NIT Hamirpur23

 r,   1
Q.No.24.: If x  r cos  , y  r sin  , prove that J   .
 x, y  r
Sol.: Given that x  r cos  (i)
And y  r sin  (ii)
From (i) and (ii), we get
y
r  x 2  y 2 and   tan 1
x
r 2x x
So we get    cos 
x 2 x 2  y 2 x 2  y2

r 2y y
And    sin 
y 2 x 2  y 2 x 2  y2

  y 
1 y
Similarly   2    2 2
x y  x  x  y2
1  
x
 1 1 x
And  2
.  2
y y x x  y2
1  
x
Let L. H. S.
r r
 r,   x y r   r x x y y
J    .  .  .  .
 x , y    x y x y 2 2 2
2
x 2  y2 x  y x 2  y2 x  y
x y

x2 y2 x 2  y2 1 1
    
2
x y 2
x y2 2 2
x y 2 2
x y 2
x y2 2
x y2 2 2
x y 2 r

= R. H. S..
Hence this proves the result.
x, y, z 
Q.No.25.: If x   cos , y   sin  , z = z, show that  .
, , z 
Sol.: Let x   cos , y   sin  and z  z
x y z
  cos  ,  sin  and 0
  
Partial Differentiation: Composite Functions Prepared by: Dr. Sunil, NIT Hamirpur24

x y z
  sin  ,   cos  and 0
  
x y z
and  0,  0 and 1
z z z
Taking L. H. S., we get
x x x
cos    sin  0
  z
x, y, z  y

, , z  
y

y
z

 sin   cos  0  1  cos 2    sin 2  
z z z
0 0 1
  z

 
  cos 2   sin 2  =  = R. H. S..
Hence this proves the result.
Q.No.26.: If x  f u, v  , y  u , v  are two functions which satisfy the equations
f  f 
 ,  and z is a function of x and y, then prove that
u v v u

 2z   2 z  2 z   f  2  f  2 
 2z
 2   2  2        .
u 2
v  x y   u   v  

z z x z y
Sol.: Given that  .  .
u x u y u

z f z    z  
g .  .    g
x u y u u  u  u
g x g y
 .  .
x u y u

f  f  2 z z  2f  2z   2 z 
 . 2 .  .  .
u  u x x xu xy u xu y 

  f  2 z z  2f  2 z   2 z 
 .  .  2.  . .
u  u xy x yu y u yu y 

 2f  2f   f   2
Now we have     1  0   f .
xu ux u  x  u yv
Partial Differentiation: Composite Functions Prepared by: Dr. Sunil, NIT Hamirpur25

 2  2
Similarly, we can have 0 .
yu yv
So that

 2z f  f  2z  2z   2 z    f  2u z f   2 z 
 . 2 .  .  .  .  . 2 (i)
u 2 u  u x xy v xu y  u  u xy x yu u y 

Similarly, we can find

 2z f  f  2z  2 z   2 z    f  2 u z  2f   2 z 
 . 2 .  .  .  .  . 2 (ii)
v 2 v  v x xy v xu y  v  v xy x yv v y 

f  f 
Since  and 
u v v u
Taking L. H. S., we get

 2z f  f  2z  2 z f
 2z  2f z  f  f  2z  2f z f  2 z 
  .  .  .  .  .  .
u 2 v 2 u  u x 2 xy v xv y  v  u xy yu x v y 2 

f  f  2 z  2z f  2f z  f  f  2z  2f z f  2 z 
 . 2 .  .  .  .  .
v  v x xy u xu y  u  v xy yv x u y 2 

22 2
2 22 2 2
 f   z  f   z  f   z  f   z
   . 2   . 2   . 2   . 2
 u  x  u  y  v  x  v  y

f  z  2f z  2f  f  z  2f z  2f 
  .  .   .  .
v  x uy y ux  u  x vy y vx 

 f f 
 ux  vx  0
 

  2 z  2z   f 2  f  2  f  z  2  f  z  2 
  2  2                
 x  y   u   v   v  x vy  u  x uy 
      

  2u  2 
    0
 vy uy 

  2 z  2z   f 2  f 2 
  2  2        = R. H. S..
 x y   u   v  

Partial Differentiation: Composite Functions Prepared by: Dr. Sunil, NIT Hamirpur26

Hence this proves the result.


z
Q.No.27: If z  u 2  v 2 , x  u 2  v 2 and y  uv . Find the value of .
x
z z u z v z u v
Sol.: Here  .  .   2u  2 v
x u x v x x x x

Now u 2  v 2  x .
Differentiating w.r.t. to x, we get
u v x u v
2u  2v   2u  2v 1 , (i)
x x x x x
and v u  y .
Differentiating w. r. t. to x, we get
v u
u.  v.  0 . (ii)
x x
Solving (i) and (ii), we get
u v
x  x  1
0  u  v  0 2u 2  2 v 2
u u u v v v
    
2

x 2 u  v 2 2zand 2
x 2 u  v 2

2z 
z u v u 2  v2 x
  2u.  2v.  
x 2z 2z z z
z x
Hence  . Ans.
x z

*** *** *** *** ***


*** *** ***
***
Partial Differentiation: Composite Functions Prepared by: Dr. Sunil, NIT Hamirpur27
Differential Calculus

Taylor’s and Maclaurin’s Infinite Series


Prepared by:
Dr. Sunil
NIT Hamirpur (HP)
(Last updated on 12-09-2009)
Latest update available at: http://www.freewebs.com/sunilnit/

(65 Solved problems and 12 Home assignments)


Maclaurin’s Theorem:
Statement: If f(x) can be expanded in ascending powers of x, then
x2 x3 xn n
f ( x )  f (0)  xf (0)  f (0)  f (0)  .......  f (0)  ........
2! 3! n!

Proof: Suppose f x   a 0  a1x  a 2 x 2  a 3x 3  ...........  a n x n  .......... (i)


where a0 , a1 , a2 ,a3 ,...... are constants to be evaluated.
Differentiating (i) w. r. t. x, we get
f ( x )  a 1  2a 2 x  3a 3 x 2  4a 4 x 3  ......  na n x n 1  ...... (ii)
Differentiating (ii) w. r. t. x, we get
f ( x )  2a 2  3.2a 3 x  4.3a 4 x 2  ......  n (n  1)a n x n  2  ...... (iii)
Differentiating (iii) w. r. t. x, we get
f ( x )  3.2.1a 3  4.3.2a 4 x  ......  n (n  1)(n  2)a n x n 3  ...... (iv)
Similarly, if we go on differentiating, we get

f n ( x )  n ( n  1)( n  2)...... 3.2.1.a n  terms containing x (v)


Putting x = 0 in (i) to (v) , we get
f (0) f (0) f n (0)
a 0  f (0) , a 1  f (0) , a 2  , a3  ,........, a n  .
2! 3! n!
Differential Calculus: Taylor and Maclaurin’s Theorems 2
Prepared by: Dr. Sunil, NIT Hamirpur (HP)

Putting these values of constants in (i), we get


x2 x3 xn n
f ( x )  f (0)  xf (0)  f (0)  f (0)  .......  f (0)  ............ .
2! 3! n!
This completes the proof.

Taylor’s Theorem:
Statement: If f (x + h) can be expanded in ascending powers of x, then
x2 x3 xn n
f ( x  h )  f (h )  xf (h )  f (h )  f (h )  .......  f (h )  ............ .
2! 3! n!

Proof: Suppose f ( x  h )  a 0  a 1 x  a 2 x 2  a 3 x 3  ......  a n x n  ...... (i)


where a0 , a1 , a2 ,a3 ,...... are constants to be evaluated.
Differentiating (i) w.r.t. x, we get
f ( x  h )  a 1  2a 2 x  3a 3 x 2  4a 4 x 3  ......  na n x n 1  ...... (ii)
Differentiating (ii) w.r.t. x, we get
f ( x  h )  2a 2  3.2a 3 x  4.3a 4 x 2  ......  n (n  1)a n x n  2  ...... (iii)
Differentiating (iii) w.r.t. x, we get
f ( x  h )  3.2.1a 3  4.3.2a 4 x  ......  n (n  1)(n  2)a n x n 3  ...... (iv)
Similarly, if we go on differentiating, we get

f n ( x  h )  n ( n  1)( n  2)......3.2.1.a n  terms containing x (v)


Putting x = 0 in (i) to (v), we get

f (h ) f (h ) f n (h )
a 0  f (h ) , a 1  f (h ) , a 2  , a3  ,........, a n  .
2! 3! n!
Putting these values of constants in (i), we get
x2 x3 xn n
f ( x  h )  f (h )  xf (h )  f (h )  f (h )  .......  f (h )  ............ .
2! 3! n!
This completes the proof.

Remarks:
(i) Put h = 0, we get
Differential Calculus: Taylor and Maclaurin’s Theorems 3
Prepared by: Dr. Sunil, NIT Hamirpur (HP)

x2 x3 xn n
f ( x )  f (0)  xf (0)  f (0)  f (0)  .......  f (0)  ............ .
2! 3! n!
x2 x3 xn
y  y(0)  xy1 (0)  y 2 (0)  y3 (0)  .......  y n (0)  ............ .
2! 3! n!
which is a Maclaurin’s expansion.
(ii) If we interchange x and h, we get
h2 h3 hn n
f (h  x )  f ( x )  hf ( x )  f ( x )  f ( x )  .......  f ( x )  ............ .
2! 3! n!
(iii) If we replace x by a and h by (x - a), we get
(x  a) 2 (x  a ) 3 (x  a) n n
f ( x )  f (a )  ( x  a )f (a )  f (a )  f (a )  .......  f (a )  ......
2! 3! n!
Taylor’s Theorem for functions of two variables:
Statement: Prove that
2
    1   
f x  h , y  k   f x, y    h  k f   h  k  f  ...... .
 x y  2!  x y 

Proof: Considering f  x  h, y  k  as a function of a single variable x, we have

f x, y  k  h 2  2f x , y  k 
f x  h , y  k   f  x , y  k   h   ...... ...(i)
x 2! x 2
Now expanding f  x, y  k  as a function of y only, we obtain

f x , y  k 2  2f x , y 
f x , y  k   f x , y   k   ...... ...(ii)
y 2! y 2

(i) takes the form


f x , y  k 2  2f x , y 
f x  h , y  k   f  x , y   k   ......
y 2! y 2

  f x, y  k 2  2f x , y  
h f  x , y   k   ..... 
x  y 2! y 2 

h 2  2  f x, y  k 2  2f x , y  
 f  x , y   k   ...... 
2! x 2  y 2! y 2 
Differential Calculus: Taylor and Maclaurin’s Theorems 4
Prepared by: Dr. Sunil, NIT Hamirpur (HP)

f f 1  2  2f  2f 2  f 
2
Hence f x  h , y  k   f x , y   h k  h  2hk k   ...... .
x y 2!  x 2 xy y 2 

In symbols we write it as
2
    1   
f x  h , y  k   f x, y    h  k f   h  k  f  ...... .
 x y  2!  x y 

Now let us expand some well known functions by using these theorems:

x 2 x3 xn
Q.No.1.: Prove that e x  1  x    .......   ........ .
2 ! 3! n!
Sol.: Let f x   e x , f  x   e x , f  x   e x ,............., f n x   e x

 f 0   1 ,  f  0   1 , f  0  1 ,..........., f n 0   1

x2 xn n
Maclaurin’s expansion is f x   f 0   xf  0  f  0   ......  f 0 ......
2! n!
Substituting all the above values in this equation, we get

x 2 x3 xn
ex  1  x    .......   ........
2 ! 3! n!
Note: If we replace x by –x, we get

x 2 x3
e x  1  x    ............... ..
2 ! 3!
x 2 x4 x6
Q.No.2.: Prove that cosh x  1     ........ .
2! 4! 6!

Sol.: Since cosh x 


1 x
2

e  e x 
1  x 2 x3   x 2 x3 
 1  x    ........   1  x    ........ 
2  2 ! 3!   2 ! 3! 
  

x 2 x4 x6
Hence, cosh x  1     ........ .
2! 4! 6!
x3 x5 x7
Similarly, sinh x 
1 x
2
 
e  e x  x   
3! 5! 7 !
 ........ .

x3 x5 x7
Q.No.3.: Prove that sin x  x     ........ .
3! 5! 7 !
Differential Calculus: Taylor and Maclaurin’s Theorems 5
Prepared by: Dr. Sunil, NIT Hamirpur (HP)

 n 
Sol.: Let y  sin x  y n  sin  x  
 2 
 y0  0 , y1 0  1 , y 2 0  0 , y3 0  1 , y n 0  0 and so on
Now Maclaurin’s series is

x2 x3
f x   f 0  xf  0   f  0  f  0   ........
2! 3!
x2 x3
 y0  xy1 0  y 2 0  y3 0  ......
2! 3!
x3 x5 x7
x    ........ .
3! 5! 7 !
x 2 x 4 x6
Similarly, cos x  1     ........ .
2! 4! 6!
x3 2 5
Q.No.4.: Prove that tan x  x   x  ........ .
3 15
Sol.: Let y  f x   tan x .

 y1  f  x   sec 2 x  1  tan 2 x .

 
y 2  f  x   2 tan x. sec 2 x  2 tan x 1  tan 2 x  2 tan x  2 tan 3 x


y3  f  x   2 sec2 x  6 tan 2 x sec2 x  2 sec 2 x 1  3 tan 2 x 
   
 2 1  tan 2 x 1  3 tan 2 x  2 1  4 tan 2 x  3 tan 4 x . 
y 4  f iv x   16 tan x  40 tan3 x  24 tan5 x .

    
y5  f v x   16 1  tan 2 x  120 tan 2 x 1  tan 2 x  120 tan 4 x 1  tan 2 x . 
Putting x  0 , we get
y0  0 , y1 0  1 , y 2 0   0 , y3 0  2 , y 4 0   0 , y5 0  16 and so on.
Now, Maclaurin’s series is

x2 x3 x 4 iv x5
f x   f 0  xf  0   f  0  f  0  f 0  f v
0 ...........
2! 3! 4! 5!
x3 x5
 tan x  x  .2  .16  ..........
3! 5!
Differential Calculus: Taylor and Maclaurin’s Theorems 6
Prepared by: Dr. Sunil, NIT Hamirpur (HP)

x3 2 5
 tan x  x   x  ........ .
3 15
x2 x3 x 4 x5
Q.No.5.: Prove that log1  x   x      ............ .
2 3 4 5
Sol.: y  log1  x 
1
 y1   1  x 1 , y 2   11  x 2 , y3   1 21  x 3
1  x 
y 4   1 2  31  x 4 , y5   1 2  3 4 1  x 
5

Putting x  0 , we get
y0  0 , y1 0  1 , y 2 0   1 , y3 0  2 , y 4 0  6 , y5 0  24,...........
Now, Maclaurin’s series is

x2 x3 x4 x5
f x   y0  xy1 0  y 2 0   y3 0  y 4 0   y5 0 ..........
2! 3! 4! 5!
x 2 x3 x4 x5
x  .2  .6  .24...........
2 ! 3! 4! 5!
x 2 x3 x 4 x5
 x    .......... .. .
2 3 4 5

Q.No.6.: Expand tan 1 x in powers of x by Maclaurin’s theorem.

Sol.: Here f x   tan 1 x

 f  x  
1
1 x 2

 1  x2 
1
 1  x 2  x 4  x 6  x 8 ........... [by Binomial theorem]

f  x   2 x  4 x 3  6 x 5  8 x 7 .......... ....

f x   2  12 x 2  30 x 4  56 x 6 .......... ....

f iv x   24 x  120 x 3  .......... ....

f v x   24  360 x 2  ..............
Putting x  0 , we get

f 0   0 , f  0  1 , f  0  0 , f  0   2 , f iv 0   0 , f v 0   24 .

x2 x3 x 4 iv x5 v
f x   f 0  xf  0  f  0   f  0  f 0  f 0 ........
2! 3! 4! 5!
Differential Calculus: Taylor and Maclaurin’s Theorems 7
Prepared by: Dr. Sunil, NIT Hamirpur (HP)

x3 x5
 x  ......... .
3 5
x4 x6
Q.No.7.: Prove that e x sin x  1  x 2    .......... .
3 120
or

Find the expansion of e x sin x as far as the terms in x 6 .

Sol.: Here if we use Maclaurin’s theorem, we need successive derivatives of e x sin x ,


which is inconvenient to obtain. We, therefore, make use of the two standard series.

x x 2 x3 x3 x5 x7
e  1 x   .......... , sin x  x     ............
2 ! 3! 3! 5 ! 7 !

e x sin x  1  x sin x  
x sin x 2  x sin x 3  x sin x 4  ..........
2! 3! 4!
x2 3 4
 1  x sin x  sin x 2  x sin x 3  x sin x 4  ............
2! 3! 4!
2
 x3 x5 x7  x2  x3 x5 x7 
 1  x x     .......   x     .......
 3! 5! 7 !  2!  3! 5! 7 ! 
   
3 4
x3  x3 x5 x7  x 4  x3 x5 x7 
 x     .......  x    .......  ......
3 !  3! 5 ! 7 ! 
 4 !  3! 5! 7 ! 

2 3
 x2 x4
2
 x4  x2 x4  x6  x2 x4 
 1 x 1  .....  1   .....  1   .....
 6 120  2  6 120  6  6 120 

4
x8  x 2 x 4 
 1   .....  ...........
24  6 120 

Now expanding by binomial theorem, we get

 x2 x4  x4   x2 x4   x6   x2 x4  
 1  x 2 1   .....   1  2  ..... ......  1  3  .....   .....  ....
 6 120  2   6 120   6   6 120  
      
Collecting terms of the same nature, we get
 1 1  1 1 1
e x sin x  1  x 2      x 4      x 6  ..........
 6 2  120 6 6 
Differential Calculus: Taylor and Maclaurin’s Theorems 8
Prepared by: Dr. Sunil, NIT Hamirpur (HP)

x 4 x6
e x sin x  1  x 2    .......... .
3 120
Differential Calculus: Taylor and Maclaurin’s Theorems 9
Prepared by: Dr. Sunil, NIT Hamirpur (HP)

ex
Q.No.8.: Apply Maclaurin’s theorem to find the expansion of as far as the term
ex  1
in x 3 .

ex 1
Sol.: Let f x   x
, f 0  
e 1 2

f  x  
e  1e  e .e  e ,  f  0  1
x x x x x

e  1 x
e  1 4 x 2

f  x  
e  1e  e .2e  1e  e  1e 1  e .e
x x x x x x x x x
 2e 2 x

e x  e 2 x  2e 2 x
e  1  x
e  1
2 2 x 4
e  1
x 3


e " e  ,  f  0  0
x 2x

e  1 x 3

f  x  
e  1 e
x 3 x
  
 2e 2 x  e x  e 2 x . 3 e x  1 .e x 
3
,  f  0   
1


 ex  1
 
 3 2 8

x2 x3
f x   f 0   xf  0  f  0  f  0  .......... ....
2! 3!
ex 1 1 1 x3
   x . x   ........ .
ex  1 2 4 4 48

x 2 1 x 4 2 x6
Q.No.9.: Prove that logsec x    .  .  ......... .
2 3 4 15 16
Sol.: Let y  log sec x ,then
dy 1 sec x tan x
 .  tan x
dx sec x 1
x3 2 5
But tan x  x   x  ........
3 15
dy x3 2 5
  x  x  ........
dx 3 15
Integrating w. r. t. x , we get
Differential Calculus: Taylor and Maclaurin’s Theorems 10
Prepared by: Dr. Sunil, NIT Hamirpur (HP)

x2 x 4 2 x6
y   .  .........  a 0
2 12 15 6
where a 0 is the constant of integration

To evaluate a 0 , But x = 0, y = 0, and  a 0  0

x 2 1 x 4 2 x6
Hence logsec x    .  .  ......... .
2 3 4 15 16
 x2 1 x4 2 x6 
Remarks: log cos x   log sec x     .  .  .........
 2 3 4 15 16 

Q.No.10.: If x 3  2 xy 2  y3  x  1 , expand y in the ascending powers of x.


Sol.: Here y is implicit function of x and so to get an expansion for y, we shall obtain first
y 0 , y 0 , y 0 , etc. by a little different method and use Maclaurin’s theorem.

Here x 3  2 xy 2  y 3  x  0 (i)

Put x = 0, we get  y 301  0  y 0  1 (ii)


Differentiating (i) w. r. t. x , we get

3x 2  2 y 2 4 xy y   3y 2 y   1  0 (iii)

Put x = 0 and y 0  1 in (iii), we get

2  3y 0  1  0  y 0  1 (iv)


Differentiating (iii) w. r. t. x, we get
2
6 x  8 yy  4 x y 2  4 xyy   6 yy 2  3y 2 y 2  0 (v)

Putting x = 0,  y 0  1 , y 0  1 in (v), we get

2
 8  6  3y 0  0  y 0   (vi)
3
Now by Maclaurin’s theorem

x2 3
y  y 0  x y 0  y 0  x y 0  .............
2! 3!
x2 x3
y  f 0  xf  0   f  0  f  0   .............
2! 3!
Substituting the values of
Differential Calculus: Taylor and Maclaurin’s Theorems 11
Prepared by: Dr. Sunil, NIT Hamirpur (HP)

y0  1 , y 0  1 , y 0   2 from (ii), (iv) and (vi) etc.


3
x2 2
Then the required expansion of y is y  1  x  . .........
2! 3
1
Hence y  1  x  x 2 .........
3
Q.No.11.: Expand log cosx  h  in powers of h by Taylor’s theorem.
Sol.: Let f x  h   log cosx  h 

 f x   log cos x
1
f  x    sin x    tan x
cos x

f  x    sec 2 x

f  x   2 sec x. sec x tan x  2 sec 2 x tan x


Now by Taylor’s series

h2 x3
f x  h   f x   hf  x   f x   f  x   .............

2! 3!
Substituting the values of f x , f  x , f  x , f  x  in the above equation, we get

log cosx  h   log cos x  h  tan x  


h2
2!

 sec 2 x h3
3!
 
 2 sec2 x tan x ..........

h2 h3
log cosx  h   log cos x  h tan x  sec 2  sec2 x tan x  .......... . Ans.
2 3

Q.No.12.: Use Taylor’s theorem to express the polynomial 2x 3  7 x 2  x  6 in


powers of x  2  .
Sol.: Let f x   f 2  x  2  , using Taylor’s theorem, we get

f 2  x  2  f 2   x  2f  2 


x  22 f  2  x  23 f  2  .............
2! 3!
Now f x   2 x 3  7 x 2  x  6

f  x   6 x 2  14 x  1

f  x   12 x  14
Differential Calculus: Taylor and Maclaurin’s Theorems 12
Prepared by: Dr. Sunil, NIT Hamirpur (HP)

f  x   12

f x   52 , f  2   53 , f  2   38 , f  2   12 , f iv
2  0

 f x   f 2  x  2   f 2  x  2f  2  
x  22 f  2  x  23 f  2  .............
2! 3!

 2 x 3  7 x 2  x  6  52  53x  2   19x  2 2  2x  2 3 .


This is the required result.

Q.No.13.: Calculate the approximate value of 10 to four decimal places using


Taylor’s expansion.
Sol.: Let f x  h   x  1  9  1  10
where x = 9 and h = 1
 f x   x  f 9   3
1 1 / 2 1
f  x   x f  9  
2 6
1 3/ 2 1
f  x   x f  9  
4 4  27
3 5 / 2 1
f  x   x f  9  
8 8  81
Now Taylor’s series is

h2 h3
f x  h   f x   hf  x   f  x   f  x   .............
2! 3!
1 1
10  f 9  1  f  9   f  9   f  9  ...........
2 6
1 1 1 1 1 1
or 10  3        ..........
6 2 4 27 6 8  81
 3  .16666  .00463  .00025
=3.1623 (app.).
This is the required result.
Q.No.14.: Expand sin x  h y  k  in powers of h and k by Taylor’s theorem.
Sol.: Here Fx  h, y  k   sin x  h y  k 
 Fx, y   sin xy (i)
Differential Calculus: Taylor and Maclaurin’s Theorems 13
Prepared by: Dr. Sunil, NIT Hamirpur (HP)

By Taylor’s theorem


Fx  h , y  k   Fx, y   hFx  kFy   1 2
2!

h Fxx  2hkFxy  k 2Fyy  (ii)

Differentiate (i) partially w. r. t. x and y, we get



Fx  sin xy   y cos xy ,
x

Fxx  y sin xy y   y 2 sin xy



Fxy  x cos xy   cos xy  x  sin xy y  cos xy  xy sin xy .
x
Putting the values of Fx  h, y  k  , Fx , Fy , Fxx , Fxy in (ii), we get

sin x  h y  k   sin xy  hy cos xy  kx cos xy

  
1 2
2!
 
h  y 2 sin xy  2hk cos xy  xy sin xy   k 2  x 2 sin xy  .......... 
 sin xy  hy  kx cos xy 
1
2!
 
2hk cos xy  hy  kx 2 sin xy  .......... . Ans.

Q.No.15.: Expand e x sin y by Taylor’s theorem in powers of x and y as for as terms of


second degree.

Sol.: Here f x , y   e x sin y , f 0, 0   0

f x x , y   e x sin y , f x 0, 0   0

f y x , y   e x cos y , f y 0, 0   1

f xx x , y   e x sin y , f xx 0, 0   0

f yy x , y   e x sin y , f yy 0, 0   0

f xy x , y   e x cos y , f xy 0, 0   1

x2 y2
f x , y   f 0, 0  xf x 0, 0   yf y 0, 0   f xx 0, 0   xyf xy 0, 0  f yy 0, 0   .....
2! 2!

 e x sin y  0  x 0   y1  xy1  0

e x sin y  y  xy  ......... . Ans.


y
Q.No.16.: Expands tan 1 in the neighbourhood of (1, 1) by Taylor’s theorem.
x
Differential Calculus: Taylor and Maclaurin’s Theorems 14
Prepared by: Dr. Sunil, NIT Hamirpur (HP)

y
Sol.: Here f x, y   tan 1 (i)
x
a  1, b  1

1 
f 1, 1  tan 1   tan 1 1  , putting x  y  1 in (i), we get
1 4
By Taylor’s theorem, we get
f x , y   f 1, 1  x  1f x 1, 1  y  1f y 1, 1


1
2!
 
x  12 f xx 1, 1  2x  1y  1f xy 1, 1  y  12 f yy 1, 1  ......... (ii)

Differentiate (i) partially w. r. t. x and y, we get

 x y 1 1
fx  tan 1    2  f x 1, 1    x  y  1
x y x  y2 11 2

 x x 1 1
fy  tan 1    2  f y 1, 1  
x y x  y2 11 2

  y  2 xy 2 1
f xx  
 2 2

  f xx 1, 1  
x  x  y  x  y2
2 4 2

  x   2 xy 2 1
f yy  
 2 2

  f yy 1, 1   
x  x  y  x 2  y2 4 2

  x  1 1
f xy     f xy 1, 1  0
x  x 2  y 2  4

Putting the values of f 1, 1, f x 1, 1, f y 1, 1, f xy 1, 1 etc. in (ii), we get

y  1 1 1
tan 1   x  1  y  1 
x 4 2 2 2!
 1  1 
 x  12 .  2x  1y  1.0  y  12     ..........
 2  2 

1 y   x  1   y  1  1  x  12  y  12 
Hence, tan        ........... . Ans.
x 4  2   2  2 !  2 

Q.No.17.: Expand x 2 y  3y  2 in powers of x  1 and y  2  using Taylor’s


Theorem.
Differential Calculus: Taylor and Maclaurin’s Theorems 15
Prepared by: Dr. Sunil, NIT Hamirpur (HP)

Sol.: Expansion f(x, y) in powers of x  a  and x  b  is given by


 
f x , y   f a , b   x  a f x a , b   y  b f y a , b 


1
2!

x  a 2 f xx a, b  2x  a y  bf xy a, b  y  b2 f yy a , b 
x  a 3 f xxx a , b   3x  a 2 y  b f xxy a , b  
1   ....
 (i)
3!   
 3 x  a y  b  f xyy a , b   y  b  f yyy a , b 
3 3

Here f ( x , y)  x 2 y  3y  2 , a = 1, b  2 .

f 1,  2   12   2   3 2   2  10

f x  2 xy , f x 1,  2   21(2)  4 ; fy  x2  3 , f y 1,2   12  3  4

f xx  2 y , f xx  (1,  2)  2 2  4 ; f xy  2 x , f xy (1,  2)  2(1)  2

f yy  0 , f yy  (1,  2)  0 ; f xxx  0 , f xxx (1,  2)  0

f xxy  2 , f xxy  (1,  2)  2 ; f xyy  0 , f xyy (1,  2)  0

f yyy  0 , f yyy  (1,  2)  0 .

All higher order partial derivatives vanish.

From (i), we get x 2 y  3y  1  f ( x , y)

 10  x  1 4   y  2 4  


1
2

x  12  4  2x  1y  2(2)  y  22 0 

1
6

x  13 0  3x  12 y  22  3x  1y  22 0  y  22 0 
 10  4x  1  4y  2   2x  12  2x  1y  2   x  12 y  2  .

Now let us solve some more problems:

Q.No.1.: Use Maclaurin’s theorem to expand esin x in a power series up to term

containing x 7 .

x 2 x3
Sol.: Since we know that ex  1  x    .......... ......
2 ! 3!
Differential Calculus: Taylor and Maclaurin’s Theorems 16
Prepared by: Dr. Sunil, NIT Hamirpur (HP)

x2 x3 x7
and sin x  x     .......... ........
3! 5 ! 7 !

e sin x
 1  sin x 
sin x 2 sin x 3
  .........................
2! 3!
2
 x3 x5 x7  1  x3 x5 x7 
 1  x     ........   x     ........
 3! 5! 7 !  2 !  3! 5! 7 ! 
3
1 x3 x5 x7 
 x     ........  ..........
3 !  3! 5! 7 ! 
2
 x2 x4 x6  x2  x2 x4 x6 
 1  x 1     .........  1     .........
 3! 5 ! 7 !  2 !  3 ! 5 ! 7 ! 
3
x3  x2 x4 x6 
 1     .........  ...........
3 !  3! 5 ! 7 ! 
Using Binomial expansion, we get

 x2 x4 x6  x2   x2 x4 x6  
e sin x  1  x 1     .........  1  2    .........  .....

 3 ! 5 ! 7 !  2 !   3! 5 ! 7 !  

x3   x2 x4 x6  
 1  3    .........  ....  ..............

3 !   3! 5 ! 7 !  
Collecting terms of the same nature, we get

x2 1 4
e sin x  1  x   x  ................... .
2 8
This is the required expansion.
Q.No.2.: Obtain by Maclaurin’s theorem the first five terms in the expansion of
log1  sin x  .

x2 x3
Sol.: We know f x   f 0  xf  0  f  0   f  0   ............. (i)
2! 3!
Here f x   log1  sin x   f 0  log1  0  0  f 0  0
1 cos x
f  x   . cos x   f  0  1
1  sin x 1  sin x
Differential Calculus: Taylor and Maclaurin’s Theorems 17
Prepared by: Dr. Sunil, NIT Hamirpur (HP)

f  x  
1  sin x  sin x   cos2 x   sin x  1  f  0   1
1  sin x 2 1  sin x 2

f  x  
1  sin x 2  cos x   21  sin x cos x  sin x  1  f  0  1
1  sin x 2
Similarly f iv
0   2 and f v
0   5 and so on.
Substituting these values in (i), we get

x2 3 4 5
log1  sin x   0  x   1  x 1  x  2  x 5  .............
2! 3! 4! 5!
x 2 x3 x 4 x5
x     .............. .Ans.
2 6 12 24
Q.No.3.: Expand log1  log1  x  in powers of x by Maclaurin’s theorem as for as the

term containing x 3 .
Sol.: Since we know that

x 2 x3 x 4
log1  x    x     .................... (i)
2 3 4
2
t t3 t 4
log1  t    x     .................... (ii)
2 3 4
Putting t  log1  x  in (ii), we get
2
 x 2 x3 x4  1 x 2 x3 x 4 
log1  log1  x    x     ..........   x     ..........
 2 3 4  2  2 3 4 

3 4
1 x 2 x3 x 4  1 x2 x3 x 4 
 x     .......   x     .......  ........
3  2 3 4  4  2 3 4 
2
 x x 2 x3  x 2  x x 2 x3 
 x 1     ..........  1     ..........
 2 3 4  2  2 3 4 
3 4
x3  x x 2 x3   x x 2 x3 
 1     ......  x 4 1     .......  ......
3  2 3 4   2 3 4 
Collecting terms of the same nature, we get
Differential Calculus: Taylor and Maclaurin’s Theorems 18
Prepared by: Dr. Sunil, NIT Hamirpur (HP)

x3
log1  log1  x   x   ................. .
3!
This is the required expansion.

Q.No.4.: Expand cos3 x using Maclaurin’s series.


1
Sol.: f x   cos3 x  cos 3x  3 cos x   f 0  0
4
1
f  x    3sin 3x  3 sin x   f  0   0
4
3
f  x    cos 3x  cos x   f  0   3
4
3
f  x   9 sin 3x  sin x   f  0   0
4
3
f iv x   27 cos 3x  cos x   f iv 0   21
4
Now Maclaurin’s series is

x2 x3 x 4 iv x 2 n 2n
f x   f 0   xf  0   f  0  f  0   f 0   ..........  f 0 ........
2! 3! 4! 2n!
Substituting these values, we get

x2 x4 3 7
cos3 x  1  3  21  ...............  1  x 2  x 4  ................ .
2! 4! 2 8
This is the required expansion.
Q.No.5.: Use Maclaurin’s theorem or otherwise show that:
 x2 x4 
(a) ecos x  e 1   4  ...................
 2 ! 4 ! 

(b) eax cos bx  1  ax 


1 2
x
  1
 
a  b 2 x 2  a a 2  3b 2 x 3  .............
6
x2 x3
x
(c) a  1  x log a  log a   log a 3  .............
2
2! 3!
Sol.: (a) It is required to prove that
 x2 x4 
ecos x  e 1   4  ...................
 2 ! 4 ! 
Differential Calculus: Taylor and Maclaurin’s Theorems 19
Prepared by: Dr. Sunil, NIT Hamirpur (HP)

f x   ecos x  f 0  e

f  x   ecos x  sin x   f x  sin x   f  0   0


f  x   f  x  sin x   f x cos x  f  0   e
f  x   f  x  sin x   f  x cos x  f  x cos x  f x sin x  f  0  0

f iv x   f  x  sin x   f  x cos x  2f  x cos x  f  x  sin x 

 f  x sin x  f x cos x

 f  x  sin x   f  x cos x  2f  x cos x  2f  x  sin x 

 f  x sin x  f x  cos x  f iv 0   4e
Now Maclaurin’s series is

x2 x3 x 4 iv x 2 n 2n
f x   f 0   xf  0  f 0   f 0 
  f 0  ..........  f 0  ........
2! 3! 4! 2n!
Substituting, we get

x2 x3 x4
e cos x
 e  x.0   
e   
0  4e   ..................
2! 3! 4!
x2 x4
e e 4e ..................
2! 4!
 x2 x4 
 e 1   4.................. .
 2 ! 4 ! 
This is the required expansion.
(b): It is required to prove that

eax cos bx  1  ax 
x

1 2
 1
 
a  b 2 x 2  a a 2  3b 2 x 3  .............
6

f x   eax cos bx  f 0   1

f  x   eax cos bx  eax  b sin bx 

 af x   e ax  b sin bx   f  0  a


f  x   af  x   b aeax sin bx  eax b cos bx   f  0   a 2  b 2

  
f  x   af  x   b a aeax sin bx  be ax cos x  b aeax ciosbx  be ax sin x 
Differential Calculus: Taylor and Maclaurin’s Theorems 20
Prepared by: Dr. Sunil, NIT Hamirpur (HP)

   
 f  0   a a 2  b 2  bab  ab   a a 2  b 2  2 b 2a  a a 2  3b 2  
Now Maclaurin’s series is

x2 x3 x 4 iv x 2 n 2n
f x   f 0   xf  0   f  0  f  0   f 0   ..........  f 0 ........
2! 3! 4! 2n!
Substituting, we get

  x2 !  a a  x3 !  ...........
2 3
eax cos bx  1  ax  a 2  b 2 2
 3b 2

This is the required expansion.

(c): f x   a x
Now Maclaurin’s series is

x2 x3 x 4 iv x 2 n 2n
f x   f 0   xf  0   f  0  f  0   f 0   ..........  f 0 ........
2! 3! 4! 2n!
Now f x   a x , f 0  1

f  x   a x log a , f  0   log a

f  x   a x log a 2 , f  0   log a 2

f  x   a x log a 3 , f  0   log a 3


Substituting the values, we get

x2 3
a x  1  x log a  loga 2  x log a 3  .......... .......... ........
2! 3!
This is the required expansion.
 
Q.No.6.: Obtain the Maclaurin’s expansion of tan   x  and hence find the value of
4 
46  30 to four decimal places.
  
Sol.: Let f x   tan   x  , f 0   tan 1
4  4

  
f  x   sec 2   x  , f  0   sec2 2
4  4

   
f  x   2 sec2   x  tan   x  ,  2
f  0   2 2 .1  4
4  4 
Differential Calculus: Taylor and Maclaurin’s Theorems 21
Prepared by: Dr. Sunil, NIT Hamirpur (HP)

      2
f  x   4 sec 2   x  tan 2   x   2 sec 4   x  , f  0  4 2 .1  2 2    4  16
4  4  4 
Now Maclaurin’s series is

x2 x3 x 4 iv x 2 n 2n
f x   f 0   xf  0   f  0  f  0   f 0   ..........  f 0 ........
2! 3! 4! 2n!
2 3
  4x 16 x
 tan   x   1  2 x    .......... ..
4  2 3

 1   14 
Now tan 46  30  tan 45  1.   tan   
 2  4 120 
2
14   
Hence tan 46  30  1  2.  2.   ..........  1.0537 . Ans.
120  120 

Q.No.7.: Expand log e  x  x 2  1  up to first four terms by Maclaurin’s theorem. Hence


 
calculate the value of loge 2 by putting x = 0.75 in the expansion.

Sol.: Let f x   loge  x  x 2  1  , f 0  log1  0


 


1 x  1
f  (x) 1   , f  0  1
   2 
2
x   x  1  x  1  x2  1
 
x
f  (x)  , f  0  0
x  1
2 3/ 2

1 3x.2 x
f  (x)   , f  0   1
x  1
2 3/ 2

2 x2  1 
5/ 2

3.2x 6x 15x 2 .2x


f iv x     , f iv 0   0

2 x2  1  3/ 2
x  1
2 5/ 2

2 x2  1  7/2

Hence by Maclaurin’s series, we get

x3 9x5
log e  x  x 2  1   x    .................... ....
  3! 5!
To calculate loge 2 , put x = 0.75

log 2  .75 
.753  .755  ................  0.6974 .Ans.
3! 5!
Differential Calculus: Taylor and Maclaurin’s Theorems 22
Prepared by: Dr. Sunil, NIT Hamirpur (HP)

Q.No.8.: Expand sin 1 x is a series of power of x.

Sol.: f x   sin 1 x  f ' x  


1

 1  x2 
1 / 2

1  x2
 1

f ' ' x      1  x 2
 2

3 / 2
 2x   x 1  x 2 
3 / 2


f ' ' ' x   1  x 2 
3 / 2  3
 x   1  x 2
 2
 
5 / 2
 2x   1  x 2 
3 / 2
 3x 2 1  x 2 
5 / 2

x     3 1  x 2     5
 
5 / 2 5 / 2 5 / 2
f iv
( 2 x )  6 x 1  x 2  3x 2  1 x
2
 2x 
 2   2 

 
 3x 1  x 2
5 / 2

 6x 1  x 2 5 / 2

 15x 2 1  x 2  7 / 2

 9 x 1  x  2 5 / 2
  7 / 2
 15x 3 1  x 2

( x )  91  x       
2 5 / 2  5 7 / 2 7 / 2  7 9 / 2
fv  9x    1  x 2 (2x )  45x 2 1  x 2  15x 3    1  x 2  2x 
 2  2
 9 x 1  x  2 5 / 2
 90x 2 1  x 2   105x 4 1  x 2 
7 / 2 9 / 2

Now put x = 0 in all, we get

f(0) = 0, f ' (0)  1 , f ' ' (0)  0 , f ' ' ' (0)  1 , f iv (0)  0 , f v (0)  9
Now by Maclourins theorem

x2 x3 x4 5
f x   f 0   xf ' 0  f ' ' 0  f ' ' ' 0  f iv
0  x f v
0  ....................
2! 3! 4! 5!
Substituting the values of f(0) = 0, f ' (0)  1, f ' ' (0)  0 , f ' ' ' (0)  1 ,

f iv (0)  0 , f v (0)  9 , we get

1 x3 9
sin x  0  x  0   0  x 5   .......... .........
3! 5!
1 3 1.3 5
 sin 1 x  x  x 0 x  ................... . Ans.
2.3 2.4.5
Other Method:

f x   sin 1 x  f ' x  
1

 1  x2 
1 / 2

1  x2
By binomial expansion
Differential Calculus: Taylor and Maclaurin’s Theorems 23
Prepared by: Dr. Sunil, NIT Hamirpur (HP)

 1  1   1  1  1 
     1      1   2 
 1 2  2  4  2  2  2
f ' (x)  1     x 2   x   x 6  ...........
 2  2! 3!

x2 3 4 5 6
 1  x  x  ...................
2 8 16
2x 12 3 5  6 5 3 15
f ' ' (x)   x  x  ................  x  x 3  x 5  ...................
2 8 16 2 8
9 75
f ' ' ' x   1  x 2  x 4  ........................
2 8
75 3
f iv ( x )  9x  x  ...................
2
225 2
fv  9 x  ....................
2
Now put x = 0 in all, we get

f(0) = 0, f ' (0)  1 , f ' ' (0)  0 , f ' ' ' (0)  1 , f iv (0)  0 , f v (0)  9

x2 x3 x4 5
f x   f 0   xf ' 0  f ' ' 0   f ' ' ' 0   f iv
0  x f v
0  ....................
2! 3! 4! 5!
Substituting the values of f(0) = 0, f ' (0)  1, f ' ' (0)  0 , f ' ' ' (0)  1 ,

f iv (0)  0 , f v (0)  9 , we get

x3 9
sin 1 x  0  x  0   0  x 5   ...................
3! 5!
1 3 1.3 5
 sin 1 x  x  x 0 x  ................... . Ans.
2.3 2.4.5

Q.No.9.: Prove that sin 1 x  2


 x2 
x4
3
 .....................


Sol.: Let f x   sin 1 x 2

Now Maclaurin’s series is

x2 x3 x 4 iv x 2 n 2n
f x   f 0   xf  0   f 0  f 0  
  f 0   ..........  f 0 ........
2! 3! 4! 2n!


f x   sin 1 x , 2
f 0  0
Differential Calculus: Taylor and Maclaurin’s Theorems 24
Prepared by: Dr. Sunil, NIT Hamirpur (HP)


f  x   sin 1 x .  1
2
, f  0   0
1 x
  1
f  x   2 sin 1 x    1  x 2
 2
 3 / 2
 2x   1  x 2 
1 / 2

.1  x2 
1 / 2 

 

 2  x sin 1 x 1  x 2

 
3 / 2

 1  x2 1 
 f  0   2


 x 1  x 
2 1 / 2
 
1  x2
3 / 2   3
 x sin 1 x  
 1 x
2
 
5 / 2 

f  x   2  2  

  2x   sin 1 x 1  x 2
3 / 2
  1 1  x 2   
2
 2x 

 2x 1  x 2

 
2
 3x 2 sin 1 x 1  x 2  
5 / 2

 sin 1 x 1  x 2 
3 / 2

 1  x2 
2
2x 

f  0   0
Substituting the values, we get

x2 x3 x4
f x   0  x.0  .2  .0  .8  .............................
2! 3! 4!

sin x 
1 2
x 
x4
3
2
 .........................

This is the required expansion.

sin 1 x 1
Q.No.10.: Prove that  x  x 3  .........
1 x 3 2

sin 1 x
Sol.: Let f x  
1  x2
Now Maclaurin’s series is

x2 x3 x 4 iv x 2 n 2n
f x   f 0   xf  0   f  0  f  0   f 0   ..........  f 0 ........
2! 3! 4! 2n!
sin 1 x
f x   , f 0  0
1  x2

 1
f  x   sin 1 x    1  x 2
 2
  3 / 2
 2x   1  x 2 
1 / 2
1  x 2 1 / 2


 x  1  x 2 3 / 2
sin 1 x  1  x 2   1 / 2
, f  0   1
Differential Calculus: Taylor and Maclaurin’s Theorems 25
Prepared by: Dr. Sunil, NIT Hamirpur (HP)

f  x   3x 2 sin 1 x 1  x 2    sin x 1  x   3x1  x 


5 / 2 1 2 3 / 2 2 2
, f  0  0


f  x   3x 2 1  x 2  1  x   3x sin x  52 1  x 
1 / 2 2 5 / 2 2 1 2 7 / 2
 2x 

 32 x sin  x 1  x 2 5 / 2



 1  x2  1  x 
1 / 2 2 3 / 2  3

 sin 1 x   1  x 2
 2

6 / 2
 2x 


 3x  1 1  x 2 
3
 2x   31  x 2 
2
, f  0   4
Substituting the values, we get
sin 1 x x2 x3
f x    0  x.1  .0  .4  ....................
1  x2 2! 3!

2
 x  x 3  ................
3
This is the required expansion.

cos x 1 1 1
Q.No.11.: Prove that  1  x  x 2  x 3  ............... .
1 x 2 8 16

x2 x4
Sol.: Since we know that cos x  1    .......... ........
2 24
1 3 5
and 1  x 1 / 2  1  x  x 2  x 3  .................... [by Binomial theorem]
2 8 16
 x2 x4  x 3 5 
 cos x 1  x  1 / 2
 1    ...........1   x 2  x 3  ...............
 2 24 
  2 8 16 

x 3 2 5 3 x 2 x3 3 4 5 5
 1  x  x    x  x  .......... .
2 8 16 2 4 16 32
x 3 1 1 5 
 1  x 2     x 3     .......... .
2 8 2  4 16 

x x 2 x3
 1    .......... ......... .
2 8 16
Which is the required expansion.

x3 x 4
x
Q.No.12.: Prove that e cos x  1  x    ................... .
3 6
Differential Calculus: Taylor and Maclaurin’s Theorems 26
Prepared by: Dr. Sunil, NIT Hamirpur (HP)

Sol.: Since we know that

x 2 x3 x4
ex  1  x     .................
2 ! 3! 4 !
x2 x4
and cos x  1    .......... .......
2! 4!
 x 2 x3 x 4  x 2 x 4 
x 
 e cos x  1  x     .......1    ............ 
 2 6 24  2 ! 4! 
  

x2 x4 x3 x5 x 2 x 4 x6 x3 x5
 1  x      
2 24 2 24 2 4 48 6 12
x7 x4 x6 x8
     ......................
24.6 24 48 242

x2 x4 x3 x 2 x 4 x3 x 4
 1  x      ...............
2 24 2 2 4 6 24
1 1
 1  x  .x 3  x 4  ..............
3 6
x3 x 4
 1 x    .......... .... ,
3 6
which is the required expansion.


Q.No13.: Prove that sin e x  1  x   x2 5 4
 x  ...................... .
2 24
x 2 x3 x 4
Sol.: Since we know that e x  1  x     .................
2 6 24

  
 sin e x  1  sin 1  x 

x 2 x3 x 4
2
 
6 24

 ......................  1

 
 x 2 x3 x 4 
 sin  x     ......................
 2 6 24 
 
  x 2 x 3 x 4 
 sin x1     .............
  2 6 24 

Since we know that
Differential Calculus: Taylor and Maclaurin’s Theorems 27
Prepared by: Dr. Sunil, NIT Hamirpur (HP)

z3 z5
sin z  z    .......... .......
6 120

 x

 x x 2 x3  x3  3
 sin e  1  x 1    
 2 6 24  6  2
3 3 
1  x  x 2  x 3  ...............  .....
6 24 
 
Applying the Binomial theorem, we obtain

x 2 x3 x 4 x3  3 3 3 3 
x    1  x  x 2  x  .......... ........ 
2 6 24 6  2 6 24 

x 2 x3 x 4 x3 x 4 x5 x 6
x        ....................
2 6 24 6 4 12 48
x2 5 4
x  x  ........................ ,
2 24
which is the required expansion.

x 3 11 4
Q.No.14.: Prove that e x cos x  1  x  x 2   x  ................ .
3 24
Sol.: Let x cos x  y

y y 2 y3 y 4
e  1 y     .................... (i)
2 6 24
Put y  x cos x in (i), we get

x2 x3 x4
e x cos x  1  x cos x  cos 2 x  cos3 x  cos 4 x  ................... (ii)
2 6 24
Using the expansion of cos x

x2 x4
cos x  1    ....................
2 24
2 3
 x7 x4  x2  x2 x4  x 3  x 2 x 4 
e x cos x 
 1 x 1 
 ..........   1  
 .......  1   ..........
 2 24  2  2 24  6  2 24 
    
4
x 4  4 x 2 4x 4 
 1   .............  ...................
24  2 24 

Using Binomial expansion, we get

x3 x5 x 2  2 x 2 2 x 4 
 1 x    ........  1   ........
2 24 2  2 24 

Differential Calculus: Taylor and Maclaurin’s Theorems 28
Prepared by: Dr. Sunil, NIT Hamirpur (HP)

x 3  3x 2 3x 4  x 4  4x 2 4x 4 
 1   ........  1    ........... 
6  2 24  24 
  2 24 

x 3 x 5 x 2 x 4 x 6 x 3 x 5 x 7 x 4 4x 6 4x 8
 1 x             .....
2 24 2 2 24 6 4 48 24 48 24 2

x3 x 2 x 4 x3 x 4
 1 x       ...........
2 2 2 6 24
x 2 x 3 11x 4
 1 x     .............. ,
2 3 24
which is the required expansion.

x x2 x
Q.No.15.: Prove that x  1   .................. .
e 1 2 12
Sol.: Since we know that

x 2 x3
ex  1  x    .............
2 6
x x x
  
ex  1  2 3  x 2 x3
1  x  x  x  ...............  1 x   ........
 2 6  2 6
 
x 1
 
 x x 2   x x2 
x 1    ........... 1    ......... 
 2 6   2 6 
   
1
 x x2 
 1    ........ (i)
 2 6 
 
Also we know that

1  z 1  1  z  z 2  .......... ..............


Apply this expansion in (i), we get
2
x  x x2   x x2 
 1     .......    ......... .....................
ex  1 2 6  2 6 
   

x x 2 x 2 x 4 x3 x  1 1
 1      ....................  1   x 2    .........
2 6 4 36 6 2  6 4
Differential Calculus: Taylor and Maclaurin’s Theorems 29
Prepared by: Dr. Sunil, NIT Hamirpur (HP)

x x2
 1   .......... ...... ,
2 12
which is the required expansion.

Q.No.16.: Prove that log 1  e x  log 2    x x2 x4


 
2 8 192
 ............ .

Sol.: Let f x   log 1  e x    f 0  log 2


1 1
f  x   x
.e x  f  0  
1 e 2

f  x  
1  e e  e
x x 2x

e x
 e2 x  e2 x 
1  e  x 2
1  e 
x 2

ex 1
  f  0  
1  e  x 2 4

f  x  
1  e  e x 2 x
 e2x  2 1  ex    1  e 2x

 2e2 x e x  2e 2 x  2e3x
1  e  x 4
1  e x 4

e x  e3x  2e 2 x  2e 2 x  2e3x e x  e3 x
   f  0  0
1  e  1  e 
x 4 x 4

x   1  e  e  3e  e  e 41  e  e
x 4 x 3x x 3x x 3 x
1
f iv
 f iv 0   
1  e  x 8 8

Putting the values, we get

x2 x3 x 4 iv
f x   f 0   xf  0  f  0   f  0  f 0  .................
2! 3! 4!
x x2 x4
 log 2     .................
2 8 192
which is the required expansion.

xe x x x2 x4
Q.No.17.: Prove that log     ................. .
ex  1 2 24 2880

xe x xe x
Sol.: f x   log x
 a  Lim  ea
e 1 x 0 e x 1
Applying L – Hospital’s rule
Differential Calculus: Taylor and Maclaurin’s Theorems 30
Prepared by: Dr. Sunil, NIT Hamirpur (HP)

e x  xe x
Lim x
 1  ea  1
x 0 e
a  0  f 0  0

f  x  
  
e x  1 e x  1 xe x  e x  xe.e x e x  x  1

xe x
.
x
e 1
2
 
x ex  1  
Applying L – Hospital’s rule

ex  x  1 ex  1 ex 1
 Lim  Lim 
Lim
x 0 
x e 1 x
 x 0 x
xe  e  1 x x 0 x
xe  2e x 2

1
f  0  
2

f  x  
    
x e x  1 e x  1  e x  x  1 e x  xe x  1    e2x  x 2ex  2ex 1
x xe  1 x 2e 2 x  x 2  2 x 2e x
2 x 2

Applying L – Hospital’s rule

 e2 x  x 2e x  2e x 1
Lim
x 0 x 2e 2 x  x 2  2 x 2e x
Putting x = 0, we get
1
f  0   
12
Applying Maclaurin’s theorem and putting the values, we get

x2 x3 x 4 iv
f x   f 0   xf  0  f 0   f 0 
  f 0  .................
2! 3! 4!
x x2 x4
 0  0  .......... .
2 24 2880
x x2 x4
    ................. ,
2 24 2880
which is the required expansion.

 sin x  x2 x4 x6
Q.No.18.: Prove that log     ......... .
 x  6 180 2835
Differential Calculus: Taylor and Maclaurin’s Theorems 31
Prepared by: Dr. Sunil, NIT Hamirpur (HP)

 3 5 7 
 x  x  x  x  ........... 
 sin x   3! 5! 7 ! 
Sol.: log   log 
 x  x
 
 
  x2 x4 x6 
 log1      ...........  
  3! 5 ! 7 ! 
  
2 3
 x 2 x 4 x6  1  x2 x 4  1  x2 x4 
        .........     .........
 3! 5 ! 7 !  2  3! 5 !  3  3! 5! 
     

x2 x4 x6 1  x 4 x8 x 6 
     
6 120 5040 2  36 14400 360 

x2 x4 x6 x4 x6
    
6 120 5040 72 720
x2  1 1   1 1 
  x4    x6  
6  120 72   5040 720 

x2 x4 x6
   ............
6 180 2835
which is the required expansion.

 sinh x  x2 x4 x6
Q.No.19.: Prove that log     ......... .
 x  6 180 2835

 3 5 6 
 x  x  x  x  .......... . 
 3! 5 ! 7 ! 
 sinh x   
Sol.: log   log
 x  x

  x3 x5 x 6 
 log1      ...........  
  3! 5 ! 7 ! 
  
2
 x 2 x4 x6  1  x 2 x 4 x6 
       .........
 3! 5! 7 !  2  3! 5! 7 ! 
   

x2 x4 x6 1  x4 x8 x 6 
      
6 120 5040 2  36 14400 360 
Differential Calculus: Taylor and Maclaurin’s Theorems 32
Prepared by: Dr. Sunil, NIT Hamirpur (HP)

x2 x4 x6
    ..........
6 180 2835
which is the required expansion.

Q.No.20.: Prove that sinh 1 x  log  x  x 2  1


 

Sol.: Let f x   log  x  x 2  1  f 0  0


 
1
f  x   1  .2 x  f  0  1
2
2 x 1

f  x   
2

1 2
x 1
3 / 2

.2 x   x x 2  13 / 2
  f  0  0

 3

f  x    x   x 2  1
 2
5 / 2

.2 x  x 2  1 
3 / 2

 1  f  0   1

Applying Maclaurin’s theorem and putting the values, we get

x2 x3 x 4 iv
f x   f 0   xf  0  f  0   f  0  f 0  .................
2! 3! 4!
x3 3x 5
 0x0 0  .......... ..
6 40
x 3 3x 5
 x   .......... ...
6 40
which is the required expansion.

x3 x5
Q.No.21.: Prove that tanh 1 x  x    ..................... .
3 5
1   1  x  1
Sol.: tanh 1 x  log   log1  x   log1  x 
2   1  x  2
Since we know that

x 2 x3 x 4 x5
log1  x   x      ............
2 3 4 5
 x 2 x3 x4 x5 
and log1  x     x      ............
 2 3 4 5 
Differential Calculus: Taylor and Maclaurin’s Theorems 33
Prepared by: Dr. Sunil, NIT Hamirpur (HP)

1  x 2 x3 x 4 x5   x 2 x3 x 4 x5 
 tanh 1 x   x      .........    x      ......... 
2  2 3 5 5   2 3 5 5 
  

x3 x5
x   .....................
3 5
which is the required expansion.

Q.No.22.: Expand e x sec x .


1
1  x 2 x3 x 4 
Sol: sec x   1     ...............
cos x  2! 3! 4! 

 x 2 x3 x 4 
Consider y as     ...............  1  y 1
 2! 3! 4! 
 
By binomial theorem

 1  y  y 2  y 3  y 4  .......... .......... ...


2 3
 x 2 x3 x 4   x 2 x3 x 4   x2 
 1     .....      ......    .........
 2! 3! 4!   2! 3! 4!   2! 
     
2
 x 2 x3 x 4   x2 
 1     ......    ........
 2! 3! 4!   2! 
   

x 2 x3 4 1 1 x 2 x3 5 4
 1   x     1   x
2! 3!  4! 4  2! 3! 6

 x 2 x 3 5 4  x x 2 x 3 x 4 
e x sec x  1    x 1      ...............
 2! 3! 6  1 2! 3! 4! 
 
2
 1  x  x 2  x 3  ..................
3
2
 e x scx  1  x  x 2  x 3  .................. . Hence proves the result.
3
Another method:

Let f x   e x sec x

f ' x   e x sec x  e x sec x tan x  e x sec x 1  tan x 

 f ' x   f ( x )tan x  1
Differential Calculus: Taylor and Maclaurin’s Theorems 34
Prepared by: Dr. Sunil, NIT Hamirpur (HP)

f ' ' x   f ' x 1  tan x   f ( x ) sec 2 x

f ' ' ' x   f ' x sec 2 x  f ( x ) 2 sec 2 x tan x  f ' ' x 1  tan x   f ' ( x ) sec 2 x

 2f ( x ) sec 2 x tan x  2f ' x sec 2 x  f ' ' x 1  tan x  .


Now put x = 0 in all
f(0) = 1, f ' (0)  1 , f ' ' (0)  2 , f ' ' ' (0)  4 ,

x2 x3
f x   f 0  xf ' 0   f ' ' 0   f ' ' ' 0   ....................
2! 3!
Substituting the values of f(0) = 1, f ' (0)  1, f ' ' (0)  2 , f ' ' ' (0)  4 , we get
2 4
e x sec x  1  x  x 2   x 3  ..................
2! 3!
2
e x scx  1  x  x 2  x 3  .................. . Hence proves the result.
3
x ex  1 1 x2 1 x4
Q.No.23.: Prove that  1   ..................
2 ex  1 6 2 ! 30 4!
 2   2 
1  x  x  .............  1  2  x  x  .............
x  2! 
 x
 2! 

Sol: f ( x ) 
2 2 3  2  2 
1  x  x  x  .........  1 x x
x 1    .........
 2! 3!   2! 3! 
   
1
1 x 2 x3  x 2 x 3 
 2  x    .............1    .............
2  2! 3! 
 2! 3! 

  x x 2  
2
 x x2 x3   x x2

 1    
 ...... 1    
 .............     ....... 

 2 4 12    2! 3!   2! 3!  
      

 x x 2 x3   x x 2 x3  x x 2 x 3 

 1    
 ......  1     ......    ......
 2 4 12   2 4 12  2 6 24 
    
2
 x x 2 x3  x x 2 x 3 
1     ......1     ......  .......
 2 4 12  2 6 24 
  

x x 2 x3 x 4 x x 2 x3 x 4 x 2 x3 x 4 x3 x 4
 1            
2 4 12 48 2 4 8 12 6 12 48 8 24
Differential Calculus: Taylor and Maclaurin’s Theorems 35
Prepared by: Dr. Sunil, NIT Hamirpur (HP)

 x x 2 x3  x 2 x 3 x  x 2 x3 
 ...........1       2.    ......
 2 4 12   4 12 2  6 24 
  

x 3 x 2 3x 4 x 4 x3 x 4  x x4 
 1    .....    ...............  1    ............ 
4 6 24 4 8 16  2 4 
 
2
 x 4 x3  x 2  x3  x3 x 4
   ........  2.   .........    .........
 36 24  36  24  6 24
  

x ex  1 1 x2 1 x4
 1    .................. . Hence proves the result.
2 ex  1 6 2 ! 30 4!

x3 5 4
Q.No.24.: Prove that 1  x x  1  x 2   x  .......... . .
2 6

Sol.: Let f x   1  x x
log f x   x log1  x 

x 2 x3 x 4
Since log1  x   x     ......................
2 3 4
x3 x 4 x5
 log f x   x log1  x   x  2
  .................
2 3 4
f  x  3x 2 4 x 3 5 x 4
 2x     .......... ......
f x  2 3 4

f  x   x 1  x x 1

f  x   1  x x 1  x x  11  x x  2

f  x   x  11  x x  2  x  11  x x  2  x 1  x x  2  x x  1x  2 1  x x 3

f iv x   x x  1x  2 1  x x  2  .......... ....


Now Maclaurin’s series is

x2 x3 x 4 iv x 2 n 2n
f x   f 0   xf  0   f  0  f  0   f 0   ..........  f 0 ........
2! 3! 4! 2n!
x3 5 4
 1  x2   x  ...................
2 6
Differential Calculus: Taylor and Maclaurin’s Theorems 36
Prepared by: Dr. Sunil, NIT Hamirpur (HP)

x3 5 4
 1  x x  1  x 2   x  .......... ......... ,
2 6
which is the required expansion.

3 2 7 4 1  32 n  3 2 n
Q.No.26.: Prove that cosh 3 x  1  x  x  ................   x .
2 8 4 n  0 2n!

Sol.: We know that cosh 3x  4 cosh3 x  3 cosh x


1
 cosh 3 x  cosh 3x  3 cosh x 
4
1
Let f x   cosh 3x  3 cosh x   f 0   1
4
Differentiating f(x), w. r. t. x
1
f  x   3 sinh 3x  3 sinh x   f  0   0
4
Again differentiating w. r. t. x, we get
1
f  x   9 cosh 3x  3 cosh x   f  0   3
4
Again differentiating w. r. t. x, we get
1
f  x   27 sinh 3x  3 sinh x   f  0   0
4
Again differentiating w. r. t. x, we get

f iv
x   1 81cosh 3x  3 cosh x  f iv
0   21
4

f 2 n x  
4

1 2n
3 cosh 3x  3 cosh x   f 2n 0 
4

1 2n
3 3
Now Maclaurin’s series is

x2 x3 x 4 iv x 2 n 2n
f x   f 0   xf  0   f 0  f 0  
  f 0   ..........  f 0 ........
2! 3! 4! 2n!
x2 x3 x4
 1  x.0  .3  .0  .21  ............. 

1 32n  3 x 2n 
2! 3! 4! 4 2n!
3x 2 7 x 4
 1  ........................
2 8
Differential Calculus: Taylor and Maclaurin’s Theorems 37
Prepared by: Dr. Sunil, NIT Hamirpur (HP)


1 32 n  3 2 n
n  0 x . Ans.
4 2n!

 1  x2  1 1 
   x  x  x  .................. .
3 5
Q.No.27.: Prove that tan 1
 x  2 3 5 
   

 1  x2 1
Sol.: Let y  tan 1 , Put x  tan 
 x 
 
 sec   1  
 f x   y  tan 1 
 tan   2
dy 1 dx
  , x  tan  ,   sec2 
d 2 d
dy 1 1
 
 
dx 2 1  tan  2 1  x 2
2
  
1
 f  0 

2 1  x2 
x
f  0  

2 1  x2 2

6x 2  2 3x 2  1
f  0   

2 1  x2  1  x 
3 2 2

12 x  12 x 3  1
f iv 0  
1  x  2 4

f v 0  

12 1  10 x 2  26 x 4 
1  x  2 5

Now Maclaurin’s series is

x2 x3 x 4 iv x5 v
f x   f 0  xf  0   f  0  f  0   f 0   f 0 
2! 3! 4! 5!

1  x3 x5 
 x   .................
2  3 5 

This completes the proof.
Differential Calculus: Taylor and Maclaurin’s Theorems 38
Prepared by: Dr. Sunil, NIT Hamirpur (HP)

 x 1  1
x

 x3 x5 
.
Q.No.28.: Prove that cos    2 x    ...........
 x  x 1   3 5 
   
 x  x 1 
Sol.: Let f x   y  cos 1 , x  tan  .
 x  x 1 
 

 tan   cot   1  sin 2   cos 2  


y  cos 1   cos     2    2 tan 1 x
 tan   cot    1 
 
2
 f  x  
1  x2
  2x  4x
f  x   2  
 2

 1  x2  1  x2
    2

f  x  
 26x 2  2
1  x 2 3

x    212 x  12 x  36 x    48x  48x


3 3 3
iv  12 x
f
1  x  2 4
1  x 
2 4

x    481  10 x  26x 
2 4
fv
1  x  2 5

Now Maclaurin’s series is

x2 x3 x 4 iv x5 v
f x   f 0   xf  0  f  0   f  0   f 0  f 0   ................
2! 3! 4! 5!
 x3 x5 
   2 x    ............... .
 3 5 
 
This completes the proof.
2
Q.No.29.: Prove that sec2  1  x 2  x 4  .................
3
Sol.: Let f x   sec 2 x

f  x   2 sec 2 x tan x


f  x   4 sec 2 x tan 2 x  2 sec 4 x  2 sec 2 x 2 tan 2 x  sec 2 x 

f  x   4 sec x. sec x tan x  2 tan 2 x  sec 2 x  2 sec 2 x 4 tan x sec 2 x  2 sec 2 x tan x 
Differential Calculus: Taylor and Maclaurin’s Theorems 39
Prepared by: Dr. Sunil, NIT Hamirpur (HP)

Now Maclaurin’s series is

x2 x3 x 4 iv x5 v
f x   f 0  xf  0   f  0  f  0   f 0   f 0 
2! 3! 4! 5!
2
 1  x 2  x 4  ..................
3
This completes the proof.

x4 x 3
Q.No.30.: Prove that e  x   .............
6

Sol.: Let f x   e x sin 2 x


f  x   e x sin 2 x  sin 2 x 
f  x   e x sin 2 x  sin 2 x   e x sin 2 x  2 cos 2 x   e x sin 2 x  2 sin 2 x  2 cos 2 x 

f  x   e x sin 2 x  2 sin 2 x  2 cos 2 x   e x sin 2 x  4 cos 2 x  4 sin 2 x 

 e x sin 2 x  3 sin 2 x  4 sin 2 x  6 cos 2 x   e x sin 2 x  12 sin 2 x  4 cos 2 x 

f iv x   e x sin 2 x  12 sin 2 x  4 cos 2 x  4 cos 2 x 


Now Maclaurin’s series is

x2 x3 x 4 iv x5 v
f x   f 0  xf  0   f  0  f  0   f 0   f 0 
2! 3! 4! 5!
x4
 x 2  x3  .................
6
This completes the proof.

y 2 y3 y 4
Q.No.32.: If x  y     ................ ,
2 3 4
x 2 x3 x 4
Prove that y  x     ..................
2 ! 3! 4 !
y 2 y3 y 4
Sol.: Given x  y     ..........  log1  y   1  y  e x  y  e x  1
2 3 4

Let f x   y  e x  1

Then f  x   e x , f  x   e x , f  x   e x , f iv x   e x , f v x   e x .
Now put x = 0, we get
Differential Calculus: Taylor and Maclaurin’s Theorems 40
Prepared by: Dr. Sunil, NIT Hamirpur (HP)

f  0   0, f  0   1, f  0   1, f iv 0   1, f v 0   1
Now by Maclaurin’s theorem

x2 x3 x 4 iv x5 v
f x   f 0  xf  0   f  0  f  0   f 0   f 0 
2! 3! 4! 5!
x 2 x3 x 4
x   ...................
2 ! 3! 4 !
This completes the proof.
x 26 2
Q.No.33.: If x 3  y 3  xy  1  0 , Prove that y  1   x  ............... .
2 81
Sol.: Here x 3  y 3  xy  1  0 (i)

Putting x = 0, we get y 3  1 y 0  1 (ii)


Differentiating (i) w. r. t. x, we get

3x 2  3y 2 y  xy1  y  0 (iii)

Put x = 0 and y 0  1 in (iii), we get


1
3y0  1  y0   (iv)
3
Differentiating (iii) w. r. t. x, we get

6 x  6 yy2  3 y 2 y  y  xy  y  0 (v)

Putting x = 0, y 0  1, y0   1 in (v), we get


3
1 1 1
6.1.  3y0   0.y   0
9 3 3
y0  0 (vi)
Differentiating (v) w. r. t. x, we get

6  12 yyy  6y3  6 yyy  3y 2 y  y  xy  y  y  0 (vii)

Putting x = 0, y 0  1, y0   1 , y0  0 in (vii), we get


3

y0   52 (viii)
27
Now by Maclaurin’s theorem
Differential Calculus: Taylor and Maclaurin’s Theorems 41
Prepared by: Dr. Sunil, NIT Hamirpur (HP)

x2
   
3 4 5
y  y 0  x y 0  y  0  x y  0  x yiv  x y v 0  ...........
2! 3! 4! 5!
x x2 x 3 52
 1  .0  .  ........
3 2! 1.2..3 27
x 26x 3
 1  ............... .
3 81
This completes the proof.
Q.No.34.: Expand log sin x  h  in powers of h by Taylor’s theorem.
Sol.: Let f x  h   log sin x  h 
1
f  x   . cos x  cot x
sin x

f  x    cos ec 2 x

f x   2 cos ecx cos ecx cot x  2 cos ec 2 x cot x


Now by Maclaurin’s theorem

x2 x3 x 4 iv x5
f x  h   f 0   xf  0   f  0   f  0   f 0   f v 0 ..................
2! 3! 4! 5!

log sin x  h   log sin x  h cot x 


h2
2!
 h3

 cos ec 2 x  2 cos ec2 x cot x  .............
3!
h 2 cos ec2 x 2h 3
 log sin x  h cot x   cos ec2 x cot x  .............. .
2! 3!
This completes the proof.
Q.No.35.: Prove that

h h2 x h 3 1  2x 2
sin 1x  h   sin  x     ............ .
1  x2 
2! 1  x2 
3/ 2
 
3! 1  x2 5 / 2

Sol.: Let y  sin 1 x  h 


1
y 
1  x  h 2

xh
y 
1  x  h  
2 3/ 2
Differential Calculus: Taylor and Maclaurin’s Theorems 42
Prepared by: Dr. Sunil, NIT Hamirpur (HP)

1  x  h  
2 3/ 2 3
 1/ 2
 x  h . 1  x  h 2  2x  h 
2

y 
1  x  h   2 3/ 2


1  x  h    3x  h  .1  x  h  
2 3/ 2 2 2 1/ 2

1  x  h   23


1  x  h   3x  h  .
2 2

1  x  h   2 5/ 2

Put x = 0 in all, we get

1 x 2x 2  1
y 0  sin 1 h, y1 0  , y0  , y0 
1  x2 1  x 
2 3/ 2
1  x 
2 5/ 2

Now by Maclaurin’s theorem

x2 3
y  y 0  x y 0  y 0  x y  0  ...........
2! 3!
h h2 x h3 2x 2
 sin 1 h    .
1  x2 
2 ! 1  x2 
3/ 2
 
3! 1  x 2 5 / 2

This completes the proof.

Q.No.36.: (i) Expand 4x 2  5x  3 in power of x  1 ,

(ii) Expand x 3  2x 2  5x  7 in power of x  1 .

Sol.: f x   4 x 2  5x  3 , f  x   8x  5 , f  x   8

f x   f 1  x  1
Using Taylor’s theorem, we get

f 1  x  1  f 1  x  1f 1 


x  12 f  1  x  13 f  1  ........
2! 3!
f 1  412  51  3  12

f  1  8  5  13 , f 1  8 ,

f 1  x  1  12  13x  1 
 x  12 .8
 12  13x  1  x  12 .4 .
2
which is the required expansion.
Differential Calculus: Taylor and Maclaurin’s Theorems 43
Prepared by: Dr. Sunil, NIT Hamirpur (HP)

(ii): f x   x 3  2 x 2  5x  7

f  x   3x 2  4 x  5 , f  x   6x  5 , f  x   6
Using Taylor’s theorem, we get

f 1  x  1  f 1  x  1f 1 


x  12 f  1  x  13 f  1  ........
2! 3!
f 1  13  2  5  7  3

f  1  3  4  5  4 , f 1  2 , f  1  6

f 1  x  1  3  x  1.4 
x  12 .2  x  13 .6
2! 3!
 3  4x  1  x  12  x  33 .
which is the required expansion.

Q.No.37.: Arrange 7  x  2   3x  2 3  x  2 4  x  2 5 in power of x using


Taylor’s theorem.

Sol.: f x   7  x  2   3x  2 3  x  2 4  x  2 5 , f 0   7  2  3.8  16  32  17

f  x   1  3.3x  2 2  4x  2 3  5x  2 4 ,

 
 1  9 x 2  4 x  4  4x  2 3  5x  2 4 , f  0   1  36  32  80  11

 
f  x   18 x  36  12 x 2  4 x  4  20x  2 3 , f  0  36  48  160  76

f  x   18  24 x  48  60x  2 2 , f  0   18  48  249  174

f iv x   24  120 x  240 , f iv 0   24  240  216

f v x   120 f v 0   120
Now by Maclaurin’s theorem

x2 x3 x 4 iv x5 v
f x   f 0  xf  0  f 0   f 0  
  f 0  f 0   .................
2! 3! 4! 5!
Putting these values, we get

x x2 x3 x4 x5
 17  .  11  .  76  .  174  .  216  .  120
1! 2 6 24 120
Differential Calculus: Taylor and Maclaurin’s Theorems 44
Prepared by: Dr. Sunil, NIT Hamirpur (HP)

 17  11x  38x 2  29x 3  9x 4  x 5


which is the required expansion.


Q.No.38.: Prove that log 1  x  x 2   x   x2 2 3 1 4 1 5 1 6
 x  x  x  x .
2 3 4 5 3

Sol.: f x   log 1  x  x 2 
2x  1
f  x   , f  0   1
1  x  x2

f  x  
1  x  x 2  2x  1
2 2
, f  0   1
1  x  x  2 2

10 x 4  2 x 2  4  28x 3  12 x  8x 5
f  x   , f  0   4
1  x  x  2 4

Similarly f iv 0   6 , f v 0   24 , f vi 0   240


Now by Maclaurin’s theorem

  
f log 1  x  x 2  f 0  xf  0 
x2
2!
x3
f  0  f  0  
3!
x 4 iv
4!
f 0  
x5 v
5!
f 0   .................

x2 x3 x4 x5 x6
 0  x  .1  .4  .6  .  24  .  240
2! 3! 24 120 6.120
x2 2 3 1 4 1 5 1 6
 x   x  x  x  x .
2 3 4 5 3
which is the required expansion.
Q.No.39.: Use Taylor’s theorem to prove that
sin z sin 2z sin 3z
tanh 1 x  h   tan 1 x  h sin z .  h sin z 2  h sin z 3  ....... ,
1 2 3

where z  cot 1 x .

Sol.: Given f x  h   tan 1 x  h   f x   tan 1 x

Now f  x  
1 x
1
2

1  cot z
1
2
 sin 2 z  f  x   cot z  z  cot x 
1

 1.2x 2x  2 cot z
f  x      2 sin 3 z cos z and so on
1  x 
2 2
1  x  1  cot z 
2 2 2 2

Putting these value in the expansion of Taylor’s theorem


Differential Calculus: Taylor and Maclaurin’s Theorems 45
Prepared by: Dr. Sunil, NIT Hamirpur (HP)

h2 h3
f x  h   f x   hf  x   f  x   f  x   .......... ...
2! 3!

 tan 1 x  h sin 2 z 
h2
2!
 
 2 sin 3 z. cos z  ...............

sin z sin 2z
 tan 1 x  h sin z .  h sin z 2 .  .............. .
1 2
This completes the proof.
 
Q.No.40.: Expand cos x in powers of  x   .
 2
Sol.: f x   cos x
2 3
   
x   x  
          
f    x     f     x  f     
2
f     
2
f     .............
2  2  2  2 2 2! 2 3!  2

f  x    sin x , f  x    cos x , f  x   sin x

   


f    0 , f     1 , f     0 , f     1
2 2 2 2
3 5
   
x   x  
     
 f    x      x    
2

2
 ....................... .
2  2   2 3! 5!
which is the required expansion.
 
Q.No.41.: Obtain the expansion of tan 1 x in the powers of  x   .
 4

   
Sol.: f x   tan 1 x  f    x  
4  4 
Using Taylor’s theorem
2 3
   
x   x  
        4  4 
f    x     f     x  f      f      f     .............
4  4  4  4 2 2!  4 3! 4

f x   tan 1 x
Differential Calculus: Taylor and Maclaurin’s Theorems 46
Prepared by: Dr. Sunil, NIT Hamirpur (HP)

1  1
f  x   2
 f   
1 x 4 2
1
16


 2x 
f  x    f     2
1  x  2 2    2  2
4
1  
 16 
 
 32 
2  1
f  x  
 
2 3x 2  1 
 f     
 16 

1  x  2 3 4
    
1 
2

3

 16 
 
Substitute the values, we get

  2 x    3  1


2
  
x    x  
4  16 
    1     4 4 
 f x   f    x    tan      2
 3
  .......
4  4  4    2
 41     2 
2
1  3 1  
 16   16   16 
     
x 1  ax 
Q.No.42.: Find the value of a and b such that the expansion of loge 1  x   in
1  bx

ascending powers of x may begins with terms containing x 4 and show that
1 4
this is  x .
36
x 2 x3 x 4 x5 x 6
Sol.: log e 1  x   x       .....................
2 3 4 5 6

1  bx 1  1  bx  bx 2  bx 3  bx 4  bx 5  bx 6  .......... ......


 
x 1  ax 1  bx 1  x  ax 2 1  bx  bx 2  bx 3  .......... .......... 
 x  bx 2  b2 x 3  b3x 4  ax 2  abx3  ab2 x 4  ab3x 5  ..........

x 1  ax   x 2 x3 x 4 
Now log e 1  x    x    ......  x  x 2 b  a 
1  bx  2 3 4 
 

  

1
 x 3 ab  b 2  x 4  b3  ab 2    .......... ........
4
Differential Calculus: Taylor and Maclaurin’s Theorems 47
Prepared by: Dr. Sunil, NIT Hamirpur (HP)

 1 1   1
 x 2  b  a    x 3   ab  b 2   x 4  b 2  ab 2    .......
 2 3   4
Applying the given conditions, we get
1 1
ba   0  ba 
2 2
1 1
 ab  b 2  a  ab  b 2  
3 3
1
a  b
2
 1 1 b 1 2 1
b b    b 2    b 2   b 2    b  , a 
 2 3 2 3 3 6

 1
Terms containing x 4  x 4  b  ab 2  
 4

2 1 4 1 x4
 x 4   .     . Ans.
3 6 9 4 36
Q.No.43.:Prove that

1 x3 x5 v
f a  2x   f a   xf a  x   f a  x   f a  x   ..............
 
2 3! 5!
Sol.: Let us first expand f a  2x 
y  f a  2 x   f x  a  x 

x3 x5 v
 f a  x   xf a  x   f a  x   f a  x   .............. (i)
3! 5!
Similarly expanding f(a) by Taylor’s theorem, we get
y  f a   f  x  a  x 

x2 x3 x 4 iv
 f a  x   xf a  x   f a  x   f a  x   f a  x   .............. (ii)
2! 3! 4!
Adding (i) and (ii) and dividing by 2, we get

x2 x3 x 4 iv
f a  2x   f a   f a  x   xf a  x   f a  x   f a  x   f a  x   ......
2! 3! 4!
x2 x3 x 4 iv
 f a  x   xf a  x   f a  x   f a  x   f a  x   ......
2! 3! 4!
Differential Calculus: Taylor and Maclaurin’s Theorems 48
Prepared by: Dr. Sunil, NIT Hamirpur (HP)

1 1 x3 x5 v 
 f a  2 x   f a   2xf a  x   2 f a  x   2 f a  x   ......
 
2 2  3! 5! 

x3 x5
 xf a  x   f a  x   f v a  x   .... .
3! 5!
which is the required expansion.
1
Q.No.44.: Prove that log x  x  1  x  12  1 x  13  .............
2 3


 x  1 1  x  1  1  x  1 

2

3
 ............ .
   
x 2 x  3 x 
Sol.: Let f x   log x  f 1  x  1
By Taylor’s theorem, we get

 f 1  x  1f x  


x  12 f x   x  13 f x   ..............
2! 3!

 0  x  1.1 
x  12 .  1  x  13 .2  x  14 .  6  ............
2! 3! 4!

 x  1 
 x  12 x  13 x  14
   ............... .
2 3 4
 x 1
Now log x  f x   1     f x 
 x 
2 3
 x 1  x 1  x 1
 f x   f 1   f 1    f 1    f 1  .............
 x   x   x 
2 3
 x 1  x 1 1  x 1 1
 f x      .   .  .................. .
 x   x  2  x  3
which is the required expansion.
2
Q. No.45.: If y  log x  a 2  x 2  , Expand y up to four terms by Maclaurin’s
  
theorem.
2
Sol.: y  log x  a 2  x 2 
  
Differential Calculus: Taylor and Maclaurin’s Theorems 49
Prepared by: Dr. Sunil, NIT Hamirpur (HP)

 a2  x2  x 
2 2
yi   . log x  a 2  x 2  = log x  a 2  x 2 
2 2  2 2    2 2  
a x x a x  a x

2  2 log x  a 2  x 2 . x 2  a 2
 
 1 / 2
.x
yii  2
a x 2
 

2 x2  a2 
3/ 2 2

 x x  a
2

1 / 2
   

 log x  a 2  x 2   2 x log x  a 2  x 2 .3 x 2  a 2 
1/ 2
.x
yiii 
x 2
 a2 
3

Putting x = 0, we get

y0  loge a 2

2
yi 0  loge a 
a
2
yii 0  
a2
2 log a
yiii 0  
a3
Now by Maclaurin’s Theorem, we get
2
x x3 x 4 iv x5 v
f x   f 0  xf  0   f  0   
f 0   
f 0   f 0   .................
2! 3! 4! 5!
2 2  1 
log x  a 2  x 2   log e a 2  log 2 a x  2 .x 2   2 loge a  x 3  ........... .
  a a  3a 
This completes the proof.
Q.No.46.: Apply Maclaurin’s theorem to prove that

x2 x4 x6
log sec x    .
2 12 45
Sol.: Here f x   log sec x
f  x   tan x

f  x   1  tan 2 x


f  x   2 tan x 1  tan 2 x 
Differential Calculus: Taylor and Maclaurin’s Theorems 50
Prepared by: Dr. Sunil, NIT Hamirpur (HP)

  
f iv x   2 sec 2 x 1  tan 2 x  2 tan x 2 tan x sec 2 x 

f v x   4 tan x sec 2 x 4  6 tan 2 x 
f vi x   16 sec 2 x  48 tan 2 x sec 2 x  72 tan 2 x sec 2 x  120 tan 4 x sec 2 x

f 0   0 : f  0   0 ; f  0  1 ; f  0   0 ; f iv 0   2 ; f v 0   0 ; f vi 0   2 .
Now by Maclaurin’s Theorem, we get
x2 x3 x 4 iv x5 x 6 vi
f x   f 0   xf  0  
f  0  f  0  f 0  f v 0  f 0  .................
2! 3! 4! 5! 6!
Substituting the values from above, we get

x2 x4 x6
log sec x     ..........
2 12 45
This completes the proof.
Q.No.47.: Expand cosx  h y  k  by Taylor’s theorem.
Sol.: f x  h y  k   cosx  h y  k 
f x   cos xy
By Taylor’s Theorem, we get


Fx  h, y  k   Fx, y   hFx  kFy   
1 2
2!
h Fxx  2hk  Fxy  k 2 Fyy 
Fx   sin xy.y , Fy   sin xy.x , Fxx   y 2 cos xy , Fyy   x 2 cos xy ,

Fxy  xy cos xy  sin xy

Putting the values, we get

Fx  h, y  k   cos xy  sin xyhy  kx  


2!

1 2 2
h y cos xy  k 2 x 2 cos xy 
 2hk xy cos xy   2hk sin xy

 cos xy  sin xyhy  kx  


1
2!

2hk sin xy  hy  kx 2 cos xy . 
This completes the proof.
 
Q.No.48.: Expand e x cos y in the neighbourhood at f 1,  .
 4

Sol.: e x cos y
Differential Calculus: Taylor and Maclaurin’s Theorems 51
Prepared by: Dr. Sunil, NIT Hamirpur (HP)

Differentiating, we get

f x  e x cos y , f xx  e x cos y ,

f xy 
 x
y
 
e cos y  e x sin y , fx 
 x
y
 
e cos y  e x sin y

f yy 

y
 
 e x sin y  e x cos y

f a , b   f a , b   x  1f x a , b   y  1f y a , b 


1
2!
 
x  12 f xx a, b   2x  1y  1f xy a, b   y  12 f yy a , b   ........

  e e     e 
f 1,    x  1   y   
 4 2 2  4  2 

1     e 
2
2 e     e  
  x  1  2 x  1 y   
  y      ..............
2 !  2  4  2   4   2 

e    1     
2
   
 1  x  1   y    x  12  2x  1 y     y     .......... .
2  4  2 !   4  4   

This completes the proof.

Q.No.49.: Expand e x log 1  y  by Maclaurin’s theorem, in powers of x and y upto terms


of third degree

Sol.: Let f x , y   e x log 1  y   f  0, 0   0

f x x , y   e x log 1  y   f x  0, 0   0

ex
f y  x, y    f y  0, 0   1
1 y

f xx x , y   e x log 1  y   f xx  0, 0   0

ex
f xy x , y    f xy  0, 0   1
1  y 

f yy x , y  
1  y 0  e x 1   e x  f yy  0, 0   1
1  y 2 1  y 2
f xxx  x, y   e x log 1  y   f xxx  0, 0   0
Differential Calculus: Taylor and Maclaurin’s Theorems 52
Prepared by: Dr. Sunil, NIT Hamirpur (HP)

ex
f xxy  x, y    f xxy  0, 0  1
1  y 

f xyy  x, y  
1  y  0   ex 1  e x  f xyy  0, 0   1
1  y  1  y 
2 2

2e x
f yyy  x, y    f yyy  0, 0   2
1  y 
3

Now Maclaurin’s theorem of f(x, y) gives


1 2
f  x, y   f  0,0   xf x  0,0   yf y  0,0    x f xx  0,0   2xyf xy  0,0   y 2 f yy  0,0  
2! 
1 3
  x f xxx  0,0   3x 2 yf xxy  0,0   3xy 2f xyy  0,0   y3f yyy  0,0    ....
3!
1 2
 0  x  0   y 1   x  0   2xy 1  y 2  1 
2!
1 3
  x  0   3x 2 y 1  3xy 2  1  y3  2    ....
3! 
y2 1 2 1

 y  xy   x y  xy 2  y3  .... Ans.
2 2 3


Q.No.50.: Find Taylor’s series for log cos x about the point .
3
Sol.: We know that by Taylor’s expansion

h2 h3
f x  h   f x   hf x   f x   f x   ...............
2! 3!
If x  a , and h  x  a then,

f x   f a   x  a f a  
x  a 2 f a   x  a 3 f a   ...............
2! 3!
Now f x   log cos x (i)

 
f a   f    log cos   log 2 (ii)
3 3

 
f  a   f    tan   3 (iii)
3 3
Differential Calculus: Taylor and Maclaurin’s Theorems 53
Prepared by: Dr. Sunil, NIT Hamirpur (HP)

 
f  a   f      sec 2  4 (iv)
3 3
From (i), (ii), (iii), (iv), we get
4
log cos x   log 2  x  a  3  x  a 2  ...........
2!
 4 
  log 2  x  a  3  x  a 2  ........... .
 2! 
4
log cos x   log 2  x  a  3  x  a 2 ...........
2!
This completes the proof.

Q.No.51.: Expand
x  h y  k  in powers of h, k up to second degree terms.
xhyk

Sol.: Here f x  h, y  k   x  h y  k 


xhyk
xy
Put h = k = 0, we have f x, y  
xy

fx 
x  y .y  xy.1  y 2 , fy 
x2
, by symmetry
x  y 2 x  y 2 x  y 2
2 y2 2x 2
f xx   , f yy  
x  y 3 x  y 3

f xy 
x  y 2 .2x  x 2 .2x  y   2x x  y   2x 2  2xy
x  y 4 x  y 3 x  y 3

x  h y  k   f x  h, y  k   f x, y   hf  kf   1 h 2f 
 2hkf xy  k 2f yy  .......
x y xx
xhyk 2!

xy  y2 x2 
   h.  k 
x  y  x  y 2 x  y 2 
1  2  2 y2 2 xy 2  2x
2 
 h .  2hk k .   .............
2  x  y 3 x  y 3 x  y 3 
xy y2 x2 y2
  .h  .k  .h 2
x  y x  y 2 x  y 2
x  y 3
Differential Calculus: Taylor and Maclaurin’s Theorems 54
Prepared by: Dr. Sunil, NIT Hamirpur (HP)

2xy x2
 .hk  .k 2  .......................... .
x  y 3
x  y  3

This completes the proof.


 
Q.No.52.: Expand f x , y   sin xy in powers of x  1 and  y   as for as terms of 2nd
 2
degree.
     
Sol.: sin xy  sinx  1  1  y     
 2   2 

Here h =1, and y 
2
We know that

  21! h 2f xx  2hk xy  k 2f yy 
f x  h , y  k   f x, y   hf x  kf y  (i)

        
fx  sin x  1 y     y   cosx  1 y  
x   2   2  2

      
fy  sin x  1 y    x  1cosx  1 y  
x   2   2

     
f xy   y   cosx  1 y  
x  2  2 

 x      
 cosx  1 y     y    x  1sin x  1 y  
 2  2   2 
2
   
f xx   y   sin x  1 y  
 2  2

 
f yy  x  12 sin x  1 y  
 2

  
 f x  h , y  k   sinx  1  1 y   
 2 2 
Substituting the values in (i), we get

          
sin xy  sin x  1 y     y  . cosx  1 y    x  1cosx  1 y  
 2   2  2 2  2 
Differential Calculus: Taylor and Maclaurin’s Theorems 55
Prepared by: Dr. Sunil, NIT Hamirpur (HP)

1   
2
 
   y   sin x  1 y  
2 !   2  2 

       
  cosx  1 y     y  x  1sin x  1 y  
  2  2  2 

 2   
  x  12 sin x  1 y   .
 4  2 

This completes the proof.


Q.No.53.: State and prove Taylor’s theorem. Use it to expand f x   log sin x in powers
of x  2  .

Sol.: Taylor’s Theorem:


Statement: If f (x + h) can be expanded in ascending powers of x , then

x 2 // x3 xn n
f ( x  h )  f (h )  xf / (h )  f (h )  f /// (h )  .......  f (h )  ............ .
2! 3! n!

Proof: Suppose f ( x )  a 0  a1x  a 2 x 2  a 3 x 3  ......  a n x n  ...... (i)


where a0 , a1 , a2 ,a3 ,...... are constants to be evaluated.
Differentiating (i) w.r.t. x, we get

f / ( x  h )  a1  2a 2 x  3a 3x 2  4a 4 x 3  ......  na n x n 1  ...... (ii)


Differentiating (ii) w.r.t. x, we get

f // ( x  h )  2a 2  3.2a 3x  4.3a 4 x 2  ......  n (n  1)a n x n  2  ...... (iii)


Differentiating (iii) w.r.t. x, we get

f /// ( x  h )  3.2.1a 3  4.3.2a 4 x  ......  n (n  1)(n  2)a n x n  3  ...... (iv)


Similarly, if we go on differentiating, we get

f n ( x  h )  n ( n  1)( n  2)......3.2.1.a n  terms containing x (v)


Putting x = 0 in (i) to (v) , we get

f // (h ) f /// (h ) f n (h )
a 0  f ( h ) , a1  f / ( h ) , a 2  , a3  ,........, a n  .
2! 3! n!
Putting these values of constants in (i), we get
Differential Calculus: Taylor and Maclaurin’s Theorems 56
Prepared by: Dr. Sunil, NIT Hamirpur (HP)

x 2 // x 3 /// xn n
f ( x  h )  f (h )  xf / (h )  f (h )  f (h )  .......  f (h )  ............ .
2! 3! n!
This completes the proof.
(b): f x   log sin x  log sinx  2   2
Now by Taylor’s expansion, we get

log sinx  2   2  f 2   x  2 f  2 


x  22 f  2  ...............
2!

 log sin 2  x  2cot 2 


 x  2 2
cos ec2 2  .........
2!

 log sin x  log sin 2  x  2 cot 2 


x  2 cos ec2 2  ..............
2!
This completes the proof.

Q.No.54.: Prove that ea sin


1
x
 1  ax 
ax 2  a 12  a 2   x 3  ................. ,
2! 3!
sin 2  2 3
Hence show that e  1  sin    sin   ............. .
2! 3!

Sol.: We will first expand sin 1 x by Maclaurin’s theorem


1
f x   sin 1 x , f 0  0 , f  0    1 , f  0  0 , f  0   1
1  0  2

According to Maclaurin’s theorem

x2 x3
f x   f 0   f  0  f 0   f  0   .................

2! 3!
x3 x5
 sin 1 x  x    ...........
3! 5 !
ax 3 ax 5
a sin 1 x  ax    ...........
3! 5!
We know that

x 2 x3
ex  1  x    ....................
2 ! 3!

e a sin 1 x

 1  a sin 1
x 
a sin  1 2
 .................
2!
Differential Calculus: Taylor and Maclaurin’s Theorems 57
Prepared by: Dr. Sunil, NIT Hamirpur (HP)

2 3
 x3 x5  a2  x3 x5  a3  x3 x5 
 1  a x    .....   x    ......   x    ......
 3! 5!  2 !  3! 5 !  3 !  3! 5! 
Using Binomial theorem

 x3 x5  a 2x 2   x2 x4  a 3x 3   x 2 x 4 
 1  a x    .....  1  2   ...... 
 1 3   ..... 
 3! 5!  2 !   3! 5!  3 !   3 ! 5 ! 

Combining the terms of similar powers of x

ea sin
1
x
 1  ax 
ax 2  a 12  a 2  x 3  a 2 22  a 2  x 4  .............. . (i)
2! 3! 4!
Hence this proves the result.

(b): In equation (i) put a  1 , and sin 1 x   , so that x  sin 


Substituting the values in (i), we get

sin 2  2 5
e  1  sin    sin 3   sin 4   .............. .
2! 3! 4!
Hence this prove the result.

 
QNo.55.: Expand sin m sin 1 x in ascending powers of x up to x 5 .

Sol.: Let f x   sin m sin 1 x  


f  x  

m cos m sin 1 x 
1  x2

 
 1  x 2  sin m  sin 1 x m m cos m sin 1 x  2 x  
m  
 
 1 x 2
1 x 2 
f  x  
1  x2

 m  m sin m sin 1 x   
cos m sin 1 x x 

 
 1  x2   mmf x   f  x x 


1  x 2
1  x2  


 m 2f x   f  x x 
1  x  2

 1  x m f  x   f  x   xf  x   m f x   f  x .x.  2x 


2 2 2
f  x  
1  x  2 2
Differential Calculus: Taylor and Maclaurin’s Theorems 58
Prepared by: Dr. Sunil, NIT Hamirpur (HP)


     
 1  x 2 f  x  m 2  1  f  x x  m 2f x   f  x .x.  2x 
1  x 
2 2

Now f 0  0 , f  0   m , f  0  0 , f  0  



m m2  1 
1
According to Maclaurin’s theorem

x2 x3
f x   f 0   f  0  f  0   f  0   .................
2! 3!
Substituting the values, we get

f x   mx 
  
m m 2  1 3 m m 2  1 m 2 32 5
x 

x  ................. .
3! 5!
Hence this prove the result.


Q.No.56.: If y  sin log x 2  2 x  1 , prove that
2 3
y  2x  x 2  x 3  x 4  ..........
3 2
Sol.: y  sin log x  12  sin 2 log x  1  f x 
2 cos 2 logx  1
f  x  
x 1
 x  14 sin 2 logx  1
2 cos 2 logx  1
x 1  4f x  f  x 
f  x    
x  12
x  12 x  1

f x  

 4 x  12 f  x   2x  1f x    x  1 f  x   f  x 
2

x  14 x  12
Now f 0  0 , f  0   2 , f  0  2 , f  0  4
According to Maclaurin’s theorem

x2 x3
f x   f 0   f  0  f  0   f  0   .................
2! 3!
2 3
 2x  x 2  x 3  x 4  .............
3 2
Hence this prove the result.

Q.No.57.: Expand x in powers of x  1 up to involving x  14 .


Differential Calculus: Taylor and Maclaurin’s Theorems 59
Prepared by: Dr. Sunil, NIT Hamirpur (HP)

Sol.: f x   x

Let f x   f 1  x  1
According to Maclaurin’s theorem

f 1  x  1  f 1  x  1f  1  f  1


 x  12 x  13
 f  1  ................. (i)
2! 3!
Now f x   x , f 1  1
1 1
f  x   , f  1 
2 x 2
1 1
f  x    3/ 2
, f  1  
4x 4
1 3 1 3
f  x       5 / 2 , f  1 
4 2 x 8
Substituting the values in (i), we get

f x   1 
 x  1 1 x  12 3 x  13 15 x  14
    ..........
2 4 2! 8 3! 16 4!
Hence this proves the result.
 
Q.No.58.: Expand tan 1 x   about the point x = 0.
 2

 x 
Sol.: Let f x  h   tan 1 x   , where h 
 2 2
According to Maclaurin’s theorem

h2 h3
f x  h   f x   hf  x   f x   f  x   .................
 (i)
2! 3!

Now f x   tan 1 x , f 0  0


1
f  x   , f  0  1
1  x2
2x
f  x    f  0  0
1  x  2 2
Differential Calculus: Taylor and Maclaurin’s Theorems 60
Prepared by: Dr. Sunil, NIT Hamirpur (HP)

  
 1  x 2 2 .2  8x 1  x 2 
  
f  x     , f  0   2
1 x 2 2

8x 8 6x 2
f iv x     , f iv 0   8
1  x  1  x  1  x 
2 3 2 3 2 4

Substituting these values in (i), we get

   3
1  5
f x  h   tan  x     2  2  ................... at x = 0.
 2  2 2 .3! 2 .5!
Hence this prove the result.

Q.No.59.: Expand y x in the neighborhood of point (1, 1) up to the second degree term.

Sol.: Let f x , y   y x

 f 1,1  11  1

f x x , y   y x log y , f x 1, 1  0

f y x , y   xy x 1 , f y (1,1)  1

f xx x , y   y x log y 2 , f xx 1, 1  0

f yy x , y   x x  1y x  2 , f yy (1,1)  0

yx
f xy x , y   xy x 1 log y  , f xy (1,1)  1
y
By Taylor’s theorem

f x, y   f a , b   x  a f x a , b   y  b f y a , b  
x  a 2 f a, b  x  a y  b f a, b 
xx xy
2!


 y  b 2
f yy a , b   ....................
2!
Substituting a = 1, and b = 1, we get

f x, y   f 1,1  x  1f x 1,1  y  1f y 1,1 


x  12 f 1,1  x  a y  1f 1,1
xx xy
2!


y  12 f 1,1  ...........
yy
2!
Differential Calculus: Taylor and Maclaurin’s Theorems 61
Prepared by: Dr. Sunil, NIT Hamirpur (HP)

f x, y   1  x  1.0  y  1.1 


 x  12
.0  x  1y  1.1  0  0
2!
 y x  1  xy  x .
Hence this proves the result.
Q.No.60.: Expand sin x  b  in a series in ascending power of b and complete the value

when x  and b = 0.01radian.
3
Sol.: Let f x  b   sin x  b 
From Taylor’s series

b2 b3
f x  b   f x   bf  x   f  x   f  x   .......... .......
2! 3!
Now f x   sin x , f  x   cos x , f  x    sin x
Substituting the values, we get

b2 b3 b4
sin x  b   sin x  b cos x  sin x  cos x  sin x  ......................
2! 3! 4!

Putting x  , and b = 0.01radian
3

    0.012 
sin   0.01  sin  0.01cos  sin  ......................
3  3 3 2! 3

3 1 0.0001 3
 sin 0.01   0.01.  .  ............  0.8710 . Ans.
2 2 2 2
2
Q.No.61.: Is Maclaurin’s expansion of e 1 / x valid in any interval ? Give reason.
1
Sol.: Let f x   e 1 / x , f 0  
2
0
1/ x2
e
According to Maclaurin’s theorem

x2 x3
f x   f 0  xf  0  f  0   f  0   .......... .......
2! 3!
2
Now f  x   e 1 / x . f  0   
2
,
x3
Differential Calculus: Taylor and Maclaurin’s Theorems 62
Prepared by: Dr. Sunil, NIT Hamirpur (HP)

4 6 4  6x 2
f  x   e1 / x . f  0   
2 2

6
 e1 / x . 4
 2
,
x x e1 / x x 6
8 24 12 24
f  x   e 1 / x . f  0   
2 2 2 2

9
 e 1 / x . 7
 e1 / x 7
 e 1 / x ,
x x x x5

Hence f x  becomes equal to infinity when we apply Maclaurin’s series to e 1 / x for any
2

2
interval of x. Hence the Maclaurin’s expansion of e 1 / x is not valid in any interval.

Q.No.62.: Use Taylor’s theorem to expand f x, y   x 2  xy  y 2 in powers of x  1

and y  2  .

Sol.: Differentiating f x, y   x 2  xy  y 2 partially w.r.t. x and y, we get

f x  2 x  y , f y  x  2 y , f xy  1 , f xx  2 , f yy  2 , f xxx  0 , f xxy  0 , fyyx= 0,

fyyy = 0.
The Taylor’s series expansion of f(x, y) in powers of x  1 and y  2  is


f ( x , y)  f (1,2)  x  1f x 1,2   y  2 f y 1,2    1
2!
x  12 f xx 1,2  2x  2y  2f xy 1,2
 y  2 2 f yy 1,2   31! x  1 f 3
xxx 1,2   3x  1
2
y  2f xxy 1,2

 3x  1y  2 2 f yyx 1,2  y  2 3 f yyy 1,2   ......... 


Here f(1, 2) = 7, fx(1, 2) = 4, fy(1, 2) = 5, fxy(1, 2) = 1, fxx = fyy = 2, etc.
Substituting these values

f x , y   7  4x  1  5y  2 
1
2!

2x  12  2x  1y  2  2y  2  2y  22  0  ....... 
Q.No.63.: Expand f ( x, y)  e x  y in Taylor’s series up to terms up to terms of second

degree in the form a 0  b1x  b 2 y  c1 x 2  c 2 xy  c 3 y 2  .........


(a). by direct use of Taylor’s theorem.
(b). by expanding ex+y in a series of powers of x + y.
(c). by multiplying together the separate expansion of ex and ey.

Sol.: a. f  e x  y , f x  e x  y , f y  e x  y , f xx  e x  y , f yy  e x  y , f xy  e x  y etc.

Since the series in powers of x and y, the expansion is about (0, 0) (i.e. Maclaurin’s
series) so f = fx = fy = fxx = fyy = fxy at (0, 0) = 1.
Differential Calculus: Taylor and Maclaurin’s Theorems 63
Prepared by: Dr. Sunil, NIT Hamirpur (HP)

By Taylor’s theorem

xy x 2  2 xy  y 2
e 1 x  y   .........
2!
b. Expanding in powers of x + y

ex y 

x  y n  1  x  y  
x  y 2  x  y 3  ..........
 n! 2! !
n 0

c. Term wise series multiplication


  x n   y n 
e x  y  e x .e y      
 n!  n! 
 n 0  n  0 
 x 2 x3  y 2 y3 

 1 x   
 ......... 1  y    .........
 2! 3!  2! 3! 
  

x2 y2
 1  x  y    xy   ..........
2! 2!

Q.No.64.: Expand f ( x , y)  e y In (1  x ) in powers of x and y and verify the result by


direct expansion.

Sol.: f  e y In(1  x )

1 ey  ey
fx  ey , f y  e y In (1  x ) , f xy  , f xx  , f yy  e y In1  x  ,
1 x 1 x 1  x 2

2e y  ey ey
f xxx  , f yyy  e y In 1  x  , f xxy  , f yyx 
1  x 3 1  x 2 1 x

Evaluating these derivatives at x = 0, y = 0,


f(0, 0) = 0, fx(0, 0) = 1, fy(0, 0) = 0, fxy = 1, f xx 0,0   1 , fyy(0, 0) = 0,
fxxx(0, 0) = 2, fyyy(0, 0) = 0, f xxy (0,0)  1 , fyyx(0, 0) = 1.

The Taylor’s series expansion up to third degree terms is

e y In(1  x )  f 0,0   xf x 0,0   yf y 0,0  


2!

1 2
x f xx (0,0)  2xyf xy 0,0   y 2 f yy 0,0  

3!

1 3

x f xxx 0,0   3x 2 yf xxy 0,0  3xy 2 f yyx 0,0   y 3 f yyy 0,0   ........
Differential Calculus: Taylor and Maclaurin’s Theorems 64
Prepared by: Dr. Sunil, NIT Hamirpur (HP)

 0  x.1  0 
1
2!
  1
 x 2  2.1xy  0  2 x 3  3x 2 y  3xy 2  0
3!

x2 x 3 x 2 y xy 2
x  xy     ............
2 3 2 2
Verification by series multiplication:

xn
We know that e x   n! and
n 0

x 2 x3
In1  x   x    ............
2 3
 y 2 y3   x 2 x3 
so e y In(1  x )  1  y    .........   x   ...........
 2! 3!   2 3 
   
Multiplication term by term up to third degree

x2 x3 x 2 y xy 2
e y In(1  x )  x    xy    .........
2 3 2 2

Q.No.65.: Find Taylor’s expansion of f x , y   cot 1 xy in powers of (x + 0.5) and

y  2 up to second degree terms. Hence compute f  0.4, 2.2 


approximately.

Sol.: Here f x , y   cot 1 xy

y x 2xy 3 2x 3 y
fx  , fy  , f xx  , f yy  ,
1  x 2y2 1  x 2y2 1  x 2 y 2 2 1  x 2 y 2 2
f xy 
x 2 y 2  1
1  x 2 y 2 2
1
Evaluating these derivatives at the point x   , y = 2
2
1
f x , y   cot 1 x , y  at x   .y  2
2
 1   1  3
f   .2   cot 1   .2   cot 1  1 
 2   2  4
Differential Calculus: Taylor and Maclaurin’s Theorems 65
Prepared by: Dr. Sunil, NIT Hamirpur (HP)

1 1
f x  1 , fy  , fxy = 0, f xx  2 , f yy  
4 8

Expanding cot 1 xy in Taylor’s series in powers of (x + 0.5) and y  2  , we get

f x , y   cot 1 xy  f  0.5,2   x  0.5f x  0.5,2   y  2f y  0.5 


1
2!
x  0.52


 f xx  0.5  2x  0.5y  2   f xy  0.5,2   y  2 2 f yy  0.5,2   .........

3 y2 1 1 
  x  0.5    2x  0.52  y  2 2   .........
4 4 2 8 
Put x  0.4 and y = 2.2 to compute
3 .2 1
cot 1  0.4 , 2.2   f  0.4,2.2    0.1   0.12  .22  2.29369 .
4 4 16
*** *** *** *** ***
*** *** ***
***
Differential Calculus: Taylor and Maclaurin’s Theorems 66
Prepared by: Dr. Sunil, NIT Hamirpur (HP)

Home Assignments
Q.No.1.: Expand f x , y   x 3  y 3  xy 2 in powers of x  1 and y  2  using Taylor’s
series.

Ans.: 13  7x  1  16y  2  3x  12  4x  1y  2   7y  2 2  x  13

 x  1
. y  2 2  y  2 3 .

Q.No.2.: Obtain Taylor’s expansion of 1  x  y 1 in powers of ( x  1) and ( y  1) .

Ans.: 1  x  y  x 2  2xy  y 2  ..........


Q.No.3.: Expand cos x cos y in powers of x and y up to fourth degree terms.

Ans.: 1 
2

1 2
x  y2  
1 4
24
 
x  6x 2 y 2  y 4  ..........

Q.No.4.: Obtain the expansion of e xy in powers of ( x  1) and ( y  1) .


Ans.: e 1  x  1  y  1 
x  12  x  1y  1  y  12  ..........

 2! 2! 


Q.No.5.: Find the Taylor’s expansion of e x cos y about the point x = 1, y  .
4
  
2 
  y   
e     x  12    4
Ans.: 1  x  1   y     x  1 y     ........... .
2   4  2!  4  2! 
 
 

Q.No.6.: Find the Maclaurin’s expansion of e x In1  y  up to terms of 3rd degree.


Differential Calculus: Taylor and Maclaurin’s Theorems 67
Prepared by: Dr. Sunil, NIT Hamirpur (HP)

Ans.: y  xy  

y 2 x 2 y  xy 2 y 3


 ...........
2 2 3

Q.No.7.: Expand e ax sin by about origin up to 3rd degree term.

Ans.: by  abxy  


6

1 2 2

3a bx y  b 3 y 3  .........

Q.No.8.: Find Taylor’s expansion x y about (1, 1).


1
Ans.: 1  x  1  x  1y  1  x  12  ............
2

Q.No.9.: Expand
xy  hk  hy  xk  in powers of h and k up to second degree terms.
x  y  h  k 
xy y2 x2 y2 2xy x2
Ans.:  h k h2  hk  k 2  ..........
x  y x  y 3
x  y 2
x  y 3
x  y 3
x  y 3

Q.No.10.:Calculate 
In 1.031 / 3  0.981 / 4  1  approximately by using Taylor’s
expansion up to first order terms.
Ans.: 0.005.
 0.9 
Q.No.11.: Compute tan 1   approximately.
 1.1 
Ans.: 0.6904.

Q.No.12.: Find Taylor’s expansion of 1  x  y 2 in powers of x  1 and y  0 .

 x  1 x  12 y 2 
Ans.: 2 1     .......... .
 4 32 4 

*** *** *** *** ***


*** *** ***
***
Differential Calculus

Errors and Approximations


Prepared by:
Dr. Sunil
NIT Hamirpur (HP)
(Last updated on 01-08-2009)
Latest update available at: http://www.freewebs.com/sunilnit/

(39 Solved problems and 00 Home assignments)


Definition: Suppose u is a function of x and y i. e. u  f x , y  . If there is a change in
the value of x from x to x  x  and a change in the value of y from y to y  y 
(where x and y are small and may be positive or negative), then there will be a
change in the value of u (say) from u to u  u . We may call this change in the value of x
i.e. x as ‘increment in x’ or ‘error in x’.
Similarly, y may be called as ‘increment in y’ or ‘error in y’ and so u is the
‘increment in the value of u’ or ‘error in the value of u’.
Now we have u  f x , y  . (i)

 u  u  f x  x , y  y 
 u  f x  x , y  y   f x , y  . (ii)
Expanding f x  x , y  y  by Taylor’s theorem on two variables

 f f  1   2 f  2f  2f 
f x  x, y  y   f x , y    x  y    2 x 2  2xy  y 2 2 .....
 x y  2!  x xy y 
Differential Calculus: Errors and Approximations Prepared by: Dr. Sunil, NIT Hamirpur 2

We know that, Taylor’s series for a function of one variable is


x2
f x  h   f h   xf h   f h   ............
2!
h2
 f x   hf x   f x   .............
2!
Also, Taylor’s series for a function of two variables is

   1  2  2 2 2  
2
f x  h, y  k   f x, y   h  k f x, y  h
   2hk k f x, y  ......
 x y  2!  x 2 x y y 2
 

Substituting the expansion of f x  x , y  y  in (ii), we obtain

  f f  1   2 f  2f  2f  
u  f x , y    x  y    2 x 2  2xy  y 2 2 .....  f x, y  .
  x y  2!  x xy y  
As x and y are supposed to be very-very small, therefore their squares and higher
powers can be neglected.
f f
Thus u  x  y .
x y
Replacing x , y, u by dx, dy, dz respectively, we have
f f u u
du  dx  dy  u  x  y  u  f x, y  .
x y x y
This formula is used in calculating the effect of small errors or increments in measured
quantities and is useful in correcting the effect of small errors.
u u u
Remarks: If u  f x, y, z,...... then, u  x  y  z  ..........
x y z

Percentage error:
x
Definition:  100 is called percentage error in the value of x, where x is the change
x
or actual error in the value of x.
y
Similarly,  100 is called the percentage error in y, and
y
u
 100 is called the percentage error in u.
u
Differential Calculus: Errors and Approximations Prepared by: Dr. Sunil, NIT Hamirpur 3

where y and u are actual errors in y and u respectively.


x
Relative error: If x is the error in x, then relative error  .
x
Now let us solve some problems related to errors and approximations:

Q.No.1.: Find the percentage error in the area of an ellipse, when an error +1 percent
is made in semi-major axis and  1 is made in measuring the semi-minor
axis.
Sol.: Since, the area A of an ellipse is given by the relation A  ab ,
where a, b are its semi-major and semi-minor axis.
Here error in a and b are given, therefore we will treat a and b variables. Since, when a
and b are treated as variables  A is also a variable.
Taking differentials, we get
d A   dab   d ab   da .b  a.db 
 dA  b . da  a . db
dA b a da db  A A
  .da  .db =   a  b , b  a 
A A A a b  
dA da db
  100   100   100 = percentage error in a + percentage error in b
A a b
  1   1  0 .
Hence percentage error in the area of an ellipse is zero.
2nd method:
Since A  ab
Taking logarithms on both sides, we get
log A  logab   log   log a  log b .
Now taking differentials, we get
1 1 1
.dA  0  .da  .db
A a b
dA da db
  100   100   100   1   1  0 .Ans.
A a b
Q.No.2.: The edge of a cube is measured with a positive error of 0.05 cm. Find the
Differential Calculus: Errors and Approximations Prepared by: Dr. Sunil, NIT Hamirpur 4

relative error in the computed volume, when the edge is found to be 7.5 cm.
Also find percentage error in the computed volume.
Sol.: Let x be the edge of the cube.

 volume V of the cube  x 3 . (i)


Taking log on both sides, we get

log V  log x 3  3 log x . (ii)


Taking differentials, we get
1 1
.dV  3. dx . (iii)
V x

3V 3x 3
 Error in the computed volume  dV  dx  dx  3x 2dx
x x

 dV  3  7.52  0.05  8.44 cubic cm.


dV 3dx 3  0.05
Thus, relative error in the computed volume =    0.02 . Ans.
V x 7 .5
dV
Now, percentage error in the computed volume   100  0.02  100  2% . Ans.
V
Q.No.3.; The diameter and altitude of a can in the shape of right circular cylinder are
measured as 4 cm and 6 cm respectively. The possible error in each
measurement is 0.1 cm. Find approximately the maximum possible error in
the value computed for the volume and lateral surface.
Sol.: Let the diameter and altitude of the can be denoted by D and H respectively.
D
Then radius  .
2
 2
(i) The volume V of the can is given by V  r 2 h  D H  f (D, H)
4

 dV 
V
D
dD 
V
H


dH  2DHdD  D 2dH
4



4
  
2  4  6  0.1  4 2  0.1  6. 4   1.6 cubic cm. Ans.
4
(ii) The lateral surface S of the can is given by S  2rh  DH  f (D, H)
S S
 dS  dD  dH  HdD  DdH   6  0.1  4  0.1   sq. cm. Ans.
D H
Differential Calculus: Errors and Approximations Prepared by: Dr. Sunil, NIT Hamirpur 5

Q.No.4.: The height of a tower is determined by observing the elevation  and  of


its summit from two points in a direct line with the foot of the tower and at a
distance ‘a’ apart. Show that the error in the calculated height due to small


errors d and d is approximately a sin 2 d  sin 2 d cos ec2    . 
Sol.: Let h be the height of the tower AB and C and D, the two points of observation
so that B
CD  a , ACB   ,
ADB   . Let AC = x
h
From right angle BAC , x  h cot  (i)
From right angle BAD , x  a  h cot  (ii)  
D
Subtracting (i) from (ii), we get a C x A
 cos  cos    sin  cos   cos  sin   h sin    
a  h cot   cot   h     h    sin  sin 
 sin  sin    sin  sin  
a sin  sin 
h  f ,  (iii)
sin   
Taking log on both sides, we get log h  log a  log sin   log sin   log sin   
dh cos  cos  cos  
Taking differentials, we get  0 d  d  d  d
h sin  sin  sin   

dh  cos  cos     cos  cos   


    d     d
h  sin  sin      sin  sin    
sin   cos   cos  sin  sin   cos   cos  sin 
 d  d
sin  sin    sin  sin   
sin     sin    
 d  d
sin  sin    sin  sin   

sin   sin  sin 2 d  sin 2 d


 d  d 
sin  sin    sin  sin    sin  sin  sin   
 sin     sin 
sin 2 d  sin 2 d a sin  sin  sin 2 d  sin 2 d
 dh  h.  . [using (iii)]
sin  sin  sin    sin    sin  sin  sin   

 
 a sin 2 d  sin 2 d cos ec2    .
Differential Calculus: Errors and Approximations Prepared by: Dr. Sunil, NIT Hamirpur 6

Hence prove.
Q.No.5.: If an error committed in measuring the side of a square be 2%. Find the error
in calculating the area.

Sol.: Since the area A of a square is A  x 2 , where x is the side of a square.


Taking log on both sides, we get

log A  log x 2  2 log x .


Taking differentials on both sides, we get
1 1
dA  2. dx
A x
dA  dx 
  100  2   100
A x 
 %age error in A  2% age error in x   2  2  4% .Ans.
Q.No.6.: Find the % error in the area of an ellipse, when an error of +1% is made in
measuring the semi-major and semi-minor axis.
Sol.: Since A  ab , where a, b are its semi-major and semi-minor axis.
Taking logarithms on both sides, we get
log A  logab   log   log a  log b .
Now taking differentials, we get
1 1 1
.dA  0  .da  .db
A a b
dA da db
  100   100   100   1   1  2% .Ans.
A a b
Q.No.7.: In some torsion experiment an error of 0.5%, was made in measuring the
diameter x. Calculate the corresponding %age error in the stress f, where

T f x3  0 .
16
  16 T Here only f and x will be treated as 
Sol.: Here T  f x3  0  f   variables as error occurs in these. 
16 11 x 3  
Taking log on both sides, we get
  16 
log f  log   log T  3 log x
 11 
Differential Calculus: Errors and Approximations Prepared by: Dr. Sunil, NIT Hamirpur 7

Taking differentials on both sides, we get


df dx df dx
 003   100  3  100  3  0.5  1.5% .Ans.
f x f x
Q.No.8.: In an experiment carried out to find the value of g error of 0.5% and 1% are
possible in the value of t and  respectively. Show that the maximum error in
the calculated value of g could not be more than 2%.


Sol.: Time period of pendulum is given by T  2 (i)
g

Where T = time period,  = length of pendulum, g = acceleration due to gravity.


On squaring (i), we get
 
T 2  4 2  g  4 2 2 . (ii)
g T
On differentiating, we get

 4   d
2
1 2 dT 
dg  42  2 d  2 dT   2    2 
T T  T  T

 d dT 
dg  g   2 
 T
dg  d dT 
   2  (iii)
g  T
d
Given, %age error in length,  100  1% (iv)

dT
%age error in Time period,  100  .5% (v)
T
Putting the values from (iv) and (v) in (iii), we get
dg
 100  1  2 .5  1  1%  2% .
g
Therefore maximum error in the value of g = 2%. Ans.
Q.No.9.: The time of swing t, of a pendulum, of length  , under certain conditions is
2
  r 
given by t  2 , where g  g  . Find the %age error in t due to the
g rh
errors of p% in h and q% in  .
Differential Calculus: Errors and Approximations Prepared by: Dr. Sunil, NIT Hamirpur 8


Sol.: Given t  2
g

1 1
Taking log on both sides, we get log t  log 2  log   log g
2 2
2
1 1  r 
 log t  log 2  log   log g 
2 2 rh
1 1
 log t  log 2  log   log g  log r  log r  h 
2 2
Taking differentials, we get
dt 1 d 1
  0 00 dh
t 2  rh
dt 1  d   dh 
  100    100     100 
t 2   rh 
1 1  dh  1 ph 
 %age error in t  q   100 h   q   % .Ans.
2 rh h  2 rh

Q.No.10.: In measuring the value of angle  , an error of 0.10 was made. Find the
corresponding error in the value of the sine of the angle.
0. 1  
Sol.: Given d  0.10  Radian  0.00175 Radian
180
dy  ? where y  sin 
 dy  cos d  0.00175 cos  .Ans.
Q.No.11.: If H. .P. required to propel a steamer is proportional to the cube of its velocity
and square of its length, prove that a 2% increase in velocity and 3% increase
in length will require approximately a 12% increase in H. P.

Sol.: Given P  v32  P  kv32 , where k is the constant of proportionality.


Taking log on both sides, we get
log P  log k  3 log v  2 log  .
Taking differentials, we get
dP dv d
 03 2
P v 
Differential Calculus: Errors and Approximations Prepared by: Dr. Sunil, NIT Hamirpur 9

dP  dv   d 
  100  3  100   2  100   3  2   2  3  6  6  12 % .Ans.
P  v    
Q.No.12.: The indicated horse power I of an engine is calculated from the formula
PLAN 
I , where A  d 2 . Assuming that error of r % may have been
33000 4
made in measuring P, L, N and d. Find the greatest possible error in I.
PLAN
Sol.: Given I  .
33000
Taking log on both sides, we get
log I  log P  log L  log A  log N  log 33000
Taking differentials, we get
  2  2
dI dP dL d d  dN  log A  log 4 d   log 4  log d 
    2.  0    
I P L dN N   
  log  2 log d 
 4 
dI dP dL d d  dN
  100   100   100  2  100   100
I P L d N
 r  r  2r  r  5r% . Ans.

Q.No.13.: The time period of a simple pendulum is given by t  2. .
g

Find the error in t due to error  and g in  and g. What is the max. %age
error in t if there is an error of 1% in  and g.


Sol.: Given t  2. .
g

Taking log on both sides, we get


1 1
log t  log 2  log   log g .
2 2
Taking differentials, we get
dt 1 d 1 dg
 0 
t 2  2 g
Differential Calculus: Errors and Approximations Prepared by: Dr. Sunil, NIT Hamirpur 10

 dt  1  d  1  dg 
   100     100     100 
 t  2   2 g 
1
(i)   1  1  1  0% (Not max). Ans.
2 2
1
(ii)   1  1  1  1% (Max)
2 2
1
(iii)   1  1  1  1% (Max). Ans.
2 2
1
(iv)   1  1  1  0% (Not Max)
2 2
 Max. %age error in t =  1 % . Ans.

w 
Q.No.14.: The slope of a hanging rod of uniform strength is given by y  A exp.  x  ,
f 
where y is the radius at any height x above a fixed point at A is constant. Find
the change in y produced by small changes w in w and f in f. Show that the
wx
%age error in y is times the difference in the %age errors in w and f.
f

Sol.: Given y  A e w x / f .
wx
Taking log on both sides, we get log y  log A  log e .
f
Taking differentials on both sides, we get
y  fw  wf 
 0  x 
y  f2 
xy  fw  wf  wxy  fw  wf 
 y      . Ans.
f  f  f  wf 
y  fy  wf  wx  fw  wf 
Also  100  x    100    100
y  f 2
 f  wf 
wx  w f 
=   100   100 . Ans.
f w f 
Q.No.15.: The area S of a triangle is calculated from the length of sides a, b, and c. If a
be diminished and b be increased by small amounts x, prove that the
Differential Calculus: Errors and Approximations Prepared by: Dr. Sunil, NIT Hamirpur 11

S 2a  b x
consequent change in area is given by  2 .
S c  a  b 2

Sol.: Hero’s Formula: A formula connecting the area of a  with its sides
abc
  ss  a s  b s  c  , s  is semi-parameter.
2

 a  b  c  a  b  c  a  b  c  a  b  c 
 Area S     a   b   c
 2  2  2  2 

a  b  c b  c a c  a  ba  b c


  . . . .
 2  2  2  2 
Taking log on both sides, we get
1
log S  loga  b  c   logb  c  a   logc  a  b   loga  b  c   4 log 2
2
Taking differentials, we get
S 1  a  b b  a a  b a  b 
    
S 2  a  b  c  b  c  a  c  a  b  a  b  c 


1
0
2x

 2x   0

2 b  c  a  c  a  b  
x x  1 1 
   x 
b  c  a  a  c  b   c  a  b  c  a  b 
c  a  b  c  a  b  2a  b x
 x   . Ans.
 c 2  a  b   c 2  a  b 
2 2

E
Q.No.16.: If R  , find the max. error and the %age error in R if C  20 with a
C
possible error of  0.1 and E  120 with a possible error of  0.05 .
E
Sol.: Given R  .
C
Taking log on both sides, we get
log R  log E  log C .
Taking differentials on both sides, we get
R E C  E C   0.05   0.1 
   R  R     6   =0.0324.(max)
R E C  E C   120  20 
Differential Calculus: Errors and Approximations Prepared by: Dr. Sunil, NIT Hamirpur 12

which is the required max. error in R.


R E C
Now  100   100   100
R E C
R  0.05  0.1 5 1
  100   100   100    0.54% (max). Ans.
R 120 20 120 2
which is the required max. % error in R.
Q.No.17.: Basing your method on the theory of small errors, find an approximate value
of f 10.02, 40.05, 29.97  where f x, y, z   x y z .
Sol.: Let x  10, x  0.02, y  40, y  0.05, z  30, z  0.03 .
Now f x , y, z   xyz .
Taking log on both sides, we get
log f  log x  log y  log z .
Taking differentials, we get
f x y z 0.02 0.05  0.03
       0.002  .00125   0.001  0.00225
f x y z 10 40 30
 f  0.00225 f , but f  10  40  30  12000 . f  12000  0.00225  27 .
 Approximate value of f  f  f  12000  27  12027 .Ans.
Actual value = 12026.991
Q.No.18.: In calculating the volume of a right circular cone, errors of +2% and minus
one percent are made in the height and radius of the base respectively. Find the
%age error in the volume. What is the percentage error in calculating value of
the surface area of the cone ?
Sol.: Given %age error in height = 2% and %age error in radius  1% .
1 2
Since we know that the volume of a right circular cone is V  r h .
3
1
log V  log  log   2 log r  log h
3
Taking differentials, we get
dV 2dr dh dV 2dr dh
    100   100   100 .
V r h V r h
 %age error in volume  2 1  2  0% . Ans.
Differential Calculus: Errors and Approximations Prepared by: Dr. Sunil, NIT Hamirpur 13

Q.No.19.: In estimating the cost of a pile of bricks measured as 6 by 50 by 4 feet, the


tape is stretched 1% beyond the standard length. If the count is 12 bricks to

1foot3 and bricks cost Rs. 100 per 1000, find the approximate error in the cost.
Sol.: Let  , b and h feet be the length, breadth and height of the pile so that its volume
V  bh .
Taking log on both sides, we get
log V  log   log b  log h .
Taking differentials, we get
V  b h
   .
V  b h
 b h
Since V  6  50  4  1200 ft 3 and  100   100   100  1% .
 b h
 3  3
 V  1200   36 ft .
 100 
Number of bricks in V  36  12  432 .
100
Thus error in the cost  432   Rs. 43.20 ,
1000
which is less to the brick seller.

Q.No.20.: If f x, y, z   x  y m z n and errors of p %, q % and r % are made in measuring


x, y, z respectively. Find the error in f x, y, z  .

Sol.: Given f x, y, z   x  ym z n .


Taking log on both sides, we get
log f   log x  m log y  n log z .
Taking differentials, we get
f x y z
 m n
f x y z

f  x   y   z 
  100    100   m  100   n   100 
f  x   y   z 
Hence %age error in f x, y, z   p  qm  r n % . Ans.
Q.No.21.: Two quantities x1 and x 2 are related to each other by the formula,
Differential Calculus: Errors and Approximations Prepared by: Dr. Sunil, NIT Hamirpur 14

x 2  a x1 n , where a and n are constant quantities. Small errors of p % and


q% are made in measuring a and n, show that the calculated value of x 2 for a

given value of X of x1 will have a percentage error of p  nq loge X .


Sol.: Given that %age error in a = p%. %age error in n = q%

Since given x 2  a x1 n .

Taking log on both sides, we get log x 2  log a  n log x1 .


dx 2 da ndx1
Differentiating on both sides, we get   dn log x1 
x2 a x1

dx 2 da  dn  ndx1
  100   100    100 n log x1   100
x2 a  n  x1
dx 2
  100  p  qn loge X  0  p  nq loge X .
x2

Thus %age error in x 2  p  nq loge X% .

r 2w 2
Q.No.22.: The acceleration of a piston is equal to rw 2 cos   cos 2 . In

measuring   30 and w small error minus 1 percent each was detected.
Prove that calculated value of acceleration is minus 1.5%. Take 4r   .

2 r 2w 2
Sol.: Given acceleration of a piston a  rw cos 2  cos 2 . (i)

Putting 4r   in (i), we get

rw 2
a  rw 2 cos   cos 2
4
r cos 2 rw 2
a  2rw cos w  rw 2  sin   2 ww  2 sin 2
4 4
  w     cos 2  w  sin 2    
a  rw 2 2 cos     sin       
  w     2  w  2    

  w     cos 2  w  sin 2    
rw 2 2 cos    sin       
a   w    2  w  2    

a  cos 2 
rw 2 cos  
 4 
Differential Calculus: Errors and Approximations Prepared by: Dr. Sunil, NIT Hamirpur 15

w 
Dividing and multiplying by 100 and putting   1% ,  1% ,   30 , we get
w 
 cos2  30 sin 60  1%  30 
  2cos 301%  sin 30  1%  30  1%  
a
   
2 2
a  cos 60  
cos 30 
 4 
 

 3   1    1 1   3 1  
  2       
  2 6   2 2   2  2  6 
a  2   %  1.5% . Hence prove.
     

a  3 1
    
  2 8 
 
Q.No.23.: In a plane triangle ABC, if the sides and angles receive small variations, prove
that a cos C  c cos A  0 ; b, B being constant.
Sol.: To prove: a cos C  c cos A  0 , b and B as constants
Here using projection formula: b  a cos C  c cos A .
Differentiating, we get
db  da cos C  a  sin C dC  dc cos A  c sin A dA
0  da cos C  a  sin CdC  dc cos A  c sin AdA
Now A  B    C
dA  dB  dC  dA  dC
0  da cos C  a sin CdA  dc cos A  c sin AdA
sin A sin C
Now using sin formula:   c sin A  a sin C
a c
 0  da cos C  dc cos A  c sin AdA  c sin AdA
 da cos C  dc cos A  0 .
Hence this proves the result.
Q.No.24.: The side a of a triangle ABC is calculated from b, c, A. Small errors db, dc, dA
occur in the measured values of b,c, and A respectively. Prove that the error in
a is given by da  cos Bdc  cos Cdb  b sin CdA .
Sol.: To prove: da  cos Bdc  cos Cdb  b sin CdA .
Here using projection formula: a  b cos C  c cos B .
Differentiating, we get
Differential Calculus: Errors and Approximations Prepared by: Dr. Sunil, NIT Hamirpur 16

da  db cos C  dc cos B  b sin CdC  c sin BdB .


Using sine formula: c sin B  b sin C
 da  db cos C  dc cos B  b sin CdB  dC 
But B  C    A  dB  dC  dA .
 da  db cos C  dc cos B  b sin CdA .
Hence this proves the result.
1 1 1
Q.No.25.: Given the formula   . If x and y are both in the error by r %, prove
z x y
that z is also in the error of r %.
1 1 1
Sol.: Since   .
z x y

Taking differentials, we get  z 2dz   x 2dx  y 2dy

1  dz  1  dx  1  dy  r r 1 1 r
    100      100     100      r    
z z  x x  y y  x y x y z

 dz 
   100   r % . Ans.
 z 
Hence z is also in the error of r %.
Q.No.26.: The quantity Q of water flowing over a notch is given by
8
Q  0.64  2g  H 5 / 2 , where H is the head at the notch. What is the
15
% age error in Q caused by measuring H as 0.198 instead of 0.2?
8
Sol.: Since Q   0.64  2g  H 5 / 2 .
15
Taking log on both sides, we get
8 5
log Q  log  log 0.64  log 2g  log H .
15 2
Q 5 H
Taking differentials, we get  000 .
Q 2 H
Q 5  H  5  0.002  5
  100  .  100   .  100    H  0.2  0.198  0.002
Q 2 H  2  0.2  2
Hence % age error in Q  2.5 % . Ans.
Differential Calculus: Errors and Approximations Prepared by: Dr. Sunil, NIT Hamirpur 17

Q.No.27.: A closed rectangular box with unequal sides a, b, c has its edges slightly
altered in length by amount a , b and c respectively. Show that its volume
a b c
and surface area remain unchanged then   .
a 2 b  c  b 2 c  a  c 2 a  b 
Sol.: Given volume of a rectangular box is V  abc .
Taking log on both sides, we get
log V  log a  log b  log c
Taking differentials, we get
V a b c a b c
   . Now since V  0    0
V a b c a b c
a b c a a 
    a   b  c 
a b c b c 
Also S  2ab  bc  ca 
Taking differentials, we get
0  2ab  ba  bc  cb  ca  ac  ( since S  0 )
 a  c b  a  b c  b  c a  0

 a  
a  cb  a  b c  a b  a c  a  cb  a  b c
b  c b c b  c
a a  c a  b a 
   b     c
b b  c b  c c
b c
  .
b c  a 
2
c a  b 
2

a c
Similarly  .
a b  c 
2
c a  b 
2

a b c
Hence   . Hence prove.
a b  c 
2
b c  a 
2
c a  b 
2

Q.No.28.: The height h and semi-vertical angle a of a cone are measured and from then A
the total area of the surface of the cone including the base is calculated. If h
and a are in error by small quantity h and a respectively. Find the

corresponding error in the area. Show further that, if   , an error of +1%
6
Differential Calculus: Errors and Approximations Prepared by: Dr. Sunil, NIT Hamirpur 18

in h will be approximately compensated by an error of  0.33o in  .


Sol.: Base radius r  h tan  .
Slant height   h sec  . 

Area of base  r 2 .

Area of curved surface  r . h

Total surface area A  r 2  r  r r     r r  h 2  r 2 


  r

 .h tan  h tan   h 2  h 2 tan 2    h tan h tan   h sec  


 

 h 2 tan tan   sec   .  f h,  

A A
 A  h  
h 

  
 2h tan 2   tan  sec  h  h 2 2 tan  sec2   sec3   tan  sec  tan   

 2h tan tan   sec  h  h 2 sec  2 tan  sec   sec2   tan 2   ,  (i)
which gives the error in A.
 h
Putting   and h  1% of h  in (i), we have
6 100

1  1 2  h 2 2  2 2 4 1 2h 2
A  2h.   .  h .  .      2 3h 2 .
3 3 3  100 3  3 3 4 3 100
Since the error in h is to be compensated by the error in   A  0
1 1
  3   0     radians
100 100 3

   
0.01
1.732
 57.3 degree 1 radian  57.3
o
nearly 
 0.33 degree.
Q.No.29.: At a distance of 30 meter from the foot of the tower the elevation of its top is
30 . If the possible error in measuring the distance and elevation are 2cm.
and 0.05degrees. Find the approximate error in calculating the height.
Sol.: h  x tan  .
Taking log on both sides, we get
Differential Calculus: Errors and Approximations Prepared by: Dr. Sunil, NIT Hamirpur 19

log h  log x  log tan x


Differentiating, we get

h x sec 2  h sec 2 
  .x  h  x  h .x  tan .x  x sec2 .
h x tan  x tan 

Given x  0.02 ,   0.05  0.05. rad.
180
  
h  tan 300.02   30.sec 2 30 0.05.   0.0464 m = 4.64 cm. Ans.
 180 
Q.No.30.: Find the %age error in the area of an ellipse if one % error is made in
measuring the major and minor axes.
Sol.: Area of an ellipse A  ab .
Taking log on both sides, we get
log A  log   log a  log b
Differentiating, we get
A a b A a b
    100   100   100  1%  1%  2% .
A a b A a b
 %age error in area of an ellipse = 2%. Ans.
Q.No.31.: Two sides a, b of a triangle and included angle C are measured. Show that the
error c in the computed length of third side c due to a small error in the angle
C is given by c  a sin BC .

Sol.: Given c2  a 2  b 2  2ab cos C


Differentiating, we get
2cc  2aa  2bb  2a.b cos C  ab cos C  ab sin C C
As a  b  0
 2cc  2ab sin C C  cc  ab sin C.C

By sin law in ABC , b sin C  c sin B


 cc  ac sin B.C  c  aC.sin B .
Hence this proves the result.


Q.No.32.: Let T  2 . Find the maximum %age error in T due to possible error of
g

1% in  and g respectively.
Differential Calculus: Errors and Approximations Prepared by: Dr. Sunil, NIT Hamirpur 20


Sol.: Given T  2 .
g

Taking log on both sides, we get


1 1
log T  log 2  log   log g .
2 2
Differentiating, we get

T 1  1 g 1   g 
 0     
T 2  2 g 2  g 

T 1   g 
 100  %age error in T      100 .
T 2  g 

1   g  1
Maximum %age error in T     .100   1  2   1.5% . Ans.
2  g  2

V 2 sin 2
Q.No.33.: Let R  , find the %age error in R due to an error of 1% in v and
g
1
% in  .
2
V 2 sin 2
Sol.: Given R  . (i)
g
dv d 1
Given  100  1% ,  100  %
v  2
Taking log on both sides of (i), we get
log R  2 log V  log sin 2  log g
Differentiate on both sides, we get
dR dV cos 2
2  .2d .
R V sin 2
Multiplying by 100, we get
dR  dV   d  1 
 100  2  100    cot 2  100   2.1   cot 2.  2  cot 2. Ans.
R  V     2 2
A
Q.No.34.: If S  , find the maximum relative error in S and maximum error in S.
AW
If the values of A and W are 1.1 and 0.6 respectively with possible error in
Differential Calculus: Errors and Approximations Prepared by: Dr. Sunil, NIT Hamirpur 21

0.01 and 0.02 in A and W respectively.


A
Sol.: Given S  . (i)
AW
Now differentiating (i) w. r. t. to A, we get
S A  W .1  A.1 A  W  A W
   (ii)
A A  W  2
A  W  A  W 2
2

Differentiating (i) w. r. t. W, we get


S A  W .0  A. 1 A
  (iii)
W A  W  2
A  W 2
S S
We know that dS  dA  dW (iv)
A W
Now putting the values of (ii) and (iii) in (iv), we get
W A
dS  dA  dW
A  W  2
A  W 2
Now givenA = 1.1, W = 0.6, dA  0.01 , dW  0.02 .
Maximum error in S=
.6 1.1 0.6 1. 1
dS   0.01   0.02   0.01   0.02
1.1  0.6 2
1.1  0.62 0.25 0.25

 0.024  0.088  0.112 .


 Maximum error in S = 0.112. Ans.
dS
Maximum relative error in S is given by .
S
A 1.1 1.1
Now S     2.2 .
A  W 1 .1  0 .5 0 .5
dS 0.112
  0.0509  0.51
S 2 .2
 Maximum relative error in S = 0.51. Ans.
Q.No.35.: Use differentials to compute f 0.9,  1.2  approximately where

f x, y   tan 1xy  .


Sol.: Given f 0.9,  1.2  .
Let x  1 , x  0.1 ,
Differential Calculus: Errors and Approximations Prepared by: Dr. Sunil, NIT Hamirpur 22

y   1 ,  y   0 .2 .

 f x, y   tan  1
1

3
Let f x , y    ,   .
4
3
Now tan 1 xy     .
4

Taking log on both sides, we get log   log tan 1xy  .


Differentiating, we get
 1 xy  yx 1  0.2  0.1 1  0.2  0.1 0.1
        0.05 .
 tan xy  1  x y
1 2 2
tan xy  1  1
1  11 2

3
 f 0.9,  1.2   f x  x , y  y        0.05  2.307 . Ans.
4
Q.No.36.: If the sides and angles of a triangle ABC vary in such a manner that its
da db dc
circum-radius remains constant, prove that    0.
cos A cos B cos C
da db dc
Sol.: To prove:    0.
cos A cos B cos C
We know that, the circum-radius R of a ABC is given by
a b c
R   .
2 sin A 2 sin B 2 sin C
Now a  2R sin A . [R is constant]
da
Differentiating, we get da  2R cos AdA   2RdA .
cos A
db
Similarly db  2R cos BdB   2RdB .
cos B
dc
dc  2R cos CdC   2RdC .
cos C
da db dc
Adding, we get    2R dA  dB  dC   2Rd A  B  C  (i)
cos A cos B cos C
Also A  B  C   .  dA  B  C  0 .
da db dc
 From (i), we get    0.
cos A cos B cos C
Differential Calculus: Errors and Approximations Prepared by: Dr. Sunil, NIT Hamirpur 23

This completes the proof.


Q.No.37.: The area of a rectangle is found from measurements of side a and angle B and
C. Prove that error in the calculated value of area due to small error
2 c B b C 
a , B, C is given by  a    .
a a sin B a sin C 
1
Sol.: We know that area   ab sin C (i)
2
sin A sin B sin C a sin B
Now   b
a b c sin A
Putting in (i), we get
1  a sin B  1 2 sin B sin C
 a   sin C  a (ii)  A  B  C  
2  sin A  2 sin  B  C 
Taking log of (ii) on both sides, we get
1
log   log  2 log a  log sin B  log sin C  log sin B  C  (iii)
2
 sin  B  C  sin(B  C)
Differentiating (iii), we get
 2a cos B cos C cos(B  C)
  B  C  B  C 
 a sin B sin C sin( B  C)

2a  cos B cosB  C    cos C cosB  C  


  B    C   
a  sin B sin( B  C)   sin C sin( B  C) 

2a  cos B sin( B  C)  sin B cos(B  C) 


  B 
a  sin( B  C) sin B 
 cos C sin( B  C)  sin C cos(B  C) 
 C  
 sin( B  C) sin C 
2a sin C C sin B
  B  . (iv)
a sin B sin B  C  sin C sin(B  C)
A  B  C    B  C    A  sin( B  C)  sin(   A )
Putting this value in (iv), we get
 2a B sin C C sin B
   . . (v)
 a sin B sin A sin C sin A
Differential Calculus: Errors and Approximations Prepared by: Dr. Sunil, NIT Hamirpur 24

sin A sin B sin C


According to sine formula:   .
a b c
c sin C b sin B
  , 
a sin A a sin A
Putting these values in (v), we get
 2a c B b C
  
 a a sin B a sin C
2 c B b C 
   a    , which is the required proof.
a a sin B a sin C 
Q.No.38.: In a plane triangle, if the sides a, b be constant, prove that the variations of its
angles are given by the relations
dA dB dC
  ,
a 2  b 2 sin 2 A b 2  a 2 sin 2 B c

the letters have their usual significance.


a b
Sol.: By sine formula,   a sin B  b sin A (i)
sin A sin B
Taking differentials, we get a cos B.dB  b cos A.dA
dA dB dA  dB  a c a c
   . (ii)  If  then each 
a cos B b cos A a cos B  b cos A  b d b  d 

Now a cos B  a 1  sin 2 B  a 2  a 2 sin 2 B  a 2  b 2 sin 2 A [using (i)]

b cos A  b 1  sin 2 A  b 2  b 2 sin 2 A  b 2  a 2 sin 2 B [using (i)]


a cos B  b cos A  c [By projection formula]
Also A  B  C    A  B    C so that dA  dB  dC
dA dB dC
 From (ii),   .
2 2
a  b sin A 2 2 2
b  a sin B 2 c

Q.No.39.: If there is a small error c in measuring the side c in a triangle, show that
relative error in the area of the triangle is equal to
1 1 1 1  c
s  s  a  s  b  s  c  4 .
 
or
Differential Calculus: Errors and Approximations Prepared by: Dr. Sunil, NIT Hamirpur 25

If A be the area of a triangle, prove that the error in A resulting from a small error
in ‘c’ is given by

A 
4

A 1

s  s  a 1  s  b 1  s  c 1 c .

A
Sol.: Let A is the error in A, then relative error in A = .
A
A 1 1 1 1  c
Now to prove:     .
A  s s  a s  b s  c  4

or A 
4

A 1

s  s  a 1  s  b 1  s  c 1 c

Since, we know that A  ss  a s  b s  c .


Taking log on both sides, we get
1 1 1 1
log A  log s  log s  a   log s  b   log s  c  .
2 2 2 2
Differentiating on both sides, we get
A 1  s s s s  c 
     . (i)
A 2 s sa sb s  c 
abc c
Since s   s  .
2 2
Putting in (i), we get
A 1  c c c c  c 1 1 1 1 
         
A 2  s s  a s  b s  c 4  s s  a s  b s  c 

 A 
4

A 1

s  s  a 1  s  b 1  s  c 1 c .

Hence this proves the result.

*** *** *** *** ***


*** *** ***
***
Differential Calculus
Indeterminate Forms
0 
, 0  , ,   , 0 0 ,  0 , 1
0 
Prepared by:
Dr. Sunil
NIT Hamirpur (HP)
(Last updated on 01-08-2009)
Latest update available at: http://www.freewebs.com/sunilnit/

(06 Solved problems and 00 Home assignments)


Indeterminate forms:
If the value of a function f(x) when x = a takes one of the following forms, i.e.
0 
, 0  , ,   , 00 ,  0 , 1 ,
0 
then the function is said to be in an indeterminate form. The value of the function is
obtained by finding the limit of f(x) as x approaches or tends to a. All the indeterminate
0
forms, by a little arrangements or (simplification), can be brought to the form .
0
(i) Problems solved by using different algebraic laws
First of all, we will discuss some problems which can be solved easily by
simplifying the given function with the help of different algebraic laws:

5
Q.No.1.: Prove that Lt  x 2  3x  1  x 2  2 x  8   .

x   
 2

Sol.: L.H.S.  Lt  x 2  3x  1  x 2  2 x  8     form


x  
 
Indeterminate forms: Problems solved by using different algebraic laws 2
Prepared by: Dr. Sunil, NIT Hamirpur (HP)

Multiplying and dividing by  x 2  3x  1  x 2  2 x  8  , we get


 

 x 2  3x  1  x 2  2 x  8   x 2  3x  1  x 2  2 x  8 
   
L.H.S.  Lt 
x   x 2  3x  1  x 2  2 x  8 
 

 Lt
x 2
 
 3x  1  x 2  2 x  8 
x 
x2
 3x  1  x 2
 2x  8 
5x  7
 Lt .
x 
x 2
 3x  1  x 2
 2x  8 
Divide the numerator and denominator by x, we get
7
5
x 50 5 5
L.H.S.  Lt    = R.H.S.
x  3 1 2 8 1 0  0  1 0  0 11 2
1  2  1  2
x x x x
This completes the proof.

1  4x  5  2x 1
Q.No.2.: Prove that Lim  .
x 2 x2 3
1  4x  5  2 x 0
Sol.: L.H.S.= Lim . [ form]
x 2 x2 0
Multiplying and dividing by 1  4 x  5  2x , we get

1  4x  5  2x 1  4x  5  2x
L.H.S.= Lim 
x 2 x2 1  4x  5  2x
1  4x   5  2x   Lim 2 x  2 
x  2 x  2  1  4 x  5  2 x  x  2 x  2  1  4 x  
 Lim
5  2x
2 2 1
 Lim
x 2  1  4x  5  2x 
  = R.H.S.
6 3
This completes the proof.
5 x
Q.No.3.: Prove that Lim  5 .
x 5 6x  5  4 x  5
5 x
Sol.: L.H.S.= Lim .
x 5 6x  5  4x  5
Indeterminate forms: Problems solved by using different algebraic laws 3
Prepared by: Dr. Sunil, NIT Hamirpur (HP)

Multiplying and dividing by 6 x  5  4 x  5 , we get

5x 6x  5  4 x  5
L.H.S.= Lim 
x 5 6x  5  4x  5 6x  5  4 x  5

 Lim
5  x  6x  5  4x  5   Lim 5  x  6x  5  4x  5 
x 5 6x  5  4x  5 x 5  25  x 

 Lim
 6x  5  4x  5

55 
 5  R.H.S.
x 5 2 2
This completes the proof.

12  22  32  ......  x 2
Q.No.4.: Evaluate Lim .
x  x3
12  22  32  ......  x 2 x x  12 x  1  
Sol.: Lim 3
 Lim 3   form
x  x x  6x
 1  1
x 3 1   2  
x  x 1
 Lim  3
 . Ans.
x  6x 3

Q.No.5.: Evaluate Lim


 n3 .
n  n4

 n n  1 
2 2
 1
n 4 1  
n 3
Sol.: Lim 4  Lim 
 2  
 Lim
n n  1
2 2
n 1
 Lim  4   . Ans.
n  n n  4 n  4 n  4
n 4n 4n
x3  a 3
Q.No.6.: Evaluate Lim .
x a x2  a2

Sol.: Since Lim


x3  a 3
 Lim
x  a x 2  ax  a 2 
x a x2  a2 x a x  a x  a 

 Lim
x 2
 ax  a 2 
x a x  a 
3a 2 3
  a. Ans.
2a 2
*** *** *** *** ***
*** *** ***
***
Differential Calculus
Indeterminate Forms
Cauchy’s Rule or L’Hospital’s Rule

Prepared by:
Dr. Sunil
NIT Hamirpur (HP)
(Last updated on 01-08-2009)
Latest update available at: http://www.freewebs.com/sunilnit/

(51 Solved problems and 00 Home assignments)


0 
(ii) Indeterminate forms-Problems of , , 0.
0 
Cauchy’s Rule or L’Hospital’s Rule:
Suppose we are interested to find the value of
 f x  
 x   at x  a , where f x x a  f a   0 (i)
 
x x a  a   0 . (ii)

 f x   0
Then   is of the form .
 x   x a 0

Then by L’Hospital’s Rule, “we differentiate the numerator and denominator w.r.t. x
0
separately. If once again, we find indeterminate form , we have further repetition of the
0
process till we get some definite result”.
 f x   f x 
Proof: The limiting value of    Lt
 x   x a x a x 
Indeterminate Forms- 0 ,  , 0   Prepared by: Dr. Sunil, NIT Hamirpur (HP) 2
0 

 f x  
Putting x  a  h in Lt   , we have when x  a then h  0
x a  x  

 f x   f a  h 
 Lt    Lt
x  a  x   h  0 a  h 

Using Taylor’s Theorem, we get

h2
f a   hf  a  
f  a   ........
f a  h  2 !
 Lt  Lt
h 0 a  h  h 0 h2
a   h  a     a   .....
2!

h2
hf  a  f  a   ........
2!  f a   0 and 
 Lt a   0 from (i) and (ii) 
h 0 h2  
h  a     a   .........
2!
As h  0 , we have
h
f  a   f  a   ........
2 ! f  a  f  x 
Lt   Lt
h   a  x a   x 
  a     a   .........
h 0
2!
In case both f  a  and   a  are zero, the above process can be repeated and we shall get
f x  f  a  f  x 
Lt   Lt and like this we can have further repetition of the
x  a  x    a  x a   x 
process till we get some definite results.

Note: Cauchy’s rule is also be applicable to form.

x cos x  logx  1
Q.No.1: Evaluate Lim .
x 0 x2
x cos x  log1  x  0 
Sol.: Lim 2
.  0 form 
x 0 x  
Apply Cauchy’s Rule (i.e. differentiate the numerator and denominator w.r.t. to x
separately), we get
1
cos x  x sin x 
1 x 0 
 Lim  0 form 
x 0 2x  
Indeterminate Forms- 0 ,  , 0   Prepared by: Dr. Sunil, NIT Hamirpur (HP) 3
0 

Again apply Cauchy’s Rule, we get


1
 sin x  sin x  x cos x 
 Lim
1  x 2 
1
. Ans.
x 0 2 2
e x  e  x  2 cos x
Q.No.2: Evaluate Lim .
x 0 x sin x
e x  e  x  2 cos x 0 
Sol.: Lim  0 form 
x 0 x sin x  

e x  e  x  2 cos x x
 Lim 
x 0 x2 sin x

e x  e  x  2 cos x  x 
 Lim 2  Lim  1
x 0 x  x 0 sin x 

e x  e  x  2 sin x 0 
 Lim  0 form 
x 0 2x  

e x  e  x  2 cos x 1  1  2
 Lim   2 . Ans.
x 0 2 2

xx  x
Q.No.3: Evaluate Lim .
x 1 1  x  log x

xx  x 0 
Sol. Lim  0 form 
x 1 1  x  log x  
Apply Cauchy’s Rule, we get
x x 1  log x   1 0 
 Lim  0 form 
x 1 1  
0 1
x
 Let y  x x 
 
x x 1 / x   1  log x x x 1  log x  log y  log x x  log x 
 Lim  
x 1
00 2
1 Differentiate w. r. t. to x 
x  dy 
  y1  log x   x 1  log x 
x
 dx 
 2 Ans.

ae x  b cos x  ce  x
Q.No.4: Find the values of a, b and c so that Lim  2.
x 0 x sin x
Indeterminate Forms- 0 ,  , 0   Prepared by: Dr. Sunil, NIT Hamirpur (HP) 4
0 

ae x  b cos x  ce  x
Sol.: Lim .
x 0 x sin x
0
This is of form, if a  b  c  0 . (i)
0
Apply Cauchy’s Rule, we get
ae x  b sin x  ce  x
Lim
x 0 x cos x  sin x
0
This is of form, if a  c  0 . (ii)
0
ae x  b cos x  ce  x ae x  b cos x  ce  x a  b  c
 Lim  Lim =  2 (given)
x  0 x  sin x   cos x  cos x x  0  x sin x  2 cos x 2
 a  b  c  4. (iii)
Solving (i), (ii) and (iii), for a, b, c, we get
a = 1, b = 2, and c = 1. Ans.
Q.No.5: Evaluate Lim log x sin x .
x0

log e sin x  
Sol.: Lt .   form 
x 0 log e x  
Applying Cauchy’s Rule, we get
1
 cos x
sin x x  x 
 Lt
1
 Lt x cot x = Lt
x  0 tan x
 1 . Ans.  xLt  1
x 0 x 0  0 tan x 
x

Q.No.6: Evaluate Lt sec . log x .
x 1 2x

Sol.: Lt sec . log x   0form
x 1 2x
log x 0 
 Lt .  0 form 
x 1     
cos 
 2x 
Applying Cauchy’s Rule, we get
Indeterminate Forms- 0 ,  , 0   Prepared by: Dr. Sunil, NIT Hamirpur (HP) 5
0 

1
x 2  1 x 2 2 x 2
 Lt  Lt  Lt  . Ans.
x 1      1  x 1  x  x 1    
 sin       2    x  sin    sin  
 2x  2  x   2x   2x 
1 
Q.No.7: Evaluate Lt  2  cot 2 x  .
x 0 x 

x3 2x5
Sol.: We know that tan x  x    ........
3 15

1 1  1  x 3 2x 5  
2

 Lt  2    Lt  2   x  3  15  ...... 
x 0 x tan 2 x  x 0  x   

1  
2
2  x2 2 4

 Lt  2  x 1  
 x  ...... 
x 0 x
  3 15  

1 1   x2 2 4  
 Lt  2  2 1  2  x  .....   terms of higher powers of x 
x 0 x
 x   3 15  

1 1 2  2
 Lt  2  2   terms containing x   . Ans.
x 0  x x 3  3
 1 
Similar Problem: Evaluate Lt  cot 2 x  2     form.
x 0
 x 

 1   1 1  x 2  tan 2 x
Sol.: Lt  cot 2 x  2   Lt     Lt
x 0
 x  x 0  tan 2 x x 2  x 0 x 2 tan 2 x
2
x 2  tan 2 x  x 
 Lt  
x 0 x4  tan x 
x 2  tan 2 x 2  x 
 Lt 1  xLt  1
x 0 x4 0 tan x

x 2  tan 2 x 0 
 Lt  form 
x 0 x4 0 

2x  2 tan x sec 2 x 2x  2 tan x 1  tan 2 x 


 Lt  Lt
x 0 4x 3 x 0 2x 3
x  tan x  tan 2 x 0 
 Lt  form 
x 0 2x 3 0 
Indeterminate Forms- 0 ,  , 0   Prepared by: Dr. Sunil, NIT Hamirpur (HP) 6
0 

1  sec 2 x  3 tan 2 x sec 2 x


 Lt
x 0 6x 2

 Lt
 
1  1  tan 2 x  3 tan 2 x 1  tan 2 x  
2
x 0 6x
1  1  tan 2 x  3 tan 2 x  3 tan 4 x
 Lt
x 0 6x 2
4 tan 4 x  3 tan 4 x
 Lt
x 0 6x 2
2
4  3 tan 2 x  tan x 
  Lt  
x 0 6  x 
4  0 3  4 2
 1   . Ans.
6 6 3

Q.No.8: Find the value of Lt


x 3.e x
4
/4
 sin 3 / 2 x 2  .
7
x 0 x
0
Sol.: As x  0 , the required limit takes the indeterminate form . The denominator
0

here is x 7 and the application of Cauchy’s Rule will required us to differentiate the
nominator and denominator at least seven times to come to the true value of the limit,
which will be cumbersome.
We therefore, use the method of expansion by Macaulurin’s Theory, which is
very convenient.

Thus, using the expansion e x


x4 2
x 4 1  x 4  4 8
 ..........  1  x  x  ..........
e4  1 
4 2!  4 
 4 32

And using the series for sin x

   
3/ 2 3/ 2
 2 3 2 5 
sin x 
2 3/ 2
 x 

2 x
3!

x
5!
 ......

3
 x 1 
x4
6

 ........ ,
   

Now using Binomial Theorem, we get


Indeterminate Forms- 0 ,  , 0   Prepared by: Dr. Sunil, NIT Hamirpur (HP) 7
0 

 3  x4 
sin x 
2 3/ 2
 x 3 1  

  x4
 .......   .....  x 3 1 


 ........
 2  6    4 

 x 4 x8
3
 3
 x4 
x 1    .......  x 1   .........
 Lt
x 3 .e x
4
/4
 sin 3 / 2 x 2   Lt  4 32   4 
x 0 x 7 x 0 x7
1  1
 Lt   terms containing x   . Ans.
x 0  2  2

e x  1  sin x
Q.No.9.: Evaluate Lim .
x 0 x2
e x  1  sin x 0 
Sol.: Lim 2
.  0 form
x 0 x  
 Using L’Hospital Rule, we get

e x  cos x 0 
 Lim .  0 form
x 0 2x  
 Again using L’Hospital Rule, we get

e x  sin x 1
 Lim  .Ans.
x 0 2 2
e ax  e  ax
Q.No.10.: Evaluate (a) Lim ,
x  0 log 1  bx 

e x  e  x  2 x
(b) Lim .
x 0 tan x  x
ea x  e  a x 0 
Sol.: (a) Lim .  0 form
x  0 log 1  bx   
 Using L’Hospital Rule, we get

 Lim
ea x a   e a x  a 
 Lim

a ea x  e a x 1  bx  2a
 .Ans.

x 0 1 x 0 b b
.b
1  bx

e x  e  x  2 x 0 
(b) Lim .  0 form
x 0 tan x  x  
 Using L’Hospital Rule, we get
Indeterminate Forms- 0 ,  , 0   Prepared by: Dr. Sunil, NIT Hamirpur (HP) 8
0 

e x  e x  1  2 ex  e x  2 0 
 Lim 2
 Lim 2
.  0 form
x 0 sec x  1 x 0 sec x  1  
 Using L’Hospital Rule, we get

e x  e  x  1 e x  e x 0 
 Lim  Lim .  0 form
x 0 2 sec x. sec x tan x x 0 2 sec 2 x tan x  
 Using L’Hospital Rule, we get

e x  e  x  1
 Lim
x 0 
2 sec2 x sec2 x  tan x.2 sec x sec x tan x .
ex  e x 11
 Lim
x 0  4
2 sec x  2 sec x tan x 2 2
 
21  0
 1 . Ans.

x  tan x
Q.No.11.: Evaluate Lim .
x 0 x3
x  tan x 0 
Sol.: Lim 3  0 form
x 0 x  
 Using L’Hospital Rule, we get

1  sec2 x 0 
= Lim 2  0 form
x 0 3x  
 Again using L’Hospital Rule, we get

 2 sec x. sec x tan x  sec 2 x tan x 0 


= Lim = Lim  0 form
x 0 6x x 0 3x  
 Using L’Hospital Rule, we get

sec 2 x. sec 2 x  tan x.2 sec x. sec x tan x


= Lim 
x 0 3
sec 4 x  2 sec 2 x tan 2 x 1 0 1
= Lim     . Ans.
x 0 3 3 3

xe x  log1  x 
Q.No.12.: Evaluate Lim .
x 0 x2
xe x  log1  x  0 
Sol.: Lim 2  0 form
x 0 x  
 Using L’Hospital Rule, we get
Indeterminate Forms- 0 ,  , 0   Prepared by: Dr. Sunil, NIT Hamirpur (HP) 9
0 

1
x.e x  e x .1 
x 1 0 
 Lim  0 form
x 0 2x  
 Again using L’Hospital Rule, we get
1 1
x.e x  e x .1  e x  2
0.e0  e 0 .1  e 0 
( x  1) (1  0) 2 3
 Lim   . Ans.
x 0 2 2 2

x3
sin x  x 
Q.No.13.: Evaluate Lim 6 .
x 0 x5

x3
sin x  x 
6 0 
Sol.: Lim 5  0 form
x 0 x  
 Using L’Hospital Rule, we get

3x 2
cos x  1 
6 0 
 Lim 4  0 form
x 0 5x  
 Again using L’Hospital Rule, we get
 sin x  x 0 
 Lim 3  0 form
x 0 20x  
 Again using L’Hospital Rule, we get
 cos x  1 0 
 Lim 2  0 form
x 0 60x  
 Again using L’Hospital Rule, we get
sin x 0 
 Lim  0 form
x 0 120 x  
 Again using L’Hospital Rule, we get
cos x 1
 Lim  . Ans.
x  0 120 120

sin 2 x  2 sin 2 x  2 sin x


Q.No.14.: Evaluate Lim .
x 0 cos x  cos2 x
sin 2x  2 sin 2 x  2 sin x 0 
Sol.: Lim 2  0 form
x 0 cos x  cos x  
Indeterminate Forms- 0 ,  , 0   Prepared by: Dr. Sunil, NIT Hamirpur (HP)10
0 

 Using L’Hospital Rule, we get


2 cos 2 x  4 sin x cos x  2 cos x
 Lim
x 0  sin x  cos x sin x
2 cos 2x  2 sin 2 x  2 cos x 0 
 Lim  0 form
x 0  sin x  sin 2 x  
 Again using L’Hospital Rule, we get
 4 sin 2 x  4 cos 2 x  2 sin x  0  4.1  0
 Lim   4 . Ans.
x 0  cos x  2 cos 2 x 1 2
log x
Q.No.15.: Evaluate Lim n  0 .
x  xn
log x  
Sol.: Lim n   form
x  x  
 Using L'Hospital Rule, we get
1
1 1
= Lim xn 1  Lim n   0 . Ans.
x   nx x   nx 
x  sin x
Q.No.16.: Evaluate Lt .
x 0 tan 3 x
x  sin x
Sol.: Lt
x 0 tan 3 x
Applying Cauchy’s rule, we get
1  cos x
Lt
x 0 3 tan 2 x.sec2 x

Again applying Cauchy’s rule, we get


sin x sin x
Lim
x 0 3sec 4
 
x.2 tan x  tan x 2 sec x 3 2
  Lim
x 0 6 tan x sec 2

x tan 2 x  sec2 x 
1 1
 Lim
x 0 6 sec 3
 2
x tan x  sec x 2
 
6
. Ans.

 1 
Q.No.17.: Evaluate Lt  2  cos ec 2 x  .
x 0 x 
 1   1 
Sol.: Lt  2  cos ec 2 x   Lt  2  sin  2 x 
x 0 x  x 0 x 
Indeterminate Forms- 0 ,  , 0   Prepared by: Dr. Sunil, NIT Hamirpur (HP)11
0 

 2
   2 
 1  x3 x5   1 1  x2 x4
1 
 
 Lim 2  x    ...............   Lim 2  2   ............... 
   
x 0  x
  3! 5 !   x 0  x x  3! 5 !  
 1 1   x2 x4  
 Lim  2  2 1  2   ............... 
x 0  x x   3! 5 !  
   

 2 2x 4  2 1
 Lim   ...............     . Ans.
 
x 0
 3! 5 !  3! 3

1 
Q.No.18.: Evaluate Lt   cot x  .
x  0 x 
1   1 cos x  sin x  x cos x sin x  x cos x
Sol.: Lt   cot x   Lt     Lt  Lt
x  0 x  x 0 x sin x  x 0 x sin x x 0  sin x 
x2 
 x 
sin x  x cos x
 Lt
x 0 x2
0
Applying Cauchy’s rule,  above equation is   form, we get
0
cos x  cos x  x sin x sin x
 Lt  Lt  0 . Ans
x 0 2x x 0 2

a x
Q.No.19.: Evaluate Lt   cot  .
x  0 x a

 x  x x
 cos  a sin  x cos
a x a a a a
Sol.: Lt   cot   Lt     Lim
x  0 x a  x  0 x sin x  x 0 x
  x sin
 a  a
x x x x x x
a sin  x cos a sin  x cos a 2 sin  ax cos
 Lim a a  Lim a a  Lim a a
x 0 x x 0 x 2 x 0 x 2
sin
x2 a a
a x
a
0
Applying L hospital’s rule,  above equation is   form, we get
0
Indeterminate Forms- 0 ,  , 0   Prepared by: Dr. Sunil, NIT Hamirpur (HP)12
0 

x x x x
a cos  a cos  x sin x sin
 Lim a a a  Lim a  0 . Ans.
x 0 2x x 0 2 x

1
cot x 
Q.No.20.: Evaluate (a) Lim x , (b) Limx  1 tan  x .
x 0 x x 1 2
1
cot x 
x   
Sol.: (a) Lim  0 form 
x 0 x  
cos x 1

 Lim sin x x  Lim x cos x  sin x  Lim  x cos x  sin x . x 
x 0 x x 0 x 2 sin x x 0 
 x3
  sin x 
 
 x cos x  sin x   x  0 
 Lim  .  Lim   1.  0 form
x 0  x3  x  0  sin x 
  
 Using L’Hospital Rule, we get
  x sin x  cos x  cos x   sin x  1
 Lim  2   Lim     .Ans.
x 0  3x  x  0  3x  3
x
(b) Limx  1 tan (0  )form
x 1 2

 Lim
x  1 0 
x  0 form
x 1
cot  
2
 Using L’Hospital Rule, we get
1 1 2
 Lim    . Ans.
x 1   x  
 cos ec 2 
2 2 2

Q.No.21.: Evaluate Lt y 2 1  e 2gx / y  .


2

y   

Sol.: Lt y 2 1  e 2gx / y 


2

y   
1
Substituting Lim y 2  Lim
y  n 0 n2
1  e 1gxn 2 
 Lim  2

n 0 n
Indeterminate Forms- 0 ,  , 0   Prepared by: Dr. Sunil, NIT Hamirpur (HP)13
0 

 Using L’Hospital Rule, we get


2
2gx.2n.e  2gxn
Lim  2gx . Ans.
n 0 2n
 x
Q.No.22.: Evaluate Lim log 2   cot x  a  .
x a  a

 x
Sol.: Lim log 2   cot x  a   (0   )form
x a  a

 x
log 2  
0 
 Lim 
a
x a tan x  a   0 form
 
 Using L’Hospital Rule, we get
1  1
 
 x  a
2  
1
 Lim 
a
  . Ans.
x a sac x  a 
2 a

x1 / 2 tan x
Q.No.23.: Evaluate Lt .
x 0
e  1
x 3/ 2

tan x
1/ 2 x3 / 2
x tan x x x1 / 2 tan x
Sol.: Lt  Lt  Lim
x 0 x

e 1
3/ 2

x 0 x
e 1
3/ 2 x 0 
x 

x 2

x 
3 
 ........... 
3/ 2

 2 ! 3 ! 
 

x1 / 2 tan x 1
 Lim  Lim  1 . Ans.
x 0  2  x 0  2 
x x
x.x1 / 2 1    ........... 1  x  x  ........... 
 2 ! 3!   2 ! 3! 
   

e1 / n  e 2 / n  3 / n .............  e n / n
Q.No.24.: Prove that Lim  e 1
n  n
e1 / n  e 2 / n  3 / n .............  e n / n
Sol.: Taking L.H.S.  Lim
n  n
Here, the series given in numerator is in geometric progression,

where, first term, a  e1 / n ,

common ratio, r  e1 / n  1 ,
Indeterminate Forms- 0 ,  , 0   Prepared by: Dr. Sunil, NIT Hamirpur (HP)14
0 

number of terms = n.
The sum of series given in numerator is,  n


a r 1n e1 / n  e1 / n  1
 
n
 
  e e  1
1/ n
 n  
r 1 e1 / n  1 e1 / n  1
e1 / n e  1 e1 / n e  1
 Lim
So, L.H.S.= Lim
n  e1/ n
1 n n   
1 
1

1
 ......... 
1  
  1.n
2
 1!n 2 !n n !n n  

e1/ n e  1 e1 / n e  1
 Lim  Lim
n  1 1 1  n   1 1 
  2
 .........  .n 1   .........  
 n 2 !n n !n n   2 !n n !n n -1 

e0 e  1
 Lim  e  1 . = R.H.S.
n  1  0  ................

Hence this completes the proof.

12  2 2  32  ......  x 2 1
Q.No.25.: Prove that Lim 3
 .
x  x 3

Sol.: Lim
12  22  32  ......  x 2
 Lim
 x2  Lim
x x  12x  1
3 3
x  x x  x x  x3
 1  1
x 3 1   2  
x  x 2 1
 Lim    .Ans.
x  x3 6 3

ax 1
Q.No.26.: Prove that Lt sin 1 cos ec a 2  x 2  .
x a ax 2a

ax
sin 1
ax ax
Sol.: Taking L.H.S.  Lt sin 1 cos ec a 2  x 2  Lim
x a ax x a sin a  x 2
2

0
The given equation is in the form   . So, apply “Cauchy’s Rule” ( i. e. differentiate
0
numerator and denominator w. r. t. x separately)
Indeterminate Forms- 0 ,  , 0   Prepared by: Dr. Sunil, NIT Hamirpur (HP)15
0 

1
.
1
.
a  x  1  a  x 1
 ax 
2
2
ax a  x 2
1   
 ax
 ax 
L.H.S.  Lim
1
. 2 x 
n
cos a 2  x 2 .
2 2
2 a x

ax a  x  2a 
. .
2 x 2 a  x a  x 2
 Lim
x a
cos a 2  x 2 .
 x 
a  x a  x 

 Lim
x  a . 2a . a  x . a  x  Lim
a. a  x
x a x. 2x .2 a  x .a  x 2 . cos a 2  x 2 x a x. 2 x .a  x . cos a 2  x 2

a . 2a 1
  = R.H.S.
a. 2a .2a . cos 0 2a
Hence this completes the proof.

x y  yx
Q.No.27.: Evaluate Lim .
xy x x  yy

x y  yx 0 
Sol.: Lim x y  0 form
xy x y  
 Using L’Hospital Rule, we get

y y y 
x y    y x log y  y    y y log y 
y
x y y y 1  log y  1  log y
 Lim x    y  y  . Ans.
x  y x 1  log x   0 y 1  log y  y 1  log y  1  log y
Q.No.28.: Determine a, b, c such that
a  b cos   c sin 
Lt  1.
0 5
a  b cos   c sin 
Sol.: Lt 1
0 5
0
The given equation is in the form   . So, apply “Cauchy’s Rule” , we get
0
a  b cos   0  b sin   c cos 
Lt
0 5 4
Indeterminate Forms- 0 ,  , 0   Prepared by: Dr. Sunil, NIT Hamirpur (HP)16
0 

a  b  c  0  a  b  c . (i)
Again apply “Cauchy’s Rule” , we get
0  b sin   b sin   b cos   c sin 
Lt
0 20. 3
0
The above equation is in the form   . So, apply “Cauchy’s Rule” , we get
0
 2b cos   b cos   b sin   c cos 
Lt
0 60.2
 3b  c  0  c  3b . (ii)
0
The above equation is in the form   . So, apply “Cauchy’s Rule” , we get
0
2b sin   b sin   b sin   b cos   c sin 
Lt
0 120.
 4b sin  b cos  c sin   4b b c
       1 (given)
120  120. 120.  120 120 120
 5b  c  120 . (iii)
 From (ii) and (iii), we get
b  60, c  180, a  120. Ans.
x 1  a cos x   b sin x
Q.No.29.: Find the values of a and b such that Lim  1.
x 0 x3
  x2 x4   x3 x5 
x 1  a 1    ......   b x    ...
   
x 1  a cos x   b sin x   2! 4!   3! 5! 
Sol.: Lim  Lim
x 0 3 3
x x 0 x

1  a  b x    a  b  x 3  ...
 2 6 0 
 Lim  0 form
x 0 x3  
Since the given limit is equal to 1, we must have
1 a  b  0 (i)
a b
and    1. (ii)
2 6
Solving (i) and (ii), we get
Indeterminate Forms- 0 ,  , 0   Prepared by: Dr. Sunil, NIT Hamirpur (HP)17
0 

5 3
a , b   . Ans.
2 2
 
log   
 2 1  2 cos x
Q.No.30.: Evaluate (a) Lim , (b) Lim .
 tan     

2
 
3
sin x  
 3

 
log   
 2  
Sol.: (a) Lim .   form

 tan   
2

 Using L’Hospital Rule, we get


1

x
2 cos 2 x 0 
 Lim  Lim  0 form
2
 sec 
 
 x 
  
2 2 2
 Using L’Hospital Rule, we get
2 cos x  sin x 
 Lim  Lim  sin 2x   0 . Ans.

 1 

2 2

1  2 cos x 2 sin x
(b) Lim  Lim
 sin 
      
3  x   3
cos x  
 3  3
 Using L’Hospital’s Rule, we get

 3
2. 

2
    3 . Ans.
cos 0
logx  a 
Q.No.31.: Evaluate Lt
x a log e x   ea .
 1 
 
logx  a   xa e x  ea
Sol.: Lt
x a log e x   ea    Lim x
 e x  x a e x  a 
 
 e x  ea 
 
 Using L’Hospital Rule, we get
Indeterminate Forms- 0 ,  , 0   Prepared by: Dr. Sunil, NIT Hamirpur (HP)18
0 

ex ea
 Lim   1 . Ans.
x a e x x  a   e x 0  ea
log sin 2 x
Q.No.32.: Evaluate Lt .
x 0 sin x

log sin 2 x log2 sin x cos x  log2 sin x   log cos x


Sol.: Lt  Lt  Lt
x 0 sin x x 0 sin x x 0 sin x
 log2 sin x  log cos x  log2 sin x  log cos x
 Lim     Lim  Lim
x 0  sin x sin x  x 0 sin x x 0 sin x

0
The second limit is of form and can be evaluated with the L’ Hospital’s rule
0
log cos x  tan x
 Lim  Lim  0 . Ans.
x  0 sin x x  0 cos x

Q.No.33.: Evaluate Lim x log sin x .


x0

Sol.: Lim x log sin x [ 0   form]  Lim log x  


x0  x 0 

log sin x  
 Lim   form 
x 0 1  
x
Using L’Hospital Rule, we get
cos x
x2 0 
 Lim sin x   Lim x 2 cot x   Lim  0 form
x 0 1 x 0 x  0 tan x  
 2
x
 Using L’Hospital Rule, we get
2x
 Lim  0 . Ans.
x 0 sec2 x

Q.No.34.: Evaluate Lim x log x .


x 0

Sol.: Lim x log x [ 0   form]


x0

log x  
 Lim   form
x0 1  
x
 Using L’Hospital Rule, we get
Indeterminate Forms- 0 ,  , 0   Prepared by: Dr. Sunil, NIT Hamirpur (HP)19
0 

1
 Lim x  Lim x   0 . Ans.
x 0 1 x 0
 2
x
1
Q.No.35.: Evaluate Lt x tan   .
x  x

1   1 
sin   sin      x  
1  x 
   Lt
x 1  1 
Sol.: Lt x tan   Lt x   Lt   1 . Ans.
x  x x   1   1 0  1    x   1   0
cos   x    cos  x 
x   x   x

 1 
Q.No.36.: Evaluate Lt  a x  1 x .
x   
 

 1 
Sol.: Lt  a x  1 x
x   
 
1
Let  y  x  , y  0 .
x
 1 
Then Lt  a x  1 x = Lt
ay 1 
 Lt
a y log a 
 log a . Ans.
x    y 0 y y 0 1
 
A  sin kx k 
Q.No.37.: Evaluate Lt  .
x 0 x 2 
 sin x  
A  sin kx k  A   sin kx  k sin x 
Sol.: Lt    Lt
x  0 x 2  sin x   x 0 x 2 
  sin x 

 k 3 x 3 k 5 x 5   k3x 3 k5 x 5 
  kx    ........    k x    ........  
A 3! 5!   3! 5! 
 Lt 2     

 2 
  x   x   x .................... 
x 0 x 4 3 6 5
 
  3 ! 5 !  
   

  k 2x k 4x3    2 x k 4 x 5 
     ........    
  3!
  ........  

Ak   3 ! 5!   5! 
 Lt 2  
x 0   2 
  x   x   x .................... 
4 3 6 5
 
  3! 5!  
   
Indeterminate Forms- 0 ,  , 0   Prepared by: Dr. Sunil, NIT Hamirpur (HP)20
0 

Using L’Hospital Rule, we get


  k 2 3k 4 x 2  2 5 4 x 4 
   ............    ............ 
Ak 3 ! 5! 3! 5!
 Lt  
x 0   3 x
2 2
5 x
4 4

 1  ............. 
 3! 5! 


Ak   2 k 2  Ak 2
  
  6 6  6 !

  k 2 . Ans. 
 x 1 
Q.No.38.: Evaluate Lim  .
x 1  x  1 log x 

 x 1   x log x  x  1 
Sol.: Lim    Lim 
x 1  x  1 log x  x 1  x  1log x 

 Using L’Hospital Rule, we get

   1 
 1  log x  1   
1
Lim    Lim x   . Ans.
x 1   x  1   x 1  1  1  2

 x    log x   2 
  x x

 f  x  1 
Q.No.39.: Find Lt   .
x a  f x   f a  x  a 

 f  x  1   f  x x  a   f x   f a  
Sol.: Lt     Lt  
x a  f x   f a  x  a  x a  f x   f a x  a  
 f  x .x  f  x .a  f x   f a  
 Lt  
x a  f  x .x  f a x  f x a  f a .a 

 x.f  x   f  x   af x   f  x   f  a  
 Lt  
x a  x.f  x   f x   xf  a   f a   af  x   af  a  

 x.f  x   f  x   af  x   f  a  


 Lt  
x a  x.f  x   f  x   f  x   xf  a   f  a   f  a   af  x   af  a  

Applying the limits, we get


 a.f  a   f  a   af  a   f  a  
 
 a.f  a   f  a   f  a   af  a   f  a   f  a   af  a   af  a  
2f  a  f  a 
  . Ans.
4f  a  2f  a 
Indeterminate Forms- 0 ,  , 0   Prepared by: Dr. Sunil, NIT Hamirpur (HP)21
0 

1 1 
Q.No.40.: Evaluate Lim   x  .
x 0  x e  1
1 1 
Sol.: Lim   x     form
x 0  x e  1
 (e x  1)  x  0 
= Lim    0 form
x  0  x (e x  1)   
 
 Using L’Hospital Rule, we get
 ex  1  ex  1 0 
= Lim  x   Lim  0 form
x  0  x.e  (e x  1).1 x
  x 0 ( x  1)e  1  

 Using L’Hospital Rule, we get

ex 1 1
 Lim x x
  .Ans.
x 0 ( x  1).e  e 11 2

1 1 
Q.No.41.: Evaluate Lim   2 log1  x  .
x 0  x x 
1 1 
Sol.: Lim   2 log1  x  (  )form
x 0  x x 
1 1  x 2 x3  1 1  1
 Lim   2  x    ......   Lim   x  ...  . Ans.
x 0  x
 x  2 3   x 0  2 3
  2

Q.No.42.: Prove that Lim


1  x 1 / x  e   e .
x 0 x 2

1  x 1 / x  e . 0 
1  x 1 / x  e
Sol.: Lim
x  0 form  Lim 
x 0   x  0

We first evaluate 1 x 1 / x .

Let y  1  x 1 / x . 

1 1 x 2 x3   x x2 
log y  log1  x    x    ..   1      ..   1  z ,
x x  2 3 

 2 3


x x2
where z     ... .
2 3
Indeterminate Forms- 0 ,  , 0   Prepared by: Dr. Sunil, NIT Hamirpur (HP)22
0 

1 z z  z2 
ye  e.e  e 1  z   .......
 2! 

  2  1  x x2 
2 
x x
 e 1      ...      ...  ......
  2 3  2!  2 3
 

 
 
 x 11 2 
 e 1   x  ...... .
 2 24 
 x x2 
e1    ...  e
1  x 1 / x e  2 3 
   1 11  1
 Lim  Lim  Lim e   x  ...   e .Ans.
x 0 x x 0 x x 0  2 24  2
This completes the proof.

e x sin x  x  x 2
Q.No.43.: Evaluate Lt .
x 0 x 2  x log1  x 

e x sin x  x  x 2
Sol.: Lt
x 0 x 2  x log1  x 

Using the expansion of e x sin x and log1  x  , we get

 2 3  3 
1  x  x  x  ............  x  x  ............   x  x 2
 2 ! 3!  3! 
 Lt   
x 0  x 2
x 3 
x 2  x  x    ............. 
 2 3 
 
 3   x3 
 x  x 2  x  x 4  ............   x  x 2   x 4  ............ 
 3  3  2
 Lt    Lt    . Ans.
x 0  x 3
x 4  x  0  1 x  3
2  2
x  x    .............     .............
 2 3   2 3 
 

1  x 1/ x  e  ex
Q.No.44.: Prove that Lim 2  11e .
x 0 2 24
x

1  x 1 / x  e  ex 0 
Sol.: Lim 2 .  Lim1  x 1 / x  e
2  0 form  x 0 
x 0 x  
Indeterminate Forms- 0 ,  , 0   Prepared by: Dr. Sunil, NIT Hamirpur (HP)23
0 

We first evaluate 1 x 1 / x .

Let y  1  x 1 / x .

1 1 x 2 x3   x x2 
 log y  log1  x    x    ..   1      ..   1  z ,
x x  2 3 

 2 3


x x2
where z     ... .
2 3
 z2 
 y  e1 z  e.e z  e 1  z   .......
 2! 
  2  1  x x2 
2 
 x x

 e 1   
 ...      ...  ......

  2 3  2!  2 3
 

 
 
 x 11 2 
 e 1   x  ...... .
 2 24 

 x 11  ex
1  x 1 / x  e  ex e1   x 2  ...  e 
 Lim 2  Lim  2 24  2
x 0 x2 x 0 x 2

 
 11  11
 Lim  e  terms containing powers of x   e.
x  0 24
  24
 
This completes the proof.
tanh x  2 sin x  x
Q.No.45.: Evaluate Lim .
x 0 x5
tanh x  2 sin x  x
Sol.: Lim
x 0 x5
 3 5 
 x  x  x  .........
 
  2 x  x  x  .........  x
3! 5! 3 5

 x2 x4   3! 5! 
1    .........  
 2! 4! 
 Lt  
x 0 x5
Indeterminate Forms- 0 ,  , 0   Prepared by: Dr. Sunil, NIT Hamirpur (HP)24
0 

 3 5   3 5  2 4   2 4 
 x  x  x  .........  2 x  x  x  .........1  x  x  .........  x1  x  x  .........
 3! 5!   3! 5!     
 Lt     2 ! 4 !   2! 4! 
x 0  x 2
x 4 
x 5 1    .........
 2! 4! 
 
1  2  1x   1  1  1  1 x 3   1  2  2  2  1 x 5  ...x 6  ............
6 3 2  120 24 12 120 24 
 Lt
x 0  x2 x4 
x 5 1    .........
 2! 4! 
 
14 5
x  ....x 6  ..............
14 7
 Lt 120   . Ans.
x 0 x5 120 60

x sin(sin x )  sin 2 x
Q.No.46.: Evaluate Lim .
x 0 x6
Sol.: We make use of one standard series to obtain this limit
x3 x5
sin x  x    .........
3! 5 !
2
 x3 x5   x3 x5 
x sin x    ......   x    ......
x sin(sin x )  sin 2 x  3! 5!   3! 5 ! 
Now Lim = Lim    
x 0 6 x 0 6
x x
  1 
3
 
5 
 x3 x5 x 3
x 5
1 x 3
x 5

x x    ......   x    ......   x    ......  ....
 3! 5!     
  3!  3! 5 !  5!  3! 5 !  

2
 x3 x5 
 x    ......
 3! 5 ! 
 Lim  
x 0 x6
  x4  x2 x4 
3 6  
5 
 2 x 4 x 6  1  x x 2
x 4

 x    ......   ......   1    ......   ....
3 ! 5 !  3!  3! 5!  
5!  3 ! 5 ! 
     

2
 x3 x5 
x    ...... 
 3! 5! 
 Lim  
x 0 x6

Expanding by Binomial expansion, we get


Indeterminate Forms- 0 ,  , 0   Prepared by: Dr. Sunil, NIT Hamirpur (HP)25
0 

 4
x6  x4   x 2 x 4   x 6   x 2 x 4   
 2 x     ...   
 x    .... 1
   3   ...... 1
   5   ....  .....  ..
3! 5 !  3!   3! 5!   5!   3! 5!   
    

  2 4   x2 x4  
2
2
x x
 x 1 2   ......     ...... 
   3! 5 ! 
  3! 5!    
 Lim
x0 x6
 1 1 1 2 1
x6        .........x7  ....
120 12 120 120 36
 Lim 
x0 x6
1 6
x  .........x7  ....
1
 Lim18 6
 . Ans.
x0 x 18

Q.No.47.: Evaluate Lim



e x sin x  cosh x 2 .
x 0 4
x

Sol.: Lim
 
e x sin x  cosh x 2
x 0 x4
We make use of two standard series to obtain this limit

e x sin x  1  x sin x 
x sin x 2  x sin x 3  x sin x 4  .............
2! 3! 4!
x2 3 4
 1  x sin x  sin x 2  x sin x 3  x sin x 4  .............
2! 3! 4!
Now using expansion of sin x
2
 x3 x5  x2  3 5 
e x sin x
 1  x x    ...........    x  x  x  ...........
 3! 5!  2!  3! 5 ! 
   
3 4
x 3  x3 x5  x4 
  x3 x5 
 x   ...........  x   ...........
3 !  3! 5 ! 
 4 !  3! 5 ! 

2
2
x2 x4

 x 4  x 2 x5 
 1 x 1   ...........  1    ........... 
 6 120  2  6 120 
   
3 4
x 6  x 2 x 4  x8  x 2 x 4 
 1   ...........  1    ........... 
6  6 120  24 
  6 120 

Indeterminate Forms- 0 ,  , 0   Prepared by: Dr. Sunil, NIT Hamirpur (HP)26
0 

Now expanding by Binomial theorem,


 x2 x4  x4   x 2 x5 
e x sin x  1  x 2 1    ...........  1  2   .......... 
 6 120  2   6 120 
    

x6   x2 x4 
 1  3   ..........   ................

6   6 120 
Collecting terms of same type

x4 x6
e x sin x  1  x 2   ............. (i)
3 120
x 2 x 4 x6
cosh x  1     ................
2! 4! 6!

 
cosh 2 x  1 
2 x 2 4 x 4 8x 6
2!

4!

6!
 .......... ......

x4 x6
 1  x2    .......... ...... (ii)
6 90

Lt
e x sin x  cosh 2 x  
4
x 0 x
 4 6   4 6 
1  x 2  x  x  ......  1  x 2  x  x  ......
 3 120   6 90 
 Lt   
4

x 0 x
 x4 x6   x4 x6 
   ......     ......
 3 120   6 90 
 Lt    
4
x 0 x
Neglecting terms having powers more than 4
1 1
x4  
 Lt  3 6   1 . Ans.
x 0 x4 6
2
2 x 2  2e x  2 cos x 3 / 2  sin 3 x
Q.No.48.: Prove that Lim  1 .
x 0 x4
2
2 x 2  2e x  2 cos x 3 / 2  sin 3 x
Sol.: Lim  1
x 0 x4
Indeterminate Forms- 0 ,  , 0   Prepared by: Dr. Sunil, NIT Hamirpur (HP)27
0 

We use sin x series for expansion of sin 3 x


3 3
3

sin x   x 
 x 3 x 5

 3
 x 3 x 5
 ..........  x 1  

 ..........
 3! 5!   6 120 
Using Binomial Theorem
 x 3 x 5
3

 x 1    ..........
 2 40 
Similarly

cos x 3/ 2
 1
x   x   x 
3/ 2 2 3/ 2 4 3/ 2 6
 ................  1 
x3 x6
  ................
2! 4! 6! 2 24
2
2x 2  2e x  2 cos x 3 / 2  sin 3 x
 Lim
x 0 x4
 x4 x6   x3 x 6   x2 x4 
2x 2  21  x 2    ......  21    .........  x 3 1    ........
 2 ! 3!   2 24   2 40 
 Lim      
x 0 4
x
 x 6  x 6   x x3 
  x4   ......    .........  x 4    ........
 3   24   2 40 
 Lim   
4
    1 . Ans.
x 0 x

1  x cos x  cosh x  log(1  x )


Q.No.49.: Evaluate Lim .
x 0 tan x  x
1  x cos x  cosh x  log(1  x )
Sol.: Lim
x 0 tan x  x
 x2 x4   x2 x4   x 2 x3 x 4 

1 x 1   
 ......  1    
 ......  x     ......
 2! 4!   2! 4!   2 3 4 
 Lim      
x 0 3 5
x 2x
x   x........
3 15
 x2 x4   x4 x6   x3 x 4 
x    ......     ......     ......
 2! 4!   4! 6!  3 4 
 Lim   
3 5
  
x 0 x 2x
  x......
3 15
Indeterminate Forms- 0 ,  , 0   Prepared by: Dr. Sunil, NIT Hamirpur (HP)28
0 

Neglecting terms greater than x 3


 x3 x3 
  
 2 3  5
 Lim  3
  . Ans.
x 0 x 2
3
Q.No.50.: The current i in a circuit containing an inductance L, a capacitance C and an
alternator of angular frequency  and maximum e. m. f. E., is given by
E 1
i cos t  cos nt  where

L n 2 2
 n
LC
. Find the limiting form of the

expression for i, when   n .


E 0 
cos t  cos nt 
Sol.: Since Lim
 n 
L n  2 2
  0 form
 
 Using L’Hospital Rule, we get
E cos t  cos nt   Et  sin t 
 Lim
 n  2L
Ecos nt  cos nt   Ent  sin nt 

 2nL
E  0  Ent sin nt  Et
  sin nt . Ans.
 2nL 2L
Q.No.51.: A column of length  has a vertical load P and horizontal load F at the top,
and the transverse deflection is given by

F  tan m  2 P F 3
D  1 , where m  . Show that as P  0 , D  .
P  m 
 EI 3EI
F  tan m  P
Sol.: Given D    1 , where m 2  .
P  m  EI

 m33 2 5 5  

  m   m   ......  
F  tan m  F  3 15  
Now Lim D  Lim  1  Lim    1

P 0 P 0 P  m  P 0 P m
 
 
 

F  m 2 2 2 4 4   F  m 2 2 2 4 4 
 Lim 1   m   ......  1  Lim   m   ......
P 0 P  3 15   P 0 P  3 15 
    
Indeterminate Forms- 0 ,  , 0   Prepared by: Dr. Sunil, NIT Hamirpur (HP)29
0 

F  1 P 2 2  P  4 
2
F P 2  2 P 2 
 Lim       ......  Lim  1   ......
P 0 P  3 EI
 15  EI   P 0 P 3EI  05 EI 

F3  2 P 2  F
3
 Lim 1    ...... 
 3EI .
P 0 3EI 
 05 EI 

F 3
Thus as P  0 , D  .
3EI

*** *** *** *** ***


*** *** ***
***
Differential Calculus
Indeterminate Forms
00 ,  0 , 1

Prepared by:
Dr. Sunil
NIT Hamirpur (HP)
(Last updated on 01-08-2009)
Latest update available at: http://www.freewebs.com/sunilnit/

(22 Solved problems and 00 Home assignments)

Indeterminate forms-Problems of 00 , 0 , 1 :
1
 sin x  x
Q.No.1.: Evaluate Lt   .
x 0 x 

1
 sin x  x
Sol.: Let y  Lt 
x 0 x 
 1

form 
Taking log of both sides, we get

1  sin x 
log 
 sin x  x 1  sin x   x  0 
log y  Lt log   Lt log   Lt  0 form 
x 0  x  x 0 x  x  x 0 x  
Now apply Cauchy’s Rule, we get
 x  x cos x  sin x   x 
log y  Lt     xLt  1
x  0 sin x  x2   0 sin x 
x cos x  sin x 0 
 Lt 2  0 form 
x 0 x  
Indeterminate forms-Problems of 00 , 0 , 1 Prepared by: Dr. Sunil, NIT Hamirpur (HP) 2

Applying Cauchy’s Rule again, we get


cos x  x sin x  cos x  x sin x  sin x
log y  Lt  Lt = Lt 0
x 0 2x x 0 2x x 0 2
log y  0

 y  e0  1 . Ans.

 
1
Q.No.2.: Evaluate Lt 1  x 2 log1 x  .
x 1

  0 form
1
2 log 1 x  0
Sol.: Let y  Lt 1 x
x 1

Taking log of both sides, we get

 
1
log y  Lt log 1  x 2 log 1 x 
x 1

 Lt
1
x 1 log1  x 
. log 1  x 2    
  form
 
Applying Cauchy’s Rule, we get
1
2
 2x  2 x 1  x  2x
1  x  Lt
log y  Lt
x 1 1
 1 x 1 1  x 2
 Lt

x 1 1  x 
1

1 x
loge y  1

 y  e1  e . Ans.
1
 a1x  a 2x x x
 ..........  a n
Q.No.3.: Evaluate Lt   .
x 0 
 n 
1
 a1x
1 form
x x
 a 2x  ..........  a n 
Sol.: Let y = Lt  
x 0 
 n 
Taking log of both sides, we get
1
 a1x  a 2x x x
 ..........  a n
log y  Lt log  
x 0
 n 
Indeterminate forms-Problems of 00 , 0 , 1 Prepared by: Dr. Sunil, NIT Hamirpur (HP) 3

 a x  a 2x  ..........  a nx 
log  1 

 n 
 Lt
x 0 x
0 
 0 form
 
Applying Cauchy’s Rule, we get

log y  Lt
n

a
x x
1 log a1  a 2 log a 2 ...........  a nx log a n 
x 0 a x  a 2x  .......  a nx n
1

n 1
  . log a1  log a 2  .........  log a n 
n n
1
log y  loga1a 2 ...........  a n n

d 1 du 
 dx log u   u dx 
 
1
y  a1.a 2 ..........a n n . Ans.

Q.No.4.: Evaluate Lim1  tan x cot x .


x 0

Sol.: Let y  Lim 1  tan x cot x . [ 1 form]


x 0

Taking log on both sides, we get

 x 0   x 0
   x 0

log y  log Lim1  tan x cot x  = Lim log1  tan x cot x   Limcot x log1  tan x 


 log1  tan x   1  tan 2 x tan 3 x 


 Lim   Lim
 x  0 tan x  tan x    ......... 
x 0  tan x   2 3 

 tan x tan 2 x 
 Lim 1    .........   1 .
x 0 
 2 3 

 y  e1  e .

Hence Lim1  tan x cot x  e . Ans.


x 0
Indeterminate forms-Problems of 00 , 0 , 1 Prepared by: Dr. Sunil, NIT Hamirpur (HP) 4

1
Q.No.5.: Evaluate Limcos x  x 2 .
x 0

1
Sol.: Let y = Limcos x  x 2 . [ 1 form]
x 0

Taking log on both sides, we get


1 1 logcos x 
 log y  Lim logcos x  x 2  Lim 2
logcos x   Lim .
x 0 x 0 x x 0 x2
0 
 0 form

Apply Cauchy’s rule, we get


 1 
 cos x sin x 
log y  Lim    Lim  tan x  . 0 
   0 form
x 0 2x x 0  2x 
Apply Cauchy’s rule, we get
 sec 2 x 
log y  Lim    1.
x 0  2  2

 y  e 1 / 2 .
1
Hence Lim cos x  2  e  1 / 2 . Ans.
x0 x

tan x
1
Q.No.6.: Evaluate Lim  .
x 0  x 

tan x
1
Sol.: Let y = Lim  . [  0 form]
x 0 x 

Taking log on both sides, we get


tan x
1 1  tan x  1
log y  Lim log   Lim tan x log   Lim x log 
x 0 x x 0  x  x 0  x  x

1   tan x  
 Lim x log   Lim    1
x 0 x  x 0 x  
Indeterminate forms-Problems of 00 , 0 , 1 Prepared by: Dr. Sunil, NIT Hamirpur (HP) 5

1
log 
x  
 Lim   .   form
x 0  1   
 
x
Apply Cauchy’s rule, we get
 1 
x  2 
log y  Lim 
x   Limx   0 .
x 0  1  x 0
 2 
 x 

 y  e0  1 .
tan x
1
Hence Lim   1 .Ans.
x 0  x 

1
 tan x  x 2
Q.No.7.: Evaluate Lim  .
x 0 x 

1
 tan x  x 2
Sol.: Let y = Lim  . [ 1 form]
x 0 x 

Taking log on both sides, we get

1  tan x 
log 
 tan x  x 2 1  tan x   x  0 
log y  Lim log   Lim 2 log   Lim .  0 form
x 0  x  x 0 x  x  x 0 x2
Apply Cauchy’s rule, we get
 x x sec2 x  tan x 
 . 
 tan x
 x2   Lim x . x sec x  tan x 
2
log y  Lim
x 0 2x x  0  tan x 2x 3 
 

x sec2 x  tan x 0    x  
 Lim 3
.  0 form  Lim    1
x 0 2x    x 0 tan x  
Again, apply Cauchy’s rule, we get

sec2 x  x.2 sec x sec x tan x  sec2 x 2 x sec2 x tan x


log y  Lim  Lim .
x 0 6x 2 x 0 6x 2
Indeterminate forms-Problems of 00 , 0 , 1 Prepared by: Dr. Sunil, NIT Hamirpur (HP) 6

sec 2 x tan x sec 2 x tan x sec 2 0 1


 Lim  Lim .   .
x0 3x x 0 3 x 3 3
  tan x  
 Lim    1
 x 0 x  

 y  e1 / 3 .
1
 tan x  x 2 1/ 3
Hence Lim   e .Ans.
x 0  x 

Q.No.8.: Evaluate Limcot x sin x .


x 0

Sol.: Let y = Limcot x sin x . [  0 form]


x 0

Taking log on both sides, we get


logcot x 
log y  Lim logcot x sin x  Lim sin x logcot x   Lim .
x 0 x 0 x  0 cos ecx

 
  form
 
Apply Cauchy’s rule, we get
1
cos ec 2 x
cot x cos ecx sin 2 x sin x
log y  Lim  Lim  Lim  Lim  0.
x 0 cos ecx cot x x0 cot 2 x x0 sin x cos2 x x 0 cos2 x

 y  e0  1 .

Hence Limcot x sin x  1 .Ans.


x 0

1
a b x x x
Q.No.9.: Evaluate Lim  .
x 0 
 2 
1
a b x x x
Sol.: Let y = Lim  . [ 1 form]
x 0 2 
 
Taking log on both sides, we get
Indeterminate forms-Problems of 00 , 0 , 1 Prepared by: Dr. Sunil, NIT Hamirpur (HP) 7

1
a b x  x x
x x
log y  Lim log   Lim 1 log a  b  . 0 
x 0  2  x 0 x  2   0 form
   
Apply Cauchy’s rule, we get
 a x log x  b x log x  log a  log b 1
2
log y  Lim    logab   logab 1 / 2 .
x
x 0 a  b x  2  2 2
 

 y  elog ab 
1/ 2
 ab .
1
 a x  bx  x
Hence Lim   ab . Ans.
x 0  2 
 
1
 tan x  x
Q.No.10.: Evaluate Lim  .
x 0 x 

1
 tan x  x
Sol.: Let y = Lim  . [ 1
x 0 x 

form]
Taking log on both sides, we get

1  tan x 
log 
 tan x  x
1  tan x   x  0 
log y  Lim log   Lim log   Lim .  0 form
x 0  x  x 0 x  x  x 0 x
Apply Cauchy’s rule, we get
 x x sec2 x  tan x 
 . 
 tan x
 x2   Lim x . x sec x  tan x 
2
log y  Lim
x 0 2 x 0  tan x 2x 2 
 

x sec2 x  tan x 0    x  
 Lim 2
.  0 form  Lim    1
x 0 2x    x 0 tan x  
Again, apply Cauchy’s rule, we get
sec 2 x  x.2 sec x sec x tan x  sec 2 x 2 x sec 2 x tan x
log y  Lim  Lim .
x 0 4x x 0 4x
x sec 2 x tan x 0. sec 2 0   tan x  
 Lim .   0.  Lim    1
x 0 2 x 2  x 0 x  
Indeterminate forms-Problems of 00 , 0 , 1 Prepared by: Dr. Sunil, NIT Hamirpur (HP) 8

 y  e0  1 .
1
 tan x  x
Hence Lim   1 .Ans.
x 0 x 

1
 sinh x  x 2
Q.No.11.: Evaluate Lim  .
x 0  x 

1
 sinh x  x 2
Sol.: Let y = Lim  . [ 1 form]
x 0  x 

Taking log on both sides, we get

1  sinh x 
log 
 sinh x  x 2 1  sinh x   x 
log y  Lim log   Lim 2 log   Lim .
x 0  x  x 0 x  x  x 0 x2
0 
 0 form
 
Apply Cauchy’s rule, we get
 x x cosh x  sinh x 
 sinh x . 
log y  Lim  x2   Lim x . x cosh x  sinh x 
 
x 0 2x x 0  sinh x 2x 3 
x cosh x  sinh x 0    x  
 Lim 3
.  0 form  Lim    1
x 0 2x  x 0 sinh x  
Again, apply Cauchy’s rule, we get
x sinh x  cosh x  cosh x x sinh x 1 sinh x 1
log y  Lim  Lim  Lim  .
x 0 6x 2 x 0 6x 2 x 0 6 x 6

 y  e1 / 6 .
1
 sinh x  x 2 1/ 6
Hence Lim   e .Ans.
x 0  x 

1 xx
Q.No.12.: Evaluate Lim .
x 0 x log x

1 xx 0 
Sol.: Let y  Lim  0 form 
x  0 x log x  
Indeterminate forms-Problems of 00 , 0 , 1 Prepared by: Dr. Sunil, NIT Hamirpur (HP) 9

Applying Cauchy’s Rule, we get


 x x 1  log x 
y  Lim  Lim x x
x 0 1  log x  x 0

Taking log of both sides, we get


1

log x x  Lim x  0
log y  Lim x log x  Lim   Lim
x 0 x 0 1 x 0 1 x 0
 2
x x
log y  0
 y  1 . Ans.
x
tan
 x 2a
Q.No.13.: Prove that Lim 2    e2 /  .
x a  a
x
tan
 x 2a
Sol.: Let y = Lim 2   . [ 1 form]
x a  a
Taking log of both sides, we get
 x   x
log y  Lim tan   log 2   [   0 form]
x a  2a   a

 x
log 2  
a 0 
 Lim   0 form
x a  x 
cot 
 2a 
Apply Cauchy’s rule, we get
1  1
.  
 x  a
2  
a 2
log y  Lim   .
x a   x  
 cos ec2  
2a  2a 

 y  e2 /  .
x
tan
 x 2a
Hence Lim 2    e 2 /  .Ans
x a  a
Indeterminate forms-Problems of 00 , 0 , 1 Prepared by: Dr. Sunil, NIT Hamirpur (HP) 10

 
1
3 log 2  x 
Q.No.14.: Prove that Lt 8x  e.
x 2

 
1
Sol.: Let y  Lt 8  x 3 log 2  x 
x 2

Taking log on both sides, we get

log y  Lt 
log 8  x 
1
3 log 2  x 
 Lim

log 8  x 3 
x 2 
x  2 log 2  x 
Apply Cauchy’s rule, we get

 3x 2
3 3x 2 12
log y = Lim 8  x  Lim  1
x 2  1 x  2 4  3x  x 2 12
2x
log y  1
 y  e . Ans.
1 1
 2cosh x  1  x 2
 e .
Q.No.15.: Prove that Lt  12
x 0  2
x 
1
 2cosh x  1  x 2
Sol.: Let y  Lt  
x 0  x2 
1
 2  x2 x4  x 2
 y  Lt  2 1    .......... .  1
x 0 x  2 4 
  
Taking log on both sides, we get
 x2   x x3 
log1   ............     .........
 12  6 3 
log y     Lim  
2 x c  x 2 4 
x x
2 x 1   ........
 12 360 
 
1
log y  .
12
1
y e 12 . Ans.
Indeterminate forms-Problems of 00 , 0 , 1 Prepared by: Dr. Sunil, NIT Hamirpur (HP) 11

N2 1
 2
 
Q.No.16.: Prove that Lt cos   e 2 .
N   N
N2
 
Sol.: Let y  Lt cos 
N   N
N2
 2 4 
y  Lim. 1  2
 4
 ...........
N 
 2!N 4!N 
Taking log on both sides, we get
 2 4 
log y  Lim. N 2 log 1  2
 4
 ...........
N   2! N 4! N 

  2 4 
 log y  Lim.N log 1  2
2
 4
 ...........  
  2!N 4!N
N 

  2 4  1  2 4 
2

 log y   Lim. N 2  2
 4
 ...........    2
 4
 ...........   .........
N 
 2!N 4!N  2  2!N 4!N  

  2 4  4  2 
2

 log y   Lim. N  2
 2
4
 ...........   4 
1  2
 ...........   .........
N 
 2!N 4!N  8N  12N  

 2 4  4  2 2  
 log y   Lim. N 2  2
 4
 ...........   4  1  2
 ...........   .........
 2!N 4!N  8N  12N
N 
 

 2 14 
 log y   Lim. N 2  2
 4
 ...................
N 
 2!N 12N 

  2 1 4  2
 log y   Lim.   2
 ...................   log y  
N 
 2! 12N  2

2

ye 2 . Ans.
x 2
 ax  1 
Q.No.17.: Prove that Lt    ea .
x  ax  1 

x
 ax  1 
Sol.: Let y  Lt   [ 1 form]
x  ax  1 

Taking log on both sides, we get


Indeterminate forms-Problems of 00 , 0 , 1 Prepared by: Dr. Sunil, NIT Hamirpur (HP) 12

 1  1  1
log 1   log 1    log 1  
 ax  1   ax   Lt  ax   ax 
log y  Lt x log   Lt
x   ax  1  x  1 1  1  x  1

x  ax  x

1 1 1   1 1 1 
 ax  2 2  3 3  .......... ...   ax  2 2  3 3 ..........
 2a x 3a x   2a x 3a x 
1
x
2 1 
log y  Lim 1  2 2  ........
x  a  3a x 
2
log y  .
a
2
y  ea . Ans.
1
1  a x  x a
Q.No.18.: Evaluate Lt    
 .
x   2  x a 

1
1  a x  x a
Sol.: Let y  Lt    

x   2  x a 

Taking log on both sides, we get
1  a x 
log    

 3  
1
2
  x a  2 xa x  a    x 2 2a
log y    ax 2    0
xa ax  a  2a
 
log y  0 .
 y  1 . Ans.

1  x sin x
Q.No.19.: Prove that Lt  1 .
x  0 x log x

1  x sin x
Sol.: Let y  Lt
x  0 x log x

1  x sin x .x x 1  xx  x x 1  log x 
y  Lt  Lim  Lt
x 0 x log x x  0 x log x x 0 1  log x
Indeterminate forms-Problems of 00 , 0 , 1 Prepared by: Dr. Sunil, NIT Hamirpur (HP) 13

y  Lim  x x
x 0

Taking log on both sides, we get


 log x  
log y  Lim  x log x  Lim   form 
x 0 x 0 1  
x
Applying Cauchy’s rule, we get
1
log y  x  x  0 .
1
x2
 y  1 . Ans.
m
 x
Q.No.20.: Evaluate Lt  cos  .
m   m
m
 x
Sol.: Let y  Lt  cos 
m   m
Taking log on both sides, we get
 x
log cos 
 x  m
log y  Lt m log cos   Lt
m   m  m  1/ m

0 
 0 form 
 

1 x  1 
  sin   2 
x
cos  m  m 
m  x
log y  Lt  Lt   tan   0
m  1 m   m
 2
m

 y  e0  1 . Ans.
1 cos x
1
Q.No.21.: Evaluate Lt   .
x  0 x 

1 cos x
1
Sol.: Let y  Lt  
x 0 x 

Taking log on both sides, we get


Indeterminate forms-Problems of 00 , 0 , 1 Prepared by: Dr. Sunil, NIT Hamirpur (HP) 14

log y  Limcos x  1log x  Lim


cos x  1 x log x   Lim cos x  1 0 
x x  0 form 
x 0 x 0 x 0  
Apply Cauchy’s rule, we get
 sin x
log y  Lim 0
x 0 1

log y  0 y  e0  1 . Ans.

Q.No.22.: Evaluate Lim tan x cos x .



x
2

Sol.: Let y = Lim tan x cos x [  0 form]



x
2

Taking log on both sides, we get


log y  Lim cos x logtan x  [ 0   form]

x
2

logtan x   
 Lim   form
x
 sec x  
2

Apply Cauchy’s rule, we get


1
. sec 2 x
tan x cos x
 Lim  Lim  0.
 sec x. tan x  sin 2 x
x x
2 2

 y  e0  1 . Ans.

***
*** *** ***
*** *** *** *** ***
Differential Calculus

Maxima and Minima


(Maxima and minima in case of two or more variables)
Prepared by:
Dr. Sunil
NIT Hamirpur (HP)
(Last updated on 11-09-2009)
Latest update available at: http://www.freewebs.com/sunilnit/

(26 Solved problems and 10 Home assignments)

Maxima and Minima of functions of two variables:


Let z  f x , y  be a function of two independent variables x and y.
Relative maximum: f x , y  is said to have a relative maximum at a point a , b  if
f a , b   f a  h , b  k   f a  h , b  k   f a , b   0
for small positive or negative values of ‘h’ and ‘k’
or
If the value of the function f (x, y) at a , b  , i.e., f a , b  is greater than the value
of the function f at all points in some small neighbourhood of a , b  , then f x , y  is said
to have a relative maximum at a point a , b  .
Relative minimum: f x, y  is said to have a relative minimum at a point a , b  if

f a , b   f a  h , b  k   f a  h , b  k   f a , b   0
for small positive or negative values of ‘h’ and ‘k’.
or
Maxima and minima: Maxima and minima in case of two or more variables 2
Prepared by: Dr. Sunil, NIT Hamirpur (HP)

If the value of the function f at a , b  , i.e., f a , b  is smaller than the value of the
function f at all points in some small neighbourhood of a , b  , then f x, y  is said to have
a relative minimum at a point a , b  .
Notation: Denoting f a  h, b  k   f a , b  by f a , b  or simply by 
i.e.   f a  h , b  k   f a , b  .
Then, f x , y  has the maximum at a , b  if  has the negative sign for all small values
of h, k; i.e.   0 .
Similarly, f x, y  has the minimum at a , b  if  has the positive sign for all small
values of h, k; i.e.   0 .

Extremum:
Extremum is a point, which is either a maximum or a minimum. The value of the
function f at an extremum (maximum or minimum) point is known as extremum
(maximum or minimum) value of the function f.
Geometrically, z  f x , y  represents a surface. The maximum is a point on the
surface (hill top) from which the surface descends (comes down) in every direction
towards the xy-plane. The minimum is the bottom of depression from which the surface
ascends (climbs up) in every direction towards the xy-plane. In either case, tangent plane
to the surface at a maximum or minimum point is horizontal (parallel to xy-plane) and
perpendicular to z-axis.

z-axis z-axis
z = f(x, y) Maximum

z = f(x, y)
Minimum y-axis y-axis

x-axis x-axis
Maxima and minima: Maxima and minima in case of two or more variables 3
Prepared by: Dr. Sunil, NIT Hamirpur (HP)

Saddle point:
Saddle point or minimax is a point where function is neither maximum nor
minimum. At such point, f is maximum in one direction while minimum in other
direction.
Geometrically, such a surface (looks like the leather seat on back of a horse)
forms a ridge rising in one direction and falling in another direction.

z-axis

Saddle point
y-axis

x-axis

Example: z = xy, hyperbolic paraboloid has a saddle point at the origin.

Necessary and sufficient conditions for extrema of a function f of two


variables:
Since   f a  h , b  k   f a , b .
Expanding f a  h , b  k  by Taylor’s theorem, we obtain
 
f a  h , b  k   f a , b   hf x a , b   kf y a , b 
1
 
 h 2 f xx a , b   2hkf xy a , b   k 2 f yy a , b   ......
2!
(i)

Since h and k are small, so neglecting higher order terms of h 2 , hk, k 2 , etc. Then the
above expression reduces to

f a  h, b  k   f a, b  hf x a, b  kf y a, b 
Thus   f a  h , b  k   f a , b   hf x a, b  kf y a, b . (ii)

Necessary conditions:
The necessary condition for f x, y  has the maximum or minimum at a , b 
Maxima and minima: Maxima and minima in case of two or more variables 4
Prepared by: Dr. Sunil, NIT Hamirpur (HP)

i.e.  has the negative or positive sign for all small values of h, k is
f x a, b  0 and f y a, b  0 ,

even though ‘h’ and ‘k’ can take both positive and negative values.

Method for developing the sufficient conditions:


With f x a, b  0 and f y a, b  0 ,


  f a  h, b  k   f a , b   hf x a , b   kf y a , b 
1

 h 2 f xx a , b   2hkf xy a , b   k 2 f yy a , b 
2!

reduces to

  f a  h, b  k   f a , b  
1 2
2!
 
h f xx a , b   2hkf xy a , b   k 2 f yy a , b 

Denote f xx a, b  r, f xy a, b  s, f yy a, b  t , we get

  f a  h, b  k   f a , b  
1 2
2!

h r  2hks  k 2 t .  (iii)

From this expression, we observe that the nature of the sign of  depends on the nature

of sign of h 2 r  2hks  k 2 t .

 

 h 2 r 2  2hkrs  k 2 rt 
Thus sign of   sign of h 2 r  2hks  k 2 t  sign of 
r




 hr  ks 2  k 2 rt  s 2
 sign of 
 . (iv)
 r 

If rt  s 2  0 , then the numerator of RHS of (iv) is positive.


In this case, sign of  = sign of r.

Thus   0 if rt  s 2  0 and r  0

and   0 if rt  s 2  0 and r  0 .
Therefore, the sufficient (Lagrange’s) conditions for extrema are:

I. f attains (has) a maximum at (a, b) if rt  s 2  0 and r  0

II. f attains (has) a minimum at (a, b) if rt  s 2  0 and r  0


Maxima and minima: Maxima and minima in case of two or more variables 5
Prepared by: Dr. Sunil, NIT Hamirpur (HP)

III. Saddle point: If rt  s 2  0 , then   0 or   0 depending on ‘h’ and ‘k’.

Therefore, f has a saddle point (minimax) at (a, b) if rt  s 2  0 .

IV. Failure case: If rt  s 2  0 , then further investigation is needed to determine


the nature of the function f.
FINAL CONCLUSIONS:
A function of two variables u  f x , y  is said to have a maximum or minimum if
f f
(1)  0,  0,
x y
which gives the values of x say x = a and y say y = b, for which the function is
maximum or minimum.
2
 2f  2f   2f 
(2) .  at these values i.e. x = a and y = b.
x 2 y 2  xy 

  2 x a  0 i.e. rt  s 2  0 .
i.e. f xx .f yy  f xy
y b

(3a) Then the function f x , y  will have a maximum value if apart from the above

 2f
conditions (1) and (2), also  0 i.e. r  0 at x = a and y = b.
x 2
(3b) Similarly, the function f x , y  will have a minimum value if apart from the above

 2f
conditions (1) and (2), also  0 i.e. r  0 at x = a and y = b.
x 2
Remarks:
2
 2f  2f   2f 
(a) If . 2  i.e. rt  s 2  0 at x = a and y = b,
2  
x y  xy 
Then the function f x , y  is neither a maximum nor a minimum.
2
 2f  2f   2f 
(b) Again if . 2  i.e. rt  s 2  0 at x = a and y = b,
2  
x y  xy 
2
 2f  2f   2f 
Then such a point where .  holds, is called a “Saddle point”.
x 2 y 2  xy 
Maxima and minima: Maxima and minima in case of two or more variables 6
Prepared by: Dr. Sunil, NIT Hamirpur (HP)

Method of finding Extrema of f(x, y):


1. Solving f x  0 and f y  0 yields critical or stationary point P of f.

2. Calculate r  f xx , s  f xy , t  f yy at the critical point.

3. a. f attains (has) a maximum at (a, b) if rt  s 2  0 and r  0

b. f attains (has) a minimum at (a, b) if rt  s 2  0 and r  0

c. Saddle point: If rt  s 2  0 , then   0 or   0 depending on ‘h’ and ‘k’.

Therefore, f has a saddle point (minimax) at (a, b) if rt  s 2  0 .

d. Failure case: If rt  s 2  0 , then further investigation is needed to determine


the nature of the function f.

Remarks: Extrema occur only at stationary points. However, stationary points need not
be extrema.
Now let us solve some problems, where we have to evaluate the maxima or
minima of the given function:

Q.No.1.: Find the values of x and y for which u  x 2  y2  6x  12 has a minimum


value and find this minimum value.

Sol.: Since ux, y  x 2  y2  6x  12 .


u u
Then  2x  6 and  2y .
x y
u u
For u (x, y) to be minimum or maximum, we have  0 and  0.
x y
 2x  6  0 and 2 y  0 .
 x  3 and y  0 .

  2u    2u    2u 
Also  2   2 ,  2  2 and    0.
 x  3,0  y  3,0   xy  3,0
2
 2 u  2 u   2 u 
 2. 2   2  2  0  4  0 at x  3 and y  0 .
x y  xy 
 
Maxima and minima: Maxima and minima in case of two or more variables 7
Prepared by: Dr. Sunil, NIT Hamirpur (HP)

  2u 
And also  2   2  0.
 x 3,0
Thus the function is minimum at x  3 and y  0 .

 
And this minimum value u x , y min  x 2  y 2  6 x  12 3,0  9  18  12  3 . Ans.

Q.No.2.: If f x , y   6xy  9 , find the values of x and y for which f x, y  has a stationary
value.
Sol.: Since f x , y   6 xy  9 .
f f
Then  6 y and  6x .
x y
f f
For f (x, y) to be minimum or maximum, we have  0 and  0.
x y
 6 y  0 and 6 x  0 .
 y  0 and x  0 .

  2f    2f    2f 
Also  2  0 ,  2  0 and    6.
 x  0,0  y 0,0  xy 0,0

2
 2 u  2 u   2u 
 2. 2   0  62  36  0 at x  0 and y  0 .
x y  xy 
 
Thus x  0 and y  0 does not stand for maxima and minima.
Thus the given function has a stationary value at x  0 and y  0 .
Q.No.3.: Find the maximum and minimum values of

(a) x 3  y3  3axy .
(b) sin x.sin y.sin x  y  .

Sol.: (a) Let u  x 3  y3  3axy .


Partially differentiating u w. r. t. x and y, we get
u u
 3x 2  3ay and  3y 2  3ax
x y
u u
For u (x, y) to be minimum or maximum, we have  0 and  0.
x y
Maxima and minima: Maxima and minima in case of two or more variables 8
Prepared by: Dr. Sunil, NIT Hamirpur (HP)

u
 0  3x 2  3ay  0  3x 2  3ay  x 2  ay . (i)
x
u
 0  3y 2  3ax  0  3y2  3ax  y2  ax . (ii)
y

Squaring (i), we get x 4  a 2 y2 (iii)

Substituting the value of y2 from (ii) in (iii), we get x 4  a 2ax  x 3  a 3  x  a

 x 2  ay  a 2  ay  y  a .

 2u  2u  2u
 6x ,  6y ,  3a
x 2 y 2 xy
2
  2u    2 u    2u 
  2 . 2     = 6x . 6y  9a 2  36xy  9a 2
 x   y   xy 
    

Substituting xy as a 2 , we get
2
  2u    2 u    2 u 
 .   = 36a 2  9a 2  27a 2  0 .
 x 2   y 2   xy 
    
To find max. and min. value which u will attain.

 2u
Put  0 , we get
x 2
 2u
 6 x  6a
x 2
6a  0 if a  0 and 6a  0 if a  0 . Ans.
(b) Let P( x, y)  sin x.sin y.sin x  y 
F
Fx   sin ycos x.sin x  y   sin x. cosx  y   sin y. sin(2x  y) (i)
x
F
Fy   sin xcos y.sin x  y   sin y. cosx  y   sin x. sin(x  2 y) (ii)
x
For max. or min. values of F(x, y), we have (i) and (ii) equal to zero.
Fx  0  sin y. sin(2x  y)  0 (iii)
Fy  0  sin x. sin(x  2 y)  0 (iv)

From (iii) and (iv), we have


Maxima and minima: Maxima and minima in case of two or more variables 9
Prepared by: Dr. Sunil, NIT Hamirpur (HP)

sin y  0 and sin x  0


x0 and y = 0, which is not possible.
We have also
x  2 y   or 2  (v)
2x  y   or 2  (vi)
Taking x  2 y  2
2 x  y  2
2 2
Solving the above equation we have x  , y
3 3
Now taking x  2 y  
2x  y  
 
Solving the above equations, we get x  , y .
3 3
   2 2 
So the points of extremum are  ,  and  ,  .
3 3  3 3 
Let r  Fxx , t  Fyy and s  Fxy .

So Fxx  sin y.sin( 2x  y).2  2 sin y. cos 2 x  y 


Fyy  2 sin y. cosx  2 y 

Fxy  cos y. sin 2 x  y   cos2 x  y . sin y  sin 2x  2 y  .

 
Now at points  ,  , we have to calculate the
3 3
2
Fxx .Fyy  Fxy

3
So Fxx  2   1   3
2
3
Fyy  2   1   3
2
 3
Fxy  .
2
Maxima and minima: Maxima and minima in case of two or more variables 10
Prepared by: Dr. Sunil, NIT Hamirpur (HP)

2

Now  3 .  3   
 3

3 9
  3   0

 2  4 4
2
So that Fxx .Fyy  Fxy  0 and Fxx   3 .

  
Hence F(x, y) has maximum at  x  , y   .
 3 3

   3 3
Maximum value of F(x, y) sin .sin .sin  .
3 3 3 8
 2 2 
Now taking points  ,  , we have to calculate
 3 3 
2
Fxx .Fyy  Fxy

3
So Fxx  2  1
2
3
Fyy  2  1
2
3
Fxy  .
2
2
 3  3   3 3 9
Now  2   1 2   1.     3   0

 2  2   2  4 4
2
So that Fxx .Fyy  Fxy  0 and also Fxx  3  0 .

 2 2 
Hence the point  ,  is the point of minimum.
 3 3 

2 2 4  3 3
Minimum value of F(x, y) sin .sin .sin  .
3 3 3 8
3 3 3 3
So the maximum and minimum values of function F(x, y) is and
8 8
respectively.

a3 a3
Q.No.4.: Show that the minimum value of f ( x , y)  xy   is 3a 2 .
x y
Maxima and minima: Maxima and minima in case of two or more variables 11
Prepared by: Dr. Sunil, NIT Hamirpur (HP)

a3 a3
Sol.: Given function is f ( x , y)  xy   .
x y

f a3 f a3
 fx   y  2 , fy  x 2 .
x x y y
For f (x, y) to be minimum or maximum, we have f x  0 and f y  0 .

a3 a3
If f x  0  y  0y (i)
x2 x2
a3 a3
If f y  0  x  0x  (ii)
y2 y2
From (i) and (ii), we get

a3 a 3.x 2  x3 
x 0x  0  x 1    0
2
a 3.a 3  a3 
 a3   
 
 x2 
 

x3
As x  0  1   0 , because when x = 0, y can not be defined.
a3

 x3  a 3  x  a

a3
 From (i), we get y   a.
a2
Hence extreme point is (a, a)

Now, f xx  0 
 2.a 3 
2a 3
,
x3 x3

f yy 
 2F
 0 
 2.a 3  2a 3 ,
y 2 y3 y3

 2F
f xy   1 0  1.
xy

2a 3 2a 3
At point (a, a) f xx   2, f yy   2.
a3 a3

Now, f xx  f yy  f xy 2  2  2  12  4  1  3  0 .


Also f xx  2  0 .
Maxima and minima: Maxima and minima in case of two or more variables 12
Prepared by: Dr. Sunil, NIT Hamirpur (HP)

a3 a3
 Minimum value of f(x, y) is a.a    a 2  a 2  a 2  3a 2 . Ans.
a a
Q.No.5.: Find co-ordinates of a point P(x, y) such that sum of squares of its distances
from rectangular axis of reference and line x  y  8 is minimum.
Sol.: We know that distance of a point P(x, y) from any line ax  by  c  0 is given by

ax  by  c
a 2  b2

 u  x 2 y2 
x  y  82
2
u u
For maxima or minima, we have  0 and  0.
x y
2x  y  8 2x  y  8
 2x  0 and 2y  0
2 2
3x  y  8 (i) x  3y  8 (ii)
Solving (i) and (ii), we get x = 2 and y = 2.
 At x = 2, y = 2, the given function have max. or min. value.
Also we know that for minima, we have

 2u  2u
0 and  0.
x 2 y 2

 2u   2u 
  3  0 and   3  0.
x 2  x  2 y 2  x  2
y2 y 2

 Thus the function have minimum value at x = 2, y = 2.


Hence (2, 2) be the required coordinates of the point such that sum of squares of its
distances from rectangular axis of reference and line x  y  8 is minimum. Ans.
Q.No.6.: Divide ‘a’ into three parts such that their products be a maximum.
Sol.: Let ‘a’ be the sum of three no. x, y, z.
Therefore a  x  y  z  z  a  x  y 
Their product can be written as

P  xyz  P  xya  x  y  P  xay  x 2 y  y2 x .


Maxima and minima: Maxima and minima in case of two or more variables 13
Prepared by: Dr. Sunil, NIT Hamirpur (HP)

For maximum or minimum values Fx , Fy are zero.

P
 Fx   ay  2xy  y 2  0  a  2x  y  0  a  2x  y  y  a  2x (i)
x
P
Fy   xa  x 2  2 yx  0  a  x  2 y  0  a  2 y  x  x  a  2 y (ii)
y
From (i) and (ii), we get
a
y  a  2a  2 y   y  a  2a  4 y  3y  a  y 
3
Putting this value in (ii), we get
2a a
x a x .
3 3
a a 2a a
z  a     a  z .
3 3 3 3
Differentiating Fx , Fy again

 2P 2
 Fxx  2
 2 y   a
x 3

 2P 2
Fxx   2 x   a
2
y 3

also Fxy 
 2P

  P  
 
xy y  x  y
 
ya  2 xy  y 2  a  2x  2 y .

Putting the value of x and y.

 2P 2a 2a 3a  4a  a
a   
xy 3 3 3 3
2
 2 P  2 P   2P   2a  2a a 2 4a 2  a 2 3a 2 a 2
 2. 2   .     0
x y  xy  3 3 9 9 9 3
 
Therefore the product is maximum.
a a a
Hence ‘a’ can be divided into three parts as  , ,  .
3 3 3
Q.No.7.: Divide 120 into three parts so that the sum of the products taken two at a time
shall be maximum.
Maxima and minima: Maxima and minima in case of two or more variables 14
Prepared by: Dr. Sunil, NIT Hamirpur (HP)

Sol.: Let 120  x  y  z . ...(i)


and P  xy  yz  zx . ...(ii)
From (i), we get z  120  x  y .
 P  xy  y120  x  y   x 120  x  y  .
P
Then  y  y(1)  120  2 x  y  120  2x  y
x
P
and  x  120  x  2 y  x  120  x  2 y .
y
P P
For maximum or minimum vales both  0 and  0.
x y
 120  2x  y  0 and 120  x  2 y  0 .
Solving these two equations, we get
x  40 , y  40 and z  40 .

  2P    2P    2P 
Also  2   2 ,  2   2 and    1 .
 x  40, 40, 40  y  40, 40, 40  xy  40,40,40
2
 2 P  2 P   2P 
 2. 2   (2)  (2)  (1) 2  3  0 at x  40 , y  40 and z  40 .
x y  x y 
 
  2P 
And also  2   2  0
 x  40, 40, 40

Thus the function is maximum at x  40 , y  40 and z  40 .


Q.No.8.: Prove that the rectangular solid of maximum volume which can be inscribed in
a sphere is a cube.
Sol.: Let x , y and z be the dimensions of the rectangular solid and D be the diameter of
the given sphere.
The diagonal of the solid will be diameter of the sphere.

 D2  x 2  y 2  z 2  D  x 2  y 2  z 2 . ...(i)
Let V be the volume of the solid

 
 V  xyz  xy D2  x 2  y 2  V 2  x 2 y2 D2  x 2  y2  D2 x 2 y2  x 4 y2  x 2 y4 .
Maxima and minima: Maxima and minima in case of two or more variables 15
Prepared by: Dr. Sunil, NIT Hamirpur (HP)

Let V 2  u .  u  D2 x 2 y2  x 4 y2  x 2 y4 . ...(ii)
Differentiate (ii) partially w. r. t. x and y separately, we get
u
x

 2D 2 xy2  4x 3 y 2  2xy 4  2xy2 D 2  2x 2  y 2  ...(iii)

u
y

 2D 2 x 2 y  2 x 4 y  4 x 2 y 3  2 x 2 y D 2  x 2  2 y 2  ..(iv)

u u
For max. and min. of u,  0, 0
x y


u
x

 2xy2 D 2  2x 2  y 2  0 
and
u
y

 2x 2 y D2  x 2  2y2  0 
But x  0, y  0 being lengths

D  2x  y   0
2 2 2
...(v)

D2  x2  2y2   0 ...(vi)

From (v) y2  D2  2x 2 and put in (vi), we get

D2 D
D 2  x 2  4 x 3  2D 2  0  3x 2  D 2  x 2  x .
3 3

2D 2 D2 D
Now y 2  D2  2 x 2  D2   y ; (y cannot be < 0)
3 3 3
Again differentiate (iii) partially w. r. t. x and y separately and (iv) w. r. t. y, we get

 2u
2
 2D 2 y 2  12x 2 y 2  2 y 4 ,
x
 2u
 4D 2 xy  8x 3 y  8xy3 ,
xy

 2u
2
 2D 2 x 2  2x 4  12x 2 y 2 .
y

  2u  D 2 12D 2 D 2 2D 4 8D 4
A 2  2D 2 .  .  
 x  D D  3 3 3 9 9
  , 
 3 3
Maxima and minima: Maxima and minima in case of two or more variables 16
Prepared by: Dr. Sunil, NIT Hamirpur (HP)

  2u  D D D3 D D D3 D4
B  2
 4D . . 8 . 8 .  4
 xy  D  3 3 3 3 3 3 3 3 9
  D
, 
 3 3

  2u  D 2 2D 4 12D 2 D 2 8D 4
C 2  2D .2
  . 
 y  D  3 9 3 3 9
  D
, 
 3 3

 8D 4  8D 4   4D 4  64D8 16D8
Now AC  B   
2       0 (+ ve)
 9  9   9  81 81
    
D
Also A is negative for x  y  .
3
D
 u is maximum when x  y  .
3
D
 V 2 or V is maximum value for x  y  .
3

D2 D2 D
From (i), we get z  D 2  x 2  y 2  D 2   
3 3 3
D
x  y  z  i.e. length = breadth = height.
3
Hence for maximum volume solid is a cube.

Q.No.9: The temperature T at any point (x, y, z) in space is defined by T  400xyz2 .

Find the maximum temperature on the surface of a unit sphere x 2  y2  z 2  1.

Sol.: Given equation of the sphere is x 2  y2  z 2  1  z 2  1  x 2  y2 (i)

 
Given T  400xyz2  T  400xy 1  x 2  y2  T  400xy  400x 3y  400xy3 (ii)
T
Now,  0  400 y  1200x 2 y  400 y3  0  y2  3x 2  1  0  y2  1  3x 2 (iii)
x
T
 0  400 y  400 x 3  1200 xy 2  0  x 2  3y2  1  0
y


 x 2  3 1  3x 2  1  0  [using (ii)]
1 1
 x 2  3  9x 2  1  0  x 2  x
4 2
Maxima and minima: Maxima and minima in case of two or more variables 17
Prepared by: Dr. Sunil, NIT Hamirpur (HP)

1
Putting in (iii), we get y  .
2
1
Putting in (i), we get z 2  .
2
Now to check Maxima and Minima

 2T  2T
 2400 x and  2400 y
x 2 y 2

 2T   2T 
 1200 and  2   1200
x 2 x
1   y  y 1
2 2

 2T  2T
Also  400  2400 x   800
xy xy 1
x
2

2
 2T  2T   2T 
Clearly   ,
x 2 y 2  xy 
 
  2T  1
Now  2   2400   1200  0 .
 x  1 2
 x  , y
1
2 2

 The function T has maxima at the calculated values of x, y and z 2 .

Put the values of x, y and z 2 in T  400xyz2 , we get


1 1 1
T  400     T  50 units. Ans.
2 2 2

which is the maximum temperature on the surface of a unit sphere x 2  y2  z 2  1.


Q.No.10.: A rectangular tank open at the top and is to hold a given volume . Find the
dimensions of the box requiring least material, for its construction.
Sol.: Let x , y and z ft. be the dimensions of the rectangular tank open at the top so that
material for construction will be least if surface area is least.
Let surface area, S = F(x, y, z) = xy + 2yz + 2zx . ....(i)
Also given volume = xyz = V. ...(ii)
Eliminating z from (i) with the help of (ii), we get
Maxima and minima: Maxima and minima in case of two or more variables 18
Prepared by: Dr. Sunil, NIT Hamirpur (HP)

V 1 1
S  xy  2y  x   xy  2V   .
xy x y
S 2V S 2V
  y  2  0 and  x  2  0. ...(iii)
x x y y
Solving these, we get x = y = 2z. ...(iv)

 2S 4V  2S  2S 4V
Now r   , s 1 ,t  2  3 .
x 2 x3 xy y y

24 V 4V 16V 2  x 6 64  1x 6


At x = y , rt  s  3 . 3  1   = always + ve and r is also + ve.
x x x6 x6
Henve S is minimum for x = y = 2z.
Also from (ii) , x = y = (2V)1/3. ...(v)
Thus from (iv) and (v), we get
length = breadth = twice height = (2 volume)1/3.

Q.No.11.: Find the point on the surface z2  xy  1 , nearest to the origin.

Sol.: Let  x , y , z be the point lies on the surface z2  xy  1 ,which is nearest to the
origin.

 Distance between origin and this point  x 2  y 2  z 2 . Squaring both sides, we get

(Distance)2  u(say)  x 2  y2  z 2 subject to the condition z2  xy  1 . ...(i)

 u  x 2  y2  xy  1 .
u u
Then  2x  y and  2y  x .
x y
u u
For maximum or minimum vales both  0 and  0.
x y
 2x  y  0 and 2 y  x  0 .
y x
x and y   .  x  0 and y  0 .
2 2
  2u    2u    2u 
Also  2   2 ,  2  2 and    1.
 x  0,0  y  0,0  xy  0,0
Maxima and minima: Maxima and minima in case of two or more variables 19
Prepared by: Dr. Sunil, NIT Hamirpur (HP)

2
 2 u  2 u   2 u 
 2. 2   2  2  1  3  0 at x  0 and y  0 .
x y  xy 
 
  2u 
And also  2   2  0 . Also from (i) , z  1 .
 x 0,0
Thus the function is minimum at x  0 , y  0 and z  1 .

Thus 0, 0, 1 and 0, 0,  1 are the points on the surface z2  xy  1 ,which are nearest to
the origin.
Q.No.12.: A tent having the form of a cylinder, surmounted by a cone is to contain a
given volume. Prove that for the canvas to be minimum, height of the cone is
twice that of the cylinder.
Sol.: Let the height of the cone is = y
Let the height of the cylinder is = x
Then, we have to prove , y  2x
Let v be the total volume of the tent.
1
Then, v  r 2 x  r 2 y (i)
3
If S is the total surface area of the tent, then

S  2  r x   r   S  2 r x   r r 2  y 2 (ii)
Putting the value of x from (i) in (ii), we get
 v y
S  2 r  2     r r 2  y 2
 r 3
S  2 r r
    2y .
y 3 2 r 2  y2
S
For maxima and minima,  0.
y
2
 2 r r y
2  
2 y
  0     4r 2  4 y 2  9 y 2
3 3  2 2 
r 2  y2  r y 
2
 5 y 2  4r 2  y  r (iii)
5
Maxima and minima: Maxima and minima in case of two or more variables 20
Prepared by: Dr. Sunil, NIT Hamirpur (HP)

Differentiating (ii) partially w. r. t. r, we get


S    v y  
 2 r   2      r r 2  y 2 
r r   r 3  r  

 2v 2y  r2
    r 2  y2  .
r2 3 r 2  y2

S
For max. or min., 0
r
 2v 2 y 2
    r 2  y2  0
r2 3 r 2  y2
 2 r  y  2y 2 2 r2
 x     r y  0
 r2  3 3 r 2  y2

2
Putting y  r in above equation, we get
5

4 4 9 5  1 5  6
 2 x  r r r r  0  2x    r  r
3 5 3 5 3 5 3 3 5 3 5  3 5
1 y
x r   y  2 x , which is the required proof.
5 2

 2S

r 2  y2  r
r y
y 
 r r 2  y2  y2 
Now,
y 2 r 2
 y2  r 2  y2

r 2
 y2 
3/ 2

 r3
 0 (iv)
r 2
 y2 
3/ 2

2 r r 2  y 2   r 2
 2S 2v 2 r r 2  y2
  2r  
r 2 r4 r 2  y2 r 2  y2

4v 2 r  r 3  2 r y 2
   0 (v)
r3 r 2  y2 r 2
 y2 
3/ 2

 2S S  S  S   2 r r y 
      
yr r  y  r  3 2 2
r y 

Maxima and minima: Maxima and minima in case of two or more variables 21
Prepared by: Dr. Sunil, NIT Hamirpur (HP)

r
r 2  y2   y   r y 

 2r

r 2  y2
 
 
 2  y r 2  y 2   r 2 y
3 2
r y 2 3 
r 2  y2
3/ 2

 2 y3
  .
3 
r 2  y2  3/ 2

 2S  2S  2S
Let r  , t and f 
y 2 r 2 yr
2
 r3  4v 2 r  r 2  2 r y 2    2  y2 
r t f2        
2
r y  
2 3 / 2  r3 2
r y 2 2
r y 2 3/ 2   3
   2
r y 
2 3/ 2 
  
2
at y  r
5
 
  2 4 2 
 r  r  2 r 
 r3  4v 2 r  r 2  2 r y 2  5 
r t f2   3    3 
 2 4 2
r  r 
3/ 2
r

2
r  r
4 2 r 2
 y 
2 3/ 2   4 
 r2  r2  

 5  5  5  
 

 4 3 
  r 
 2 5 5 
 
3 3/ 2 
  2 4 2 
r  r 
  5  

2 r  1 2  5 3
v at y  r and x    r2  r  r .
5 5  5 3 5 3

5 5 4 5 2 5 13 5    2 4 
r t f2         
27  3 3 3   3 27 

5 5 19 5  14
    0 (vi)
27  3  27
From (iv), (v) and (vi), we find that the conditions satisfied from these equations is for
minima.
For minimum canvas to be used y  2x . Ans.
Maxima and minima: Maxima and minima in case of two or more variables 22
Prepared by: Dr. Sunil, NIT Hamirpur (HP)

Q.No.13.: Find the stationary value of u  x 2  y 2  z 2 . If xy  yz  zx  3a 2 .


{Without Lagrange Theorem}

Sol.: u  x 2  y 2  z 2 (i)

xy  yz  zx  3a 2 (ii)
From (ii), we get
3a 2  xy
z (iii)
xy
Putting above value of z in (i), we get
2
2 3a 2  xy 
2
u  x y   .
 x  y 

u  3a 2  xy   x  y 
 2x  2  
.  y   1 3a 2  xy  
x  x  y   x  y 2 

 2x 
 
2 3a 2  y 2  xy  y 2  3a 2  xy   2x  23a 2

 y 2 y 2  3a 2 
x  y  3
x  y  3

 2x 

2 9a 4  y 4  (iv)
x  y  3

and
u  3a 2  y 2    x x  y   3a 2  xy 
 2y  2  
 
y  x  y   x  y 2 

 2y 

2 9a 4  x 4  (v)
x  y  3

u u
Now putting 0
x y

2x 

2 9a 4  y 4 0
x  y  3


 x x 3  y3  3xy 2  3x 2 y  9a 4  y 4  0   
 x 4  y 4  xy3  3x 2 y 2  3xy3  9a 4  0 (vi)

Similarly x 4  y 4  yx 3  3x 2 y 2  3yx 3  9a 4  0 (vii)


Maxima and minima: Maxima and minima in case of two or more variables 23
Prepared by: Dr. Sunil, NIT Hamirpur (HP)

Subtracting (vi) and (vii), we get


xy3  yx 3  3yx 3  3xy3  0  yx x 2  y 2  0 
 yxx  yx  y  0
Now, Case 1. x  0 . From (vi), we get

y 4  9a 4  y  3a .
Now putting above value in (iii), we get

3a 2
z  3a .
3a

Now, x 2  y 2  z 2  0  3a 2  3a 2  x 2  y 2  z 2  6a 2 (viii)
Case 2. y = 0. From (vi), we get

y 4  9a 4  y  3a .

and from (iii), we get z  3a


Now from (i), we get

Now, x 2  y 2  z 2  3a 2  0  3a 2  x 2  y 2  z 2  6a 2 (ix)
Case 3. Either x   y .
It is not possible as it does not satisfy (iii)
Case 4. If x = y.
From (vi), we get

x 4  x 4  x 4  3x 4  3x 4  9a 4  0  9 x 4  9a 4 .
3a 2  a 2
Using (iii), we get z  a.
2a
x  y  z  a (x)
Now, according to definition of stationary value f(a, b) is said to be stationary value of
f(x, y).
If f x a, b  0 and f y a , b  0

i. e. the function is stationary at [a, b].


Stationary value of function will be

u  x 2  y 2  z 2  u  a 2  a 2  a 2  3a 2 (xi)
Maxima and minima: Maxima and minima in case of two or more variables 24
Prepared by: Dr. Sunil, NIT Hamirpur (HP)

From (viii), (ix) and (xi) stationary value of u are 6a 2 and 3a 2 . Ans.
Q.No.14.: A rectangular box open at the top is to have volume equal to 32 cubic ft. Find
the dimensions of the box requiring least material for its construction.
Sol.: Let x , y and z ft. be the dimensions of the rectangular box open at the top so that
material for construction will be least if surface area is least.
Let surface area, S = F(x, y, z) = xy + 2yz + 2zx . ....(i)
Also given volume = xyz = 32. ....(ii)
Eliminating z from (i) with the help of (ii), we get
32  1 1
S  xy  2 y  x  xy  64   .
xy  x y

S 64 S 64
  y  2  0 and  x  2  0.
x x y y
Solving these, we get x = y = 4.

 2S 128  2S  2S 128
Now r   , s 1 ,t  2  3 .
x 2 x3 xy y y

At x = y = 4, rt  s 2  (2  2)  1  + ve and r is also + ve.


Hence S is minimum for x = y = 4. Then from (ii), z = 2.
Hence the dimensions of the box are x = 4ft.,y = 4ft. and z = 2ft.Ans.
1
Q.No.15.: Prove that maximum value of cos A cos B cos C  .
8
Sol.: cos A cos B cos C  cos A cos B cosW  A  B

  cos AcoaBcos(A  B)  A  B  C  W
f (A, B)  coaA cos B cosA  B
f A   cos B sin A.cos(A  B)  cos A.sin(A  B)
 cos Bsin A.cos(A  B)  cos A.sin(A  B)  cos B sin(2A  B)
f B   cos A sin B.cos(A  B)  cos B.sin(A  B)
 cos Asin B.cos(A  B)  cos B.sin(A  B)  cos A sin(A  2B)
r  f AA  2 cos B. cos(2A  B)
s  f AB   sin A sin(A  2B)  cos A cos(A  2B)  cosA  A  2B  cos(2A  2B)
Maxima and minima: Maxima and minima in case of two or more variables 25
Prepared by: Dr. Sunil, NIT Hamirpur (HP)

t  f BB  2 cos A cos(A  2B) .


For maximum and minimum values f A  0 and f B  0
cos B sin(2A  B)  0
cos A sin(A  2B)  0
From above equations, we get the following four pairs of equations
W W
(i) cos B  0 , cos A  0 , B  , A .
2 2
It is not possible as two angles in same triangle cannot be 90  .
W
(ii) cos B  0 , sin(A  2B)  0 , B  , A  2B  W  A  0 .
2
Not possible as in same triangle no angle can be zero.
(iii) Similarly, we reject pair
sin2A  B  0 , cos A  0 ,
sin2A  B  0
2A  B  0 (i)
A  2B  0 (ii)
Solving (i) and (ii), we get
W W
AB , AB
3 3
W
r  2 cos cos W  1 ,
3
4W  W  1
s  cos  cos W    ,
3  3  2
W
t  2 cos cos W  1 .
3
3
rt  s 2   0 and r  0 .
4
W
Hence f(A, B) is maximum at A  B  .
3
W W 2W 1
and maximum value   cos cos cos  . Ans.
3 3 3 8
Maxima and minima: Maxima and minima in case of two or more variables 26
Prepared by: Dr. Sunil, NIT Hamirpur (HP)

Q.No.17.: If x, y, z be the perpendicular from any point within the triangle on the sides
a, b, c of a triangle ABC of area  . Show that the minimum value of

42
x 2  y2  z 2  .
a 2  b2  c2
Sol.:  = Area of the triangle ABC = area of three small triangles with in the  ABC .
1 1 1
 ax  by  cz
2 2 2
 ax  by  cz  2 (i)

Let u  x 2  y 2  z 2 (ii)
1
From (i) cz  2  ax  by  z  2  ax  by
c
Put the value of z in (ii), we get
1
u  x 2  y2  2
2  ax  by 2 (iii)
c
Now differentiate (iii) w. r. t. x, we get
u 2
 2 x  2 2  ax  by  a  .
x c
u
For max. or min. 0
x
2
2x  2  ax  by  a   0
c2
c2x
 2  ax  by , (iv)
a
Now, again differentiating w. r. t. y, we get
u 2
 2 y  2 2  ax  by  b   0
y c

c2 y
 2  ax  by (v)
b
From (iv) and (v), we get

c2 x c 2 y b
 y x (vi)
a b a
Put the value of y in (iv), we get
Maxima and minima: Maxima and minima in case of two or more variables 27
Prepared by: Dr. Sunil, NIT Hamirpur (HP)

c2x b
 2  ax  b  x  c 2 x  2a  a 2 x  b 2 x
a a
2 a
x
a  b2  c2
2

Putting the value of x in (vi), we get


2 a b 2b
y 2 2 2
 y 2
a b c a a  b2  c2
2 c
Similarly, z 
a  b 2  c2
2

Putting these values of x, y, z in (ii), we get

u  x 2  y2  z 2


 2
2a  

 
2b
2
 

 
2c
2

 
2
42 a 2  b 2  c 2  
 a  b2  c 2   a 2  b 2  c 2   a 2  b 2  c2  a 2  b2  c2
2
 
42

a 2
 b 2  c2  . Ans.
Q.No.18.: A rectangular strip   b of metal is bent up at the sides to form a trough.
Without ends. Find the width of the side bases and the angle through which the
side must bent so that the trough may have a maximum capacity.
Sol.: Let the length  of the strip is bent up by an angle of  , the length of bent part of
strip are of equal on both sides by x.
Let h be the height and y be the base.
h y
 sin    h  x sin  , cos    h  x cos  .
x x
Let F be the capacity of the trough.
1
F    2x b  h  2   h  y  b    2x b  x sin   x sin .x cos .b
2


 b x  2x 2 sin    x2
2
b sin 2 .

F 2  b sin 2
Now, Fx   b  4x sin    b  4x sin   bx sin 2
x 2
Maxima and minima: Maxima and minima in case of two or more variables 28
Prepared by: Dr. Sunil, NIT Hamirpur (HP)

 2F
Fxx   b0  4sin   b sin 2  4 b sin   b sin 2
x 2

F 
F

 
 b x  2 x 2 . cos  
x 2b
2
 
.2 cos 2  b x  2 x 2 . cos   bx 2 cos 2

F 
 2F
 2
 
 b x  2x 2 .sin   2bx 2 sin 2

 2F
Fx   b  4 x . cos   2bx cos 2 .
x
For x be max. or min.
Fx  0 , F  0

Fx  0  b  4x sin   bx 2 sin  cos  .


b4x   
cos   .
2bx
Also, F  0

  
 b x  2 x 2 . cos   bx 2 2 cos 2   1  0 

 b x  2 x 2 .  b42xbx   2bx 2

b 2 4x   2
4x b 2 2
 bx 2  0

bx  2x 4x    b4x   2    2 x 4 x    4x   2   bx 2


   bx 2  0  b  b 
2x 2  2 2 


4x      2x   4x     x 2  4x   .
2x
 x2  
 x 2  x 4 x     0
2 2
 xx  4x     0
As x  0

x  4x     0  x  4x    0  3x    x  .
3

When x  .
3

b 4     4  3
 
3   1
cos     3    cos 60 .
 2 2 2
2b.
3 3
Maxima and minima: Maxima and minima in case of two or more variables 29
Prepared by: Dr. Sunil, NIT Hamirpur (HP)

   60  .

When   60  , x  .
3
3 3
Now Fxx  4b sin   b sin 2  4 b sin 60   b sin 120   4b   bx 
2 2
b 3  4b 3  b 3  3b 3  3 3 b
 2b 3     .
2 2 2 2

   
 3
2 
9 
2
F   b x  2 x 2 sin   2bx 2 sin 2  b .  2   sin 60  2b. sin 120
9

  2 2 2  3 2b 2 3  3 2  2 2  3 2b 2 3
 b         b    
 3 9  2 9 2  9  2 9 2
 

 b 2 3 2 3b 2  3 3b 2  3b 2


   
18 18 18 18
  
Fx  b  4x . cos   2bx cos 2  b     4.   60  2b. cos120
 3  3

    1 2b 1  b  2b  3b  b


 b       .
 3  2 3 2 6 6 2

  3 3b    3b 2    b  2 3  3b 2 2 b 2 2
Fxx  F  Fx2      
2   6   2  26 4
   

3b 2 2  b 2 2 2b 2 2
 
4 4
b 2 2
 0.
2
3 3 b
Also  0.
2
 
Hence the length have maximum capacity when x  ,   60  .
3 3
 
Hence width of bases  and angle .
3 3
Q.No.19.: Divide 24 into three parts such that the continued product of the first, the
square of 2nd and the cube of the third may be a maximum.
Maxima and minima: Maxima and minima in case of two or more variables 30
Prepared by: Dr. Sunil, NIT Hamirpur (HP)

Sol.: Let the three parts be x, y and 24  x  y

And let, f  24  x  y y 2 x 3


f
Then,  3x 2 y 2 24  x  y    1y 2 x 3  72 x 2 y 2  4 x 3 y 2  3x 2 y3 (i)
x
 2f
2
 144 xy 2  12 x 2 y 2  6 xy3
x
f
and  2 yx 3 24  x  y    1y 2 x 3  48yx3  2 yx 4  3y 2 x 3 (ii)
y

 2f
2
 48x 3  2 x 4  3yx 3
y

 2f
From (i) we can get  144 x 2 y  8x 3 y  9 x 2 y 2 (iii)
xy

f f
F has maximum and minimum, then 0 and 0
x y
From (i), we get
 f 
72x 2 y 2  4x 3 y 2  3x 2 y3  0  x  0
 
 72  4x  3y  0  4x  3y  72 (iv)
From (ii), we get
 f 
48yx 3  2 yx 4  3y 2 x 3  0   0
 y 
 48  2x  3y  0  2x  3y  48 (v)
Using (iv) and (v), we get
x  12 and y  8

Now f having maximum and minimum if at x  12 and y  8


2
 2f  2f   2f 
.  0
x 2 y 2  xy 

 
 144  12  82  12  122  82  6  12  83 48123  2124  6  8123 

 144  122  8  8  123  8  9  (12) 2  83  0 
Maxima and minima: Maxima and minima in case of two or more variables 31
Prepared by: Dr. Sunil, NIT Hamirpur (HP)

 1528754688  0
So, f having maximum and minimum at x  12 and y  8

  2f 
Now f has maximum value if  2 0
 x  x 12
y 8

 
 144  12  82  12  122  82  6  12  83  36864  0
So f having maximum and minimum if at x  12 and y  8 .

So the required three parts are 12, 8 24 - 12  8  12, 8, 4 . Ans.

Q.No.20.:. Examine f x   x 3  y3  3xy for maximum and minimum values.


Sol.: For maxima and minima values. Solve two equations
f f
0 and 0
x y

i. e. 3x 2  3y  0 and 3y 2  3x  0
Solve these equations, we get

x2  y  0 and y 2  x  0

 y 4  y  0  y  0,  1 .
Similarly, y  0,  1 .

So, we get four points (0, 0), 0,  1, (1, 0) and (1,  1) .
For maxima or minima 2nd condition is
2
 2f  2f   2f  1
. 2   6x.6 y  9  xy  .
2  
x y  xy  4

Only point (1,  1) satisfies this condition .

Now for maxima f xx  0 at point (1,  1) will  that this is maxima.

f xx  6x  6  0 at (1,  1) . So this point is a maximum value and no minimum


value of function.
Now, condition for saddle point.

 2  xy  14 .
i. e. f xx .f yy  f xy
Maxima and minima: Maxima and minima in case of two or more variables 32
Prepared by: Dr. Sunil, NIT Hamirpur (HP)

Three points (0, 0), 0,  1, (1, 0) satisfies the above equation.

Therefore these points are saddle points for the curve x 3  y3  3xy .
Q.No.21.: Find the shortest and longest distance of the point (1, 2,  1) to the sphere

x 2  y 2  z 2  24 .
Sol.: Let us consider a point A(x, y, z) on the surface of the sphere.

 AP  x  12  y  22  z  12


Let, F  x  12  y  2 2  z  12  x 2  2x  1  y 2  4 y  4  z 2  2z  1

 x 2  y 2  2 z  2 x  4 y  2z  6 (i)

Now the given equation of sphere is x 2  y 2  z 2  24 (ii)


From (i) and (ii), we get
F  24  2x  4 y  2z  6  30  2x  4 y  2z

also from (ii), we get z   24  x 2  y 2

When z  24  x 2  y 2 i. e. positive, then

F  30  2 x  4 y  2 24  x 2  y 2

2 2 x   2x 
 Fx  2   2 (iii)
2 24  x 2  y 2 24  x 2  y 2

2 2 y   2 y 
Fy  4  4 (iv)
2 24  x 2  y 2 24  x 2  y 2
For maximum or minimum.
Fx  0 and Fy  0

 2x  2  0
 2x
 2   x  24  x 2  y 2
2 2 2 2
24  x  y 24  x  y
Squaring both sides, we get

x 2  24  x 2  y 2  2 x 2  y 2  24 (v)
 2 y   4  0   y  2 24  x 2  y 2
2 2
24  x  y
Maxima and minima: Maxima and minima in case of two or more variables 33
Prepared by: Dr. Sunil, NIT Hamirpur (HP)

Squaring both sides, we get

 
y 2  4 24  x 2  y 2  4 x 2  5 y 2 96 (vi)
Multiplying (v) by 2 and subtracting from (vi), we get

3y 2  48  y 2  16  y  4 .
But from (iv) y should be negative
 y  4
1
x 24  16  4  2 .
2
But from (iii) x should be negative. x  4

 z  24   2 2   4 2  24  4  16  4  2 .

Hence the points are  2,  4, 2 .

 48  2 y 2
Now Fxx  .
24  x 2
 y2 
3/ 2

 48  32  16
Fxx at  2,  4, 2    2 .
24  4  16 3/ 2 8

 48  2x 2  48  8
Fyy at  2,  4, 2    5 .
24  x 2
 16 
3/ 2 8

 2xy  28
Fxy at  2,  4, 2    2 .
24  x 2
 16 
3/ 2 8

For maxima or minima

 2  0
Fxx .Fyy  Fxy

  2  5   2 2  0  10  4  0  6  0 , which is true.

Also Fxx  2  0 .

  2,  4, 2 are the points of maxima for F.

Fmax .   2  12   4  2 2  2  12 9  36  9  54 .

When F is maximum then AP is also maximum.


 APmax .  54 .
Maxima and minima: Maxima and minima in case of two or more variables 34
Prepared by: Dr. Sunil, NIT Hamirpur (HP)

Now, when z   24  x 2  y 2 i. e. negative.

Then, F  30  2 x  4 y  2 24  x 2  y 2

2x
 Fx  2 (vii)
24  x 2  y 2
2y
Fy  4 (viii)
24  x 2  y 2
For max. or min.
Fx  0 and Fy  0

 2x  2  0
 2x
 2   x  24  x 2  y 2
24  x 2  y 2 24  x 2  y 2
Squaring both sides, we get

x 2  24  x 2  y 2  2 x 2  y 2  24
 2 y   4  0   y  2 24  x 2  y 2
2 2
24  x  y
Squaring both sides, we get

 
y 2  4 24  x 2  y 2  4 x 2  5 y 2 96
Solving above equations, we get
x  2 and y  2
But from (vii) and (viii) x and y should be positive
 x  2 and y  2 and z   2 .

Hence the points are 2, 4,  2

 24  y 2   24  16 
Fxx at 2, 4,  2  2  .  2
3/ 2 
2
 
 24  x 2  y 2 
3/ 2

 
 24  4  16  

 24  x 2 
Fyy at 2, 4,  2  2    5.

 24  x 2  y 2
 
3/ 2


2xy 16
Fxy at 2, 4,  2 2 2
  2.
24  x  y 8
Maxima and minima: Maxima and minima in case of two or more variables 35
Prepared by: Dr. Sunil, NIT Hamirpur (HP)

Now, for max. or mini.

 2  0
Fxx .Fyy  Fxy

 10  4  0  6  0 , which is true.
Also, Fxx  2  0 .

 2, 4,  2 are the point of minimum for F.


when F is minimum AP is also minimum

 APmin .  2  12  4  22   2  12  6.

The longest distance of point (1, 2,  1) from sphere x 2  y 2  z 2  24 is 54 and the

shortest distance is 6 . Ans.

Q.No.22.: Find the extreme values of x 2  y 2  z 2 , when ax  by  cz  P .


Sol.: A maximum or minimum value is called extreme value of the function.

Let u  x 2  y 2  z 2 (i)
And also ax  by  cz  P
P  ax  by
z 
c
Putting in (i), we get
1
u  x 2  y2  2
P  ax  by 2 (ii)
c
Differentiate partially (ii) w. r. t. x, we get
u 2
 2x  2 P  ax  by   a
x c
Differentiate partially (ii) w. r. t. y, we get
u 2
 2 y  2 P  ax  by   b
y c
for max. and mini. Values both
u u
 0 and 0
x y
2
2x   a P  ax  by   0 (iii)
c2
Maxima and minima: Maxima and minima in case of two or more variables 36
Prepared by: Dr. Sunil, NIT Hamirpur (HP)

2b
2y  P  ax  by   0 (iv)
c2
Solving (iii) and (iv), we get
aP  aby
x
c2  a 2
Put in (iv), we get
 aP  aby 
c 2 y  bP  ab 2
 c a  2
2 2 2 2 2

  b y  0  c  a c  b y  abaP  aby  bP c  a
2 2
   
bP
y  ,
a  b2  c2
2

aP cP
and also x  2 2 2
, z .
a b c a  b 2  c2
2

 2u 2a 2a 2
 2u 2b 2  2u ab
Now  2  a   2  and  2 , 
x 2 c 2 2
c y 2 c2 xy 2 c2

 2u  2 u   2 u 
 2. 2   42
b 2  a 2 a 2b 2
 4 0
2
 
x y  x.y  c 2
c
 
2
 2 u  2 u   2u 
Clearly .  0
x 2 y 2  x.y 

 2u 2b 2
Also  2  0.
x 2 c2
aP bP cP
u has minimum value at x  2 2 2
, y 2 2 2
and z  .
a b c a b c a  b2  c2
2

And minimum value of x 2  y 2  z 2

a 2P 2 b 2P 2 c 2P 2 P2
    . Ans.
a 2
b c 2
 a
2 2 2 2
b c 2 2
 a 2
b c 2

2 2 a 2  b2  c2

Q.No.23.: Find the maximum and minimum values of

f ( x , y)  x 3  3xy 2  15 x 2  15 y 2  72 x .
Sol.: Differentiating f partially w.r.t. x and y, we get
f
fx   3x 2  3y 2  30x  72 ,
x
Maxima and minima: Maxima and minima in case of two or more variables 37
Prepared by: Dr. Sunil, NIT Hamirpur (HP)

f
fy   6 xy  30 y .
y
The stationary (critical) points are given by fx = 0 and fy = 0.
From f y  6 xy  30 y  0  6 y( x  5)  0 .

Thus, either y  0 or x = 5.

Since f x  3x 2  3y 2  30 x  72  0 .

For y = 0, 3x 2  30 x  72  0  x = 6 or 4.

For x = 5, 75  3y 2  150  72  0  y  1
Thus, the four stationary points are given by
(6, 0), (4, 0), (5, 1), (5,  1) .
To determine the nature of these points, calculate fxx, fyy and fxy, we obtain
f xx  A  6x  30 , f xy  B  6 y , f yy  C  6 x  30 .


Thus, AC  B 2  6 x  30 2  36 y 2  36 x  52  y 2 
1. At the stationary point (6, 0), we have A  36  30  6  0 and AC  B 2  36  0 .
So (6, 0) is a minimum point of the given function f and the minimum value of f at (6,

0) is 6 3  0  15.36  72.6  108 .

2. At (4, 0): A  24  30  6  0 and AC  B 2  36  0 . So a maximum occurs at the


point (4, 0) and the maximum value of f at (4, 0) is 112.

3. At (5, 1), A = 0, AC  B 2  36  0 . So (5, 1) is a saddle point (It is neither


maximum nor minimum).
4. At (5,1) , A = 0, AC = B2   36  0 . So (5,1) is neither a maximum nor a
minimum (it is a saddle point).

Q.No.24.: Find the shortest distance from origin to the surface xyz 2  2 .

Sol.: Given equation of surface is xyz 2  2 .


Let d be the distance from origin (0, 0, 0) to any point (x, y, z) of the given surface, then

d x  02  y  02  z  02  d2  x 2  y2  z2 .


Maxima and minima: Maxima and minima in case of two or more variables 38
Prepared by: Dr. Sunil, NIT Hamirpur (HP)

2
Eliminating z2 using the equation of the surface xyz 2  2 . So replace z 2  .
xy
2
 d2  x 2  y2   f ( x , y) .
xy
2 2
Now f x  2 x  , f y  2y  .
x2y xy 2
The stationary (critical) points are given by fx = 0 and fy = 0.
Solving fx = 0 and fy = 0, we get

x3y 1 xy 3  1
 0 and 0
x2y xy 2

 
x 3 y  1  xy 3  xy x 2  y 2  0 .
Since x  0 , y  0 , so x   y  1.

Thus, the two stationary points are (1, 1) and  1,1 .


4 4 2
Now f xx  2  , f yy  2  , f xy  .
3 3 2 2
x y xy x y
2
At (1, 1): f xx  6  0 , f xx .f yy  f xy  6.6  6  32  0 .

At  1,1 : f xx  6.f xx f yy  f xy
2
 32 .

So minimum occurs at (1, 1, 2 ) and ( 1,  1, 2) .

Hence the shortest distance is 12  12   2 2  4  2 . Ans.


Q.No.25.: The temperature T at any point (x, y, z)in space is T(x, y, z) = kxyz2, where k
is a constant. Find the highest temperature on the surface of the sphere

x 2  y2  z2  a 2 .
Sol.: Given the temperature T at any point (x, y, z) in space is T(x, y, z) = kxyz2.

Eliminating the variable z, using z 2  a 2  x 2  y 2 , we get

T ( x , y, z)  kxyz 2  kxy (a 2  x 2  y 2 )  F( x , y) .

Now Fx  ky(a 2  3x 2  y 2 ) , Fy  kx (a 2  x 2  3y 2 ) .

The stationary (critical) points are given by Fx = 0 and Fy = 0.


Maxima and minima: Maxima and minima in case of two or more variables 39
Prepared by: Dr. Sunil, NIT Hamirpur (HP)

ky (a 2  3x 2  y 2 )  0 , kx (a 2  x 2  3y 2 )  0

 3x 2  y 2  a 2 , x 2  3y 2  a 2 and x =0, y = 0
a
Solving, we get x  y   .
2


Also Fxx  6kxy , Fyy  6kxy , Fxy  k a 2  3x 2  3y 2 . 
At (0, 0), Fxx  0  Fyy , Fxy  ka 2 .

 0.0  ka 2  0  (0, 0) is a saddle point.


a a a a
At both the points  ,  and  , ,
2 2  2 2 

ka 2 2 9 a 4k 2
Fxx  6  0 and Fxx .Fyy  Fxy  k 2a 4   2k 2 a 4  0 .
4 4 4
a a a a
 T attains a maximum value at both these points  ,  and  , .
2 2  2 2 

a2  a 2  ka 4
The maximum value of T is k.   . Ans.
4  2  8
 
Q.No.26.: Find the shortest distance between the lines
x 3 y5 z7
 
1 2 1
x 1 y 1 z 1
and  
7 6 1
Sol.: The given equations of lines are
x 3 y5 z7
  (i)
1 2 1
x 1 y 1 z 1
and   (ii)
7 6 1
Equating each of the fractions of (i) to  , we get x  3   , y  5  2 , z  7   .
Thus any point P on the first line (i) is given by
3  , 5  2, 7   .
Similarly, any point Q on the second line (ii) is  1  7,  1  6,  1    .
Maxima and minima: Maxima and minima in case of two or more variables 40
Prepared by: Dr. Sunil, NIT Hamirpur (HP)

The distance between the given

PQ  3    1  7 2  5  2  1  6 2  7    1   2 .
Consider f (, )   PQ   6 2  86 2  40  116 .
2

The problem is to find minimum value of f as a function of the two variables ,  .

f   12  40 , f   172  40 .

Solving 12  40  0 and 172  40  0 , we get


  0 and   0 as the only stationary point.

f   12 , f   172 , f   40 .

Now f   12  0 and  f  f   f    12 172   40   2064  1600  464  0 .


2 2

Thus a minimum occurs at   0,   0 .


The minimum (shortest) distance is given by

PQ  4 2  6 2  8 2  116  2 29 . Ans.

Home Assignments
Q.No.1.: Test the functions for maxima, minima and saddle points:

a. x 4  y 4  x 2  y 2  1 .

b. x 2  2 y 2  3z 2  2 xy  2 yz  2 .
Ans.: a. Maximum at (0, 0)
Maximum value is 1.

Minima at four points  1 / 2 ,  1/ 2 
1
Minimum value at these 4 points is .
2
 
Saddle points at four points 0,  1 / 2 ,  1/ 2 , 0 
b. Maximum at (1, 1), minimum at  1,  1 .

Q.No.2.: Find the extrema of f(x, y) : x 2  y 2 e 6x  2x 2


.
Maxima and minima: Maxima and minima in case of two or more variables 41
Prepared by: Dr. Sunil, NIT Hamirpur (HP)

Ans.: Minima at (0, 0) (minimum value 0) and at  1, 0 (minimum value e 4 ) .

 1 
Saddle point at   , 0  .
 2 
Q.No.3.: Examine the following function f(x, y) for extrema : sin x  sin y  sin(x  y) .

  3 3
Ans.: Maximum at  ,  , maximum value .
3 3 2
Q.No.4.: Find the shortest distance from the origin to the plane x  2 y  2z  3 .

1 2 2
Ans.: Shortest distance is 1 (from 0, 0, 0) to the point  ,  ,   on the plane.
3 3 3
Q.No.5.: Find the shortest distance between the lines
x 2 y6 z5
 
3 2 2
x 5 y 3 z 8
and   .
2 1 6
Ans.: Shortest distance is 3 between the points (5, 4, 3) and (3, 2, 2).
Q.No.6.: If the perimeter of a triangle is constant, prove that the area of this triangle is
maximum, when the triangle is equilateral.
2s
Ans.: Maximum when a  b  c  .
3
Q.No.7.: Find the volume of the largest rectangular parallelopiped with edges parallel to
the axes, that can be inscribed in the:
a. Sphere

b. Equation of the ellipsoid is 4 x 2  4 y 2  9 z 2  36 .

8a 3 a
Ans.: a. Volume : , xyz
3 3 3

b. Volume: 16 3 (with a = 3, b = 3, c = 2).


Q.No.8.: Find the dimensions of a rectangular box, with open top, so that the total surface
area of the box is a minimum, given that the volume of the box is constant say
V.
Ans.: x = y = 2z = (2V)1/3 .
Maxima and minima: Maxima and minima in case of two or more variables 42
Prepared by: Dr. Sunil, NIT Hamirpur (HP)

Q.No.9.: Find the dimensions of the rectangular box, with open top, of maximum
capacity whose surface area is 432 sq. cm.
Ans.: 12, 12, 6.
Q.No.10.: If the total surface area of a closed rectangular box is 108 sq. cm., find the
dimensions of the box having maximum capacity.
Ans.: 18 , 18 , 18 .
*** *** *** *** ***
*** *** ***
***
Differential Calculus

Maxima and Minima


(Lagrange’s method of undetermined multipliers)

Prepared by:
Dr. Sunil Chaudhary
NIT Hamirpur (HP)
(Last updated on 11-09-2009)
Latest update available at: http://www.freewebs.com/sunilnit/

(33 Solved problems and 00 Home assignments)

Lagrange’s Method of undetermined multipliers:


Q.: Write a short note on “Lagrange’s Method of undetermined multipliers”.
What are the advantages and disadvantages of this method.
In many practical and theoretical problems, it is required to find the maximum or
minimum of a function of several variables, where the variables are connected by some
given relation or condition known as a constraint condition(s). Thus, if f(x, y, z) is a
function of 3 independent variables, where x, y, z are related by a known constraint
x , y, z   0 , then the problem of constrained extrema consists of finding the Extrema
of
u = f(x, y, z) (i)
subject to constraint condition x , y, z   0 . (ii)
Now these type of problems can be solved by
(a) Elimination method
(b) Implicit differentiation
(c) Lagrange’s multiplier’s method
Maxima and minima: Lagrange’s method of undetermined multipliers 2
Prepared by: Dr. Sunil, NIT Hamirpur (HP)

(a) Elimination method: In this method, the constraint condition (ii) is solved for one
variable, say z, in terms of the other variables x and y. Then z is eliminated from f(x,
y, z), we get a function of two variables x and y only. The disadvantage of this
method is that many times, (ii) may not be solvable and in case of solution also the
amount of algebra will be generally huge.
(b) Implicit differentiation: In this method, no elimination of variables is done but
derivative are eliminated by calculating them through implicit differentiation. This
method also suffers due to more labour involved.

Remarks: If u  f x, y  then the function is said to have a maximum or minimum if

f x  0 , f y  0 , at (a, b).

For a max. : f xx  0 , f xx .f yy   f xy  at (a, b).


2

For a min. : f xx  0 , f xx .f yy   f xy  at (a, b).


2

For a saddle point, we have f xx .f yy   f xy  at (a, b).


2

If f xx .f yy   f xy  at (a, b). Then further investigations are necessary.


2

In the above case, the variables x, y are independent.


There are several situations, where function to be maximized or minimized,
depend upon variables, which are not independent, but are inter-related by one or more
constraint conditions. So in that situation, we may use Lagrange’s method.
(c) Lagrange’s method: The very useful “Lagrange’s method of undetermined
multiplier’s” introduces an additional unknown constant  known as Lagrange’s
multiplier.
Let f(x, y, z) be a function of x, y, z which is to be examined for maximum or minimum
value.
Let the variables x, y, z be connected by the relation x , y, z   0 . (i)
For f(x, y, z) to have a maximum or minimum value, the necessary conditions are
f f f
 0,  0,  0.
x y z
Maxima and minima: Lagrange’s method of undetermined multipliers 3
Prepared by: Dr. Sunil, NIT Hamirpur (HP)

f f f
 dx  dy  dz  0 . (ii)
x y z
  
Also, from (i), taking differential, we get dx  dy  dz  0 . (iii)
x y z
Multiplying (iii) by a parameter  and adding to (ii), we get

 f    f    f  
   dx     dy     dz  0 .
 x x   y y   z z 

This equation will hold good if


f  f  f 
  0,   0,   0. (iv)
x x y y z z

These equations together with constraint condition (i), give the values of x, y, z and  for
a maximum or minimum.
Remarks: The above equations can be easily obtained by considering Lagrange’s
(auxiliary) function.
F( x , y, z )  f ( x , y, z )   ( x , y, z )
and considering the stationary values of F(x, y, z).
For stationary values of F(x, y, z), dF = 0.

 f    f    f  
    dx     dy     dz  0 .
 x x   y y   z z 

f  f  f 
   0,   0,   0.
x x y y z z
Advantages:
1. The stationary value f(x, y, z) can be determined from (i) and (iv) even without
determining x, y, z explicitly.
2. This method can be extended to a function of several ‘n’ variables x1 , x 2 , x 3 ,......, x n
and subject to many (more than one) ‘m’ constraints by forming the auxiliary equation
m
Fx1 , x 2 ,....., x n   f x 1 , x 2 ,....., x n     i  i x 1 , x 2 ,....., x n  .
i 1

The stationary values are obtained by solving the (n + m) equations consisting of


Maxima and minima: Lagrange’s method of undetermined multipliers 4
Prepared by: Dr. Sunil, NIT Hamirpur (HP)

F
n equations  0 , for i = 1, 2, 3, ….,n and
x i

m constraint conditions,  i  0 for i = 1, 2, 3, ….,m.


Disadvantages:
1. Nature of the stationary points cannot be determined. Further investigation
needed. This means that Lagrange’s method does not enable us to find weather
there is maximum or minimum. This fact is determined from the physical
considerations of the problem.
2. Equations (iv) are only necessary conditions but not sufficient.

Now let us find maxima and minima of some problems:

Q.No.1: A rectangular box open at the top is to have volume of 32 cubic ft. Find the
dimension of the box requiring least material for its constructions.
Sol.: Let x, y and z ft. be the edges of the box and S be its surface.
Then S  xy  2 yz  2zx (i)
and xyz = 32 (ii)
Write F  xy  2 yz  2zx  xyz  32
F
Then  y  2z  yz  0 (iii)
x
F
 x  2z  zx  0 (iv)
y
F
 2 y  2 x  xy  0 (v)
z
Multiply (iii) by x and (iv) by y and subtracting, we get
2zx  2zy  0  x  y .
[The value z = 0 is neglected, as it will not satisfy (ii)]
Again multiply (iv) with y and (v) with z and subtracting, we get y = 2z.
Hence the dimension of the box is x = y = 2z = 4. (vi)
Now let us see what happens as z increases from a small value to a large one. When z is
small, the box is flat with the large base showing that S is large. As z increases, the base
Maxima and minima: Lagrange’s method of undetermined multipliers 5
Prepared by: Dr. Sunil, NIT Hamirpur (HP)

of the box decreases rapidly and S also decreases. After a certain stage, S again starts
increasing as z increases. Thus S must be a minimum at some intermediate stage which is
given by (vi). Hence S is minimum when x = y = 4 ft. and z = 2 ft.
Q.No.2.: Of all the rectangular parallelopipeds which have sides parallel to the co-

x2 y2 z2
ordinate planes and which are inscribed in the ellipsoid    1,
a2 b2 c2
find the dimensions of that one which has the largest possible volume.
or
Find the volume of the greatest rectangular parallelopiped that can be inscribed

x2 y2 z2
in the ellipsoid    1.
a2 b2 c2
Sol.: Let the edges of the parallelopiped be 2x , 2y and 2z which are parallel to the axes.
Then its volume V = 8xyz.
Now we have to find the maximum value of V subject to the condition that

x2 y2 z2
  1  0 . ...(i)
a2 b2 c2
 x 2 y2 z2 
Write F  8xyz   2  2  2  1
a b c 
 
F  2x 
Then  8 yz   2   0 ...(ii)
x a 
F  2y 
 8zx   2   0 ...(iii)
y b 
F  2z 
 8xy   2   0 ...(iv)
z c 

x2 y2
Equating the values of  from (ii) and (iii), we get  .
a2 b2
y2 z2
Similarly from (iii) and (iv), we obtain  .
b2 c2
x2 y2 z2
 2  2  2.
a b c
Maxima and minima: Lagrange’s method of undetermined multipliers 6
Prepared by: Dr. Sunil, NIT Hamirpur (HP)

x2 1
Substituting these in (i), we get 2
 .
a 3

x2 y2 z2 1 a b c
i.e. 2
 2
 2
 . These give x  , y ,z  .Ans.
a b c 3 3 3 3
When x = 0, the parallelopiped is just a rectangular sheet and as such its volume V = 0.
As x increases, V also increases continuously.
Thus V must be greatest at the stage given by (v).
8abc
Hence the greatest volume = .
3 3
Q.No.3.: A rectangular tank open at the top is to have a volume of 4 cubic meters . Find
its dimensions so that material used is minimum.
Sol.: Let x , y and z mts. be the dimensions of the rectangular tank open at the top so that
material for construction will be least if surface area is least.
Let surface area, S = f(x, y, z) = xy + 2yz + 2zx . ....(i)
Also given volume = xyz = 4. ...(ii)
Write F  xy  2 yz  2zx  xyz  4 
F
Then  y  2z  yz  0 ...(iii)
x
F
 x  2z  zx  0 ...(iv)
y
F
 2 y  2 x  xy  0 ...(v)
z
Multiplying (iii) by x and (iv) by y and subtracting, we get
2 zx  2 zy  0  x  y .
[The value z = 0 is neglected, as it will not satisfy (ii)]
Again multiplying (iv) by y and (v) by z and subtracting, we get y = 2z .
Hence the dimensions of the box are x = y = 2z = 2. i.e. x = 2m.,y = 2m. and z = 1m.Ans.
Q.No.4.: Of all the rectangular boxes having the same surface area, find that one, which
encloses the maximum volume. Use Lagrange’s method.
Sol.: Let its sides be x, y, z
S. A.  2xy  yz  zx  , V  xyz .
Maxima and minima: Lagrange’s method of undetermined multipliers 7
Prepared by: Dr. Sunil, NIT Hamirpur (HP)

F  xyz  2 xy  2 yz  2zx  k   xyz  2xy  2yz  2zx  k


F
 yz  2y  2z  0 (i)
x
F
 xz  2x  2z  0 (ii)
y
F
 xy  2x  2y  0 (iii)
z
From (i) and (ii), we get
y  x z  2y  x   0  y  x   2  z   0
z
 y  x,  .
2
From (ii) and (iii), we get
z  y x  2z  y   0
x
 z  y,  . x  y  z.
2
Hence the rectangle box with maximum volume is a cube. Ans.

Q.No.5.: Find the maximum value of 400xyz2 on a sphere x 2  y2  z 2  1 .


or
The temperature T at any point (x, y, z) in space is T = 400xyz2. Find the

highest temperature on the surface of the unit sphere x 2  y2  z 2  1 .


Sol.: Let f(x, y, z) = 400xyz2 . ....(i)

Also the given equation of sphere is x 2  y2  z 2  1 . ...(ii)

Now f is to be maximized subject to the constraint x 2  y2  z 2  1 i.e. x , y, z   0 .


Let F  f   where  is Lagranges's multipliers.
2 2 2
 2
 F  400xyz   x  y  z  1 . 
F
Then  400 yz 2   2 x  0 ...(iii)
x
F
 400xz 2   2 y  0 ...(iv)
y
Maxima and minima: Lagrange’s method of undetermined multipliers 8
Prepared by: Dr. Sunil, NIT Hamirpur (HP)

F
 800 xyz   2 z  0 ...(v)
z
Multiplying (iii) by x and (iv) by y and subtracting, we get

 
2 x 2  y 2  0  x  y .[The value x = y is neglected] ...(vi)
Multiplying (iv) by 2y and (v) by z and subtracting, we get

 
2 2 y2  z 2  0  z  2 y . ...(vii)
1
From (vi) and (vii) , we get x  y  z .
2
Substitute these values in (ii), we get
1
x 2  x 2  2x2  1  x  .
2
1 1 1 1
x  , y , z 2.  .
2 2 2 2
2
1 1  1 
Maximum value of f(x, y, z) = 400xyz2 = 400  . .   50 . Ans.
2 2  2
Q.No.6.: Find the maximum and minimum distances of the point (3, 4, 12) from the

sphere x 2  y 2  z 2  1 .
or
Find the maximum and minimum distances of the point (3, 4, 12) from the
unit sphere with centre at origin.
Sol.: Let (x, y, z) be any point on the sphere.

Distance of the point A(3, 4, 12) from (x, y, z) is given by x  32  y  42  z  12 2
If the distance is maximum or minimum, so will be the square of the distance.

Let f x, y, z   x  32  y  42  z  122

subject to the condition that x, y, z   x 2  y 2  z 2  1  0 . (ii)


Consider Lagrange’s function
Fx , y, z   f x , y, z   x , y, z 


 x  32  y  42  z  122  λ x 2  y 2  z 2  1 
Maxima and minima: Lagrange’s method of undetermined multipliers 9
Prepared by: Dr. Sunil, NIT Hamirpur (HP)

F F F
For stationary value, dF = 0  dx  dy  dz  0 .
x y z

 2x  3  2λx dx  2y  4  2λydy  2z  12  2λzdz  0


 2x  3  2λx  0 , (iii)
2y  4   2λy  0 , (iv)
2z  12   2λz  0 . (v)
Multiplying (iii) by x, (iv) by y, and (v) by z and adding, we get

   
2 x 2  y2  z 2  6x  8y  24z  2λ x 2  y 2  z 2  0
 2  6 x  8 y  24z  2λ [using (ii)]
 3x  4 y  12z  1  λ . (vi)
From (iii), (iv) and (v), we get
3 4 12
x , y , z .
1 λ 1 λ 1 λ
Putting these values in (vi), we get
9 16 144
   1 λ  1   2  169  1    13
1 λ 1 λ 1 λ
 λ  12 or  14 .
3 4 12
When λ  12 , x , y , z .
13 13 13
3 4 12
When λ  14 , x , y , z .
13 13 113
 3 4 12   3 4 12 
Thus we get two points P , ,  and Q  ,  ,  
 13 13 13   13 13 13 
on the sphere which are at a maximum or minimum distance from the given point A.
2 2
 3  4  12 
Now AP   3     4    12    12
 13   13   13 

2 2
 3  4  12 
AQ   3     4    12    14 .
 13   13   13 

 3 4 12 
 P , ,  is at a minimum distance from A and the minimum distance = 12.
 13 13 13 
Maxima and minima: Lagrange’s method of undetermined multipliers 10
Prepared by: Dr. Sunil, NIT Hamirpur (HP)

 3 4 12 
Q  ,  ,   is at a maximum distance from A and the maximum distance = 14.
 13 13 13 

x2 y2 z2
Q.No.7.: Prove that the stationary values of u    , where
a4 b4 c4
x2 y2 z2
x  my  nz  0 and    1 are the roots of equation
a2 b2 c2
 2a 4 m2b4 n 2c 4
   0.
1  a 2u 1  b2u 1  c 2u
Sol.: Consider Lagrange’s function,
 x 2 y2 z2   x 2 y2 z 2 
Fx, y, z    4  4  4   λx  my  nz   μ 2  2  2  1 .
a b c  a b c 
  
For stationary values, dF  0
 2x 2μx   2y 2μy   2z 2μz 
  4  λ  2 dx   4  λm  2 dy   4  λn  2   0
a a  b b  c c 
2x 2μx
 4
 λ   0, (i)
a a2
2y 2μy
4
 λm   0, (ii)
b b2
2z 2μz
4
 λn   0. (iii)
c c2
Multiplying (i), (ii), (iii) by x, y, z respectively and adding, we get
 x 2 y2 z 2   x 2 y2 z 2 
2  4  4  4   λx  my  nz   2μ 2  2  2   0
a b c  a b c 
 
 2u  λ0   2μ 1  0 [From given relation]
 μ  u .

1  a u   λ  x   21λaa 2u  .
4
2x 2ux 2x
 Equation (i) becomes    0 2
a4 a2 a4

λmb 4 λnc 4
Similarly, y   , z

2 1  b2u  
2 1  c 2u .
Maxima and minima: Lagrange’s method of undetermined multipliers 11
Prepared by: Dr. Sunil, NIT Hamirpur (HP)

To eliminate λ between them, multiply these values of x, y, z by , m, n respectively

   2a 4 m2b4 n 2c 4 
and add. Then x  my  nz     
2 1  a 2 u 1  b 2 u 1  c2 u 

 2a 4 m2b4 n 2c 4
Since x  my  nz  0 , we have    0,
1  a 2u 1  b2u 1  c 2u
which is a quadratic in u and gives two stationary values of u.
Q.No.8: Use Lagrange’s method to determine the minimum distance from the origin to
the plane 3x  2 y  z  12 .
Sol.: The distance of any point P(x, y, z) on the plane 3x  2 y  z  12  0 from the

origin is given by x 2  y 2  z 2

or the square of the distance is equal to x 2  y 2  z 2 .

Let f ( x, y, z)  x 2  y2  z 2 . (i)
Now f is to be maximized or minimized subject to the constraint.
3x  2 y  z  12  0 i. e. x , y, z   0 . (ii)
Let F  f   , where  is Lagrange’s multipliers. Since f is to be extremised, we have
F f  3
0    0  2x  3  0  x  .
x x x 2
F f 
0    0  2 y  2  0  y    .
y y y
F f  
0    0  2z    0  z  .
z z z 2
Putting the values of x, y, z in (ii), we get
3  
3    2       12  0 .
 2   2 
 12 18 12 6
 x , y , z .
7 7 7 7
2 2 2
2 2  18   12   6 
2
 f ( x, y, z)  x  y  z          .
 7   7  7
Maxima and minima: Lagrange’s method of undetermined multipliers 12
Prepared by: Dr. Sunil, NIT Hamirpur (HP)

2 2 2
 18   12   6  12
Minimum distance           . Ans.
 7   7  7 14
Distance required has to be minimum, since the maximum distance can be as large as we
like i.e., infinity.

Q.No.9: Find the shortest distance between the circle x 2  y 2  1 and the straight line

y  3 2  x  .

Sol.: Let Px1, y1  and Qx 2 , y 2  be the points on the circle and straight line respectively
such that the distance ‘d’ between P and Q is given by

d 2  x 2  x1 2  y 2  y1 2 .
Now d is minimum if,
F F F F
0 and 0 ,
x1 x 2 y1 y 2
where F is the Lagrange’s function given by

    
F  x 2  x1 2  y2  y1 2  1 x12  y12  1   2 y2  3x 2  2 3 
F
 0 , gives  2x 2  x1   21x1  0 , (i)
x1
F
 0 , gives 2x 2  x1   3 2  0 . (ii)
x 2
F F
Similarly,  0 , and 0
y1 x 2

 2y 2  y1   21y1  0 , (iii)


2y 2  y1    2  0 . (iv)

The equations (i), (ii), (iii) and (iv) along with the equations of circle x12  y12  1 and

straight line y 2  3 2  x  , constitute six equations, which on solving give


x1, x 2 , y1, y 2 , 1 and  2 .
y 2 y1
We have from (i) and (iii)  . (v)
x 2 x1
From (v) it is clear that the shortest distance lies on the line passing through the origin
and meeting the point P and Q.
Maxima and minima: Lagrange’s method of undetermined multipliers 13
Prepared by: Dr. Sunil, NIT Hamirpur (HP)

From (ii) and (iv), we get


y 2  y1 1
 . (vi)
x 2  x1 3
Equation (vi) shows that the straight line joining P and Q and passing through the origin,

makes an angle of 30o with x-axis.


Thus the line where the minimum distance lies is given by
1
y x. (vii)
3

As Q lies on y  2 3  3x and also on (vii), we have coordinates of Q on solving for x


and y as

3 3 3 3
x , y  Q   , .

2 2 2 2 
1
The circle x 2  y 2  1 also meets the lines y  x.
3
1
Solving for x and y from x 2  y 2  1 and y  x.
3

3 1
We have x   , y .
2 2
 Points on the circle where the line (vii) meets are

 3 1   3 1
   
 2 , 2  and  2 , 2 
   

 3 1
 P   , 
 2 2
2
3 3  3 1
d   
2   
  2  2  42 3
 2 2   

 d  4  2 3 , the required minimum distance.

  3 1
The point  ,  gives maximum distance.

 2 2 
Q.No.10: A tent on a square base of side x, has its sides vertical of height y and the top
Maxima and minima: Lagrange’s method of undetermined multipliers 14
Prepared by: Dr. Sunil, NIT Hamirpur (HP)

is a regular pyramid of height h. Find x and y in terms of h, if the canvas for


its construction is to be minimum for the tent to have a given capacity.
Sol.: Let V be the volume enclosed by the tent and S be its surface area.
Then V = Cuboid ABCD, A' B' C' D' + Pyramid K, A' B' C' D

1  h
 x 2 y  x 2h  x 2  y  
3  3
1
S  4ABGF  4KGH  4 xy  4 x. KM 
2
 2
 4xy  x x  4h  2 2
  KM 

KL 2
 LM 2
 x 
2
 h  
2 
 
For constant V, we have

 h x2
V  2 x y  x  x 2 y   h  0 .
 3 3
For minimum S, we have

S  4 y 

x 2

 4 h 2  x.
2

1 2
x  4h 2 
1 / 2  1
.2 x  x  4xy  x. x 2  4h 2
 2
 
1 / 2
.8hh  0 .

By Lagrange’s method,

4 y 
 x 2
 
 4 h 2  x 2 x 2  4h 2 
1 / 2   h
  .2 x y  3   0
(i)

4 x  .x 2  0 (ii)


4hx x 2  4h 2 
1 / 2
 .
x2
2
0 (iii)

4
(ii) gives    . Then (iii) becomes
x


4hx x 2  4h 2 1 / 2

4x
3
 0  x  5h .

4
Now putting x  5 ,    in (i), we get
x
5 4  h 14 8h
4 y  3h  h  .2x  y    0  4 y  h  8 y  0
3 x  3 3 3
Maxima and minima: Lagrange’s method of undetermined multipliers 15
Prepared by: Dr. Sunil, NIT Hamirpur (HP)

h
y .
2
5xyz
Q.No.11: If xyz = 8, find the value of x, y for which u  is a maximum.
x  2 y  4 z 
5xyz
Sol.: Let f x , y, z   . ....(i)
x  2 y  4 z 
Also given xyz = 8 ...(ii)
Now f is to be maximized subject to the constraint xyz  8  0 i.e.  x, y, z  0 .

Let F  f   where  is Lagrange’s multipliers.


5xyz
 F  xyz  8 .
x  2 y  4 z 
F x  2 y  4z .5yz  5xyz
Then   yz  0 ...(iii)
x x  2 y  4z 2
F x  2 y  4z .5zx  10xyz
  zx  0 ...(iv)
y x  2y  4z 2
F x  2 y  4z .5xy  20 xyz
  xy  0 ...(v)
z x  2 y  4z 2
Multiplying (iii) by x and (iv) by y and subtracting, we get
x  2y ...(vi)
Multiplying (iv) by y and (v) by z and subtracting, we get
y  2z ...(vii)
From (vi) and (vii) , we get x  2 y  4z
Substitute these values in (ii), we get
x x
 xyz = 8  x. .  8  x 3  64  x  4
2 4
 x  4, y  2, z  1 . Ans.

Q.No.12: Find the minimum value of x 2  y 2  z 2 , given that

(i) xyz  a 3 (ii) ax  by  cz  p

Sol.: (i) Let f x, y, z   x 2  y 2  z 2 . ....(i)


Maxima and minima: Lagrange’s method of undetermined multipliers 16
Prepared by: Dr. Sunil, NIT Hamirpur (HP)

Also given xyz  a 3 ...(ii)

Now f is to be maximized subject to the constraint x, y, z   xyz  a 3  0 .


Let F  f   where  is Lagranges’s multipliers.
2 2 2 3
 F  x  y  z   xyz  a .  
F
Then  2 x  yz  0 ...(iii)
x
F
 2 y  zx  0 ...(iv)
y
F
 2z  xy  0 ...(v)
z
Multiplying (iii) by x and (iv) by y and subtracting, we get

x 2  y2 ...(vi)
Multiplying (iv) by y and (v) by z and subtracting, we get

y2  z2 ...(vii)

From (vi) and (vii) , we get x 2  y2  z 2


xyz
Substitute these values in (ii), we get

 xyz = a  x.x.x  a 2  x3  a 3  x  a
3

 x  a , y  a, z  a .

Hence the minimum value of x 2  y 2  z 2  a 2  a 2  a 2  3a 2 . Ans.

(ii) Let f x, y, z   x 2  y 2  z 2 . ....(i)


Also given ax  by  cz  p ...(ii)

Now f is to be maximized subject to the constraint x , y, z   ax  by  cz  p  0 .


Let F  f   where  is Lagranges’s multipliers.

 F  x  y  z  ax  by  cz  p  .
2 2 2

F
Then  2 x  a  0 ...(iii)
x
Maxima and minima: Lagrange’s method of undetermined multipliers 17
Prepared by: Dr. Sunil, NIT Hamirpur (HP)

F
 2 y  b  0 ...(iv)
y
F
 2 z  c  0 ...(v)
z
Multiplying (iii) by b and (iv) by a and subtracting, we get
ay  bx ...(vi)
Multiplying (iii) by c and (v) by a and subtracting, we get
az  cx ...(vii)
From (vi) and (vii) , we get ax  by  cz  p
bx cx
 ax  b c p
a a
 a 2 x  b 2 y  c 2z  ap
ab
x 2
a  b2  c2
bp cp
Similarly y  2 2 2
, z
a b c a  b2  c2
2

p2
Hence the minimum value of x 2  y 2  z 2  . Ans.
a 2  b2  c2

Q.No.13: Find the extreme values of x 2  y 2  z 2 , when ax  by  cz  p .

Sol.: Let f x, y, z   x 2  y 2  z 2


And x , y, z   ax  by  cz  p  0 (i)
 f x , y, z  is extremised.

Fx , y, z   f x , y, z   x , y, z 

 F  x 2  y 2  z 2  ax  by  cz  P 
F F F
 2 x  a ,  2 y  b and  2z  ac
x y z
Now to find for extreme values
F F F
 0,  0 and  0.
x y z
 2x  a  0 , 2 y  b  0 and 2z  c  0
Maxima and minima: Lagrange’s method of undetermined multipliers 18
Prepared by: Dr. Sunil, NIT Hamirpur (HP)

 2x  2y  2z
 ,  and  
a b c
x a y b z c
  ,  and  .
y b z c x a
Substituting these in (i), we get
ap bp cp
x 2 2 2
, y 2 2 2
and z
a b c a b c a  b2  c2
2

 f x, y, z   x 2  y 2  z 2 i. e. the minimum value.

a 2p2 b 2p 2 c 2p 2 p2
   . Ans.
a 2
 b 2  c2  a
2 2
 b2  c2  a
2 2
 b2  c2 
2
a 2  b2  c2

Q.No.14.: Find a point x 0 , y0 , z0  on the plane ax  by  cz  d  0 , which is the


nearest to the origin.
Sol.: Let L be the length from the origin.

L x 0  0 2  y0  0 2  z 0  0 2  x 02  y02  z 02

Now, L is to be minimized subject to the constant


ax 0  by0  cz0  d  0

x 0 , y0 , z0   ax 0  by0  cz0  d  0


i. e. F  L  

 F  x 02  y 02  z 02   ax 0  by 0  cz 0  d 

where  is the Langrange’s multiplier.


Since L is to be minimized, so we have
F F F
 0, 0 and 0
x 0 y0 z 0
which gives (i)
y0
 b  0 (ii)
x 02  y02  z 02

z0
and  c  0 (iii)
x 02  y 02  z 02
Maxima and minima: Lagrange’s method of undetermined multipliers 19
Prepared by: Dr. Sunil, NIT Hamirpur (HP)

 x0
Fro equation (i),  
a x 02  y 02  z 02

bx 0 cx
Hence y 0  and z 0  0
a a
Putting these values in the given equation, we get

ax 0 
b 2 x 0 c2 x 0
a

a
 d  0  a 2  b 2  c2 x 0  ad  
 ad  bd  cd
 x0  2 2 2
, y0  2 2 2
and z0 
a b c a b c a  b2  c2
2

ad bd cd
 x0  , y0  and z 0  .
a 2  b 2  c2 a 2  b2  c2 a 2  b2  c2
Putting these values in L finally

a 2d 2 b 2d 2 c 2d 2 a 2  b2  c2
L   d
a 2
 b 2  c2  a2 2
 b 2  c2  a
2 2
 b2  c2 
2
a 2
 b2  c2 
2

d
 .
2 2 2
a b c
As x 0 , y0 , z0  approach towards infinity, the distance from origin also tends to infinity

and when x 0 , y0 , z0  approach towards origin, the distance gets shorter. Hence the

values founds out of x 0 , y0 , z0  gives the minimum distance from origin and that

d
minimum distance is . Ans.
a 2  b2  c2
Q.No.15.: Find a point on the curve of intersection of surface x  y  z  1 and
z  xy  5 , which is nearest to the origin.

Sol.: Distance of point x, y, z from origin is given by d  x 2  y 2  z 2 .

So, let f ( x )  x 2  y2  z 2 (i)


As x, y, z lies on intersection of two curves it must satisfy both the equations.
x   x  y   z  1  0 (ii)

 x   z  xy  5  0 (iii)
Using Lagrange’s Multiplier method, let
Maxima and minima: Lagrange’s method of undetermined multipliers 20
Prepared by: Dr. Sunil, NIT Hamirpur (HP)

F( x )  f ( x )  1 ( x )   2 ( x )  x 2  y 2  z 2  1 ( x  y  z  1)   2 (z  xy  5)
F( x )
Now,  0  2 x  1   2 y (iv)
x
F( x )
 0  2 y  1   2 y (v)
y
F( x )
 0  2z  1   2 (vi)
z
Subtracting (v) from (iv), we get
2   2 x  y   0
Either x  y (vii)  2  2 (viii)
If x = y or z  1  2 y
2 2
 In equation (iii) z  2 y  y  5  0  y  2 y  1  3

 y  12  3 , which is not possible.


 2  2
 From equation (iv), 2x  y   1  0 (ix)
From equation (vi), 2z  2  1  0 (x)
Adding (ix) and (x), we get
2x  y  z   2  21  0 (xi)
From (ii) and (xi), we get 0  2 2  1  0 (xii)
From (x) and (xii), we get 2z  2  0  0  z  1 (xiii)
From (ix), we get x  y  0  x   y (xiv)

Using (xiii), (xiv) and (iii), we get 1  y 2  5  0  y 2  4  y  2 .

From (xiv), we get  x   y   2   2 .

 The points are 2,  2, 1 and  2, 2, 1 . Ans.


Q.No.16.: Show that all the rectangular parallelepipeds which have sides parallel to the

coordinate planes and which are inscribed in the sphere x 2  y 2  z 2  a 2 , one


which has the maximum volume is a cube.
Sol.: Let the sides be , m, n .  Volume = mn
Maxima and minima: Lagrange’s method of undetermined multipliers 21
Prepared by: Dr. Sunil, NIT Hamirpur (HP)

From figure,  2  m 2  n 2  a 2 .
By Lagrange’s method, we get

 
mn    2  m2  n 2  a 2  0
f  0  mn  2  0 (i)

f m  0  n  2m  0 (ii)

f n  0  m  2n  0 (iii)
mn n
From (i) and (ii), we get   2  m2    m
 m
n m
From (i) and (ii), we get   m2  n 2  m  n .
m n
a
   m  n  3 2  a 2     m n.
3
 The maximum volume that can be inscribed is a cube. Ans.
1 1 1
Q.No.17.: If f  a 3x 2  b3y 2  c2z 2 , where    1 , prove that stationary value of
x y z
abc abc abc
f is given by x  , y and z  .
a b c

Sol.: Given that f  a 3x 2  b3 y 2  c 2z 2 (i)


1 1 1
Also   1 (ii)
x y z
1 1 1
    1  0  x , y, z  [say]
x y z

To find the stationary value of f subject to considered x, y, z 


Let F  f   [Here  is Lagrange’s multiplier]

1 1 1 
 F  a 3x 2  b3 y 2  c 2 z 2      1 .
x y z 
Since we have to find stationary value of f.
F F F
  0, 0 and  0.
x y z
Maxima and minima: Lagrange’s method of undetermined multipliers 22
Prepared by: Dr. Sunil, NIT Hamirpur (HP)

F  
 2a 3x  2  0  2a 3x  2    2a 3x 3 (iii)
x x x
F F
 2b3 y3   (iv)  2c 3 z 3   (v)
y z
From (iii), (iv) and (v), we get

a 3x 3  b3 y3  c3z3

b c
 x    y   z .
a a
a a
Also y    x (vi) z   x (vii)
b c
Using (vi) and (vii) in (ii), we get
1 b c 1  b c abc
   1  1     1 x  .
x ax ax x  a a a

b b
x   y , z   y .
a c
Using these two equations in (ii), we get
a 1 c 1 a c abc
   1   1   1  y  .
by y by y b b b

c c
x    z , y   z .
a b
Using these two equations in (ii), we get
1 1 1 a b 1 1 a b 
  1    1     1  1
c c z cz cz z z c c 
 z   z
a b
abc
z .
c
abc abc abc
x  , y and z  for stationary value of f . Hence this
a b c
proves the result. Ans.
Q.No.18.: A wire of length b is cut into two parts which are bent in the form of a square
and circle respectively. Find the least value of the sum of the areas so found by
using Lagrange’s method.
Maxima and minima: Lagrange’s method of undetermined multipliers 23
Prepared by: Dr. Sunil, NIT Hamirpur (HP)

Sol.: Let x and y be two parts into which the given wire is cut so that x + y = b.
x
Suppose the piece of wire of length x is bent into a square so that each side is and thus
4

x x x2
the area of the square is . = .
4 4 16
Suppose the wire of length y is bent into a circle with perimeter y. So the area of this
circle so formed is
2
 y  y 2 y 2 y
radius2     2  . Since 2radius   y  radius  .
 2  4 4 2
Thus to find the minimum of the sum of the two areas subject to the constraint that sum
x +y = b.
Let F  f   , where  is Lagranges's multipliers.

 x 2 y2 
F( x , y        x  y  b  .
 16 4 
 
F x
Then  0
x 8
F y
   0
y 2
Solving, we get x  8 , y  2 .
Substituting these values in the constraint condition x +y = b, we get
b
 8  2  b     .
8  2
8b 2b
Thus x  8  and y  2  .
8  2 8  2
Thus the least value of the sum of the areas of the square and circle is
 x 2 y2  64b 2 4 2 b 2 4b 2 b 2
F( x , y      8b    
 16 4  x 
 822b 168  2 48  2 2 44  2 44  2
2

y
8 2 

4b 2  b 2 b2
  . Ans.
44  2 44  

Q.No.19.: Find (by the Lagrange’s method) the maximum value of x m y n z p when
Maxima and minima: Lagrange’s method of undetermined multipliers 24
Prepared by: Dr. Sunil, NIT Hamirpur (HP)

xyz  a.

Sol.: This is a constraint maximum problem where the function f ( x , y, z)  x m y n z p is


given, subjected to the constraint condition x  y  z  a .
So consider the Lagrange’s auxiliary function
F  f   , where  is Lagrange’s multiplier.

 F( x , y, z)  x m y n z p  x  y  z  a  . (i)
Differentiating (i) w.r.t x, y, z and equating to zero, we get
F
Fx   mx m 1 y n z p    0 , (ii)
x
F
Fy   nx m y n 1z p    0 , (iii)
y
F
Fz   px m y n z p 1    0 . (iv)
z
m mf
Solving for x, y, z, we obtain f  0  x  .
x 
nf pf
Similarly, we can obtain y   and z   .
 
Substituting these values in the given constraint condition, we have
m n p
x  y  z     f  a .
   

Solving, we get the value of  as   


m  n  p  f .
a
Using this  , we get
mf mf . a  am
x   .
 f m  n  p  m  n  p
nf an pf ap
Similarly, y    and z    .
 mnp  mnp
Thus, the maximum value is
m n p
m n p  am   an   ap  a m  n  p .m m .n n .p p
x y z         . Ans.
mn p mnp mnp m  n  p m  n  p
Q.No.20.: Find (by the Lagrange’s method) the maximum and minimum distances from
Maxima and minima: Lagrange’s method of undetermined multipliers 25
Prepared by: Dr. Sunil, NIT Hamirpur (HP)

the origin to the curve 3x 2  4xy  6 y 2  140 .


Sol.: The distance d from the origin (0, 0) to any point (x, y) is given by

x 2  y 2  f ( x , y) .
To find: The extrema of f (x, y) subject to the condition that the point (x, y) lies on the

curve 3x 2  4xy  6 y 2  140 .


So consider the Lagrange’s auxiliary function
F  f   , where  is Lagrange’s multiplier.

  
 F( x , y)  x 2  y 2   3x 2  4 xy  6 y 2  140 .  (i)
Differentiating (i) w.r.t x, y and equating to zero we get
Fx  2 x  6 x  4 y   0 ,

Fy  2 y  12 y  4x   0 .

x y
Solving for     ,
3x  2 y  (6 y  2x )
x2 y2 x2  y2
   
3x 2  2xy 6 y 2  2xy 3x 2  4 xy  6 y 2
f
   .
140
Substituting  in Fx  0 and Fy  0 , we get

(140  3f ) x  2fy  0 ,
 2fx  (140  6f ) y  0 .

140  3f  2f
This system has non-trivial solution if 0
 2f 140  6f

 (140  3f )(140  6f )  4f 2  0

 14f 2  1260f  140 2  0  f 2  90f  1400  0


 f  70f  20  0  f  70, 20 .

Thus, the maximum and minimum distances are 70 , 20 . Ans.


Q.No.21.: Find (by the Lagrange’s method) the dimensions of a rectangular box of
Maxima and minima: Lagrange’s method of undetermined multipliers 26
Prepared by: Dr. Sunil, NIT Hamirpur (HP)

maximum capacity whose surface area is given when


(a) box is open at the top (b) box is closed.
Sol.: Let x, y, z be the dimensions of the rectangular box so that its volume V is
V = xyz . (i)
The total surface area of the box is
nxy  2 yz  2zx  S  given constant (ii)
Here n = 1, the box is open at the top
n = 2, the box is closed (on all sides)
The constrained maximum problem is to maximize V subject to constraint condition (ii).
So consider the Lagrange’s auxiliary function
F  f   , where  is Lagrange’s multiplier.

 F( x , y, z)  xyz  nxy  2 yz  2zx  S . (iii)


Differentiating (iii) w.r.t x, y, z and equating to zero, we get
Fx  yz  ny  2z   0 , (iv)

Fy  xz  nx  2z   0 , (v)

Fz  xy  2 y  2 x   0 . (vi)
Multiplying (iv), (v) and (vi) by x, y, z respectively and adding, we get
3xyz  2nxy  2 yz  2zx   0
 3.V  2.S  0 (using (i) and (ii))
3V
 . (vii)
2S
Substituting the value of  from (vii) in (iv), (v) and (vi), we get
3V
yz  ny  2z   0  yz  3xyz ny  2z   0  nxy  2 xz  2S . (viii)
2S 2S 3
2S
Similarly, we can obtain nxy  2 yz  . (ix)
3
2S
2 yz  2 zx  (x)
3
From ( viii )  (ix ) , we get x = y . (xi)
From (ix )  ( x ) , we get ny = 2z. (xii)
Maxima and minima: Lagrange’s method of undetermined multipliers 27
Prepared by: Dr. Sunil, NIT Hamirpur (HP)

Substituting (ix) and (xii) in the given constraint condition (ii), we obtain
nx S
n.x.x  4 x  S  3nx 2  S  x 2  .
2 3n
Case a: When box is open at the top: n = 1.

S S
Then x 2  x .
3 3

S 1 S
Also since x = y and ny = 2z. Thus x  y  , z .
3 2 3

S 1 S
Hence the dimensions of the open top box are x  y  , z . Ans.
3 2 3
Case b: When the box is closed: n = 2.

S S
Then x 2  x ,
6 6

S
Also since x = y and ny = 2z. Thus x = y = z = .
6

S
Hence the dimensions of the closed box are x = y = z = . Ans.
6
Q.No.22.: Suppose a closed rectangular box has length twice its breadth and has constant
volume V. Determine the dimensions of the box requiring least surface area
(sheet metal) by the Lagrange’s method..
Sol.: Let x be the breadth so that the length is 2x and y be the height of the closed
rectangular box.

Its volume is x. 2 x.y  2 x 2 y  V (given).

The surface area (6 faces) S is given by S  22 x.x   2(2x.y)  2( x.y)  4 x 2  6 xy .

Thus, the problem is to minimize f ( x , y)  4 x 2  6 xy , (i)


V
subject to the constraint condition x 2 y  (known). (ii)
2
So consider the Lagrange’s auxiliary function
F  f   , where  is Lagrange’s multiplier.
Maxima and minima: Lagrange’s method of undetermined multipliers 28
Prepared by: Dr. Sunil, NIT Hamirpur (HP)

 V
 Fx , y   4 x 2  6 xy   x 2 y   . (iii)
 2
Differentiating (iii) w.r.t x and y and equating to zero, we get
Fx  8x  6 y  2xy  0 , (iv)

Fy  6 x  x 2  0 . (v)

6 6
Solving (v),    x . (vi)
x 
Substituting x from (vi) in (iv), we get
 48 8
 6 y  12 y  0  y   . (vii)
 
Substituting (vi) and (vii) in the given constraint condition (ii), we get
1/ 3
576  576 
3        . (viii)
V  V 
Using (viii), from (vi) and (vii), we get
1/ 3 1/ 3
6  V   3V 
x  6.    , (ix)
  576   8 
1/ 3 1/ 3
8  V   8V 
y  8.    . (x)
  576   9 
The least surface area with these dimensions (ix) and (x) is
2/3 1/ 3 1/ 3
 3V   3V   8V 
S  4 x 2  6 xy  4.   6.    .
 8   8   9 
On simplification


S  35 V 2 
1/ 3

 243V 2
1/ 3
. Ans.
Q.No.23.: Find the extremum values of x 2  y 2 when 13x 2  10xy  13y 2  72 by

the Lagrange’s method.


Sol.: F  x 2  y 2   13x 2  10 xy  13y 2  72 
F 2x
   26 x  10 y 
x 2 x 2  y 2
Maxima and minima: Lagrange’s method of undetermined multipliers 29
Prepared by: Dr. Sunil, NIT Hamirpur (HP)

F
 0 for maxima or minima
x
x 1
  (i)
x 2  y2 26x  10 y 
F 2y
   26 y  10 x 
y 2 x 2  y 2

F
 0 for maxima or minima
y
y 1
  (ii)
x 2  y2 26 y  10 x 
From (i) and (ii), we get
x 1 y 1
  
x 2  y2 26x  10 y  x 2  y2 26 y  10 x

  
x 2  y 2 26xy  10 y 2  x 2  y 2 26 xy  10 y 2 
 y2  x 2  x  y

For 13x 2  10 xy  13 y 2  72
When x = y

13 y 2  13 y 2  10 y 2  72
9
16 y 2  72  y 2 
2
when x   y

13x 2  10 x 2  13 y 2  72

36x 2  72  x 2  2

x 2  y 2  2 y 2  9  3 and 2x 2  2  2  2

Maximum value = 3,
Minimum value = 2.

Q.No.24.: Find the minimum value of x 2  y 2  z 2 subject to the condition


Maxima and minima: Lagrange’s method of undetermined multipliers 30
Prepared by: Dr. Sunil, NIT Hamirpur (HP)

1 1 1
   0 by the Lagrange’s method.
x y z

 1 1 1
Sol.: Fx , y, z   x 2  y 2  z 2     
x y z
F  F  F 
 2x  2 ,  2y  2 ,  2z  2 .
x x y y z z
F F F
For maxima and minima,   0
x y z

 x 3  y3  z3 
2
1 1 1 1 1 1
  1   1
x y z x x x
3
 1 x  3
x
x  y  z  3

 x 2  y 2  z 2  3 2  3 2  3 2  27
when, x = y = z = 0

 x 2  y 2  z 2  27 is the minimum value.


Q.No.25.: Divide 24 into three parts such that the continued product of the first, square of
the second and the cube of the third may be maximum.
 yx 2 z 3  max . 
Sol.: F  yx 2 z 3   y  x  z  24   
 y  x  z  24

F F F
 2 yxz3   ,  x 2z3   ,  3yx 2 z 2   .
x y z
F F F
For maxima and minima,   0
x y z

  2 yxz 2

   x 2 z 3  x 2 z 3  2 yxz 3  x  2 y

  3 yx 2 z 2  3yx 2 z 2  x 2 z 3  3y  z
y  x  z  24
Maxima and minima: Lagrange’s method of undetermined multipliers 31
Prepared by: Dr. Sunil, NIT Hamirpur (HP)

2 y  y  3y  24
y  4 , x = 8, z = 12.
 maximum value of yx2 z2 = 4.82 . 123.
Q.No.26.: Determine the perpendicular distance of the point (a, b, c) from the plane
x  my  nz  0 by the Lagrange’s method.
Sol.: Perpendicular distance of a point (a, b, c) from a point (x, y,z) can be calculated as

D x  a 2  ( y  b ) 2  ( z  c ) 2
f  D 2  x  a 2  ( y  b ) 2  ( z  c ) 2
  x  my  nz  0

F  f    x  a 2  ( y  b) 2  ( z  c) 2   x  my  nz 
F F F
 2x  a     0 ,  2 y  b    m  0 ,  2z  c   n  0 .
x y z
F F F
For maxima and minima,   0
x y z
 m  n
xa , yb , z c.
2 2 2
Also, x  my  nz  0

 2 m 2 n 2
a   mb   nc  0
2 2 2
2a  mb  nc 
 (i)
2  m2  n 2

D 2  x  a 2  ( y  b ) 2  ( z  c ) 2
  
 x a  a  a 
2 2
m  m
yb  b b 
2 2

D2 
2  2 2 m 2 2 n 2 2 2
4

4

4

4
  m2  n 2 
 2 a  mb  nc
D   m2  n 2  2  2  m2  n 2 [From (i)]
2  m n2 2
Maxima and minima: Lagrange’s method of undetermined multipliers 32
Prepared by: Dr. Sunil, NIT Hamirpur (HP)

a  mb  nc
D . Ans.
2  m2  n2
Q.No.27.: Determine the point in the plane 3x  4 y  5z  50 nearest to the origin by the
Lagrange’s method.
Sol.: Let (x, y, z) be a point on the given plane which is nearest to the origin.
Distance between (x, y, z) and (0, 0, 0) is given by

f  D2  x 2  y2  z2
  3x  4 y  5z  50

F  f    x 2  y 2  z 2    3x  4y  5z  50 

F F F
 2 x  3 ,  2 y  4 ,  2z  5 .
x y z
F F F
For maxima and minima,   0
x y z
 3z 4z  5z
x , y , z .
2 2 2
Also, 3x  4 y  5z  50
 9 16 25
   50
2 2 2
 50
  50    2
2
3 4 5
x  2  3 , y  2  4 , z  2  5 .
2 2 2
 3,  4, 5 is the point on the given plane which is nearest to the origin (0, 0, 0).

Q.No.28.: Determine the point on the paraboloid z  x 2  y 2 which is closest to the


point 3,  6, 4  by the Lagrange’s method.
Sol.: Let (x, y, z) be the required point.
Distance between (x, y, z) and 3,  6, 4  is given by

D x  32  y  6 2  z  4 2
f  D 2  x  32  y  6 2  z  4 2
Maxima and minima: Lagrange’s method of undetermined multipliers 33
Prepared by: Dr. Sunil, NIT Hamirpur (HP)

  x 2  y2  z
F  f  


F  x  32  y  6 2  z  4 2   x 2  y 2  z 
F F F
 2x  3  2x ,  2y  6  2y ,  2z  4    .
x y z
F F F
For maxima and minima,   0
x y z
3 6 z
x , y , z 4.
1  1  2
Also, x 2  y 2  z
9 36 
  4
1    2
1    2 2

45

 8
 
 1  2  2   8  90
1    2 2

   8  3  82  22  16  90

 3  102  17  82  0


   2  2  12  41  0 
2
3 6
x  1, y  2 , z  1 4  5.
3 3

 1,  2, 5 is the point on the paraboloid z  x 2  y 2 which is nearest to the point

3,  6, 4  .
Q.No.29.: a. Find (by the Lagrange’s method) the dimensions of the rectangular box,
without top, of maximum capacity, whose surface is 108 square cm.
b. What are the dimensions, when the box is closed (on all sides)?
Sol.: (a). Let x, y, z, V, A be the length, breadth, height, volume, surface area of the
rectangular box.
  A  xy  2 yz  2xz  108
A = 108 sq. cm.
Maxima and minima: Lagrange’s method of undetermined multipliers 34
Prepared by: Dr. Sunil, NIT Hamirpur (HP)

f = V = x, y, z
F  f    xyz  xy  2 yz  2xz  108 .
F F z F
 yz  y  2z ,  xz  2y  xz  2z  x ,  xy  2x  2y .
x y y z
F F F
For maxima and minima,   0
x y z
 y   yz    z  z  
and xz  2z  0  x  2
and xy  2x  0  y  2
Also, xy  2 yz  2xz  108

42  42  42  108  122  108

 2  9    3
When   3
z = 3, x = 6, y = 6.
When   3
z  3 , x  3 , z  3 .
Lengths cannot be negative.
 Dimensions of rectangular base are
z = 3, x = 6, y = 6.
Maximum Volume = 6 . 6. 3 = 108 cm3.
(b). f = V = x, y, z
  A  2xy  2 yz  2xz  108
  2xy  2 yz  2xz  108

F  f    xyz    2xy  2yz  2zx  108 

F F F
 yz  2y  2z ,  xz  2x  2x ,  xy  2y  2x .
x y z
F F F
For maxima,   0
x y z
yz  2y  2z  0 (i)
xz  2x  2z  0 (ii)
Maxima and minima: Lagrange’s method of undetermined multipliers 35
Prepared by: Dr. Sunil, NIT Hamirpur (HP)

xy  2y  2x  0 (iii)


Substituting (ii) from (i), we get
y  x z  2y  x   0
 y  x z  2   0
 y  x  z  2
Substituting (iii) from (ii), we get
x z  y   2z  y   0
 z  y x  2   0
 z  y  x  2
 x  y  z  2 .
Also, 2xy  2 yz  2xz  108

 2x 2  2x 2  2x 2  108

 x 2  18  x  18 .

 x  y  z  18 are the required dimensions of the rectangular base.


Q.No.30.: a. If the surface of the rectangular box, with open top, is 432 sq. cm, find (by
the Lagrange’s method) the dimensions of the box having maximum
capacity (volume).
b. If the box is closed (on all sides), what are the dimensions?
Sol.: (a). Let x, y, z, V, A be the length, breadth, height, volume, surface area of the
rectangular base respectively.
f  V  xyz
A  xy  2 yz  2zx
  xy  2 yz  2zx  432

F  f    xyz    xy  2yz  2zx  432 

F 2 F z F
 yz  y  2z ,  xz  x  2y ,  xy  2y  2x .
x 2 y y z
F F F
For maxima,   0
x y z
yz  y  2z  0 (i)
Maxima and minima: Lagrange’s method of undetermined multipliers 36
Prepared by: Dr. Sunil, NIT Hamirpur (HP)

xz  x  2z  0 (ii)
xy  2y  2x  0 (iii)

Substituting (ii) from (i), we get


z y  x     y  x   0
 y  x z     0
 y  x  z   .
Substituting (iii) from (ii), we get
x 2z  y   22z  y   0
 x  2 2z  y   0
x  2  y  2z  2
 x  y  2 and z   .
Also, xy  2xz  2 yz  432

42  2 2  z   2 2z  z   432

42  42  42  432

2  36    6
x = y =12 and z = 6.
 12, 12, 6  are the required dimensions.
(b). f  V  xyz
A  2xy  2 yz  2xz  432
  2xy  2 yz  2xz  432

F  f    xyz    2xy  2yz  2xz  432 

F
 yz  2y  2z (i)
x
F
 xz  2x  2z (ii)
y
F
 xy  2x  2y (iii)
z
Maxima and minima: Lagrange’s method of undetermined multipliers 37
Prepared by: Dr. Sunil, NIT Hamirpur (HP)

F F F
For maxima,   0
x y z
zy  x   2y  x   0 [From (i) and (ii)]
 z  2 y  x   0
 z  2  y  x
zz  y   2z  y   0 [From (ii) and (iii)]
 z  y x  2   0
 z  y  x  2
 x  y  z  2
A  2xy  2 yz  2xz  432

2x 2  2x 2  2x 2  432

 6x 2  432  x 2  72  x  72

 x  y  z  72 are the required dimensions of the rectangular box.


Q.No.31.: Find (by the Lagrange’s method) the length and breadth of a rectangle of

maximum area that can be inscribed in the ellipse 4 x 2  9 y 2  36 .


Sol.: Let x and y be the length and breadth of rectangle.
Area, A = xy
f = xy

  4 x 2  9 y 2  36

 
F  f    xy   4 x 2  9 y 2  36 .
F F
 y  8x ,  x  18y .
x y
F F
For maxima,  0
x y
y  8x  0 (i)
x  18y  0 (ii)
Multiplying (i) by x and (ii) by y and subtracting

 
 18 y 2  8x 2  0
Maxima and minima: Lagrange’s method of undetermined multipliers 38
Prepared by: Dr. Sunil, NIT Hamirpur (HP)

 8x 2  18 y 2  4 x 2  9 y 2  2 x  3y .

Also, 4 x 2  9 y 2  36

9
4  y 2  9 y 2  36  18 y 2  36  y 2  2  y  2
4

3 2
x
2
3 2
 Length  , Breadth = 2
2

3 2
Maximum area = xy =  2  3 sq. units.
2
Ans.: Length  3 2 / 2 , breadth: 2 . Maximum area of the rectangle is 12 square units.
Q.No.32.: Find (by the Lagrange’s method) the volume of the largest rectangular
parallelopiped that can be inscribed in the ellipsoid of revolution

4x 2  4 y 2  9z 2  36 .
Sol.: Let 2x, 2y, 2z be the dimensions of the parallelopiped

  4 x 2  4 y 2  9z 2  36
f = 8xyz


F  f    8xyz   4 x 2  4 y 2  9z 2  36 
F F F
 8 yz  8x ,  8xz  8y ,  8xy  18z .
x y z
F F F
For maxima and minima,   0
x y z
8yz  8x  0 (i)
8xz  8y  0 (ii)
8xy  8z  0 (iii)
Solving (i) and (ii), we get

 
8 x 2  y 2  0
x  y .
Solving (ii) and (iii), we get
Maxima and minima: Lagrange’s method of undetermined multipliers 39
Prepared by: Dr. Sunil, NIT Hamirpur (HP)

 
 8x 2  18z 2  0  2 x  3z .

Also, 4 x 2  4 y 2  9z 2  36

4
 4 x 2  4x 2  9    x 2  36
9

 x2  3  x  3
2
x  y  3 , z  3.
3
2 3
Volume of the parallelopiped = 8xyz  3  3   18  16 3 cu. Unit.
3
Q.No.33.: Find (by the Lagrange’s method) the dimensions of a rectangular box, with
open top, of given capacity (volume) such that sheet metal (surface area)
required is least.
Sol.: Let x and y be the dimension of rectangular box.
  V  xyz
f  A  xy  2 yz  2zx
F  f    xy  2 yz  2zx   xyz  V 
F F F
 y  2z  yz ,  x  2z  xz ,  2 y  2x  xy .
x y z

F F F
For maxima and minima,   0
x y z
y  2z  yz  0 (i)
x  2z  xz  0 (ii)
2 y  2x  xy  0 (iii)
Solving (i) and (ii), we get
y  x   zy  x   0  y  x 1  z   0
1
yx .
z
Solving (ii) and (iii), we get
22 x  y   x 2z  y   0  2  x 2z  y   0
Maxima and minima: Lagrange’s method of undetermined multipliers 40
Prepared by: Dr. Sunil, NIT Hamirpur (HP)

2
 y  2z  x 

 x  y  2z
3
x x
V  xyz  x  y    
2 2

 x  2V 1 / 3

Required dimensions are x  y  2z  2V 1 / 3 .

Home Assignments
*** *** *** *** ***
*** *** ***
***
Differential Calculus

Curve Tracing
(a) Cartesian form
(b) Polar form
(c) Parametric form
Prepared by:
Dr. Sunil
NIT Hamirpur (HP)
(Last updated on 11-09-2009)
Latest update available at: http://www.freewebs.com/sunilnit/

Introduction:
Curve tracing is an analytical method of drawing an approximate
shape of a curve, by the study of some of its important characteristics such
as symmetry, intercepts, asymptotes, tangents, multiple points, region of
existence, sign of the first and second derivatives.
Knowledge of curve tracing is useful in application of integration in
finding length, area, volume etc.
Here, we will study the procedure for tracing of standard curves in
the
(a) Cartesian form
(b) Polar form
(c) Parametric form
GENERAL PROCEDURE FOR TRACING OF CARTESIAN CURVES:
Plane algebraic curve of nth degree is represented by

 
f x , y   ay n  bx  c y n 1  dx 2  ex  f y n  2  .........  u n ( x )  0 , (i)
where a, b, c, d, f ……. are all constants and un (x) is a polynomial in x of degree n.
Differential Calculus: Curve Tracing Prepared by: Dr. Sunil, NIT Hamirpur 2

General procedure for tracing the algebraic curve consists of the study of the following
characteristics of the curve.
1. Symmetry
2. Origin
3. Tangents to the curve at the origin
4. x and y-Intercepts
5. Special points (Multiple points)
dy
6. Sign of first derivative
dx

d2 y
7. Sign of second derivative
dx2
8. Imaginary values (Imaginary region)
9. Region
10. Asymptotes
11. Shape of the curve
1. Symmetry
(i) Symmetric about x-axis:
If even and only even powers of y occur in equation of the curve, then the curve is
symmetrical about x-axis.
or
If the equation of the curve remains unchanged, when y is changed into  y , then
the curve is symmetrical about x-axis.
This means that, if f x,  y   f x, y  then the curve is symmetrical about x-axis.
y-axis y-axis

x-axis x-axis

Examples: 1. The parabola y 2  4ax is symmetrical about x-axis.

2. The circle x 2  y 2  a 2 is symmetrical about x-axis.


Differential Calculus: Curve Tracing Prepared by: Dr. Sunil, NIT Hamirpur 3

(ii) Symmetric about y-axis:


If even and only even powers of x occur in equation of the curve, then the curve is
symmetrical about y-axis.
or
If the equation of the curve remains unchanged, when x is changed into  x , then
the curve is symmetrical about y-axis.
This means that, if f  x, y   f x, y  then the curve is symmetrical about y-axis.

y-axis y-axis

x-axis x-axis

Examples: 1. The parabola x 2  4ay is symmetrical about y-axis.

2. The circle x 2  y 2  a 2 is symmetrical about y-axis.


(iii) Symmetric about both x- and y-axes:
If even and only even powers of x and y occur in equation of the curve, then the
curve is symmetrical about x- and y-axes.
or
If the equation of the curve remains unchanged, when x is changed into  x and y
is changed into  y , then the curve is symmetrical about x- and y-axes.

This means that, if f  x,  y   f x, y  then the curve is symmetrical about both
the axes.

y-axis

x-axis

Example: 1. The circle x 2  y 2  a 2 is symmetrical about both the axes.


Differential Calculus: Curve Tracing Prepared by: Dr. Sunil, NIT Hamirpur 4

(iv) Symmetric about origin:


If the equation remains unchanged, when x and y are replaced by  x and  y ,
respectively, then the curve is also symmetrical about origin.
This means that, if f  x,  y   f x, y  , then the curve is symmetrical about the
origin.

Examples: 1. The curve x 5  y 5  5a 2 x 2 y is symmetrical about origin.

2. The circle x 2  y 2  a 2 is symmetrical about origin.


(v) Symmetric about opposite quadrants:
If x and y are replaced by  x and  y , respectively, the equation of the curve
remains unchanged, then the curve is also symmetrical in opposite quadrants.
This means that, if f x, y   f  x,  y  , then the curve is symmetrical about the
opposite quadrant.

Examples: 1. xy  c 2

2. Cubic parabola y  x 3

3. Circle x 2  y 2  a 2
4. Ellipse
5. Hyperbola
Remarks: Curve symmetric about both the axes is also symmetric about origin but not
the converse (because of the presence of odd powers).

(vi) Symmetric about the line y = x:


If on interchanging x and y the equation of the curve remains unaltered, then the
curve is symmetrical about the line y = x.
This means that, if f x, y   f y, x  , then the curve is symmetrical about the line
y = x. y-axis y=x
y-axis
y=x
x-axis
x-axis
Differential Calculus: Curve Tracing Prepared by: Dr. Sunil, NIT Hamirpur 5

Examples: 1. The curve x 3  y 3  3axy is symmetrical about the line y = x.

2. The circle x 2  y 2  a 2 is symmetrical about the line y = x.


(vii) Symmetric about the line y   x :
If on changing x to  y and y to  x , the equation of a curve remains unchanged,
then the curve is symmetrical about y   x .

This means that, if f x, y   f  y, - x  , then the curve is symmetrical about the
line y   x

Example: 1. The curve x 3  y 3  3axy is symmetrical about the y   x .

2. Origin
If the co-ordinates of origin i.e. (0, 0) satisfy the given equation, then the curve
passes through the origin.
This means that, if the equation of the curve has no constant term then it will pass
through the origin.

Examples: 1. The curve x 3  y 3  3axy passes through the origin.

2. The curve y 2 a  x   x 3 passes through the origin.

3. Tangents to the curve at the origin


If the curve passes through the origin, find the equations of the tangents at the
origin.
Method for evaluating equation of tangents:
Equation of tangents is evaluated by equating the lowest degree term in the equation of
the curve to zero.

Examples: 1. y 2  4ax . Here lowest degree term 4ax equated to zero gives
x = 0 (y-axis) as tangent to the curve at origin.

2. x 3  y 3  3axy . Here lowest degree terms 3axy equated to zero gives


xy = 0 or x = 0 and y = 0 are the two tangents to the curve at origin.

 
3. a 2 y 2  a 2 x 2  x 4 . Here lowest degree term y 2  x 2 equating to zero
gives y   x as the two tangents at origin.
Differential Calculus: Curve Tracing Prepared by: Dr. Sunil, NIT Hamirpur 6

4. x and y- Intercepts
 x-intercept and y-intercept: Find the points, where the curve meets the x-axis by
substituting y = 0, in the equation of the curve. Also find the points where the curve
meets the y-axis, by substituting x  0 .
 Find the tangents at these points; this can be done easily by shifting the origin to
these points of intersection and equating the lowest degree term in the changed
equation to zero.
 If the curve is symmetrical about the line y = x or y   x . Find the points of
intersection of curve with these lines also by putting y   x , in the equation of the
curve. Find the tangents at these points.

5. Special points (Multiple points)


Multiple point (or singular point):
A point, through which r branches of the curve pass is called a multiple point of
th
the r order and has r tangents. Thus, at a double point two branches of the curve pass.
Double point is classified as a node, a cusp or an isolated (or conjugate) point.
If the tangents are real and different, then the point is called a node.
If the tangents are real and coincident, then point is called a cusp.
If the tangents are imaginary, then the point is called an isolated point.
Method for evaluating multiple points:
Multiple points are obtained by solving for (x, y) the three equations
f x x, y   0, f y x, y   0, f x, y   0 .

dy
6. Sign of first derivative
dx
 In an interval a  x  b if
dy
  0 , then the curve is increasing in [a, b]
dx
dy
  0 , then the curve is decreasing in [a, b]
dx
Differential Calculus: Curve Tracing Prepared by: Dr. Sunil, NIT Hamirpur 7

dy
 If at x  x 0 , y  y 0 ,  0 , then x 0 , y 0  is a stationary point where
dx
maxima and minima can occur
Remarks:
dy
At point where  0 , the tangent is parallel to x-axis i.e., horizontal.
dx
dy
At point where   , the tangent is parallel to y-axis i.e., vertical.
dx

d2 y
7. Sign of second derivative
dx2
 In an interval a  x  b if

d2y
  0 , then the curve is convex or concave upwards (holds water)
dx 2

d2y
  0 , then the curve is concave downwards (spills water)
dx 2

d2y
 A point at which  0 is known as an inflection point, where the curve
dx 2
changes the direction of concavity from downward to upward or vice- versa.

8. Imaginary values (Imaginary region)


Find the region, where no part of the curve lies. This region can be found by
solving the given equation of the curve for one variable in terms of the other, say y in
terms of x and then finding those values of x for which y becomes imaginary.
Thus, imaginary region is the region in which the curve does not exist. In such
region y becomes imaginary (undefined) for values of x or vice-versa.

9. Region (Region of extent)


 Consider the variation of one of the variables, say y, as other say x varies, paying
special attention, when x increases and finally approaches  .
 Similarly, observe the variation of y as x decreases and finally approaches   .
Differential Calculus: Curve Tracing Prepared by: Dr. Sunil, NIT Hamirpur 8

In other way, region of extent is obtained by in terms of x or vice-versa.


Real horizontal extent is defined by values x for which y is defined.
Real vertical extent is defined by values of y for which x is defined.
10. Asymptotes
Find the asymptote of the curve, if any. Now first let us define asymptote.
Definition:
Asymptote to a curve is a straight line, which is tangent to the curve at infinity.
or
Asymptote is a straight line related to an infinite branch of a curve, such that its
perpendicular distance, from a point, which moves to infinity along the curve, tends to
zero.
Method for evaluating asymptote:
Asymptotes parallel to x-axis:
Asymptotes parallel to x-axis are obtained by equating the coefficients of the
highest powers of x, to zero.
Asymptotes parallel to y-axis:
Asymptotes parallel to y-axis are obtained by equating the coefficients of the
highest powers of y, to zero.

Example: Let us consider the curve y 2 a  x   x 3 .


Equating to zero the coefficient of y2, the highest degree term in y.
The asymptote parallel to y-axis is x  a  0  x  a .
Oblique Asymptotes
 Let y  mx  c be an asymptote.
 Put y = mx + c in the given equation and we get the equation in terms of
descending powers of x.
 Equate the coefficients of the powers of x to zero, separately.
 Solve these equations for m and c.
 Substitute these values of m and c in y = mx + c.
 Then, we get the required equation of asymptote.
Differential Calculus: Curve Tracing Prepared by: Dr. Sunil, NIT Hamirpur 9

Note that any two such equations in m and c will be sufficient to give the values of m and
c.

Example: Let us consider the curve x 3  y 3  3axy .


Let y = mx + c be an asymptote.

Putting this value of y in x 3  y 3  3axy , we get

x 3  mx  c 3  3axmx  c   x 3  m 3 x 3  c 3  3cmx(mx  c)  3amx 2  3acx

 x 3  m 3 x 3  c 3  3cm 2 x 2  3c 2 mx  3amx 2  3acx  0

     
 x 3 1  m 3  x 2 3cm 2  3am  x 3c 2 m  3ac  c 3  0 .
This one is the equation in terms of descending powers of x.
Equate the coefficients of the powers of x to zero, separately.
Equating coefficient of x3 equal to zero, we get

1  m 3  0  m  1 .
Equating coefficient of x2 equal to zero, we get

3cm 2  3am  0  cm  a  0  cm  a  c  a .  m  1


Putting these values of m and c in y  mx  c , we get
y   x  a  x  y  a  0 is the required asymptote.

11. Approximate shape of the curve


Draw the rough sketch of the curve (approximate shape of the curve).

Important Note:
For approximate shape of the curve, make use of salient features of the above procedure.
Differential Calculus: Curve Tracing Prepared by: Dr. Sunil, NIT Hamirpur10

Now let us trace some important curves with their salient features:

1. Cissoid: y 2 a  x   x 3

2. Folium of Descartes: x 3  y 3  3axy or x 3  y 3  3axy


3. Astroid or Four cusped hypocycloid:
2/3 2/3
2/3 2/3 2/3 x  y
x y a or     1
a b
4. Cycloid: x  a   sin  , y  a 1  cos 

5. Equiangular Spiral: r  ae  cot 

Q.No.1: Trace the following curve giving the salient points y 2 a  x   x 3 (Cissoid).

y-axis
Double tangent
x=a

x-axis

Sol.: The given curve is y 2 a  x   x 3 . (i)


1. Symmetry:
The curve is symmetrical about x-axis as these are even and only even powers of y in
the equation of the curve.
2. Origin:
As we know, if the co-ordinates of origin i.e. (0, 0) satisfy the given equation, then
the curve passes through the origin. Here this curve passes through the origin.
3. Tangents at origin:
The equation of given curve can be written as

ay 2  xy 2  x 3  x 3  xy 2  ay 2  0 .
Differential Calculus: Curve Tracing Prepared by: Dr. Sunil, NIT Hamirpur11

Equating lowest degree term to zero, we get ay 2  0  y 2  0  y  0, y  0 .


 The two tangents are real and coincident therefore the origin is cusp, in the present
problem.
4. Intersection with coordinate axis:
When x = 0  y  0 , when y  0  x  0 .
 The curve does not intersect the coordinate axis.
The curve meets the coordinate axes only at origin.
5. Special points:
From the equation of the curve

x3 / 2
y [taking positive sign for discussion]
ax
3 1/ 2 1
x a  x 1 / 2  a  x 1 / 2 x 3 / 2
dy 2 2 x 3a  2 x 
 
dx a  x  2a  x 3 / 2
3a
=0 when x = 0 or x  .
2
3a 3a
Rejecting the value of x  , because y is imaginary when x  .
2 2
The tangent at x = 0 is parallel to x-axis.
6. Imaginary values:

If x < 0, y 2 becomes negative and y is imaginary.


Hence no part of the curve lies in the second and third quadrants.
Also, if x > a, y becomes imaginary.
Hence no part of the curve lies beyond the point x = a.
7. Asymptotes:
Equating to zero the coefficient of y2, the highest degree term in y.
The asymptote parallel to y-axis is x  a  0  x  a
There is no asymptote parallel to x-axis.
Differential Calculus: Curve Tracing Prepared by: Dr. Sunil, NIT Hamirpur12

8. Shape:
y-axis
x=a

x-axis

Q.No.2.: Trace the following curve x 3  y 3  3axy , by studying its important


features/characteristics.. y-axis y=x
(Folium of Descartes).  3a 3a 
 , 
 2 2 
xya 0
x-axis
(0, 0)

Sol.: The given curve is x 3  y 3  3axy .


1. Symmetry:
If we interchange x and y, the given equation does not alter, thus there is
symmetry about the line y = x.
2. Origin:
As we know, if the co-ordinates of origin i.e. (0, 0) satisfy the given equation,
then the curve passes through the origin. Here this curve passes through the origin.
3. Tangent at origin:
Tangents at (0, 0) are given by equating lowest degree terms to zero. Thus
tangents at (0, 0) are given by
3axy  0  x  0 and y = 0 a  0
4. Intersection of the curve with co-ordinate axes:

The curve is x 3  y 3  3axy


When x = 0, we get y = 0
Differential Calculus: Curve Tracing Prepared by: Dr. Sunil, NIT Hamirpur13

When y = 0, we get x = 0.
Thus the curve does not intersect the co-ordinate axes.
5. Region in which the curve lies.

The given curve is x 3  y 3  3axy .


When x and y both are positive, the equation is satisfied when x and y both are negative
the equation is not satisfied, thus the curve does not exist in the third quadrant. When x is
positive and y is negative, the equation is satisfied. When x is negative and y is positive,
the equation is satisfied. Thus the curve exist in the first, second and fourth quadrant.
6. Asymptotes:
The curve has
(a). No asymptote parallel to x-axis.
(b). No. asymptotes parallel to y-axis.
(c). Let y = mx + c be an asymptote.

Putting this value of y in x 3  y 3  3axy

We get x 3  mx  c 3  3ax mx  c 

 x 3  m 3 x 3  c 3  3cmx(mx  c)  3amx 2  3acx

 x 3  m 3 x 3  c 3  3cm 2 x 2  3c 2 mx  3amx 2  3acx  0

     
 x 3 1  m 3  x 2 3cm 2  3am  x 3c 2 m  3ac  c 3  0 .
Equating coefficient of x3 equal to zero, we get

1  m 3  0  m  1
Equating coefficient of x2 equal to zero, we get

3cm 2  3am  0  cm  a  0  cm  a  c  a  m  1


Putting these values of m and c in
y  mx  c we get y   x  a
 x  y  a  0 is the required asymptote.
7. Intersection of the curve with the line y = x

x 3  y 3  3axy (i)
y=x (ii)
Differential Calculus: Curve Tracing Prepared by: Dr. Sunil, NIT Hamirpur14

Solving for x and y from (i) and (ii)

2 x 3  3ax 2
2x  3a
3a 3a
x and y  .
2 2

Differentiate x 3  y 3  3axy w.r.t. x, we get

dy  dy  dy dy
3x 2  3y 2  3a  x  y  x 2  y 2  ax  ay
dx  dx  dx dx

 dxdy  ay  x 2  dxdy  ayy 2 xax .


2
 y 2  ax

3a 9a 2 3 2 9 2
 a a  a
dy  3a 3a  2 4 2 4
The value of at  ,     1 .
dx  2 2  9a 2 3a 9a 2
3 2
a  a
4 2 4 2
 3a 3a 
This means that the tangent to the curve at  ,  makes an angle of 135o with the
 2 2
positive direction of x-axis, keeping in view all the points.
8. Shape: The shape of the curve is given as follows.

y-axis y=x
 3a 3a 
 , 
 2 2 
xya 0
x-axis
(0, 0)

Q.No.3.: Trace the following curve giving the salient points x 2 / 3  y 2 / 3  a 2 / 3


(Astroid or Four cusped hypocycloid).

Sol.: The given equation x 2 / 3  y 2 / 3  a 2 / 3 can be written in the parametric form as

x  a cos 3  and y  a sin 3  .


Differential Calculus: Curve Tracing Prepared by: Dr. Sunil, NIT Hamirpur15

1. Symmetry:

Parametric form: x  a cos 3  , y  a sin 3  .


Here x is even function and y is odd function.  Symmetrical about x-axis.
The parametric form can be written as

x  a sin 3  , y  a cos 3  .
( The value of x changes as  is changed to   but the value of y remains unaltered as
 is changed to   ).
Here x is odd function and y is even function  Symmetrical about y-axis.

If we interchange x and y in x 2 / 3  y 2 / 3  a 2 / 3 , the equation remains the same.


 there is symmetry about the line y = x.
2. Origin:
As we know, if the co-ordinates of origin i.e. (0, 0) satisfy the given equation,
then the curve passes through the origin. Clearly, the curve does not pass through the
origin.
3. Intersection of the curve with co-ordinate axes:
When x = 0, we get y =  a.
When y = 0, we get x =  a.
Thus, the curve meets x-axes at  a ,0 and y-axes at 0,a  .
4. Shape of curve at (a, 0) and (0, a):

x 2 / 3  y2 / 3  a 2 / 3
Differentiating w.r.t. x, we get
2 1 / 3 2 1 / 3 dy dy
x  y  0  x 1 / 3  y 1 / 3 0
3 3 dx dx

x 1 / 3
1/ 3
1 / 3 dy dy y
y   x 1 / 3    
dx dx 1 / 3 x
y

 dy 
   0 , the tangent is x-axis and (a, 0) is cusp as there is symmetry about x-
 dx  a ,0 

axis.
Differential Calculus: Curve Tracing Prepared by: Dr. Sunil, NIT Hamirpur16

 dy 
    , the tangent is y-axis and (0, a) is cusp as there is symmetry about y-axis.
 dx  0, a 

5. Region in which the curve lies:

x 2 / 3  y2 / 3  a 2 / 3

  2
y 2 / 3  a 2 / 3  x 2 / 3  y1 / 3  a 2 / 3  x 2 / 3 .
When x > a, y will be imaginary,  the curve does not exist when x > a. Similarly, the
curve does not exist when y > a.
6. Shape: Shape of the curve is given as follows:

y-axis
cusp (0, a)

x-axis
(a, 0)
cusp cusp

cusp

Q.No.4.: Trace the following curve giving the salient points


x  a   sin 
y  a 1  cos  from cusp to cusp. (Cycloid)
Sol.:
1. Symmetry:
Here x is an odd function and y is an even function
( The value of x changes as  is changed to   but the value of y remains unaltered as
 is changed to   ).
 The curve is symmetrical about y-axis.
2. Origin:
There is no value of  for which both x and y become zero simultaneously.
Hence, the curve does not pass through (0, 0).
3. Tangents at   0 and    :
Since x  a   sin  , y  a 1  cos  .
Differential Calculus: Curve Tracing Prepared by: Dr. Sunil, NIT Hamirpur17

dx dy
 a 1  cos  ,  a sin 
d d
dy dy d  a sin  
      tan .
dx d dx a 1  cos   2
When   0 : x = 0, y = 2a
dy
i.e. at (0, 2a),   tan 0  0 .
dx
i.e. the tangent at (0, 2a) is parallel to x-axis.
When    : x  a , y = 0
dy 
i.e. at a,0 or    ,   tan   .
dx 2
i.e. the tangent is parallel to y-axis.
4. Table for values of  , x and y is:
    0   0
  
3 6 6 3
x  a  3  1 0  1  3 a
 a     a   a   a   
3 2   6 2  6 2 3 2 
   
 1.913a
y 0 3a  3 2a  3 3a 0
a 1   a 1  
2
 2   2  2

5. Shape: One arch will be traced with this table.


With the aid of all the points discussed before, shape of the cycloid will be as under.

y-axis
vertex
(0, 2a)
cusp
cusp

x-axis

 a, 0 Base a, 0


*******************************************************
Differential Calculus: Curve Tracing Prepared by: Dr. Sunil, NIT Hamirpur18

PROCEDURE FOR TRACING OF POLAR CURVES:


The equation of a curve in the form r  f () or f r,   0 is known as polar
equation of the curve.
The procedure for tracing of such curves is fundamentally the same as for the
tracing of polar curves is given by as under.
1. Symmetry
(a). If the equation of a curve remains unaltered when  is changed to   ,
then the curve is symmetrical about the initial line.
(b). If the equation of a curve remains unchanged when  is changed to    or  is
changed to   and r to  r , then the curve is symmetrical about the line through the

pole and perpendicular to the initial line i.e. about the line   .
2

(c). If the equation of the curve remains unaltered when  is changed to ,
2

then the curve is symmetrical about the line   .
4
3
(d). If the equation of a curve remains unchanged when  is changed to ,
2
3
then the curve is symmetrical about the line   .
4
(e). If the equation of a curve is remains unchanged when r is replaced by  r ,
then the curve is symmetrical about the pole.
2. Pole
(a). Find if the pole lies on the curve. The pole will lie on the curve if for some real value
of  , we have r = 0.
(b). If the pole lies on the curve, the values of  for which r = 0, give the tangents to the
curve at the pole.
3. Determination of 
[The angle between the radius vector and the tangent to the curve at a point on the curve]
d
(a). Find tan   r . Then  gives the direction of the tangent of the curve at a point.
dr
Differential Calculus: Curve Tracing Prepared by: Dr. Sunil, NIT Hamirpur19


(b). Find the points on the curve for which  is 0 or . The tangent being parallel or
2
perpendicular to the initial line.
4. Limitation of the curve
(a). Let the least and the greatest value of r be a and b respectively, then the curve lies
with in a circle of a radius b but outside the circle of radius a.
(b). Solve the given equation of the curve for r in terms of  and find for what value of
 , r is imaginary. Let for      , the value of r be imaginary then no part of the
curve lies between the lines    and    .

5. Asymptotes
If for some value of  , r   , then the asymptotes exist.
6. Region
(a). Find the variation of r for positive and negative values of  , making values of  for
which r attains a maximum, minimum or zero value, when r is a periodic function of
 , the negative values of  which need not to be considered and curve is traced for
one period only.
(b). Giving suitable values of  , find the corresponding values of r to get some points on
the curve. Find also  for these values of  .

7. Approximate shape of the curve


Draw the rough sketch of the curve (approximate shape of the curve).

Q.No.5.: Trace the equiangular or Logarithmic spiral r  ae  cot  , where a and  are
constants.

Sol.: Given equation of curve is r  ae  cot  .


1. Symmetry:
No symmetry about initial line and no symmetry about pole.
2. Origin:
When   0 , r = a i.e. curve does not pass through origin.

3. r  ae  cot 
Differential Calculus: Curve Tracing Prepared by: Dr. Sunil, NIT Hamirpur20

Suppose  to be positive, as the magnitude of  increases the corresponding values of r


will increase.
Again suppose  to be negative, as the magnitude of  increases in the negative
direction the corresponding values of r will decrease.
4. Angle between radius vector and tangent at any point is constant:
d
We know that tan   r ,
dr
where  is the angle between radius vector and tangent at any point of the curve.
dr
Since r  aee  cot  .   ae  cot   cot   r cot 
d
d 1 tan 
   ,
dr r cot  r
d
and r  tan  .
dr
 tan   tan      ,
which means angle between radius vector and tangent at any point is constant.
Shape of the curve is shown as follows:

2/3 2/3
x  y
Q.No.3.: Trace the following curve giving the salient points     1
a b
(Astroid or Four cusped hypocycloid).
2/3 2/3
x  y
Sol.: The given equation is      1.
a b
1. Symmetry:

The parametric form can be written as x  a cos 3  , y  b sin 3  .


Differential Calculus: Curve Tracing Prepared by: Dr. Sunil, NIT Hamirpur21

Here x is even and y is odd.  symmetric about x-axis.

Also the parametric form can be written as x  a sin 3  , y  b cos 3  .


Here x is odd and y is even.  symmetric about y-axis.
Thus, the curve is symmetrical about both the axis, as there are even and only even
powers of both x and y in the equation of the curve.
2. Origin:
As we know, if the co-ordinates of origin i.e. (0, 0) satisfy the given equation,
then the curve passes through the origin. Clearly, the curve does not pass through the
origin.
3. Intersection with the axes:
When x = 0, we get y =  b.
When y = 0, we get x =  a.
Thus, the curve meets x-axes at  a ,0 and y-axes at 0, b  .
2/3 2/3
 y x
4. Special Points: From the equation of the curve   1  
b a
1 / 3 1 / 3
2 y 1 dy 2x 1
Differentiating w.r.t. x, we get     
3b b dx 3a  a
1/ 3
 y
   b2 y 
1/ 3
dy b  b   
  
dx a  x 1 / 3  a 2x 
 
 
a
 
dy
Now  0 , when y = 0
dx
From (i), when y = 0, x  a .
Hence, the tangents are parallel to x-axis at the points  a ,0
Also (i), when x = 0, y  b

Hence, the tangents are parallel to y-axis at the points 0,b 


5. Imaginary values:
From (i), we have
Differential Calculus: Curve Tracing Prepared by: Dr. Sunil, NIT Hamirpur22

2/3
y
when x  a ,    0,  y is imaginary.
b
Hence, no part of the curve lies beyond the lines x  b .
6. Asymptotes:
The curve has no asymptotes.
7. Region: From (i), when x = 0, y   b
y = 0, x   a .
Also as x increases from 0 to a, y decreases b to 0 in the first quadrant.
6. Shape: Shape of the curve is given as follows:

y-axis
cusp (0, b)

x-axis
(a, 0)
cusp cusp

cusp

*** *** *** *** ***


*** *** ***
***

Vous aimerez peut-être aussi